Download as pdf or txt
Download as pdf or txt
You are on page 1of 302

Pre Sea Review Materials

Deck

DP-TRN-01-F13/Rev.1/06Nov17
Confidentiality

This document is commercially confidential to UMTC Inc. and is provided to all recipients in
circumstances of confidence.
In particular, no part of this document may be disclosed or provided to any person or organization
without the express written consent of UMTC Inc.
Further, for the purpose of distribution control, no part of this document may be reproduced, by
photocopying, scanning, or otherwise without the express written consent of UMTC Inc.

Copyright

Copyright  UMTC Inc. 2018

Acknowledgement

Designed, written and produced by:


UMTC Inc.

2120 Leon Guinto St.


Malate
Philippines

Confidential to United Marine Training Center, Inc. Page a


Responsible Departments

UMTC – General Education Department


UMTC – Deck Prospective Officer Department

Authors

Document Preparation, Review and Approval

Section Name Signature Date

Anthony James Noakes


Self-Study Guidelines 11 / 5 / 2018
Peter Grunau

Note Taking and Interview Tips Regine Branzuela 11 / 5 / 2018

Ester Dinawanao
Mathematics 11 / 5 / 2018
Christian Kevin Villanueva

Physics Karizsa Denise Salvador 11 / 5 / 2018

Terrestrial Navigation for Randy Diamante 11 / 5 / 2018


Victor Rene Carlos

Confidential to United Marine Training Center, Inc. Page b


Document Control

Copy Status:
Document Location:
Developed by:
Last modified by:
Course Owner:
Print Date:
Total Pages: 301

Confidential to United Marine Training Center, Inc. Page c


Revision History

Date Version Author Comments and Authorizations

28 September 2018 0.01 P. Grunau Submission of contents for Self-Study


Guidelines
28 September 2018 0.02 R. Branzuela Submission of contents for Note Taking
and Interview Tips
28 September 2018 0.03 E. Dinawanao Submission of contents for Mathematics
C. K. Villanueva
28 September 2018 0.04 K. D. Salvador Submission of contents for Physics
28 September 2018 0.05 R. Diamante Submission of contents for Terrestrial
Navigation
02 October 2018 0.06 A. J. Noakes Review of the contents of Self-Study
Guidelines and Note Taking and
Interview Tips
04 October 2018 0.07 P. Grunau Review of the contents of Mathematics,
Physics and Terrestrial Navigation
08 October 2018 0.08 E. Dinawanao Revision of Mathematics and Physics
K. D. Salvador based on the review of version 0.07
C. K. Villanueva - Fixed references and captions

15 October 2018 0.09 C. K. Villanueva Consolidated all reviewed contents in the


UMTC Template
17 October 2018 0.10 E. Dinawanao Review and first proof reading of
K. D. Salvador consolidated contents
C. K. Villanueva
17 October 2018 0.11 C. K. Villanueva Revision based on the review of version
0.10
Draft for comment
18 October 2018 0.12 C. K. Villanueva Update bibliography and second proof
reading of consolidated contents
24 October 2018 0.13 E. Dinawanao Third proof reading of consolidated
K. D. Salvador contents

25 October 2018 0.14 C. K. Villanueva Revision based on the review of version


0.12 and version 0.13

Confidential to United Marine Training Center, Inc. Page d


Revision History

Date Version Author Comments and Authorizations

30 October 2018 0.15 C. K. Villanueva Revision based on the review and


feedback of Capt. Svilen
- Removed Panel Interview Tips
- Fixed formatting
- Added discussion for “Radical” under the
subtopic “Review of Arithmetic”
- Fixed figure for Example 3-90
- Replaced the symbol ∅ (phi) for central
angle with symbol 𝜃 (theta)
- Elaborated on the formula of
circumference under the subtopic
“Methods of Measuring Angles”
- Fixed notation for Course as 𝐶𝑜

5 November 2018 0.16 C. K. Villanueva Attached Introductory Letter for


candidates

Confidential to United Marine Training Center, Inc. Page e


Distribution

Location Responsibility

Course Owner, Subject Matter Expert Review and revise as required


Senior Faculty Approve revisions

Confidential to United Marine Training Center, Inc. Page f


Confidential to United Marine Training Center, Inc. Page g
Table of Contents
Section 1 ................................................................................................................. 1-1
Self-Study Guidelines ...................................................................................................1-1
What is Self-study? ....................................................................................................... 1-1
Aim of this Self-study Package .................................................................................... 1-1
Self-study Tips .............................................................................................................. 1-1

Section 2 ................................................................................................................. 2-8


Note Taking ...................................................................................................................2-8
Effective Note Taking.................................................................................................... 2-8

Section 3 ................................................................................................................. 3-1


Mathematics ..................................................................................................................3-1
Applied Algebra ............................................................................................................ 3-1
Review of Arithmetic................................................................................................ 3-1
Factors, Products, Exponents and Radicals ..................................................... 3-1
Fractions and Decimals ................................................................................... 3-4
Percentage.................................................................................................... 3-15
Scientific Notation.......................................................................................... 3-19
Linear Equation in One Variable ............................................................................ 3-22
Properties of Equality .................................................................................... 3-22
Strategies for Solving Linear Equation ........................................................... 3-24
Quadratic equations .............................................................................................. 3-28
Strategies for Solving Quadratic Equations .................................................... 3-28
Formula Transformation ........................................................................................ 3-38
Variation ............................................................................................................... 3-43
Types of Variation ......................................................................................... 3-43
Interpolation .......................................................................................................... 3-54
Plane Trigonometry and its Maritime Application ..................................................... 3-61
Definition of Angles ............................................................................................... 3-61
Units of Angle Measurement ................................................................................. 3-66
Review of Geometry of Circle ........................................................................ 3-66
Methods of Measuring Angles ....................................................................... 3-67
Angle Conversions – Degrees and Radians ................................................... 3-71
Plane Triangle Trigonometry ................................................................................. 3-76
Right Triangle................................................................................................ 3-76
Circular Functions ......................................................................................... 3-95
Oblique Triangles ........................................................................................ 3-101
Applications of Plane Trigonometry ..................................................................... 3-113
Angle of Elevation and Angle of Depression ................................................ 3-113
Directions and Bearings .............................................................................. 3-120
Sailings ....................................................................................................... 3-132
Plane Sailing ....................................................................................... 3-135
Parallel Sailing..................................................................................... 3-142
Mercator Sailing .................................................................................. 3-148
Spherical Trigonometry and its Maritime Application ............................................. 3-158

Confidential to United Marine Training Center, Inc. Page h


Right Spherical Triangle ...................................................................................... 3-164
Oblique Spherical Triangle .................................................................................. 3-171
Application of Spherical Trigonometry ................................................................. 3-178
Great Circle Sailing ..................................................................................... 3-178

Section 4 ................................................................................................................. 4-1


Physics ..........................................................................................................................4-1
Measurement................................................................................................................. 4-1
Vector Addition ............................................................................................................. 4-7
Kinematics .................................................................................................................... 4-8
Forces and Newton’s Laws ........................................................................................ 4-12
Work, Energy and Power ............................................................................................ 4-23
Work ..................................................................................................................... 4-23
Mechanical Energy................................................................................................ 4-25
Potential Energy ............................................................................................ 4-25
Kinetic Energy ............................................................................................... 4-27
Power ................................................................................................................... 4-28
Fluids........................................................................................................................... 4-31

Section 5 ............................................................................................................... 5-43


Terrestrial Navigation .................................................................................................5-43
Magnetic Compass ..................................................................................................... 5-43
Why is a Magnetic Compass Required in the Age of Electronic Navigation? .......... 5-43
Magnetic Compass Location and Installation ......................................................... 5-44
Variation and Deviation ......................................................................................... 5-46
Swinging the Compass, Swinging the Ship ............................................................ 5-47
Why Does a Compass Card Sometimes Tilt? ........................................................ 5-49
Why Engage a Professional, Qualified Compass Adjuster? ................................... 5-50
Legislative Requirements ...................................................................................... 5-51
SOLAS Chapter V ................................................................................................. 5-51
Problems involving Compass Error ........................................................................ 5-52
Gyro Compass ............................................................................................................ 5-54
Brief History .......................................................................................................... 5-54
Nautical Charts and Publications ............................................................................... 5-57
Chart Constructions .............................................................................................. 5-57
Chart Projections........................................................................................... 5-57
Classifications of Chart Projections ................................................................ 5-57
Information on Nautical Charts ...................................................................... 5-59
Chart Scales ................................................................................................. 5-60
How Navigational Aids Are Charted ............................................................... 5-60
Chart Correction ............................................................................................ 5-61
IALA...................................................................................................................... 5-63
Position Fixing Method ............................................................................................... 5-63
Lines of Position.................................................................................................... 5-63
Ranges ......................................................................................................... 5-64
Position Fix ................................................................................................... 5-65
Estimated Position......................................................................................... 5-66
Dead Reckoning............................................................................................ 5-67
Running Fix ................................................................................................... 5-68

Confidential to United Marine Training Center, Inc. Page i


Sailings........................................................................................................................ 5-69
Methods of Calculations ........................................................................................ 5-69

Section 6 ................................................................................................................. 6-1


Bibliography ..................................................................................................................6-1

Figures
Figure 2-1: Cornell method of note taking – Source: https://www.bcbe.org/Page/9514 2-8
Figure 2-2: T-note lay-out – Source:
https://www.westernsydney.edu.au/__data/assets/pdf_file/0008/1082663/Note-taking_techniques.pdf
2-10
Figure 2-3: Concept map lay-out – Source: http://www.leeds.ac.uk/educol/documents/188101.pdf 2-14
Figure 3-1: Mathematical and visual representation of fraction – Source: Christian Kevin Villanueva 3-
4
Figure 3-2: Equivalent fractions – Source: Christian Kevin Villanueva 3-4
Figure 3-3: Proper fractions – Source: Christian Kevin Villanueva 3-4
Figure 3-4: Improper fractions – Source: Christian Kevin Villanueva 3-5
Figure 3-5: Mixed numbers – Source: Christian Kevin Villanueva 3-5
Figure 3-6: Place values – Source: Christian Kevin Villanueva 3-10
Figure 3-7: Fractions as percentages – Source: Christian Kevin Villanueva 3-15
Figure 3-8: Speed versus time graph with equation 𝑠 = 1000𝑡; – Source: Christian Kevin Villanueva,
graphed using https://www.desmos.com/calculator 3-45
Figure 3-9: Interpolated data point between two known points – Source: Christian Kevin Villanueva,
Graphed using https:2//www.desmos.com/calculator 3-54
Figure 3-10: Angle between two rays – Source: Christian Kevin Villanueva 3-61
Figure 3-11: Angles in standard position – Source: Christian Kevin Villanueva 3-62
Figure 3-12: Positive and negative angles – Source: Christian Kevin Villanueva 3-62
Figure 3-13: Reference angle – Source: Christian Kevin Villanueva 3-62
Figure 3-14: Summary of solution for reference angle in each quadrant – Source: Christian Kevin
Villanueva 3-64
Figure 3-15: Definition of a circle – Source: Christian Kevin Villanueva 3-66
Figure 3-16: Radius, diameter and circumference of a circle – Source: Christian Kevin Villanueva 3-66
Figure 3-17: Central angle, minor arc and major arc of a circle – Source: Christian Kevin Villanueva 3-
67
Figure 3-18: Revolution as an angular measurement – Source: Christian Kevin Villanueva 3-68
Figure 3-19: Six equilateral triangles arranged inside a circle – Source: Christian Kevin Villanueva 3-
68
Figure 3-20: Analogy of degree measurement to pizza slice – Source: Christian Kevin Villanueva 3-69
Figure 3-21: Radian measure – Source: Christian Kevin Villanueva 3-69
Figure 3-22: Measure of central angle in radian for one revolution – Source: Christian Kevin
Villanueva 3-70

Confidential to United Marine Training Center, Inc. Page j


Figure 3-23: Degree-radian conversion chart - Source:
https://commons.wikimedia.org/wiki/File:Degree-Radian_Conversion.svg, Edited by: Christian Kevin
Villanueva 3-71
Figure 3-24: A right triangle – Source: Christian Kevin Villanueva 3-76
Figure 3-25: Reference figure for Example 3-71 – Source: Christian Kevin Villanueva 3-77
Figure 3-26: Reference figure for Example 3-73 – Source: Christian Kevin Villanueva 3-78
Figure 3-27: Reference figure for Example 3-74 – Source: Christian Kevin Villanueva 3-79
Figure 3-28: (a) A 45°- 45°- 90° special triangle and (b) a 30°- 60° -90° special triangle – Source:
Christian Kevin Villanueva 3-80
Figure 3-29: Reference figure for Example 3-75 – Source: Christian Kevin Villanueva 3-81
Figure 3-30: Right triangle with labelled angles – Source: Christian Kevin Villanueva 3-85
Figure 3-31: Right triangle with reference angle alpha (𝛼) – Source: Christian Kevin Villanueva 3-86
Figure 3-32: Reference figure for Example 3-79 – Source: Christian Kevin Villanueva 3-90
Figure 3-33: Reference figure for Example 3-33 – Source: Christian Kevin Villanueva 3-90
Figure 3-34: Reference right triangle for cofunctions – Source: Christian Kevin Villanueva 3-91
Figure 3-35: Unit Circle – Source: Christian Kevin Villanueva 3-95
Figure 3-36: Unit circle with angle 𝜃 – Source: Christian Kevin Villanueva 3-95
Figure 3-37: The coordinates of the points in the unit circle – Source: Christian Kevin Villanueva 3-97
Figure 3-38: Coordinates in a unit circle – Source: Christian Kevin Villanueva 3-98
Figure 3-39: Unit circle with angles and point coordinates equivalent - Source:
https://commons.wikimedia.org/wiki/File:Unit_circle_angles_color.svg 3-99
Figure 3-40: Signs of the 𝑥 and 𝑦 coordinates in the rectangular coordinate system – Source: Christian
Kevin Villanueva 3-99
Figure 3-41: Pneumonic for the signs of trigonometric functions – Source: Ester Dinawanao 3-100
Figure 3-42: (a) Acute triangle and (b) obtuse triangle – Source: Christian Kevin Villanueva 3-101
Figure 3-43: Two angles and one side case – Source: Christian Kevin Villanueva 3-102
Figure 3-44: Two sides and an angle opposite either sides – Source: Christian Kevin Villanueva 3-104
Figure 3-45: Two sides and an included angle – Source: Christian Kevin Villanueva 3-108
Figure 3-46: Three sides are given – Source: Christian Kevin Villanueva 3-110
Figure 3-47: Angle of depression and angle of elevation - – Source: Christian Kevin Villanueva 3-113
Figure 3-48: (a) Angle of depression and (b) Angle of elevation – Source: Christian Kevin Villanueva
3-113
Figure 3-49: Direction and bearing – Source: Christian Kevin Villanueva 3-120
Figure 3-50: Bearing – Source: Christian Kevin Villanueva 3-121
Figure 3-51: Relative and true bearing – Source: Christian Kevin Villanueva 3-122
Figure 3-52: Reciprocal bearing – Source: Christian Kevin Villanueva 3-123
Figure 3-53: Reciprocal Bearing of 330° – Source: Christian Kevin Villanueva 3-123
Figure 3-54: True bearing (blue arrow) and conventional bearing (green arrow) - Source: Christian
Kevin Villanueva 3-124
Figure 3-55: Terrestrial sphere – Source: Christian Kevin Villanueva 3-132
Figure 3-56: Lines of latitude – Source: https://kids.britannica.com/students/assembly/view/54583 3-
133
Figure 3-57: Latitude as angular measurement – Source: Christian Kevin Villanueva 3-133

Confidential to United Marine Training Center, Inc. Page k


Figure 3-58: Lines of longitude – Source: https://kids.britannica.com/students/assembly/view/54584 3-
134
Figure 3-59: Plane sailing – Source: Ester Dinawanao 3-135
Figure 3-60: Projected plane sailing triangle from globe – Source: Christian Kevin Villanueva 3-137
Figure 3-61: Orientations of the plane sailing triangles – Source: Christian Kevin Villanueva 3-137
Figure 3-62: Terrestrial Sphere – Source: The Sailings by Capt. Udo Moeller 3-142
Figure 3-63: Concentric circles – Source: The Sailings by Capt. Udo Moeller 3-142
Figure 3-64: Projection of Points A and B to Points D and E – Source: Christian Kevin Villanueva 3-
148
Figure 3-65: Meridional parts - Source: Christian-Kevin Villanueva 3-149
Figure 3-66: The DMP Triangle after projecting the DMP and DLONM from the globe 3-150
Figure 3-67: Mercator Sailing Triangle – Source: Christian-Kevin Villanueva 3-150
Figure 3-68: Breakdown of the Mercator sailing triangle - Source: Christian-Kevin Villanueva 3-151
Figure 3-69: Parts of the sphere - Source: https://www.mathalino.com/reviewer/solid-mensuration-
solid-geometry/sphere and
http://www.vikdhillon.staff.shef.ac.uk/teaching/phy105/celsphere/phy105_earth.html 3-158
Figure 3-70: Formation of the spherical triangle – Source: https://www.kisspng.com/png-spherical-
trigonometry-solution-of-triangles-spher-1024715/ 3-159
Figure 3-71: Sides and angles of a spherical triangle – Source: http://1.bp.blogspot.com/-
REiyDx5332U/UYlh9G6oRNI/AAAAAAAATZ4/WfIL20n-bdY/s1600/picture2.jpg 3-159
Figure 3-72: Right spherical triangle - Source: Christian Kevin Villanueva 3-164
Figure 3-73: Napier's circle of right spherical triangle of Figure 3-72 - Source: Christian Kevin
Villanueva 3-164
Figure 3-74: Terrestrial sphere - Source: Christian Kevin Villanueva 3-178
Figure 3-75: Terrestrial triangle with labels on its parts - Source: Christian Kevin Villanueva 3-178
Figure 3-76: Spherical Triangles with labels in Mathematics and Navigational Terms - Source:
Christian Kevin Villanueva 3-179
Figure 4-1: Distance and displacement - Source: www.marinetraffic.com 4-8
Figure 4-2: Applied (a) pushing force and (b) pulling force - Source: Karizsa Denise D. Salvador 4-12
Figure 4-3: Normal force -- Source: Karizsa Denise D. Salvador 4-13
Figure 4-4: Friction force - Source: Karizsa Denise D. Salvador 4-13
Figure 4-5: Tension force - Source: Karizsa Denise D. Salvador 4-14
Figure 4-6: Gravitational force - Source: Karizsa Denise D. Salvador 4-14
Figure 4-7: An object at rest will remain at rest – Source:
https://fairplay.ihs.com/bulk/article/4293706/divergent-fortunes-forecast-for-tanker-and-bulk-markets
4-15
Figure 4-8: Unless acted upon by an unbalanced force – Source:
https://hendersoninternational.wordpress.com/2014/12/21/improve-a-propeller-improve-a-ships-
voyage/ 4-15
Figure 4-9: An object in motion will continue with constant speed and direction – Source:
http://vansail.com/service/sea-freight/ 4-16
Figure 4-10: Unless acted upon by an unbalanced force – Source:
http://www.pinoyseamanph.com/2017/11/marina-exam-reviewer-if-ships-bow-has.html 4-16
Figure 4-11: Law of Acceleration - Source: Karizsa Denise D. Salvador 4-17
Figure 4-12: Law of accelaration_2 - Source: Karizsa Denise D. Salvador 4-17

Confidential to United Marine Training Center, Inc. Page l


Figure 4-13: Law of interaction - Source: Karizsa Denise D. Salvador 4-17
Figure 4-14: FBD - Source: Karizsa Denise D. Salvador 4-18
Figure 4-15: Zero Angle - Source: Karizsa Denise D. Salvador 4-24
Figure 4-16: 180° angle - Source: Karizsa Denise D. Salvador 4-24
Figure 4-17: 90° angle - Source: Karizsa Denise D. Salvador 4-24
Figure 4-18: Negative work - Source: Karizsa Denise D. Salvador 4-25
Figure 4-19: Gravitational potential energy – Source: https://jp.freepik.com/free-photos-vectors/vessel
4-26
Figure 4-20: Change in potential energy - Source: Karizsa Denise D. Salvador 4-26
Figure 4-21: Work-energy theorem - Source: Karizsa Denise D. Salvador 4-27
Figure 4-22: Force acting on a submerged object – Source:
http://www.atmo.arizona.edu/students/courselinks/fall16/atmo336s2/lectures/sec1/Fluid_Pressure.htm
l 4-32
Figure 4-23: Forces acting on a submerged object - Source:
http://www.atmo.arizona.edu/students/courselinks/fall16/atmo336s2/lectures/sec1/Fluid_Pressure.htm
l 4-33
Figure 4-24: Pressure applied by the fluid at the bottom of the container – Source: https://www.school-
for-champions.com/experiments/fluids_pressure_depth_relationship.htm#.W3TaNugzaUk 4-34
Figure 4-25: Enclosed fluid in a pipe with two pistons - Source:
http://pascalteam.hu/en_pascal_law.php 4-35
Figure 4-26: Fluid Flow - Source: https://www.slideshare.net/rexparis36/05-lecture-outline 4-36
Figure 4-27: Internal Pressure - Source: https://slideplayer.com/slide/3621809/ 4-37
Figure 4-28: Laminar Flow vs Turbulent Flow – Source: https://www.cfdsupport.com/openfoam-
training.html 4-37
Figure 5-1: Magnetic Compass - Source: http://www.compassadjustment.com/ 5-45
Figure 5-2: Magnetic Chart – Source: http://www.compassadjustment.com/ 5-46
Figure 5-3: Shadow Pin – Source:https//safety4sea.com/how-to-maintain-and-adjust-magnetic-
compasses/ 5-47
Figure 5-4:Signal Flag for Compass adjustment - Source: http://www.compassadjustment.com/ 5-48
Figure 5-5:Earth magnetic Field – Source: http://www.compassadjustment.com/ 5-49
Figure 5-6: Compass liquid - Source: http://www.compassadjustment.com/ 5-50
Figure 5-7: Jean- Bernard-Leon Foucault – Source:
https://de.wikipedia.org/wiki/L%C3%A9on_Foucault 5-54
Figure 5-8: Elmer A. Sperry - Source: https://de.wikipedia.org/wiki/L%C3%A9on_Foucault 5-54
Figure 5-9: Construction of a Compass - Source: https://www.marineinsight.com/marine-
navigation/gyro-compass-on-ships-construction-working-and-usage/ 5-56
Figure 5-10: Chart Projection – Source:
ttps://centrodemidias.am.gov.br/dmdocuments/18J4GEO004P2.pdf 5-57
Figure 5-11: Distortion – Source: https://www.e-education.psu.edu/geog160/node/1918 5-58
Figure 5-12: Navigational Aids - Source: Snip from MARES 5-61
Figure 5-13: Line of Position – Source: http://www.sailingissues.com/pdfs.html 5-64
Figure 5-14: To get the LOP - Source: www.sailingissues.com 5-64
Figure 5-15: Avoiding dangerous wreck - Source: http://www.sailingissues.com/pdfs.html 5-64
Figure 5-16: The position fix - Source: http://www.sailingissues.com/pdfs.html 5-65
Figure 5-17: Construction of LOP - Source: http://www.sailingissues.com/pdfs.html 5-66

Confidential to United Marine Training Center, Inc. Page m


Figure 5-18: Estimate Position - Source: http://www.sailingissues.com/pdfs.html 5-66
Figure 5-19: Dead Reckoning- Sources: http://www.sailingissues.com/pdfs.html 5-67
Figure 5-20: Running Fix - Sources: http://www.sailingissues.com/pdfs.html 5-69

Tables
Table 3-1: Diameter and circumference relationship – Source: Ester Dinawanao 3-43
Table 3-2: Table of angle of heel vs. GZ for Example 3-66 – Source: Ester Dinawanao 3-54
Table 3-3: Traverse Table for Example 2.67 – Source: Ester Dinawanao 3-58
Table 3-4: Derivation of the conversion factors radian to degree and vice –versa- Source: Ester
Dinawanao 3-72
Table 3-5: Conversion table of common angles – Source: Ester Dinawanao 3-74
Table 3-6: Table of common trigonometric functions – Source: Ester Dinawanao 3-85
Table 3-7: The definition of the three basic trigonometric function – Source: Ester Dinawanao 3-86
Table 3-8: The definition of the reciprocal of the three basic trigonometric function – Source: Ester
Dinawanao 3-86
Table 3-9: Trigonometric functions of 𝛼 and 𝜃 of triangle of Figure 3-34 – Source: Ester Dinawanao 3-
91
Table 3-10: Cofunction pair – Source: Ester Dinawanao 3-92
Table 3-11: Cofunction Identities – Source: Ester Dinawanao 3-92
Table 3-12: Quadrantal and three-figure notation – Source: Ester Dinawanao 3-120
Table 3-13: Derivation of Formulas used in Great Circle Sailing Based on the Principles of Solving
Oblique Spherical Triangles - Source: Ester Dinawanao 3-182
Table 3-14: Summary of the Derived Formulas -Source: Ester Dinawanao 3-182
Table 4-1: SI base units - - Source: Karizsa Denise D. Salvador 4-1
Table 4-2: SI derived units - - Source: Karizsa Denise D. Salvador 4-2
Table 4-3: SI prefixes - - Source: Karizsa Denise D. Salvador 4-2
Table 4-4: English units - Source: Karizsa Denise D. Salvador 4-2
Table 4-5: Conversion Factors - Source: Karizsa Denise D. Salvador 4-4

Confidential to United Marine Training Center, Inc. Page n


Pre Sea Review Materials <course code>
Version Number
Deck
5 November 2018

Section 1
Self-Study Guidelines

What is Self-study?

Self-studying is a teaching technique that involves studying without direct supervision or attendance in a
classroom. It is becoming a popular means of learning because it is self-paced and can be conducted at
the learners’ own time and discretion. If conducted correctly, it is a valuable way to learn.
All teaching methods and techniques have pros and cons. As stated, Self-study is a good way to learn,
however, there are certain requirements that you will have to commit to successfully complete this field of
study and if not complied with will unfortunately result in failure and a waste of your time.
 You must set up a disciplined regime of study time
 You must complete all examples (Moreover, you must understand what you are doing. If the
information supplied in this package is not sufficient, you must be prepared to research until you
develop meaning and understanding.)
 If you do not understand language or terminology used in this package, you must research
(dictionary) thesaurus (meaning) to fully understand the language used.
 Use whatever resources are available to you to complete all work. These could include;
 Internet (e.g. Google, YouTube, company intranet, etc.)
 Books on board (publications, SMS, logs, etc.)
 Peers (Officers)

Aim of this Self-study Package

The aim of this self-study package is to provide you underpinning information and examples of work that
will be required in the Prospective Officer program 2019. The information is based on the Pre – Sea
Phase of the training and, if completed correctly, will provide the underpinning knowledge and skills for
you to complete the entrance examination and pre-sea training. The content should not be entirely new to
you as you would have done similar in your previous schooling. The program realizes that you have been
out of the formal education system for some time and that you will need some background training to cope
up with the demands of the Marlow Prospective Officer Training Program.

Self-study Tips

There are various self-studying methods you can implement to help you improve your understanding.
 Research books and articles on a topic of study
 Use the internet to search for related material

Confidential to United Marine Training Center, Inc. Page 1-1


Pre Sea Review Materials <course code>
Version Number
Deck
5 November 2018

 Watch educational videos


 Work through practice questions to reinforce skills that are learned before attempting the
examples in the self-study package as these will be marked.

A designated study area is crucial for effective self-studying. This might mean a home office or a desk in
your cabin. No matter where it is, it should include a tidy work space – free of clutter and distractions, with
good lighting. If possible, access to computer is sometimes necessary for research. You will also need
stationery and a good scientific calculator (that you know how to use).
Stationery required:
 Pens / pencils
 Ruler
 Post-it notes
 Colored highlighters

Note-taking tools such as highlighters, colored pens, and sticky notes are useful tools for you if you are
studying on board. Keeping notes while learning will enable you to retain the information longer, and help
to build valuable organizational skills. Here are some useful tips for a successful reviewing and
preparation for examination:

1. Learn the same information in a variety of ways


The research conducted (Willis, J. 2008) shows that different media stimulate different parts of the brain.
The more areas of the brain that are activated, the more likely it is that you’ll understand and retain the
information. So to learn a specific topic, you could do the following:
 Read the class notes
 Read the textbook
 Watch a video
 Look up other online resources
 Create a mind map
 Teach someone what you’ve learned
 Do practice problems from a variety of sources
Of course, you won’t be able to do all of these things in one sitting. But each time you review the topic,
use a different resource or method – you’ll learn faster this way.

2. Study multiple subjects each day, rather than focusing on just one or two subjects
It’s more effective to study multiple subjects each day, than to deep-dive into one or two subjects. For
example, if you’re preparing for exams in math, Terrestrial Navigation, physics, and Cargo Handling, it’s
better to study a bit of each subject every day. This approach will help you to learn faster than by focusing
on just Math on Monday, Terrestrial Navigation on Tuesday, Physics on Wednesday, Cargo Handling on
Thursday, and so on.
Why?
Because you’re likely to confuse similar information if you study a lot of the same subject in one day. So to
study smart, spread out your study time for each subject. In doing so, your brain will have more time to
consolidate your learning.

Confidential to United Marine Training Center, Inc. Page 1-2


Pre Sea Review Materials <course code>
Version Number
Deck
5 November 2018

3. Review the information periodically, instead of cramming


Periodic review is essential if you want to move information from your short-term memory to your long-
term memory. This will help you get better exam grades. The optimal review interval varies, depending on
how long you want to retain the information. The following review intervals work well.
 1st review: 1 day after learning the new information
 2nd review: 3 days after the 1st review
 3rd review: 7 days after the 2nd review
 4th review: 21 days after the 3rd review
 5th review: 30 days after the 4th review

4. Don’t multitask
The data is conclusive: Multitasking makes you less productive, more distracted, and dumber. The studies
even show that people who claim to be good at multitasking aren’t actually better at it than the average
person. Effective students focus on just one thing at a time. So don’t try to study while also intermittently
replying to Facebook messages, watching a movie, and checking your Twitter feed. Here are some
suggestions to improve your concentration:
 Take everything away which might cause distraction
 Log out of all instant messaging programs
 Turn off the internet access on your computer
 Close all of your internet browser windows that aren’t related to the assignment you’re working
on
 Clear the clutter from your study area
 Turn off music player or do not use the phone for listening to music
 Concentrate on the topic you like to review
 Use other hand-outs which might help and assist you in solving the problem, like formula
booklet, nautical publication, etc.

5. Simplify, summarize, and compress the information


Use mnemonic devices like acronyms, as these are proven to increase learning efficiency.
Example 1-1

If you want to memorize the electromagnetic spectrum in order of increasing frequency, you could use
this acronym/sentence:

Raging Martians Invaded Venus Using X-ray Guns

(In order of increasing frequency, the electromagnetic spectrum is: Radiowave, Microwave, Infrared,
Visible, Ultraviolet, X-rays, Gamma rays.)

Example 1-2

Question: Stalactites and stalagmites– which ones grow from the top of the cave and which ones grow
from the ground?

Answer: Stalactites grow from the top, while stalagmites grow from the ground.

Confidential to United Marine Training Center, Inc. Page 1-3


Pre Sea Review Materials <course code>
Version Number
Deck
5 November 2018

Study smart by using mnemonic devices whenever possible. In addition, you could summarize the
information into a comparison table, diagram, or mind map. These tools will help you learn the
information much faster.

6. Take notes by hand, instead of using your laptop


Scientists recommend this, and not just because you’re more likely to give in to online distractions when
using your laptop. Even when laptops are used only for note-taking, learning is less effective.
Why?
Because students who take notes by hand tend to process and reframe the information. In contrast,
laptop note-takers tend to write down what the teacher says word-for-word, without first processing the
information. As such, students who take notes by hand perform better in tests and exams.

7. Write down your worries


 Will I do well on this exam?
 What if I forget the key concepts and equations?
 What if the exam is harder than expected?
These kinds of thoughts probably run through your head before you take an exam. But if these thoughts
run wild, the accompanying anxiety can affect your grades.
Here’s the solution.
In one experiment, researchers at the University of Chicago discovered that students who wrote about
their feelings about an upcoming exam for 10 minutes performed better than students who didn’t. The
researchers say that this technique is especially effective for habitual worriers.
Psychologist Kitty Klein has also shown that expressive writing, in the form of journaling, improves
memory and learning. Klein explains that such writing allows students to express their negative feelings,
which helps them to be less distracted by these feelings.
To be less anxious, take 10 minutes and write down all the things related to the upcoming exam that
you’re worried about. As a result of this simple exercise, you’ll get better grades.

8. Test yourself frequently


Decades of research has shown that self-testing is crucial if you want to improve your academic
performance. In one experiment, University of Louisville psychologist Keith Lyle taught the same statistics
course to two groups of undergraduates.
For the first group, Lyle asked the students to complete a four- to six-question quiz at the end of each
lecture. The quiz was based on material he’d just covered. For the second group, Lyle didn’t give the
students any quizzes.
At the end of the course, Lyle discovered that the first group significantly outperformed the second on all
four midterm exams. So don’t just passively read your textbook or your class notes. Study smart by
quizzing yourself on the key concepts and equations. And as you prepare for a test, do as many practice
questions as you can from different sources.

Confidential to United Marine Training Center, Inc. Page 1-4


Pre Sea Review Materials <course code>
Version Number
Deck
5 November 2018

9. Connect what you’re learning with something you already know


The more strongly you relate new concepts to concepts you already understand, the faster you’ll learn the
new information (McDaniel & Roedinger). For example, if you’re learning about Great Circle sailing, you
could relate it to Spherical trigonometry. Spherical trigonometry deals with Napier’s rules. Napier’s rule will
be used to find the formula to be used to calculate the course and the distance and so on. Another
example: You can think of white blood cells as “soldiers” that defend our body against diseases, which are
the “enemies”. It takes time and effort to think about how to connect new information to what you already
know, but the investment is worth it.

10. Read key information out loud.


Studies have been conducted, which demonstrate that reading information out loud helps students to
learn faster than by reading silently. What’s the reason for this?
When you read information out loud, you both see and hear it. On the other hand, when you read
information silently, you only see it. It isn’t practical to read every single word of every single set of notes
out loud. That would take way too much time. So here’s the process recommended:
This is a step by step procedure that can guide you.
Step 1: As you read your notes, underline the key concepts/equations. Use a marker for this. Don’t stop to
memorize these key concepts/equations; underline them and move on.
Step 2: After you’ve completed Step 1 for the entire set of notes, go back to the underlined parts and read
each key concept/equation out loud as many times as you deem necessary. Read each
concept/equation slowly. This might be for two times or might be also for five to six times.
Step 3: After you’ve done this for each of the underlined key concepts/equations, take a short break.
(about 5 minutes)
Step 4: When your break is over, go to each underlined concept/equation one at a time, and cover it
(either with your hand or a piece of paper). Test yourself to see if you’ve actually memorized it.
Step 5: For the concepts/equations that you haven’t successfully memorized, repeat Steps 2, 3, and 4.

11. Take regular study breaks


Taking regular study breaks enhances overall productivity and improves focus. That’s why it isn’t a good
idea to hole yourself up in your room for 3-4 hours straight to study for an exam or to review. You might
feel like you get a lot done this way, but the research proves otherwise. So take a 5- to 10-minute break
for every 45 minutes of work. During your break, refrain from playing on your computer or using your
phone. These devices prevent your mind from fully relaxing.

12. Reward yourself at the end of each study session.


Before starting a study session, set a specific reward for completing the session. By doing this, you’ll
promote memory formation and learning (Adcock RA, 2006). The reward could be something as simple
as:
 Going for a short walk
 Eating a healthy snack
 Listening to your favorite music
 Stretching
 Doing a couple of sets of exercise
 Playing a musical instrument
Reward yourself at the end of every session – you’ll study smarter and learn faster.

Confidential to United Marine Training Center, Inc. Page 1-5


Pre Sea Review Materials <course code>
Version Number
Deck
5 November 2018

13. Focus on the process, not the outcome


Successful students concentrate on learning the information, not on trying to get a certain grade. You will
later on not be able to have the full understanding. A lot of students try to learn only for an examination,
means learning for the outcome, but did not understand the process.
For example, you have to solve a mathematical problem. Your approach is:
 Find out which formula must be used.
 Do I have to rearrange the formula for the parameter I have to solve?
 What parameters are on hand?
 Which parameters are unknown?
 How to get these parameters?

If you use this approach, you are focusing of the result and not on the process. This process remains for
nearly all problems regardless what problem you have to solve. Therefore:
 Focus on effort, not the end result
 Focus on the process, not on achievement
 Believe you can improve – even in your weak subjects – as long as you put in the time and hard
work
 Embrace challenges
 Define success as pushing yourself to learn something new, not as getting straight A’s
Try to set performance goals and learning goals. What is the difference between these two types of
goals? Performance goals (e.g. getting 90% on the next terrestrial test), is about looking intelligent and
proving yourself to others. In contrast, learning goals (e.g. doing and solving three problems in physics
every other day, learning five rules in COLREGS a day) are about mastery and growth.

14. Drink at least eight glasses of water a day


You probably think you drink enough water, but studies show that up to 75% of people are in a chronic
state of dehydration. Dehydration is bad for your brain – and your exam grades and learning too.
Studies and research have found that your brain’s overall mental processing power decreases when
you’re dehydrated. Further research has shown that dehydration even causes the grey matter in your
brain to shrink.
The simple solution?
Drink at least eight glasses of water a day. Have a bottle of water present if you start your review, and
drink water before you start to feel thirsty.
Every 40 minutes or so, drink some water. This will help you stay hydrated and improve your
performance. Plus, this also acts as a short break to refresh your mind.

15. Be well rested before you start your learning activities, and don’t pull all-nighters
If the human body is well rested, the brain works much better compared to not being well rested, because
you will feel tired after a while and will be less concentrated. If you end your day job on deck (e.g. 1800
hours), start with your review work at 2000 hours. Keep a good time distance between the normal day job
and your reviews. Sleep expert Dan Taylor says that learning the most difficult material immediately
before going to bed makes it easier to recall the next day. So whenever possible, arrange your schedule
such that you study the hardest topic right before you sleep. Lastly, don’t pull all-nighters. As psychologist
Pamela Thacher’s research shows, students who pull all-nighters get lower grades and make more
careless mistakes.

Confidential to United Marine Training Center, Inc. Page 1-6


Pre Sea Review Materials <course code>
Version Number
Deck
5 November 2018

Benefits
Self-learning does take a lot of discipline and can be difficult at first, but like any endeavour, it becomes
easier with time. Self-study, when done correctly, is a very effective learning tool, so it can be helpful
when used to prepare for a test or learn an entirely new subject matter on your own. Self-study also
means self-discipline. You are the main person, the main actor of this program.

Summary
Set realistic goals. Setting work goals for yourself, ones that realistically fit in with your life and other
commitments, is important when creating self-study habits. You can set yourself up for success by
assigning only a certain number of topics to read each night, adjusting your workload according to how
hectic your schedule is in any given week, and giving yourself a mental break each week to let your mind
rest. If you think that you are not able to concentrate for any self- study at this day, do not start the
program. Take a break. Never set yourself under pressure to complete a task because you think you are
not in the time frame you have worked out for you.
Find what works for you. There are many different ways to learn, and it is important to adjust studying
techniques to find what works for your brain. Some students find reading aloud helpful, others like taking
handwritten notes rather than typing. Discover whatever works best for you, and stick with it. Review
material the same day you learn it. After taking notes for a topic or reading the next chapter in your
textbook, make sure you review all the new material, by typing up your notes, practicing your new skill, or
reading over a chapter again, to help it resonate. While this may seem tedious, it only takes a short
amount of time. Reviewing can help with long-term absorption of material, so it decreases the need of
cramming in the future.
Study in short, frequent sessions. Instead of treating your study session like a marathon, break up your
material by topic into a series of short sessions, separated by short breaks. That way, you won’t be staring
at your books or computer for too long while wearing on your focus, and your brain can absorb the
material more easily. While cramming may seem like a great way to cover a lot of material in a condensed
amount of time, studying in short, frequent sessions is a more effective way to learn subject matter and
self-study.
Prepare and maintain your study environment. When learning remotely, it is important to create a study
space for yourself. By setting aside a desk or table that is a designated environment for self-studying, you
will know to be mentally prepared to learn when you enter that space.
If you are practicing exercises, during the first attempt, use your notes and any information you have
found to solve the problem. Don’t be ashamed to use the material for the first run. It will assist you. You
can also seek advice from one of your Officers and check if your approach is correct. It is always
advisable to repeat the exercises two or three times, but the last practice should be conducted without
your study material.

Good luck!!!

Confidential to United Marine Training Center, Inc. Page 1-7


Pre Sea Review Materials <course code>
Version Number
Deck
5 November 2018

Section 2
Note Taking

Effective Note Taking

Note taking is an essential skill for studying. It allows a permanent record for revision. It also provides a
register of relevant points that one can integrate with one’s own writing and speaking. It also reduces the
risk of plagiarism and helps one distinguish where ideas came from and how one thinks about those
ideas.
Effective note taking requires recognizing the main ideas, identifying what information is relevant for one’s
task, and having a system of note taking that works for one’s purpose. It also requires reducing the
information to note and diagram format, and where possible, putting information in one’s own words.
Finally, it requires recording of the source of information. When doing notes for reading, one must be
selective. He or she must identify the purpose and function of a text and has to include one’s thoughts.
Here are some suggested note-taking methods that can be used:

Cornell method
Cornell method divides the page into three parts. The first part is the “Note Taking” section. In the note
taking section, you take down the important points of the lecture. Try to avoid writing in long sentences to
save space. The second part of the system is the “Keywords and Questions” section. This section should
be about half as wide as the “Note Taking” section. In this space you’ll want to write down any important
terms, questions you want to ask, or questions you think might be asked. This will really help you when
you’re studying your notes. The last section is the “Summary” section and it’s at the bottom of the page.
Save about 5-7 lines of paper to re-write and summarize the notes and questions from the lecture. The
lay-out of the Cornell method is shown in Figure 2-1

Figure 2-1: Cornell method of note taking – Source: https://www.bcbe.org/Page/9514

Confidential to United Marine Training Center, Inc. Page 2-8


Pre Sea Review Materials <course code>
Version Number
Deck
5 November 2018

Refer to Example 2-1 for the Cornell method of note taking.

Example 2-1

What is distance? Distance - scalar quantity which refers to “how much


ground an object has covered” during its motion.
- only has magnitude and no direction

What is displacement? Displacement - vector quantity which refers to the object’s


overall change in position.
- has both magnitude and direction

What is speed? Speed - scalar quantity that refers to “how fast an object
is moving”.
- rate of which an object covers distance
How to compute for speed? 𝑑
𝑣=
𝑡
What is velocity? Velocity - vector quantity which describes the rate at
which the position changes
𝑑⃑
How to compute for velocity? 𝑣⃑ =
𝑡
What is an acceleration? Acceleration - the rate of change of velocity with time
𝑣𝑓 − ⃑⃑⃑⃑
⃑⃑⃑⃑⃑ 𝑣𝑖
How to compute for acceleration? 𝑎⃑ =
𝑡
An object can have a change in velocity by:
When can an object accelerate? - changing speed only
- changing direction only
- changing both speed and direction
Summary:
Distance refers to how much ground and object covers while displacement refers to the object’s
change in position. Speed only tells how fast an object is moving while velocity describes how fast
the object is changing its position. Acceleration can be attained either of the three ways: change
speed, change direction or change both speed and direction.

Some of the advantages of a Cornell method are it is organized and systematic for recording and
reviewing notes; it has an easy format when pulling out major concept and ideas.

T-note method
T-note method or sometimes referred to T-method is a fairly variation of the Cornell method. It is
specifically useful for scientific and mathematical equations although widely used also in other disciplines.
The paper should be divided into two columns and the title should be on top part of the paper. The form of
this method creates a boarder that looks like letter T and thus, the name of this method suggests.
The equations or the key points are placed on the left column then the definition, working or function of
the equation and the description are placed on the opposite side or right column. The order of the
equations and the descriptions can also be interchangeable. Steps for examples or practice problems are
listed on one side of the "T", while the opposite side is used to record notes for one’s self or questions that
one needs to have answered. Figure 2-2 shows the lay-out of the T-note method. This method is also
used in this review material for the sample problems in math and in physics section.

Confidential to United Marine Training Center, Inc. Page 2-9


Pre Sea Review Materials <course code>
Version Number
Deck
5 November 2018

Figure 2-2: T-note lay-out – Source: https://www.westernsydney.edu.au/__data/assets/pdf_file/0008/1082663/Note-


taking_techniques.pdf

Example 2-2 shows the T-note method of note taking,

Example 2-2

What is the mass of HFO in a 2 000 liter capacity tank filled up to 90%? The relative density of the HFO
is 80%.

Given:

Values Distinction

2 000 𝑙𝑖𝑡𝑟𝑒𝑠 = Capacity of tank

90 % = Percentage of HFO inside the tank

80 % = Relative Density or Specific Gravity of HFO

Formula and calculation for mass,

M= ρ∙V Transformed formula for mass given density and volume

Prior using the previous formula, volume of HFO must first be


solved

Confidential to United Marine Training Center, Inc. Page 2-10


Pre Sea Review Materials <course code>
Version Number
Deck
5 November 2018

Voil = ηVtank Formula for Percentage

Where:

Voil = Volume of Oil in Tank

= Percentage of Vtank filled


η

= Volume Capacity of Tank


Vtank

The previous formulas show the practice of marine engineers that


tanks must only be filled up to 90%. This is to provide space for the
oil to move freely when the vessel is steaming and for thermal
expansion when the fuel is heated

Voil = 0.90(2 000 liters) Substitute the given values to solve for the volume of HFO in the
tank

η = 90 % = 0.90

Voil = 1 800 liters Volume of HFO

Prior solving the mass, the density of HFO must also be solved

ρliquid = SG ∙ ρfreshwater Transformed formula for density of HFO

MT Substitute the given values to solve for the volume of HFO in the
ρliquid = 0.8 ∙ 1
m3 tank

SG = 80 % = 0.80

MT Density of HFO
ρliquid = 0.8
m3

MT Substitute the solved values to the transformed formula for mass


M = (0.8 ) (1.8m3 )
m3 but make sure that the units agree with one another.

Convert liters to cubic meters,

Confidential to United Marine Training Center, Inc. Page 2-11


Pre Sea Review Materials <course code>
Version Number
Deck
5 November 2018

1 m3
Voil = 1 800 liters ( ) = 1.8 m3
1 000 liters

𝐌 = 𝟏. 𝟒𝟒 𝐦𝟑 Answer

Sentence Method
Sentence method is just simply writing down each topic as a jot note sentence. Write every new thought,
fact or topic on a separate line, numbering as one progress in studying. With this method, one simply
writes every new concept or topic on a separate line. The information can also be numbered. The usage
of some form of visual aid to group related points together is also recommended. Jotting main points helps
one determine which information is important and which is least important. Notes are also simplified for
studying and reviewing. This method is slightly more organized than a paragraph.
Example 2-2 shows the Sentence method of note taking,

Example 2-3

A free-body diagram is a way of representing forces acting on a certain object or body


using arrows which represent the forces acting on a body. The length of the arrow depicts
the magnitude of the force while the arrow head represents the direction of the force.
Figure 1 shows some of the contact forces such as normal force which is defined as a
support force acting perpendicular with the surface hence the name normal. It is a force
exerted on an object by any surface with which it is in contact. The red arrow shows the
friction force that is present whenever two bodies or two surfaces are in direct contact
with each other. Friction always opposes the direction of the motion of the object. The
green arrow represents the applied force which is defined as a force exerted by one
body to another body. When a box is being pulled by a rope and that rope applied a
tension force on the box. Tension force is a pulling force exerted by a stretched rope,
cord or chain on an object to which it is attached to.

Sample Note:

FBD – uses arrows to represent forces acting on a body

Normal force – support force perpendicular with the surface

Friction force – opposes the direction of the motion

Applied force – exerted by one body to another

Tension force – pulling force exerted by strings, ropes and alike.

Confidential to United Marine Training Center, Inc. Page 2-12


Pre Sea Review Materials <course code>
Version Number
Deck
5 November 2018

Charting Method
Charting method is great for subject matter that can be broken down into categories, such as branches,
similarities/differences, date/event/impact, pros/cons. When reviewing, it is effective in listing down the
equations per topic so that it will be easier to solve and familiarize with the equations.
Example 2-4

Angle between
𝒄𝒐𝒔 𝜽 𝒊𝒔 Work done
displacement and force

0° ≤ 𝜃 ≤ 90° positive positive


𝜃 = 90° zero zero
90° < 𝜃 ≤ 180° negative negative

The advantages of charting method include facts are organized; it is easy to review notes and; it highlights
key pieces of information for each topic.

Outlining Method
Outlining method naturally organizes the information in a highly structured, logical manner, forming a
skeleton of the textbook chapter or lecture subject that serves as an excellent study guide when preparing
for tests. Refer to Example 2-5 for an outlining method. Look at how the text on the left was transferred to
an outlining method.
Example 2-5

Physics
A. Kinematics
a. Distance & displacement
b. Speed & velocity
c. Acceleration
B. Work, Energy, Power
a. Force
1. Normal force
2. Gravitational force/weight
3. Frictional force
4. Tension force
5. Applied force
b.
Work done
1. Positive
2. Negative
3. Zero
c. Energy
1. Kinetic energy
2. Gravitational potential energy
3. Work-energy theorem
C. Fluid Statics
a. Pressure
1. Fluid pressure
2. Gauge pressure
3. Absolute pressure
b. Pascal’s Principle
1. Mechanical advantage

Confidential to United Marine Training Center, Inc. Page 2-13


Pre Sea Review Materials <course code>
Version Number
Deck
5 November 2018

Some of its advantages are it allows notes to be neatly organized; it records contents as well as
relationship; it is easy to see relationship between topics and subtopics; and it is also easy to review by
turning points into study questions.

Concept Maps
Concept maps or sometimes referred to as mind maps are one of the best ways to organize information.
It uses a two-dimensional structure, instead of the list format that is conventionally used to take notes. It
helps one to make associations easily and generate new ideas. It also helps one to break large projects
or topics down into manageable chunks, so that one can plan effectively without getting overwhelmed and
without forgetting something important.
The steps in constructing concept maps are the following:
1. Draw the first image or words that come to mind on the topic you are mind mapping. Label the
image.
2. Branch off from your image or words and create one of your main ideas. Label the branch. You
can also draw a picture for it.
3. From your main branches, draw some sub-branches, and from those sub-branches, you can
draw even more branches.
4. Draw pictures for each branch or for as many branches as possible. Make each picture as
absurd, funny and/or exaggerated as possible.
5. Keep repeating the above process. As much as possible, use curved lines as branches because
it helps in stimulating the brain. But some people are into straight lines so it is also applicable.

To make concept maps more effective:

1. Use single words or simple phrases


2. Print words
3. Use color to separate different ideas
4. Use symbols and images
5. Use cross-linkages

Figure 2-3: Concept map lay-out – Source: http://www.leeds.ac.uk/educol/documents/188101.pdf

Confidential to United Marine Training Center, Inc. Page 2-14


Pre Sea Review Materials <course code>
Version Number
Deck
5 November 2018

This method is useful for visual learners who struggle with studying from notes. It also helps in
remembering and connecting relationships between topics.

Note-Taking General Reminders

 Use abbreviations, symbols, and key words


 Develop symbols to help in recording thoughts or reactions. For example, an exclamation point
could mean “I disagree”
 Organize notes with headings, numbers, etc.
 Circle, highlight or underline concepts that are not understandable
 Notes should be accurate and complete, including all necessary diagrams, key points, theories,
definitions, formulas, and facts, so that notes can be studied even without the review material
 Record review material page numbers on notes so that it will be quicker to find the textbook
page for reference

Note: Methods of note taking presented here can be used individually or in conjunction with one another.
For instance, the charting method can be used to write the notes in the note taking portion of the Cornell
method lay-out.

Confidential to United Marine Training Center, Inc. Page 2-15


Pre Sea Review Materials <course code>
Version Number
Deck
5 November 2018

Section 3
Mathematics

Applied Algebra

Review of Arithmetic

Factors, Products, Exponents and Radicals

Multiplication
The basic idea of multiplication is repeated addition. (Pierce, n.d.)
Example 3-1

5+5+5 The number 5 is added 3 times

3∙5 The previous expression can also be written like the one on the left and is
read as 3 times 5

The dot between the 3 and 5 indicates multiplication. It directs to multiply the numbers that it
separates. In algebra, the dot is preferred over the symbol × to denote multiplication because 𝑥 is
often used to represent a variable (Ellis & Burzynski, 2008). This removes the confusion between
the 𝑥 as a letter and × as a mathematical operation.

Factors and Products


In multiplication, the numbers being multiplied are called factors. The result of the multiplication is
called the product (Ellis & Burzynski, 2008). Writing the result of Example 3-1 in a form of an
equation gives us:
Example 3-2

3 ∙ 5 = 15 The numbers 3 and 5 are factors and the number 15 is the product

Exponents
Another way of expressing multiplication is by using exponential notation. Exponent, also known as
power or index, indicates the number of times a factor is used in a multiplication.
Example 3-3

5∙5∙5 In this multiplication, the number 5 is used as a factor 3 times

53 In exponential notation, the number 5 is referred to as the base and the


number 3 is its exponent, power or index.

Note: Both mathematical expressions of Example 3-3 yield the same result of 125.

Confidential to United Marine Training Center, Inc. Page 3-1


Pre Sea Review Materials <course code>
Version Number
Deck
5 November 2018

Example 3-4

5∙5∙5∙7∙7∙7∙7∙7 In this multiplication, the number 5 is used as a factor 3 times


while the number 7 is used as a factor 5 times

53 ∙ 75 The expression on the left shows the exponential notation of the


previous multiplication

(53 )(75 ) The expression on the left makes use of parenthesis to indicate
multiplication and is mathematically the same with the previous
expression.

All three mathematical expressions of Example 3-4 yield the same result of 2 100 875. For the last
expression, the parenthesis ( ) is used to indicate multiplication. Other symbols that can be used for
multiplication are brackets [ ] or curly braces { }.
Besides numbers, algebra makes use of letters, such as 𝑥 and 𝑦, to represent random numbers.
These letters can also be factors of a product; thus, they can also be written in exponential notations.
Example 3-5

𝑥∙𝑥∙𝑥∙𝑥∙𝑥∙𝑥∙𝑥∙𝑥 In this multiplication, a random number x, is used as a factor


8 times

𝑥8 In this expression, the previous multiplication is expressed in


exponential notation where x is the base and 8 is the
exponent

Radicals
For a given expression such that 42 = 16 and (−4)2 = 16, the number is 16 is the square of both 4
and −4. Since 16 comes from squatting 4 and −4, 4 and −4 are called the square roots of 16. Thus,
16 has two square roots. To express mathematically the square root of 16, the radical symbol (√ ) is
used and is the expression is written as

Example 3-6

√16 The square root of 16

The √16 are −4 and 4.

Example 3-7

√36 The square root of 36

The √36 are −6 and 6.

Confidential to United Marine Training Center, Inc. Page 3-2


Pre Sea Review Materials <course code>
Version Number
Deck
5 November 2018

Supplementary Exercises
For the following problems, express each product in exponential notation.
Answer
1] 8∙8∙8∙8 84
2] 12 ∙ 12 ∙ 12 ∙ 12 ∙ 12 125
3] 3∙3∙3∙3∙3∙3∙4∙4∙4∙4∙4∙4∙4 36 ∙ 47
4] 8 ∙ 8 ∙ 8 ∙ 9 ∙ 9 ∙ 9 ∙ 9 ∙ 10 ∙ 10 ∙ 10 ∙ 10 83 ∙ 94 ∙ 104
5] 𝑥∙𝑥∙𝑥∙𝑥∙𝑥∙𝑥 𝑥6
6] 𝑦∙𝑦∙𝑦∙𝑦∙𝑧∙𝑧∙𝑧∙𝑧 𝑦 4 ∙ 𝑧4

For the following problems, express each number into factors of 𝟐, 𝟑, 𝟓 and 𝟕.
7] 630 2∙3∙3∙5∙7
8] 14 2∙7
9] 30 2∙3∙5
10] 100 2∙2∙5∙5
11] 80 2∙2∙2∙2∙5
12] 21 3∙7
13] 210 2∙3∙5∙7
14] 405 3∙3∙3∙3∙5
Simplify the following radicals
15] √3 ±1.732. ..
16] √81 ±9
17] √56.25 ±7.5
18] √100 ±10
19] √49 ±7
20] √169 ±13

Confidential to United Marine Training Center, Inc. Page 3-3


Pre Sea Review Materials <course code>
Version Number
Deck
5 November 2018

Fractions and Decimals

Fractions
A fraction is part of a group, number or whole. For instance, if one circle has been cut in half, one of
the halves is a fraction of the whole circle (Eather, n.d.). Refer to Figure 3-1 to see the mathematical
and visual representation of a fraction.

Numerator
Fraction Bar
1
2
Denominator

Figure 3-1: Mathematical and visual representation of fraction – Source: Christian Kevin Villanueva

Equivalent Fractions
Fractions that have the same value are called equivalent fractions (Ellis & Burzynski, 2008). Figure 3-
2 shows three fractions that are equivalent even if their numerators and denominators are different.
They represent the same part of a whole equivalent.

1 2 4
2 4 8
Figure 3-2: Equivalent fractions – Source: Christian Kevin Villanueva

Types of Fractions
There are three main types of fractions: proper fraction, improper fraction and mixed number
(Eather, n.d.).
In proper fraction, the numerator is less than the denominator.

4 1
8 3
Figure 3-3: Proper fractions – Source: Christian Kevin Villanueva

Confidential to United Marine Training Center, Inc. Page 3-4


Pre Sea Review Materials <course code>
Version Number
Deck
5 November 2018

In improper fraction, the numerator is larger than or equal to the denominator.

8 4
8 3
Figure 3-4: Improper fractions – Source: Christian Kevin Villanueva

A mixed number is a number written as a whole number with a proper fraction.

4 2
2 1
8 3
Figure 3-5: Mixed numbers – Source: Christian Kevin Villanueva

Reducing Fractions to Lowest Terms


A fraction can be reduced to its lowest term by:
1. Expressing the numerator and denominator into its factors
2. Dividing the factors of the numerator that are common to the factors of the denominator (The
division results to a quotient of 1 and does not affect the original value of the fraction. This
technique is commonly called “cancelling”.)
Example 3-8

6
18

2∙3 The numerator has 2 and 3 as its factors while the denominator has 2, 2 and 3.
2∙3∙3 The factors 2 and 3 are common to both; therefore, they are cancelled.

2̸ ∙ 3̸ The slash indicates that a factor in the numerator is cancelled by a same


2̸ ∙ 3̸ ∙ 3 factor in the denominator
2 3
𝟏 Since = 1 and = 1 , the resulting numerator is 1 and 3 is left as the
2 3
𝟑 denominator

Confidential to United Marine Training Center, Inc. Page 3-5


Pre Sea Review Materials <course code>
Version Number
Deck
5 November 2018

Example 3-9

6
18

6 The numbers 3 and 6 are factors of 18. If the denominator is factored in this
3∙6 manner, it immediately cancels the numerator


3 ∙ 6̸

𝟏
𝟑

Multiplication of Fractions
To multiply two fractions, multiply the numerators together and multiply the denominators together.
Reduce to lowest terms if possible (Ellis & Burzynski, 2008).
Example 3-10

3 1

4 6
3 1 3∙1
∙ =
4 6 4∙6

3
=
24
3 Reduce the fraction to lowest term by factoring the numerator and
=
3∙8 denominator

3̸ Cancel the common factors


=
3̸ ∙ 8

𝟏
=
𝟖

Division of Fractions
To divide fractions, multiply the first fraction (dividend) by the reciprocal of the second fraction
(divisor). Reduce if possible (Ellis & Burzynski, 2008).
𝑎 𝑏 𝑏 𝑎
The reciprocal of a fraction in the form 𝑏 is 𝑎 ; likewise, the reciprocal of 𝑎 is 𝑏 . Two numbers
whose product is 1 are reciprocals of each other. Some pairs of reciprocals are listed below.
2 7 3 4 6 1
, , ,
7 2 4 3 1 6

Confidential to United Marine Training Center, Inc. Page 3-6


Pre Sea Review Materials <course code>
Version Number
Deck
5 November 2018

Example 3-11

1 1
÷
3 6
1 1 1 6 Obtain the reciprocal of the second fraction (divisor) then multiply
÷ = ∙
3 6 3 1
6
=
3
2∙3 Reduce the fraction to lowest term by factoring the numerator
=
3

2 ∙ 3̸ Cancel the common factors


=

= 𝟐

Example 3-12

5 24
÷
6 5

5 24 5 5 Obtain the reciprocal of the second fraction (divisor) then multiply


÷ = ∙
6 5 6 24

𝟐𝟓 Already in lowest term or relatively prime (no common factors


= Except 1)
𝟏𝟒𝟒

Factoring the numerator and denominator can take place before multiplication. This method allows
quick immediate cancellation of common factors which simplifies the multiplication

Example 3-13

3 5 3 4 Obtain the reciprocal of the second fraction (divisor) then multiply


÷ = ∙
8 4 8 5

3∙4 Look for factors that are common on both numerator and
= denominator
2∙4∙5

3 ∙ 4̸ Cancel 4 since this is a common factor then multiply the


= remaining factors
2 ∙ 4̸ ∙ 5

𝟑 Already in lowest term or relatively prime (no common factors


= Except 1)
𝟏𝟎

Confidential to United Marine Training Center, Inc. Page 3-7


Pre Sea Review Materials <course code>
Version Number
Deck
5 November 2018

Addition and Subtraction of Fractions with Same Denominator


To add or subtract two or more fractions having the same denominators, add or subtract the
numerator and place the result over the common denominator. Reduce if possible (Ellis & Burzynski,
2008).

Example 3-14

3 2
+
7 7

3 2 𝟓 Add only the numerators and copy the common denominator.


+ =
7 7 𝟕

Example 3-15

7 1

10 10

7 1 6 Subtract only the numerators and copy the common


− = denominator.
10 10 10

2∙3 Reduce the fraction to lowest term by factoring the numerator


=
2∙5

2̸ ∙ 3 Cancel the common factors


=
2̸ ∙ 5

𝟑
=
𝟓

Addition and Subtraction of Fractions with Different Denominator


To add or subtract two or more fractions having different denominators, convert each fraction to an
equivalent fraction such that both fractions will have the same denominators.
A method of finding the equivalent fractions of both fractions in an addition or subtraction is
determining their least common denominator or LCD.
LCD is the smallest number that can be divided by each denominator.
Example 3-16 and Example 3-17 demonstrate how to obtain the LCD of two given fractions.

Example 3-16

1 3 To find the LCD of these given fractions, identify the common factors of
;
6 4 their denominators

6 =2∙3 List the common factors of 4 and 6 once then list the remaining factors
one by one in line with the common factor
4 =2∙2

Confidential to United Marine Training Center, Inc. Page 3-8


Pre Sea Review Materials <course code>
Version Number
Deck
5 November 2018

2, 2 𝑎𝑛𝑑 3 The first 2 in this list of number is the common factor of 4 and 6 (which is
written in blue on the previous equations); while the second 2 and the
last number 3 are the remaining factors that are not common (and are
written in green)

2 ∙ 2 ∙ 3 = 12 Multiply the numbers in the previous list

After determining the LCD, use it as the denominator of the two fractions to be added or subtracted
then solve for the equivalent numerators. Reduce if possible.

Example 3-17

1 3
+
6 4

Change the denominators of the fractions to be added and solve for the
𝑛 𝑚
+ equivalent numerators. Let 𝑛 be the 1st numerator and 𝑚 be the 2nd
12 12 numerator

𝑛 = 12 ÷ 6 ∙ 1 To solve for 𝑛, divide the LCD by the denominator of the original


fraction then multiply to the numerator of the original fraction
𝑛=2

𝑚 = 12 ÷ 4 ∙ 3 To solve for 𝑚, divide the LCD by the denominator of the original


fraction then multiply to the numerator of the original fraction
𝑚=9

2 9 Substitute the values of 𝑛 and 𝑚 then perform the addition of similar


+
12 12 fractions
2 9 1 3
The fractions and 12 are the equivalent fractions of 6 and 4
12
respectively.

𝟏𝟏
𝟏𝟐

Example 3-18

5 5

9 12

9 =3∙3 Determine the LCD of the fractions to be subtracted


12 = 2 ∙ 2 ∙ 3
𝐿𝐶𝐷 = 2 ∙ 2 ∙ 3 ∙ 3
𝐿𝐶𝐷 = 36

𝑛 𝑚 Change the denominators of the fractions to be added and solve for


+ the equivalent numerators. Let 𝑛 be the 1st numerator and 𝑚 be the
36 36
2nd numerator

Confidential to United Marine Training Center, Inc. Page 3-9


Pre Sea Review Materials <course code>
Version Number
Deck
5 November 2018

𝑛 = 36 ÷ 9 ∙ 5 To solve for 𝑛, divide the LCD by the denominator of the original


fraction then multiply to the numerator of the original fraction
𝑛 = 20

𝑚 = 36 ÷ 12 ∙ 5 To solve for 𝑚, divide the LCD by the denominator of the original


fraction then multiply to the numerator of the original fraction
𝑚 = 15

Substitute the values of 𝑛 and 𝑚 then perform the subtraction of


20 15 similar fractions
− 20 15 5 5
36 36 The fractions and are the equivalent fractions of and
36 36 9 12
respectively.

𝟏𝟓
𝟑𝟔

Converting Decimal Fractions to Fractions


Fractions are one way of expressing parts of a whole and decimal fractions are another. Decimal
fraction is a fraction of a whole part expressed or written on the right of a decimal point. The number
or numbers written on the left of the decimal point is/are the whole number values. The position of the
numbers on the left of the decimal points have corresponding place values that serve as their
denominator if converted to fraction.
Figure 3-6 shows the value of a digit placed on the left or right of a decimal point

Figure 3-6: Place values – Source: Christian Kevin Villanueva

The place value on the right suggests the denominator of the decimal if it is to be converted. This
means that a number placed in the Tenths place will have a denominator of 10. It is also
recommended to reduce the fractional part if possible.

Example 3-19

57.9

𝟗 The whole part of the number is 57. The decimal fraction


𝟓𝟕
𝟏𝟎 contains 9 in the Tenth place; thus, it will have a denominator of 10.

Confidential to United Marine Training Center, Inc. Page 3-10


Pre Sea Review Materials <course code>
Version Number
Deck
5 November 2018

Example 3-20

5.5

5 The whole part of the number is 5. The decimal fraction contains 5 in


5
10 the Tenth place; thus, it will have a denominator of 10.

𝟏 The resulting fraction is reduced to lowest term and resulting to 1


𝟓
𝟐 over 2 or is usually called a half.

Example 3-21

96.75

75 The whole part of the number is 96. The decimal fraction


96 contains 75 that occupies up to the Hundredths place; thus, it will
100 have a denominator of 100.

𝟑 The resulting fraction is reduced to lowest term


𝟗𝟔
𝟒

Example 3-22

6.8014

The whole part of the number is 6. The decimal fraction


contains 8 014.
The 8 is in the tenth place
8 014 The 0 is in the hundredths place
6
10 000 The 1 is in the thousandths place
The 4 is in the ten thousandths place
Since 8 014 occupies up to the ten thousandths place, it will have a
denominator of 10,000

𝟒 𝟎𝟎𝟕 The resulting fraction is reduced to lowest term


𝟗𝟔
𝟓 𝟎𝟎𝟎

Confidential to United Marine Training Center, Inc. Page 3-11


Pre Sea Review Materials <course code>
Version Number
Deck
5 November 2018

Converting Fractions to Decimal Fractions


Fractions are converted to decimal fractions by dividing its numerator by its denominator. (For cases
that division is non-terminating, the results are rounded up to 3 decimal places).

Example 3-23

3
4

` 0.75 Note that when dividing, align the decimal point of the quotient with
4 3.00 the dividend
28
The dividend is the numerator of the previous expression
0 20
0 20
0

3 The equivalent of
3
in decimal fraction is 0.75.
= 𝟎. 𝟕𝟓 4
4

Example 3-24

1
19
5

Set aside the whole number part of the number and perform long
` 0.2 division of the fraction part.
5 1.0
10
0

1 Copy the whole number part and replace the fraction with its
19 = 𝟏𝟗. 𝟐 equivalent decimal fraction
5

Example 3-25

5
6

` 0.833 Notice that the remainder, 20, is recurring and indicates that the
6 5.000 division is non-terminating
48
20
18
20

5 For the purpose of simplicity, only 3 decimal places will be written.


= 𝟎. 𝟖𝟑𝟑
6

Confidential to United Marine Training Center, Inc. Page 3-12


Pre Sea Review Materials <course code>
Version Number
Deck
5 November 2018

Supplementary Exercises
Reduce, if possible, each fraction to its lowest term
Answer
1] 6 3
8 4
2] 6 3
14 7
3] 18 3
12 2
4] 16 8
42 21
5] 121 11
132 12
Perform the indicated operations
6] 5 14 7

6 15 9
7] 9 20 5

16 27 12
8] 76 66 4

99 38 3
9] 3 14 6 1
∙ ∙
7 18 2
10] 14 21 45 9
∙ ∙
15 28 7 2
11] 5 5 2
÷
9 6 3
12] 9 15 3
÷
16 8 10
13] 25 4 225
÷
49 9 196
14] 24 8 3
÷
75 15 5
15] 7 10 49
÷
10 7 100
16] 3 4 7
+
11 11 11
17] 5 7 1
+
12 12
18] 11 2 9

16 16 16
19] 15 2 13

23 23 23
20] 1 1 2
+
2 6 3

Confidential to United Marine Training Center, Inc. Page 3-13


Pre Sea Review Materials <course code>
Version Number
Deck
5 November 2018

21] 3 1 13
+
4 3 12
22] 8 3 7

15 10 30
23] 1 3 3 7
+ −
16 4 8 16
24] 8 1 7 47
− +
3 4 36 18
25] 3 3 5 217
− +
4 22 24 264
Convert each decimal fraction to fractions
26] 0.06 3
50
27] 3.7 37 7
𝑜𝑟 3
10 10
28] 48.116 12 029 29
𝑜𝑟 48
250 250
29] 712.004 178 001 1
𝑜𝑟 712
250 250
30] 0.115 3
50
Convert each fraction to decimal fractions. If the decimal is nonterminating, round to 3
decimal places
31] 5 0.625
8
32] 9 0.45
20
33] 15 0.682
22
34] 7 0.636
11
35] 2 0.222
9

Confidential to United Marine Training Center, Inc. Page 3-14


Pre Sea Review Materials <course code>
Version Number
Deck
5 November 2018

Percentage

Percentage
A percentage or per cent is a fraction expressed as number out of 100 followed by the % symbol
1
(Eather, n.d.). Mathematically, the symbol % or 1 % means 100.

For instance, an area that has a capacity of a hundred same-sized boxes fitted side by side is being
loaded. If fifty boxes are placed, this means that 50% of its capacity is occupied. Similarly, if forty-two
(42) are placed, the area is 42 % occupied and if seventy-five (75) are placed, then the area
is 75 % occupied.

Figure 3-7: Fractions as percentages – Source: Christian Kevin Villanueva

Percentage to Decimal Fraction and Vice Versa


1
To convert a percentage to a decimal, replace the % with 100 . This results to dividing the number in
percentage by 100. This also amounts to moving the decimal place two (2) places to the left.

Example 3-26

48 %

1
48 % = 48 ∙
100
48
=
100

= 𝟎. 𝟒𝟖

Example 3-27

659 %

1
659 % = 659 ∙
100

659
=
100

= 𝟔. 𝟓𝟗

Confidential to United Marine Training Center, Inc. Page 3-15


Pre Sea Review Materials <course code>
Version Number
Deck
5 November 2018

Example 3-28

0.41113 %

1
0.41113 % = 0.41113 ∙
100
0.41113659
=
100

= 𝟎. 𝟎𝟎𝟒𝟏𝟏𝟑

To convert decimal to fraction, simply multiply to 100 then affix the % sign

Example 3-29

10.2345

10.2345 = 10.2345 ∙ 100

= 1 023.45 %

Example 3-30

0.025

0.025 = 0.025 ∙ 100

= 𝟐. 𝟓 %

Fraction to Percentage
A percentage can be expressed as a fraction by removing the % sign and having 100 as its
denominator. If possible, reducing to lowest term is recommended. For fractions that have 100 as their
denominator, omit the denominator then affix the % sign. For fractions with other denominators,
multiply (or divide) the denominator to a number that will make a product (or quotient) of 100. The
numerator is also multiplied by (or divided by) this number. Remove the denominator after the
multiplication (or division) and affix the % sign.

Example 3-31

3
4
3 3 ∙ 25
=
4 4 ∙ 25
75
=
100

= 𝟕𝟓 %

Confidential to United Marine Training Center, Inc. Page 3-16


Pre Sea Review Materials <course code>
Version Number
Deck
5 November 2018

Example 3-32

1
2
1 1 ∙ 50
=
2 2 ∙ 50
50
=
100

= 𝟓𝟎 %

Example 3-33

143
500

143 143 ÷ 5
=
500 500 ÷ 5
28.6
=
100

= 𝟐𝟖. 𝟔 %

Confidential to United Marine Training Center, Inc. Page 3-17


Pre Sea Review Materials <course code>
Version Number
Deck
5 November 2018

Supplementary Exercises
Convert each fraction or number to percentage
Answer
1] 2 40 %
5
2] 7 87.5 %
8
3] 1 12.5 %
8
4] 2 18.181 %
11
5] 73 97.33 %
75
6] 111 55.5 %
200
7] 756 604.8 %
125
8] 14 1 400 %
9] 12 1 200 %
10] 0.36 36 %
11] 0.1298 12.98 %
12] 4.25 425 %
13] 5.875 587.5 %
14] 86.98 8 698 %
15] 21.26 2 126 %

Confidential to United Marine Training Center, Inc. Page 3-18


Pre Sea Review Materials <course code>
Version Number
Deck
5 November 2018

Scientific Notation

Standard Form to Scientific Form


Very large numbers such as 43 000 000 000 000 000 000 and very small numbers such as
0.000000000000000000000340 are extremely inconvenient to write and read. Such numbers can be
expressed more conveniently by writing them as part of a power of 10 (Ellis & Burzynski, 2008).
To write a number as a power of 10, multiply the given number to 10𝑛 where 𝑛 dictates the number of
zeroes or movement of the decimal point
Consider Example 3-34 (A number with 11 digits) written in standard form,

Example 3-34

21 870 000 000 Standard form

21 870 000 000 × 100 Multiplied to 100


Since 100 = 1, the factor 100 can be ignored but for the purpose
of discussion, this factor is displayed

2 187 000 000 × 101 Multiplied to 101


Since 101 = 10, one zero from the standard form of the number
is omitted. The value of the number is unchanged

21 870 000 × 103 Multiplied to 103


Since 103 = 10 ∙ 10 ∙ 10 = 1 000, three (3) zeroes from the
standard form of the number are omitted. The values of the
number is unchanged

21 870 × 106 Multiplied to 106


Since 106 = 10 ∙ 10 ∙ 10 ∙ 10 ∙ 10 ∙ 10 = 1 000 000, six (6) zeroes
from the standard form of the number are omitted. The values of
the number is unchanged

2.187 × 1010 Multiplied to 1010


The standard form of the number, 21 870 000 000 can written
with a decimal point explicitly shown.
21 870 000 000.00
By multiplying to 1010, it could be concluded that that the decimal
point moved 10 places to the left and note that 𝑛 = 10

Confidential to United Marine Training Center, Inc. Page 3-19


Pre Sea Review Materials <course code>
Version Number
Deck
5 November 2018

A number written as a power of 10 with one nonzero digit to the left of the decimal point is call the
scientific form of the number. This is a convenient way of writing or reading very large or very small
numbers.

Example 3-35

0.000000745 Standard form

0.01 02 03 04 05 06 7745 The subscript on each digit starting from the decimal point to the
right is the movement required for the decimal point to pass the
first nonzero digit. The number of movements is the value of
𝑛 and movement to left is considered positive while movement
to the right is considered negative; thus, 𝑛 = −7

7.450 × 10n Substitute 𝑛 = −7

7.450 × 10−7 Scientific Form


Only one nonzero digit to the left of the decimal point

Example 3-36

2 480 Standard form

23 42 81 0.00 Three movements to the left until only one nonzero digit is to the
left of the decimal point; thus, 𝑛 = 3
The subscripts indicate the number of movements of the decimal
point going to the left

2.480 × 103 Scientific Form

Example 3-37

0.00056 Standard form

0.01 02 03 54 6 Four movements to the right; thus, 𝑛 = −4


The subscripts indicate the number of movements of the decimal
point going to the right

5.6 × 10−4 Scientific Form

Confidential to United Marine Training Center, Inc. Page 3-20


Pre Sea Review Materials <course code>
Version Number
Deck
5 November 2018

Supplementary Exercises
Convert the following number to scientific notation
Answer
1] 981 9.81 × 102
2] 54.06 5.406 × 10
3] 0.00000000004632 4.632 × 10−11
4] 0.027 2.7 × 10−2
5] 179 000 000 000 000 000 000 1.79 × 1020

Confidential to United Marine Training Center, Inc. Page 3-21


Pre Sea Review Materials <course code>
Version Number
Deck
5 November 2018

Linear Equation in One Variable

Equation
Equation is a mathematical statement that two expressions are equal.
Example 3-38

3𝑥 = 𝑥 − 5

The important thing to remember about any equation is that the equals sign represents a balance.
What an equals sign says is that what is on the left-hand side is exactly the same as what is on the
right-hand side. Whatever is done on an equation, do the same thing to both sides of that equation,
otherwise, the balance is disturbed.

Linear Equation in One Unknown


Linear Equation in one unknown is an equation that can be written in the form 𝑎𝑥 + 𝑏 = 0, where 𝑎
and 𝑏 are constants and 𝑥 is the unknown value. Linear equation is also known as first-degree
equation because the exponent on the variable is understood to be 1.
Example 3-39

7𝑥 − 4 = 10

To solve this equation means to find the numerical value of 𝑥, at which this equation becomes an
identity.
A solution or root of a linear equation is the value of the variable which makes the left-hand side
equals the right-hand side. The solution to 7𝑥 − 4 = 10 is 2.

Equivalent Equations
Two equations that have the same solutions are called equivalent equations.
Example 3-40

5 +3 = 2 + 6

Properties of Equality

Addition Property of Equality


The addition property of equality (APE) states that adding the same number to each side of an
equation gives an equivalent equation.
For all real numbers 𝑎 , 𝑏 and 𝑐 if 𝑎 = 𝑏 , then 𝑎 + 𝑐 = 𝑏 + 𝑐.

Confidential to United Marine Training Center, Inc. Page 3-22


Pre Sea Review Materials <course code>
Version Number
Deck
5 November 2018

Example 3-41

Solve for 𝑥 in 𝑥 − 10 = −1, apply APE by adding 10 to both sides of the equation.
Solution

𝑥 − 10 = −1

𝑥 − 10 + 10 = −1 + 10 Add both sides by 10

𝒙=𝟗 Answer

Subtraction Property of Equality


The subtraction property of equality (SPE) states that subtracting the same number to each side of
an equation gives an equivalent equation.
For all real numbers 𝑎 , 𝑏 , and 𝑐 if 𝑎 = 𝑏 , then 𝑎 − 𝑐 = 𝑏 − 𝑐.

Example 3-42

To solve for x in −5 = 𝑥 + 7, apply SPE by subtracting 7 to both sides of the


equation.
Solution

−5 = 𝑥 + 7

−5 − 7 = 𝑥 + 7 − 7 Subtract both sides by−7

−12 = 𝑥

𝒙 = −𝟏𝟐 Answer

Multiplication Property of Equality


The multiplication property of equality (MPE) states that multiplying each side of an equation with
the same nonzero number produces an equivalent equation.
For all real numbers 𝑎 , 𝑏 , and 𝑐 if 𝑎 = 𝑏 , then 𝑎 ∙ 𝑐 = 𝑏 ∙ 𝑐.

Example 3-43
𝑥
Solve for 𝑥 in = −7 , apply MPE by multiplying 3 to both sides of the equation.
3

Solution
𝑥
= −7
3
𝑥 Multiply both sides by 3
3[ = −7 ] 3
3
𝑥 Perform cancellation of factors
3[ = −7 ] 3
3

𝒙 = −𝟐𝟏 Answer

Confidential to United Marine Training Center, Inc. Page 3-23


Pre Sea Review Materials <course code>
Version Number
Deck
5 November 2018

Division Property of Equality


The division property of equality (DPE) tells us that dividing each side of an equation with the same
nonzero number you produce an equivalent equation.
For all real numbers 𝑎 , 𝑏 , and 𝑐 if 𝑎 = 𝑏 , then 𝑎 ÷ 𝑐 = 𝑏 ÷ 𝑐.

Example 3-44

Solve for 𝑥 in 8𝑥 = −40, apply DPE by dividing both sides of the equation by 8.
Solution

8𝑥 = −40

8𝑥 −40 Divide both sides by 8


=
8 8

𝒙 = −𝟓 Answer

Reflexive Property
Reflexive Property states that a number equals itself.
For all real numbers 𝒄, 𝒄 = 𝒄

Symmetric Property
Symmetric Property states that the order of equality does not matter.
For all real numbers a and b, if 𝑎 = 𝑏 , then 𝑏 = 𝑎 .

Example 3-45

8𝑥 = −40 is the same as −40 = 8𝑥.

Transitive Property
Transitive Property states that two numbers equal to the same number are equal to each other.
For all real numbers 𝑎 , 𝑏 , and 𝑐 if 𝑎 = 𝑏 and 𝑏 = 𝑐, then 𝑎 = 𝑐.
Note: Reflexive, symmetric, and transitive properties define an equivalence relation.

Strategies for Solving Linear Equation


Step 1: Simplify each side, if needed. This may include collecting and combining like terms, removing
grouping symbols, eliminating fractions, etc.
Step 2: Use Addition and Subtraction Properties to move the variable term(s) to one side and all
constant(s) to the other side. This can be shortened by the transposition process
(transferring term or terms from one side of the equation to the other side with its sign
changed)
Step 3: Use the multiplication and division properties to remove any values that are in front of the
variable. This can be shortened by the cross-multiplication method.
Step 4: Check the answer by substituting the obtained value to the original equation.

Confidential to United Marine Training Center, Inc. Page 3-24


Pre Sea Review Materials <course code>
Version Number
Deck
5 November 2018

Example 3-46

Solve 8𝑥 − 9 − 6𝑥 + 31 = 6𝑥 − 17 − 9𝑥 − 1
Solution

8𝑥 − 9 − 6𝑥 + 31 = 6𝑥 − 17 − 9𝑥 − 1

2𝑥 + 22 = −3𝑥 − 18 Combine like terms

2𝑥 + 3𝑥 = −18 − 22 Regroup the terms (all terms containing 𝑥 on the


left side and constants on the right side) or apply
transposition process

5𝑥 40 Divide both side by 5 (DPE)


=−
5 5

𝒙 = −𝟖 Answer
Format 𝑣𝑎𝑟𝑖𝑎𝑏𝑙𝑒 = 𝑐𝑜𝑛𝑠𝑡𝑎𝑛𝑡

Example 3-47

Solve 1 − 3(2𝑥 − 4) = 4(6 − 𝑥) − 8


Solution

1 − 3(2𝑥 − 4) = 4(6 − 𝑥) − 8

1 − 6𝑥 + 12 = 24 − 4𝑥 − 8 Remove () by the distributive law

13 − 6𝑥 = 16 − 4𝑥 Combine like terms

13 − 16 = 6𝑥 − 4𝑥 Transpose all terms containing 𝑥 to the right side and the


constants to the left side

−3 = 2𝑥 Combine like terms

−3 2x Divide both sides by 2 (DPE)


=
2 2
−3 2𝑥 Perform cancellation of factors
=
2 2
3
− =𝑥
2
𝟑 Answer
𝒙=− ;
𝟐 Apply Reflexive property to follow the format
𝑣𝑎𝑟𝑖𝑎𝑏𝑙𝑒 = 𝑐𝑜𝑛𝑠𝑡𝑎𝑛𝑡
𝒙 = −𝟏. 𝟓

Confidential to United Marine Training Center, Inc. Page 3-25


Pre Sea Review Materials <course code>
Version Number
Deck
5 November 2018

Example 3-48
𝑥+5 𝑥+1 𝑥+3
Solve 6
− 9
= 4

Solution

𝑥+5 𝑥+1 𝑥+3 Given


− =
6 9 4
𝑥+5 𝑥+1 𝑥+3 Multiply the entire equation by 36 which is the LCD of
36 [ − = ] 36
6 9 4 to get rid of the terms to remove all denominators

6(𝑥 + 5) − 4(𝑥 + 1) = 9(𝑥 + 3) Remove () by the distributive law

6𝑥 + 30 − 4𝑥 − 4 = 9𝑥 + 27 Combine like terms

2𝑥 + 26 = 9𝑥 + 27 Transpose all terms containing 𝑥 on the right side and


the constants on the left side

26 − 27 = 9𝑥 − 2𝑥 Combine like terms

−1 7𝑥 Divide both sides by 7 (DPE)


=
7 7
−1
=𝑥
7
𝟏 Answer
𝒙=− ;
𝟕 Apply Reflexive property to follow the format
𝑣𝑎𝑟𝑖𝑎𝑏𝑙𝑒 = 𝑐𝑜𝑛𝑠𝑡𝑎𝑛𝑡
𝒙 = 𝟎. 𝟏𝟒𝟐𝟖 …

Confidential to United Marine Training Center, Inc. Page 3-26


Pre Sea Review Materials <course code>
Version Number
Deck
5 November 2018

Supplementary Exercises
Solve for the value of 𝒙
Answer
1] 5𝑥 − 9 + 3𝑥 + 3 = 6 + 7𝑥 − 12 − 4𝑥 𝑥=0
2] 12𝑥 − 10 − 8𝑥 + 21 = −5𝑥 + 13 − 9𝑥 − 1 + 4𝑥 1
𝑥=
14
3] −2[𝑥 − (5 − 4𝑥)] + 4 = 3𝑥 14
𝑥=
13
4] (𝑥 + 1)(2𝑥 + 1) = (𝑥 + 3)(2𝑥 + 3) − 14 𝑥=1
5] 2(𝑥 − 1) + 𝑥 = 5(2𝑥 + 3) − 2(𝑥 + 3) 11
𝑥=−
5
6] 2 5
= 𝑥=3
3𝑥 + 1 8𝑥 + 1
7] 5𝑥 + 1 𝑥 − 2 2𝑥 + 4 3
− = 𝑥=
2 6 3 10
8] 2 𝑥 36
𝑥 + 10 = + 𝑥 = −6
3 5 5
9] 3𝑥 2 + 10𝑥 + 3 𝑥 2 − 4𝑥 + 3
= 𝑥 = −2
𝑥+3 𝑥−1
10] 4 3 8
− = 𝑥 = −2
𝑥 − 2 𝑥 + 1 (𝑥 − 2)(𝑥 + 1)

Confidential to United Marine Training Center, Inc. Page 3-27


Pre Sea Review Materials <course code>
Version Number
Deck
5 November 2018

Quadratic equations

Quadratic Equation
A quadratic equation in 𝒙 is any equation that can be written in the form 𝑎𝑥 2 + 𝑏𝑥 + 𝑐 = 0 where 𝑎,
𝑏, and 𝑐 are coefficients and 𝑎 ≠ 0. It is also known as a second degree equation because the
highest power of the unknown (𝑥) is 2.

Example 3-49

𝑥 2 + 2𝑥 = 4 This is a quadratic equation but not in standard form

𝑥 2 + 2𝑥 − 4 = 0 By transposing 4, the previous equation is now in the


standard form of a quadratic equation where:
a=1
b=2
c = −4

Example 3-50

(2 + 𝑥)(3 − 𝑥) = 0 This is a quadratic equation but not in standard form

6 − 2𝑥 + 3𝑥 − 𝑥 2 = 0 By FOIL method and properties of equalities, the


previous equation can be written in the standard form
−𝑥 2 + 𝑥 + 6 = 0
of a quadratic equation where:
𝑥2 − 𝑥 − 6 = 0
a=1
b = −1
c = −6

Note: Quadratic equations always have an 𝑥 2 term, may possibly have an 𝑥-term, and may possibly
have a constant term. If your equation has an 𝑥 2 term or will have an 𝑥 2 term after simplifying it may
be a quadratic, provided that the other terms fit the form.

Strategies for Solving Quadratic Equations


To illustrate on how to solve quadratic equations using the factoring method, extracting square root
method, and the quadratic formula, the following equations will be used all throughout the discussion.

5𝑥 2 − 125 = 0

(𝑥 + 5)2 − 36 = 0

𝑥(3𝑥 + 4) = 7

Confidential to United Marine Training Center, Inc. Page 3-28


Pre Sea Review Materials <course code>
Version Number
Deck
5 November 2018

Factoring
This method is applicable only if the expression is factorable.
Step 1: Simplify each side if needed.
Step 2: Write the equation in standard form 𝑎𝑥 2 + 𝑏𝑥 + 𝑐 = 0, if needed.
Step 3: Factor the equation using the appropriate factoring technique.
Step 4: Equate each factor to zero and solve. This is possible because of the Zero Product Law which
states that if 𝑎𝑏 = 0, 𝑡ℎ𝑒𝑛 𝑎 = 0 𝑜𝑟 𝑏 = 0

Example 3-51
Solve 5𝑥 2 − 125 = 0
Solution

5𝑥 2 − 125 = 0

5(𝑥 2 − 25) = 0 Factor out the common factor which is 5

5(𝑥 − 5)(𝑥 + 5) = 0 Factor 𝑥 2 − 25

(𝑥 + 5) = 0 ; 𝑥1 = −5 Equate each factor to zero and solve using


the appropriate properties of equality.
(𝑥 − 5) = 0 ; 𝑥2 = 5
5 = 0 𝐹𝑎𝑙𝑠𝑒 ; 𝐷𝑖𝑠𝑟𝑒𝑔𝑎𝑟𝑑

Substitute 𝑥1 = 5 to 5𝑥 2 − 125 = 0 Check by substituting the obtained values


2) of 𝑥 into the original equation.
5(5 − 125 = 0
Since the left-hand side and the right-hand
5(25) − 125 = 0
side of the equations are equal when
125 − 125 = 0 substituted by the 𝑥 values, then it is verified
that the solutions or roots of the equation
0=0✔ are 5 and−5 .
Substitute 𝑥2 = −5 to 5𝑥 2 − 125 = 0
5(−5)2 − 125 = 0
5(25) − 125 = 0
125 − 125 = 0
0=0✔

Confidential to United Marine Training Center, Inc. Page 3-29


Pre Sea Review Materials <course code>
Version Number
Deck
5 November 2018

Example 3-52

Solve (𝑥 + 5)2 − 36 = 0
Solution

(𝑥 + 5)2 − 36 = 0

𝑥 2 + 10𝑥 + 25 − 36 = 0 Square the binomial (𝑥 + 5)2 or rewrite


it as (𝑥 + 5)(𝑥 + 5), then use FOIL
method to get the product

𝑥 2 + 10𝑥 − 11 = 0 Combine like terms

(𝑥 + 11)(𝑥 − 1) = 0 Since the equation is in standard form


already , factor the expression, 𝑥2 +
10𝑥 − 11.
Factor the first term, 𝑥 2 and the last
term −11. Choose factors of −11 such
that when added gives the numerical
coefficient of the middle term which is
10.
Factors of -11
−11 = (−11)(1)
−11 = (11)(−1) ✔
Sum of the factors
−11 + 1 = −10
11 + (−1) = 10 ✔
The second pair of factors must be
chosen since it adds up to the
numerical coefficient of the middle term
which is 10.

𝑥 + 11 = 0 ; 𝒙𝟏 = −𝟏𝟏 Equate each factor to zero and solve


𝑥−1 =0 ; 𝒙𝟐 = 𝟏

Substitute 𝑥1 = 1 to (𝑥 + 5)2 − 36 = 0 Check by substituting the obtained


values of 𝑥 into the original equation.
𝑥 − 36 = 0
Since the left-hand side and the right-
(1 + 5)2 − 36 = 0 hand side of the equations are equal
when substituted by the 𝑥 values, then
(6)2 − 36 = 0 it is verified that the solutions or roots
36 − 36 = 0 of the equation are 1 and −11.
0=0✔
Substitute 𝑥2 = −11 to (𝑥 + 5)2 − 36 = 0
(−11 + 5)2 − 36 = 0
(−6)2 − 36 = 0
36 − 36 = 0
0=0✔

Confidential to United Marine Training Center, Inc. Page 3-30


Pre Sea Review Materials <course code>
Version Number
Deck
5 November 2018

Example 3-53

Solve 𝑥(3𝑥 + 4) = 7
Solution

𝑥(3𝑥 + 4) = 7

3𝑥 2 + 4𝑥 = 7 Simplify by the distributive property

3𝑥 2 + 4𝑥 − 7 = 0 Write it in standard form , 𝑎𝑥 2 + 𝑏𝑥 +


𝑐=0

(3𝑥 + 7)(𝑥 − 1) = 0 Factor the quadratic trinomial using trial


and error method. Factor the first term
3𝑥 2 and last term−7.
Possible factors are:
(3𝑥 + 7)(𝑥 − 1) = 0 , (3𝑥 − 1)(𝑥 + 7) ,
(3𝑥 − 7)(𝑥 + 1) = 0, (3𝑥 + 1)(𝑥 − 7)
The correct factors are (3𝑥 + 7)(𝑥 −
1) = 0 because the middle products
7𝑥 and −3𝑥 add up to 4x.

−𝟕 Equate each factor to zero and solve


(3𝑥 + 7) = 0 ; 3𝑥 = −7 ; 𝒙=
𝟑 using the appropriate properties of
equality.
(𝑥 − 1) = 0 ; 𝒙=𝟏
−7
Substitute 𝑥 = to 𝑥(3𝑥 + 4) = 7 Check by substituting the obtained
3
values of 𝑥 into the original equation.
7 −7 Since the left - hand side and the right -
− [3 ( ) + 4] = 7
3 3 hand side of the equations are equal
−7 when substituted by the 𝑥 values, then it
[−7 + 4] = 7 is verified that the solutions or roots of
3 −7
−7 the equation are and 1 .
3
(−3) = 7 ✔
3
7=7
Substitute 𝑥 = 1 to 𝑥(3𝑥 + 4) = 7
1[3(1) + 4] = 7
1[7]=7
7=7✔

Confidential to United Marine Training Center, Inc. Page 3-31


Pre Sea Review Materials <course code>
Version Number
Deck
5 November 2018

Extracting Square Root


This allows quadratic equation to be re written in the form 𝑥 2 = 𝑘 as 𝑥 = ±√𝑘 , where 𝑘 is some real
number. Note that the squared quantity must be isolated on one side of = before the square roots are
extracted.

Example 3-54

Solve 5𝑥 2 − 125 = 0
Solution

5𝑥 2 − 125 = 0

5𝑥 2 = 125 Isolate the squared quantity on one side of the


equation

5𝑥 2 125 Apply division property of equality to make the


= numerical coefficient of the squared term equal
5 5
to 1 or a perfect square

5𝑥 2 125 Perform cancellation of factors


=
5 5
𝑥 2 = 25

𝑥 = ±√25 Extract square roots on both sides of the equation

𝑥 = ±5 Simplify radicals
𝑥1 = 5
𝑥2 = 5

Substitute 𝑥1 = 5 to 5𝑥 2 − 125 = 0 Check by substituting the obtained values of x into


the original equation. Since the left - hand side
5(52 ) − 125 = 0 and the right - hand side of the equations are
5(25) − 125 = 0 equal when substituted by the x values, then it is
verified that the solutions or roots of the equation
125 − 125 = 0 are 5 and −5
0=0✔
Substitute 𝑥2 = −5 to 5𝑥 2 − 125 = 0
5(−5)2 − 125 = 0
5(25) − 125 = 0
125 − 125 = 0
0=0✔

Confidential to United Marine Training Center, Inc. Page 3-32


Pre Sea Review Materials <course code>
Version Number
Deck
5 November 2018

Example 3-55

Solve (𝑥 + 5)2 − 36 = 0
Solution

(𝑥 + 5)2 − 36 = 0

(𝑥 + 5)2 = 36 Isolate the squared quantity on one side of


the equation

√(𝑥 + 5)2 = ±√36 Extract square roots on both sides of the


equation

𝑥 + 5 = ±6 Simplify radicals

𝑥 = −5 ± 6 Apply subtraction property of equality


(transpose 5 to the other side of the
equation)

𝑥1 = −5 + 6 ; 𝒙𝟏 = 𝟏 Solve for the values of 𝑥


𝑥2 = −5 − 6 ; 𝒙𝟐 = −𝟏𝟏

Substitute 𝑥1 = 1 to (𝑥 + 5)2 − 36 = 0 Check by substituting the obtained values


2 of 𝑥 into the original equation. Since the left
(1 + 5) − 36 = 0
- hand side and the right - hand side of the
(6)2 − 36 = 0 equations are equal when substituted by
the x values, then it is verified that the
36 − 36 = 0 solutions or roots of the equation are
0=0✔ 1 and −11
Substitute 𝑥2 = −11 to (𝑥 + 5)2 − 36 = 0
(−11 + 5)2 − 36 = 0
(−6)2 − 36 = 0
36 − 36 = 0
0=0✔

The Quadratic Formula

Step 1: Simplify each side of the equation if needed.


Step 2: Write in standard form, 𝑎𝑥 2 + 𝑏𝑥 + 𝑐 = 0 if needed.
Step 3: Identify 𝑎, 𝑏, and 𝑐.
−𝑏±√𝑏2 −4𝑎𝑐
Step 4: Plug the values found in Step 3 into the quadratic formula; 𝑥 = .
2𝑎

Step 5: Simplify if possible.

Confidential to United Marine Training Center, Inc. Page 3-33


Pre Sea Review Materials <course code>
Version Number
Deck
5 November 2018

Example 3-56

Solve 5𝑥 2 − 125 = 0
Solution:

5𝑥 2 − 125 = 0

𝑎 = 5; Quadratic equation in standard form

𝑏 = 0; Identify the values of 𝑎, 𝑏 , and 𝑐

𝑐 = −125

−𝑏 ± √𝑏2 − 4𝑎𝑐 Plug in the values of 𝑎, 𝑏, and 𝑐 into the


𝑥= quadratic formula
2𝑎
Note: Zero (0) is neither positive nor negative
(0) ± √02 − 4(5)(−125)
𝑥=
2(5)

(0) ± √2500 50 Simplify the radicand first then divide it by the


𝑥= =± = ±5 product of the denominator to get the value of
10 5 𝑥
𝒙𝟏 = 𝟓;
𝒙𝟐 = −𝟓

Substitute 𝑥1 = 5 to 5𝑥 2 − 125 = 0 Check by substituting the obtained values of 𝑥


into the original equation. Since the left - hand
5(52 ) − 125 = 0
side and the right - hand side of the equations
5(25) − 125 = 0 are equal when substituted by the 𝑥 values,
the it is verified that the solutions or roots of
125 − 125 = 0 the equation are 5 and −5 .
0 = 0✔
Substitute 𝑥2 = −5 to 5𝑥 2 − 125 = 0
5(−5)2 − 125 = 0
5(25) − 125 = 0
125 − 125 = 0
0=0✔

Confidential to United Marine Training Center, Inc. Page 3-34


Pre Sea Review Materials <course code>
Version Number
Deck
5 November 2018

Example 3-57

Solve (𝑥 + 5)2 − 36 = 0 .
Solution

(𝑥 + 5)2 − 36 = 0

𝑥 2 + 10𝑥 + 25 − 36 = 0 Square the binomial (𝑥 + 5)2 or rewrite


it as (𝑥 + 5)(𝑥 + 5), then use FOIL
method to get the product

𝑥 2 + 10𝑥 − 11 = 0 Combine like terms

𝑎 = 1; Identify the values of 𝑎, 𝑏, and 𝑐 since


the quadratic equation is in standard
𝑏 = 10;
form already
𝑐 = −11

−𝑏 ± √𝑏 2 − 4𝑎𝑐 Plug in the values of 𝑎, 𝑏, and 𝑐 into


𝑥= the quadratic formula
2𝑎
−(10) ± √(10)2 − 4(1)(−11)
𝑥=
2(1)

−(10) ± √100 + 44 −10 ± √144 Simplify to get the values of 𝑥


𝑥= =
2 2
−10 ± 12
𝑥=
2
−10 + 12 2
𝑥1 = = = 1 ; 𝒙𝟏 = 𝟏
2 2
−10 − 12 −22
𝑥2 = = = −11 ; 𝒙𝟐 = −𝟏𝟏
2 2

Substitute 𝑥1 = 1 to (𝑥 + 5)2 − 36 = 0 Check by substituting the obtained


2 values of 𝑥 into the original equation.
(1 + 5) − 36 = 0
Since the left - hand side and the right
(6)2 − 36 = 0 - hand side of the equations are equal
when substituted by the 𝑥 values, then
36 − 36 = 0 it is verified that the solutions or roots
0 = 0✔ of the equation are 1 and −11 .
Substitute 𝑥2 = −11 to (𝑥 + 5)2 − 36 = 0
(−11 + 5)2 − 36 = 0
(−6)2 − 36 = 0
36 − 36 = 0
0 = 0✔

Confidential to United Marine Training Center, Inc. Page 3-35


Pre Sea Review Materials <course code>
Version Number
Deck
5 November 2018

Example 3-58

Solve 𝑥(3𝑥 + 4) = 7
Solution

𝑥(3𝑥 + 4) = 7

3𝑥 2 + 4𝑥 = 7 Simplify by the distributive property

3𝑥 2 + 4𝑥 − 7 = 0 Write it in standard form


𝑎𝑥 2 + 𝑏𝑥 + 𝑐 = 0

𝑎 = 3; Identify the values of 𝑎, 𝑏, and 𝑐


𝑏 = 4;
𝑐 = −7

−𝑏 ± √𝑏 2 − 4𝑎𝑐 Plug in the values of 𝑎, 𝑏, and 𝑐 into the


𝑥= quadratic formula
2𝑎
−(4) ± √(4)2 − 4(3)(−7)
𝑥=
2(3)

−(4) ± √16 + 84 Simplify to get the values of 𝑥


𝑥=
6
−(4) ± √100 −4 ± 10
𝑥= =
6 6
−4 + 10 6
𝑥1 = = = 1 ; 𝒙𝟏 = 𝟏
6 6
−4 − 10 −14 −7 𝟕
𝑥2 = = = ; 𝒙𝟐 = −
6 6 3 𝟑

−7
Substitute 𝑥 = to 𝑥(3𝑥 + 4) = 7 Check by substituting the obtained
3
values of 𝑥 into the original equation.
7 −7 Since the left - hand side and the right -
− [3 ( ) + 4] = 7
3 3 hand side of the equations are equal
−7 when substituted by the 𝑥 values, then
[−7 + 4] = 7 it is verified that the solutions or roots of
3 −7
−7 the equation are and 1
3
(−3) = 7
3
7=7✔
Substitute 𝑥 = 1 to 𝑥(3𝑥 + 4) = 7
1[3(1) + 4] = 7
1[7]=7
7=7✔

Confidential to United Marine Training Center, Inc. Page 3-36


Pre Sea Review Materials <course code>
Version Number
Deck
5 November 2018

Supplementary Exercises
Solve for the value of 𝒙
Answer
1] 2
3𝑥 = 75 𝑥 = 5; 𝑥 = −5
2] 2
𝑥 = 3𝑥 𝑥 = 0; 𝑥=3
3] 𝑥 2 + 9 = 6𝑥 𝑥 = 3; 𝑥=3
4] (𝑥 − 2)(𝑥 + 3) = 6 𝑥 = −4 ; 𝑥=3
5] 2
2𝑥 − 5𝑥 + 1 = 0 5 + √17 5 − √17
𝑥= ; 𝑥=
4 4
6] (𝑥 + 4)2 = 49 𝑥 =3, 𝑥 = −11
7] 6𝑥 2 − 17𝑥 = −12 4 3
𝑥= , 𝑥=
3 2
8] 3𝑥 2 = 7𝑥 + 20 −5
𝑥= ; 𝑥=4
3
9] (2𝑥 − 5)2 + 5 = 3 5 + 𝑖√2 5 − 𝑖√2
𝑥= ; 𝑥=
2 2
10] 𝑥+2 3 𝑥 = 25.51; 𝑥 = 1.49
+ =7
4 𝑥−1

Confidential to United Marine Training Center, Inc. Page 3-37


Pre Sea Review Materials <course code>
Version Number
Deck
5 November 2018

Formula Transformation

In the study of math and sciences, students often meet formulas. They are used to relate physical
quantities to each other and are used to find the numeric value of a variable when the other variables
are known.
A formula is an equation that involves two or more variables that have a specific relationship with
each other.
To solve for a variable in a formula, the formula can be transformed into another one in which
the selected variable is expressed in terms of other variables, with no numeric values involved. The
process is similar to solving a linear equation.

Example 3-59

Consider one of the sailing formulas in navigation to calculate departure (𝐷𝑒𝑝). Transform the
formula to solve for 𝐷𝑙𝑜.

𝐷𝑒𝑝 = 𝐷𝑙𝑜 ∙ cos(𝑙𝑎𝑡) Formula for Departure using Parallel Sailing


Where:
Dep = Departure, the distance travelled due east or
west
Dlo = The angular distance between meridians
lat = The angle from the equatorial plane to the
parallel of latitude

Solution

𝐷𝑒𝑝 = 𝐷𝑙𝑜 ∙ 𝑐𝑜𝑠(𝑙𝑎𝑡)

𝐷𝑒𝑝 𝐷𝑙𝑜 ∙ 𝑐𝑜𝑠(𝑙𝑎𝑡) Apply division property of equality


=
𝑐𝑜𝑠(𝑙𝑎𝑡) 𝑐𝑜𝑠(𝑙𝑎𝑡)

𝐷𝑒𝑝 𝐷𝑙𝑜 ∙ 𝑐𝑜𝑠(𝑙𝑎𝑡) Perform cancellation of factors


=
𝑐𝑜𝑠(𝑙𝑎𝑡) 𝑐𝑜𝑠(𝑙𝑎𝑡)

𝐷𝑒𝑝
= 𝐷𝑙𝑜
𝑐𝑜𝑠(𝑙𝑎𝑡)

𝑫𝒆𝒑 Answer
𝑫𝒍𝒐 =
𝒄𝒐𝒔(𝒍𝒂𝒕)

Example 3-60

Given the formula for finding the perimeter (𝑃) of a rectangle. Rearrange the formula to solve for 𝐿.

𝑃 = 2𝐿 + 2𝑊 Formula for the Perimeter of Rectangle


Where:
P = Perimeter of rectangle
L = Length of rectangle
W = Width of rectangle

Confidential to United Marine Training Center, Inc. Page 3-38


Pre Sea Review Materials <course code>
Version Number
Deck
5 November 2018

Solution

𝑃 = 2𝐿 + 2𝑊

𝑃 − 2𝑊 = 2𝐿 + 2𝑊 − 2𝑊 Apply subtraction property of equality or transpose 2𝑊 to


the other side of the equation to isolate 2𝐿
𝑃 − 2𝑊 = 2𝐿 + 2𝑊 − 2𝑊
𝑃 − 2𝑊 = 2𝐿

𝑃 − 2𝑊 2𝐿 Divide both sides by 2


=
2 2 Perform cancellation of factors
𝑃 − 2𝑊 2𝐿
=
2 2
𝑃 − 2𝑊
=𝐿
2

𝑃 2𝑊 Distribute the denominator 2 to each term in the


− =𝐿 numerator
2 2
𝑃 2𝑊 Perform cancellation of factors
− =𝐿
2 2
𝑃
−𝑊=𝐿
2
𝑷 Answer
𝑳= −𝑾
𝟐

Example 3-61

Transform the given formula to solve for the broken stowage (𝐵𝑆 ).

𝑣𝑐 Formula for the Volume of the Hold with Broken Stowage


𝑉𝐻 =
1 − 𝐵𝑠
Where:
VH = Volume of hold
vc = Volume of cargo stowed in hold
BS = Broken Stowage

Solution
𝑣𝑐
𝑉𝐻 =
1 − 𝐵𝑆

𝑣𝑐 Apply multiplication property of equality or cross-


(1 − 𝐵𝑠 ) [𝑉𝐻 = ] 1 − 𝐵𝑆
1 − 𝐵𝑆 multiplication method to remove fraction
(1 − 𝐵𝑠 )𝑉𝐻 = 𝑣𝑐

𝑉𝐻 (1 − 𝐵𝑠 ) 𝑣𝑐 Apply division property of equality


=
𝑉𝐻 𝑉𝐻
𝑉𝐻 (1 − 𝐵𝑠 ) 𝑣𝑐
=
𝑉𝐻 𝑉𝐻

Confidential to United Marine Training Center, Inc. Page 3-39


Pre Sea Review Materials <course code>
Version Number
Deck
5 November 2018

𝑣𝐶
(1 − 𝐵𝑆 ) =
𝑉𝐻

𝑣𝐶 By symmetric property
= (1 − 𝐵𝑆 )
𝑉𝐻 Parenthesis can also be removed since the factor
𝑣𝐶
= 1 − 𝐵𝑆 (1 − 𝐵𝑆 ) does not have any other factor beside it
𝑉𝐻
or the () is preceded by +.

𝒗𝑪 Transpose (−𝐵𝑆 ) to the left side to change its sign


𝑩𝑺 = 𝟏 − 𝑣
𝑽𝑯 to (+) and 𝐶 to the right side making its sign (−)
𝑉𝐻

𝑉𝐻 𝑣𝐶 This is another way to write the formula for broken


𝐵𝑆 = − 𝑉
𝑉𝐻 𝑉𝐻 stowage. It is obtained by rewriting 1 as 𝐻, then
𝑉𝐻
𝑽𝑯 − 𝒗𝒄 apply subtraction of similar fractions.
𝑩𝑺 =
𝑽𝑯

Example 3-62

Transform the given formula to solve for the height of the center gravity (𝐾𝐺) from the given formula.

𝐺𝑍 = 𝐾𝑁 − 𝐾𝐺𝑠𝑖𝑛∅ Formula for Righting arm considering 𝑲𝑵 and 𝑲𝑮 Values


Where:
GZ = Righting arm
KN = Righting arm at 𝐾𝐺 = 0
KG = Distance form Keel to Center of Gravity
∅ = Angle of Heel

Solution

𝐺𝑍 = 𝐾𝑁 − 𝐾𝐺𝑠𝑖𝑛∅

𝐾𝐺𝑠𝑖𝑛∅ = 𝐾𝑁 − 𝐺𝑍 Transpose −𝐾𝐺𝑠𝑖𝑛∅ to the left side and GZ to the right


side. Note that their signs change.

𝐾𝐺 𝑠𝑖𝑛 ∅ 𝐾𝑁 − 𝐺𝑍 Divide both sides by 𝑠𝑖𝑛 ∅


=
𝑠𝑖𝑛 ∅ 𝑠𝑖𝑛 ∅ Perform cancellation of factors
𝐾𝐺 𝑠𝑖𝑛 ∅ 𝐾𝑁 − 𝐺𝑍
=
𝑠𝑖𝑛 ∅ 𝑠𝑖𝑛 ∅

𝑲𝑵 − 𝑮𝒁
𝑲𝑮 =
𝒔𝒊𝒏∅

Confidential to United Marine Training Center, Inc. Page 3-40


Pre Sea Review Materials <course code>
Version Number
Deck
5 November 2018

Example 3-63

𝐿
Rearrange the formula for the period (𝑇), 𝑇 = 2𝜋√𝑔 to find an expression for 𝐿 of a simple
pendulum of length 𝐿.
Solution

𝐿
𝑇 = 2𝜋√
𝑔

2
Square both sides of the equation to remove the
𝐿 radical sign
[ 𝑇 = 2𝜋√ ]
𝑔
The exponent must be distributed to all the factors
2
𝐿
(𝑇)2 = (2𝜋)2 (√ )
𝑔

𝐿 Simplify
𝑇 2 = (2𝜋)2 ( )
𝑔
4𝜋 2 𝐿
𝑇2 =
𝑔

𝑔 2
4𝜋 2 𝐿 𝑔 𝑔
Multiply both sides of the equation by (4𝜋2) to
( 2
) [𝑇 = ] ( 2)
4𝜋 𝑔 4𝜋 solve for 𝐿
𝑔 4𝜋 4 𝐿 𝑔 Perform cancellation of factors
( 2 ) [𝑇 2 = ] ( 4)
4𝜋 𝑔 4𝜋

𝑇2𝑔
=𝐿
4𝜋 2

𝑻𝟐 𝒈 By symmetric property
𝑳=
𝟒𝝅𝟐

Confidential to United Marine Training Center, Inc. Page 3-41


Pre Sea Review Materials <course code>
Version Number
Deck
5 November 2018

Supplementary Exercises
Solve for the indicated variable
Answer
1] 𝑉 𝑉
𝑆𝑓 = ; 𝑓𝑜𝑟 𝑚 𝑚=
𝑚 𝑆𝑓
2] 𝐷𝑙𝑎𝑡 = 𝑙𝑎𝑡2 − 𝑙𝑎𝑡1 ; 𝑓𝑜𝑟 𝑙𝑎𝑡_2 𝐷𝑙𝑎𝑡 + 𝑙𝑎𝑡1 = 𝑙𝑎𝑡2
3] 𝐺𝑍 = 𝐺𝑀 𝑠𝑖𝑛 𝜃 ; 𝑓𝑜𝑟 𝐺𝑀 𝐺𝑀 = 𝐺𝑍/ 𝑠𝑖𝑛 𝜃
4] ℎ𝑎𝑣𝐺𝐶𝐷 − ℎ𝑎𝑣𝐷𝑙𝑎𝑡 ℎ𝑎𝑣𝐺𝐶𝐷 = ℎ𝑎𝑣𝐷𝑙𝑜 ∙ 𝑐𝑜𝑠𝑙𝑎𝑡1 ∙ 𝑐𝑜𝑠𝑙𝑎𝑡2 + ℎ𝑎𝑣𝐷𝑙𝑎𝑡
ℎ𝑎𝑣𝐷𝑙𝑜 = ; 𝑓𝑜𝑟 ℎ𝑎𝑣 𝐺𝐶𝐷
𝑐𝑜𝑠𝑙𝑎𝑡1 ∗ 𝑐𝑜𝑠𝑙𝑎𝑡2
5] 1 2(𝑑 − 𝑣𝑖 𝑡)
𝑑 = 𝑣𝑖 𝑡 + 𝑎𝑡 2 ; 𝑓𝑜𝑟 𝑎 𝑎=
2 𝑡2
6] 57.3𝑉 60𝑅(𝑅. 𝑜. 𝑇)
𝑅. 𝑜. 𝑇 = ; 𝑓𝑜𝑟 𝑉 𝑉=
60𝑅 57.3
7] 𝐺𝑚1 𝑚2
= 𝐹𝑔 ; 𝑓𝑜𝑟 𝑑 𝐺𝑚1 𝑚2
𝑑2 𝑑=√
𝐹𝑔

Confidential to United Marine Training Center, Inc. Page 3-42


Pre Sea Review Materials <course code>
Version Number
Deck
5 November 2018

Variation

In Mathematics, it is usual to deal with two types of quantities: (1) variable quantities (or simply
variables) and constant quantities (or simply constants). Such relationship with regards to the
change in the value of a variable when the values of the related variables change, is termed as
variation.
Variation is defined by any change in some quantity due to change in another.
It is an expression in which a variable may be related to a power, a quotient, a product, or a
combination of powers, quotients, and products of other variables.

Types of Variation

Direct Variation
Consider the table below,
Diameter, 𝒅 Circumference, 𝑪
(𝑚𝑒𝑡𝑒𝑟) (𝑚𝑒𝑡𝑒𝑟)
1 3.1416
2 6.2832
3 9.4248
4 12.5664
Table 3-1: Diameter and circumference relationship – Source: Ester Dinawanao

It can be observed that as the diameter (𝑑) is doubled, circumference (𝐶) is also doubled. If the
diameter is halved, circumference is also halved. In general, when the diameter (𝑑) decreases,
circumference (𝐶) also decreases or an increase in diameter yields a proportional increase in the
circumference.
In this case, circumference 𝐶 varies directly as diameter 𝑑 or 𝐶 is directly proportional to 𝑑. In
symbol, 𝐶 𝛼 𝑑 . Mathematically, this can be translated to an equation 𝐶 = 𝑘𝑑 , where 𝑘 is the constant
of proportionality.
𝐶
Furthermore, if you divide the circumference by its corresponding diameter, , the quotient is always
𝑑
3.1416 or 𝜋. The constant of proportionality in this case is 𝜋; thus, the circumference of the circle is
given by 𝐶 = 𝜋𝑑. The graph of circumference against the diameter is a straight line passing through
the origin.
Key notes:
 Two quantities are directly proportional to each other if an increase in one results to a
proportional increase in the other, or if a decrease in one quantity results to a corresponding
decrease in the other.
 Generally, the variable 𝑦 is said to vary directly as 𝑥 (or to be directly proportional to 𝑥),
when 𝑥 and 𝑦 are related by the equation 𝑦 = 𝑘𝑥. In other words, the variables change at the
same rate. If the value of one variable doubles, the value of other variable doubles, if one
value triples, the other triples. The graph is a line passing through the origin with slope 𝑘. In
𝑦
this case the slope 𝑘 = 𝑥 is also called the constant of variation or constant of
proportionality.

Confidential to United Marine Training Center, Inc. Page 3-43


Pre Sea Review Materials <course code>
Version Number
Deck
5 November 2018

Example 3-64

According to Hooke's Law, the amount of force (𝐹) needed to stretch a spring varies directly to the
amount of stretch (𝑥). If 50 Newton of force stretches a spring 5 m, how much will the spring be
stretched by a force of 120 Newton?
Solution

𝐹 ∝ 𝑥 The mathematical expression for the statement “Force 𝐹 varies


directly to the amount of stretch 𝑥”

𝐹 = 𝑘𝑥 Convert it into an equation by writing = sign and introducing the


constant of variation 𝑘

𝐹 Transform the equation for 𝑘


𝑘=
𝑥
50 Solve for 𝑘 by substituting the given values
𝑘= ; 𝑘 = 10
5

𝐹 = 𝑘𝑥; 𝐹 = 10𝑥 Write the specific equation by substituting the value of 𝑘 into the
original equation

𝐹 Transform the equation 𝐹 = 10𝑥 in terms of 𝑥 since it is the one


𝑥=
10 required in the problem

120 Solve for 𝑥 by substituting 𝐹 = 120 into the equation.


𝑥=
10 Thus, a force of 120 N will stretch the spring to 12 m. As expected,
𝒙 = 𝟏𝟐 the amount of stretch is longer than 5 m since the force applied
increases from 50 N to 120 N.

Alternate Solution:

𝐹 ∝𝑥 Since force 𝐹 varies directly to the amount of


𝐹
𝐹 = 𝑘𝑥 stretch 𝑥, the ratio is constant.
𝑥

𝐹
𝑘=
𝑥
𝐹1 𝐹2 Set up a proportion
=
𝑥1 𝑥2
𝑘1 = 𝑘2

50 𝑁 120 𝑁 Let 𝐹1 = 50 𝑁 , 𝑥1 = 5 𝑚 , 𝐹2 = 120 𝑁 , 𝑥2 = ?


= and substitute to the proportion
5𝑚 𝑥2

(50 𝑁)(𝑥2 ) = (120 𝑁)(5𝑚) Apply cross multiplication to solve the


equation and simplify
50 𝑁 ∙ 𝑥2 = 600 𝑁 ∙ 𝑚

50 𝑁 ∙ 𝑥2 600 𝑁 ∙ 𝑚 Divide both sides of the equation by 50 N


=
50 𝑁 50 𝑁
50 𝑁 ∙ 𝑥2 600 𝑁 ∙ 𝑚
=
50 𝑁 50 𝑁

Confidential to United Marine Training Center, Inc. Page 3-44


Pre Sea Review Materials <course code>
Version Number
Deck
5 November 2018

600 𝑚
𝑥2 =
50

𝒙𝟐 = 𝟏𝟐 𝒎 The same answer is obtained which is 12 m.

Inverse Variation
Consider the graph of speed versus time below,

1000
𝑠=
𝑡

t
1000
Figure 3-8: Speed versus time graph with equation 𝑠 = ; – Source: Christian Kevin Villanueva, graphed using
𝑡
https://www.desmos.com/calculator

The coordinates of any point on the graph represent the time and the speed respectively. Considering
two points on the graph (1, 1000) and (40, 25). It can be seen that the faster the speed the lesser the
time as represented by the coordinates of the point (1, 1000). On the other hand, the longer the time,
the slower the speed as represented by the coordinates (40, 25).
In this situation, the relationship between speed (𝑠) and time (𝑡) is said to be inversely proportional
1
or speed varies inversely with time. In symbol, 𝑠 𝛼 . Mathematically, this can be translated to an
𝑡
𝑘
equation 𝑠 = , where 𝑘 is the constant of proportionality or constant of variation.
𝑡

Moreover, if you multiply the coordinates of each point on the graph, 𝑠 ∙ 𝑡, the product is always 1000.
The constant of proportionality (𝑘) in this case is 1000 which represents the distance. The graph of
speed versus time is no longer a straight line but a curve.
Key notes:
 A relationship between two variables where as one value increases, the other decreases.
 When there is inverse variation between variables 𝑥 and 𝑦, doubling the value of 𝑥 will
always result in 𝑦 being cut in half. Also, if 𝑥 is cut in half, then 𝑦 is doubled.
 Generally, the variable 𝑦 is said to vary inversely as 𝑥 (or to be inversely proportional to 𝑥),
𝑘
when 𝑥 and 𝑦 are related by the equation 𝑦 = . The constant of proportionality (𝑘) is the
𝑥
product of 𝑥 and 𝑦, 𝑘 = 𝑥𝑦.

Confidential to United Marine Training Center, Inc. Page 3-45


Pre Sea Review Materials <course code>
Version Number
Deck
5 November 2018

Example 3-65

The time (𝑡) it takes to travel a fixed distance varies inversely with the speed (𝑠) travelled. If a ship
travelling at 12 knots can complete a certain voyage in 16 days. How many days would the ship take
to do the same voyage at a speed of 15 knots?
Solution
1 By intuition, the faster a vessel moves, the less time it
𝑡∝ would take to cover a certain distance. A relationship
𝑠
where as one value increases (s) while the other decreases
(t) shows inverse variation which is mathematically
expressed on the left.

𝑘 Since the one required in the problem is the algebraic


𝑡= → ① equation to solve for t, the proportionality symbol ∝ is
𝑠
replaced by = and the constant of variation 𝑘 is introduced.
Name this equation as equation 1

𝑘 = 𝑠∙𝑡 Transform the previous equation to solve for k

𝑘 = (12 𝑘𝑛𝑜𝑡𝑠)(16 𝑑𝑎𝑦𝑠) Substitute the given values

𝑘 = 192 𝑘𝑛𝑜𝑡𝑠 ∙ 𝑑𝑎𝑦𝑠 Constant of variation

192 𝑘𝑛𝑜𝑡𝑠 ∙ 𝑑𝑎𝑦𝑠 Substitute the constant of variation to equation 1


𝑡=
𝑠
192 𝑘𝑛𝑜𝑡𝑠 ∙ 𝑑𝑎𝑦𝑠 Substitute the given values
𝑡=
15 𝑘𝑛𝑜𝑡𝑠
192 𝑘𝑛𝑜𝑡𝑠 ∙ 𝑑𝑎𝑦𝑠 Perform cancellation of units
𝑡=
15 𝑘𝑛𝑜𝑡𝑠

𝒕 = 𝟏𝟐. 𝟖 𝒅𝒂𝒚𝒔 It will take 12.8 days for the ship to do the same voyage
with a speed of 15 knots.
Since there is an increase in the speed (from 12 knots to
16 knots), it is expected that the time to take the same
voyage would be lesser than 16 days, which is 12.8 days in
this case.

Alternate Solution
1 Since time 𝑡 varies inversely with speed 𝑠, the
𝑡 ∝
𝑠 product, 𝑠 ∙ 𝑡 is constant (𝑘).
𝑘
𝑡=
𝑠
𝑠𝑡 = 𝑘

𝑡1 𝑡2 Set up a proportion
=
𝑠2 𝑠1
𝑠1 𝑡1 = 𝑠2 𝑡2
𝑘1 = 𝑘2

Confidential to United Marine Training Center, Inc. Page 3-46


Pre Sea Review Materials <course code>
Version Number
Deck
5 November 2018

16 𝑑𝑎𝑦𝑠 𝑡2 Let 𝑡1 = 16 𝑑𝑎𝑦𝑠 , 𝑠1 = 12 𝑘𝑛𝑜𝑡𝑠 , 𝑠2 = 15 𝑘𝑛𝑜𝑡𝑠 ,


=
15 𝑘𝑛𝑜𝑡𝑠 12 𝑘𝑛𝑜𝑡𝑠 𝑡2 = ? and substitute to the proportion

12 𝑘𝑛𝑜𝑡𝑠 ∙ 16 𝑑𝑎𝑦𝑠 = 15 𝑘𝑛𝑜𝑡𝑠 ∙ 𝑡2 Cross Multiply

192 𝑘𝑛𝑜𝑡𝑠 ∙ 𝑑𝑎𝑦𝑠 = 15 𝑘𝑛𝑜𝑡𝑠 ∙ 𝑡2 Simplify


192 𝑘𝑛𝑜𝑡𝑠 ∙ 𝑑𝑎𝑦𝑠 Divide both sides by 15 knots
= 𝑡2
15 𝑘𝑛𝑜𝑡𝑠 Perform cancellation of units
192 𝑘𝑛𝑜𝑡𝑠 ∙ 𝑑𝑎𝑦𝑠
= 𝑡2
15 𝑘𝑛𝑜𝑡𝑠
12.8 𝑑𝑎𝑦𝑠 = 𝑡2

𝒕𝟐 = 𝟏𝟐. 𝟖 𝒅𝒂𝒚𝒔 The same answer is obtained which is 12.8.

Joint Variation
There are situations when more than one variable is involved in a direct variation problem.
In these cases, the relationship is called joint variation. In here, one variable depends on more than
one other variable.
For example, the volume (𝑉) of a cylinder varies jointly as the square of the radius (𝑟) and the height
(ℎ). This statement is translated as 𝑉 ∝ 𝑟 2 ℎ. This means that the volume is dependent on the radius
and height.
If radius (𝑟) is held constant, then doubling the height (ℎ) of the cylinder will also double the volume
(𝑉). If height (ℎ) is constant, doubling the radius (𝑟) of the cylinder will quadruple the volume (𝑉) of the
cylinder.
To be able to solve for any variable, the statement can be written as an equation 𝑉 = 𝑘𝑟 2 ℎ (the value
of 𝑘 is known to be 𝜋).

Example 3-66

The amount of coal (𝐴) used by a steamship travelling at a uniform speed varies jointly as the
distance (𝑑) travelled and the square of the speed (𝑠). If a steamship uses 45 metric tons of coal
travelling 80 NM at 15 knots, how many metric tons will it use if it travels 120 NM at 20 knots?
Solution

𝐴 ∝ 𝑑𝑠 2 The expression on the left is the mathematical


translation of the expression “Amount of coal
(𝐴) varies jointly as the distance (𝑑) and the
square of the speed (𝑠)”

𝐴 = 𝑘𝑑𝑠 2 Convert it into an equation by writing = sign and


2 introducing the constant of variation 𝑘
𝐴 𝑘𝑑𝑠
2
= Divide both sides by 𝑑𝑠 2
𝑑𝑠 𝑑𝑠 2
𝐴 𝑘𝑑𝑠 2 Perform cancellation of factors
=
𝑑𝑠 2 𝑑𝑠 2
𝐴
=𝑘
𝑑𝑠 2

Confidential to United Marine Training Center, Inc. Page 3-47


Pre Sea Review Materials <course code>
Version Number
Deck
5 November 2018

𝐴 Transform the equation for 𝑘


𝑘=
𝑑𝑠 2
45 Solve for 𝑘 by substituting the given values ;
𝑘=
(80)(152 ) 𝐴1 = 45 𝑡𝑜𝑛𝑠;
1
𝑘 = 0.0025; 𝑜𝑟 𝑘 = 𝑑 1 = 80 𝑁𝑀;
400
𝑠1 = 15 𝑘𝑛𝑜𝑡𝑠

𝐴 = 𝑘𝑑𝑠 2 Write the specific equation by substituting the


value of 𝑘 into the original equation
𝐴 = 0.0025𝑑𝑠 2
1
𝐴= 𝑑𝑠 2
400

𝐴 = (0.0025)(120)(20)2 Use the equation, 𝐴 = 0.0025𝑑𝑠 2 to solve the


amount of coal (𝐴) in the problem by
substituting
𝑑2 = 120 𝑁𝑀 ;
𝑠2 = 20 𝑘𝑛𝑜𝑡𝑠

𝑨 = 𝟏𝟐𝟎 𝑴𝑻 Calculate the value of 𝐴.


The amount of coal needed by a steamship to
travel 80 NM distance at a speed of 20 knots is
120 MT.
The amount of coal increases from 45 MT to
120 MT due to the increase of the distance
travelled from 80 NM to 120 NM and the
increase of the speed from 15 knots to 20 knots.

Alternate Solution

𝐴 ∝ 𝑑𝑠 2 Since amount of coal (𝐴) varies


jointly as the distance (𝑑) and
𝐴 = 𝑘 𝑑𝑠 2
the square of the speed (𝑠), the
𝐴
𝐴 𝑑𝑠 2 ratio, 2 is constant.
= 𝑘 𝑑𝑠
𝑑𝑠 2 𝑑𝑠 2
𝐴
𝑘= 2
𝑑𝑠

𝑘1 = 𝑘2 Set up a proportion
𝐴1 𝐴2
2
=
𝑑1 (𝑠1 ) 𝑑2 (𝑠2 )2

45 𝑀𝑇 𝐴2 Let:
2
=
80 𝑁𝑀 (15 𝑘𝑡𝑠) 120 𝑁𝑀 (20 𝑘𝑡𝑠)2 𝐴1 = 45 𝑀𝑇;
𝑑 1 = 80 𝑁𝑀;
𝑠1 = 15 𝑘𝑛𝑜𝑡𝑠;

Confidential to United Marine Training Center, Inc. Page 3-48


Pre Sea Review Materials <course code>
Version Number
Deck
5 November 2018

𝑑2 = 120 𝑁𝑀;
𝑠2 = 20 𝑘𝑛𝑜𝑡𝑠;
𝐴2 =?
and substitute to the proportion

45 𝑀𝑇 𝐴2 Simplify the denominator of each


2
= ratio
18000 𝑁𝑀 ∙ 𝑘𝑡𝑠 48000 𝑁𝑀 ∙ 𝑘𝑡𝑠 2

(45 𝑀𝑇 )(48 000 𝑁𝑀 ∙ 𝑘𝑡𝑠 2 ) = (18 000 𝑁𝑀 ∙ 𝑘𝑡𝑠 2 )(𝐴2 ) Apply cross multiplication to
solve the equation and simplify
2 160 000 𝑀𝑇 ∙ 𝑁𝑀 ∙ 𝑘𝑡𝑠 2 = (18 000 𝑁𝑀 ∙ 𝑘𝑡𝑠 2 )(𝐴2 )

2 160 000 𝑀𝑇 ∙ 𝑁𝑀 ∙ 𝑘𝑡𝑠 2 (18 000 𝑁𝑀 ∙ 𝑘𝑡𝑠 2 )(𝐴2 ) Divide both sides of the equation
= by 18 000 𝑁𝑀 ∙ 𝑘𝑡𝑠 2
18 000 𝑁𝑀 ∙ 𝑘𝑡𝑠 2 18 000 𝑁𝑀 ∙ 𝑘𝑡𝑠 2

2 160 000 𝑀𝑇 ∙ 𝑁𝑀 ∙ 𝑘𝑡𝑠 2 (18 000 𝑁𝑀 ∙ 𝑘𝑡𝑠 2 )(𝐴2 ) Perform cancellation of units
=
18 000 𝑁𝑀 ∙ 𝑘𝑡𝑠 2 18 000 𝑁𝑀 ∙ 𝑘𝑡𝑠 2

120 𝑀𝑇 = 𝐴2

𝑨𝟐 = 𝟏𝟐𝟎 𝑴𝑻 The same answer is obtained


which is 120 MT.

Combine Variation
A combination of direct and inverse variation.
For instance, according to Newton’s Law of Universal Gravitation, the gravitational attraction 𝐹𝑔
between two objects with masses 𝑚1 and 𝑚2 is directly proportional to the product of their masses
and inversely proportional to the square of the distance 𝑑 between their centers. Mathematically, it
is written as

𝐹𝑔 ∝ 𝑚1 𝑚2 This expression is the mathematical translation of the


statement “𝐹𝑔 between two objects with masses 𝑚1 and
𝑚2 is directly proportional”

1 This expression is the mathematical translation of the


𝐹𝑔 ∝
𝑑2 statement “𝐹𝑔 is inversely proportional to the square of the
distance 𝑑 between their centers.”
𝑚1 𝑚2 This expression is the combined direct and inverse
𝐹𝑔 ∝
𝑑2 variation of the previous two equations
𝑚1 𝑚2 To translate the variation into an equation, the ∝ is
𝐹𝑔 = 𝑘
𝑑2 replaced by the = sign and the constant of variation is
introduced

𝑘𝑚 𝑚
1 2 1
If everything else constant in the formula 𝐹𝑔 = , 𝐹𝑔 is 𝑡ℎ of the original value when 𝑑 is tripled.
𝑑2 9
Furthermore, 𝐹𝑔 is 4 times the original value if 𝑑 is halved.

Confidential to United Marine Training Center, Inc. Page 3-49


Pre Sea Review Materials <course code>
Version Number
Deck
5 November 2018

Example 3-67

The number of hours (ℎ) that it takes (𝑚) men to assemble 𝑥 machines varies directly as the number
of machines and inversely as the number of men. If four men can assemble 12 machines in 4 hours,
how many men are needed to assemble 36 machines in 8 hours?
Solution
𝑥 Translate the statement;
ℎ ∝
𝑚
The number of hours (ℎ) that it takes (𝑚) men to
assemble 𝑥 machines varies directly as the
number of machines and inversely as the
number of men
𝑥 Convert it into an equation by writing = sign and
ℎ = 𝑘
𝑚 introducing the constant of variation 𝑘

ℎ∙𝑚 Transform the equation for 𝑘


𝑘=
𝑥
(4 ℎ𝑜𝑢𝑟𝑠)(4 𝑚𝑒𝑛) Solve for 𝑘 by substituting the given values;
𝑘=
12 𝑚𝑎𝑐ℎ𝑖𝑛𝑒𝑠 𝑚1 = 4 𝑚𝑒𝑛;
ℎ𝑜𝑢𝑟𝑠 ∙ 𝑚𝑒𝑛
𝑘 = 1.333 … 𝑥1 = 12 𝑚𝑎𝑐ℎ𝑖𝑛𝑒𝑠;
𝑚𝑎𝑐ℎ𝑖𝑛𝑒𝑠
4 ℎ𝑜𝑢𝑟𝑠 ∙ 𝑚𝑒𝑛 ℎ1 = 4 ℎ𝑜𝑢𝑟𝑠
𝑘= ∙
3 𝑚𝑎𝑐ℎ𝑖𝑛𝑒𝑠

4 ℎ𝑜𝑢𝑟𝑠 ∙ 𝑚𝑒𝑛 𝑥 Write the specific equation by substituting the


ℎ = ( ∙ )∙
3 𝑚𝑎𝑐ℎ𝑖𝑛𝑒𝑠 𝑚 value of 𝑘 into the original equation

4 ℎ𝑜𝑢𝑟𝑠 ∙ 𝑚𝑒𝑛 𝑥 Transform the formula for 𝑚


𝑚 = ( ∙ )∙
3 𝑚𝑎𝑐ℎ𝑖𝑛𝑒𝑠 ℎ

4 ℎ𝑜𝑢𝑟𝑠 ∙ 𝑚𝑒𝑛 36 𝑚𝑎𝑐ℎ𝑖𝑛𝑒𝑠 Substitute the given values


𝑚 = ( ∙ )∙
3 𝑚𝑎𝑐ℎ𝑖𝑛𝑒𝑠 8 ℎ𝑜𝑢𝑟𝑠 𝑥2 = 36 𝑚𝑎𝑐ℎ𝑖𝑛𝑒𝑠;
ℎ2 = 8 ℎ𝑜𝑢𝑟𝑠

4 ℎ𝑜𝑢𝑟𝑠 ∙ 𝑚𝑒𝑛 36 𝑚𝑎𝑐ℎ𝑖𝑛𝑒𝑠 Perform cancellation of units


𝑚 = ( ∙ )∙
3 𝑚𝑎𝑐ℎ𝑖𝑛𝑒𝑠 8 ℎ𝑜𝑢𝑟𝑠

𝒎 = 𝟔 𝒎𝒆𝒏

Alternate Solution:
𝑥 Since the number of hours (ℎ) that it takes (𝑚)
ℎ 𝛼
𝑚 men to assemble 𝑥 machines varies directly as
𝑥 the number of machines and inversely as the
ℎ = 𝑘 ℎ∙𝑚
𝑚 number of men , 𝑘 = is constant.
𝑥
ℎ∙𝑚
𝑘=
𝑥

𝑘1 = 𝑘2 Set up a proportion

Confidential to United Marine Training Center, Inc. Page 3-50


Pre Sea Review Materials <course code>
Version Number
Deck
5 November 2018

ℎ1 ∙ 𝑚1 ℎ2 ∙ 𝑚2
=
𝑥1 𝑥2

(4)(4) (8)(𝑚2 ) Plug in the given information:


=
12 36 𝑚1 = 4 ;
16 8 𝑚2
= 𝑥1 = 12;
12 36
8 𝑚2 16 ℎ1 = 4;
=
36 12 𝑥2 = 36;
96 𝑚2 = (16)(36) ℎ2 = 8
96 𝑚2 = 576 and simplify.

96 𝑚2 576 Divide both sides of the equation by 96


=
96 96
96𝑚2 576
=
96 96
576
𝑚2 =
96

𝒎𝟐 = 𝟔 𝒎𝒆𝒏 The same answer is obtained which is 6.

Confidential to United Marine Training Center, Inc. Page 3-51


Pre Sea Review Materials <course code>
Version Number
Deck
5 November 2018

Supplementary Exercises
Solve the following the problems
Answer
1] The horsepower (ℎ𝑝) required to propel a ship varies directly
as the cube of its speed (𝑠). If the horsepower required for a 24 000 ℎ𝑝
speed of 15 knots is 10 125, find the horsepower required for a
speed of 20 knots.
2] The intensity of light (𝐿) from a given source is inversely
proportional to the square of the distance (𝐷) from it. The 5625
a. 𝐿 =
𝐷2
intensity of light is 225 candela at a distance of 5 m.
a. Write a specific equation for this relationship. b. 𝐿 = 25
b. Find the intensity of light at a point 15 m from the source.
3] A ship floating at water density of 1025 kg/m3 displaces 46 681
m3 of water. When she shifted to another berth she displaced a
new volume of 47 141 m3 . Find the density (𝜌) of water in the 1015 𝑘𝑔/𝑚3
new berth, considering there is no change with the ship’s
weight during shifting?
4] The speed (𝑠) of a conveyor belt varies jointly as the diameter
(𝑑) of the cog around which it passes and the number of
revolutions per second (𝑛) the cog makes. The speed of a belt (𝑎). 𝑘 = 3.142
that passes round a cog of diameter 0.3 m, revolving 20 times
(𝑏) = 14.14 𝑚/𝑠
per second, is 18.85 m/s. Find the value of (a) the constant of
variation and (b) the speed of a belt passing around a cog half
as big revolving 30 times per second.
5] A job can be completed in 15 days by 12 men. How long will it
take 5 men to finish the same job if they work at the same 1.837 𝑑𝑎𝑦𝑠
rate?
6] The gas in a cylinder canister occupies a volume of 22.5 cm3
and exerts a pressure of 1.9 kg/cm2. If the volume (𝑉) varies 2.85𝑘𝑔/𝑐𝑚2
inversely as the pressure (𝑃), find the pressure if the volume is
reduced to 15 cm3.
7] The kinetic energy (𝐾𝐸) of an object varies directly as its mass
(𝑚) and the square of its velocity (𝑣). If the kinetic energy of an
object with a mass of 2.5 kg moving at 15 m/s is 281.25 Joules, 360 𝐽
find the energy of an object with a mass of 1.8 kg moving at 20
m/s.
8] It requires 2.0 liters of paint to color a wall measuring 2.0
meters by 4.0 meters. How many liters of paint will be needed 8𝐿
to paint a wall measuring 4.0 meters by 8.0 meters?
9] The intensity of illumination (𝐼) of a light varies inversely as the
square of the distance (𝑑) from the light. At a distance of 20 m 64 𝐶𝐷
a light has an intensity of 100 candelas (𝐶𝐷). Find the intensity
of the light at a distance of 25 m.
10] The number of hours needed to assemble computers varies
directly as the number of computers and inversely as the
number of workers. If 4 workers can assemble 12 computers in 41 𝑤𝑜𝑟𝑘𝑒𝑟𝑠
9 hours, how many workers are needed to assemble 48
computers in 8 hours?

Confidential to United Marine Training Center, Inc. Page 3-52


Pre Sea Review Materials <course code>
Version Number
Deck
5 November 2018

11] The period of time (𝑡) required for one complete oscillation of a
pendulum varies directly as the square root of its length (𝑙). A a. 1.06√𝑙
pendulum 1 feet long has a period of 1.06 s.
b. 1.836 𝑠
a. Write the specific equation for this relationship.
b. What is the period of a pendulum 3 ft. long?
12] The weight of a person varies inversely as the square of the
distance from the center of the earth. If the radius of the earth 80 𝑙𝑏𝑠
is 4000 miles, how much would a 180 pound person weigh,
2000 miles above the surface of the earth?

Confidential to United Marine Training Center, Inc. Page 3-53


Pre Sea Review Materials <course code>
Version Number
Deck
5 November 2018

Interpolation

When graphical data contains gap, but data is available on either side of the gap or at a few specific
points within the gap, interpolation allows us to estimate the values within the gap.
An interpolated point is one that lies within the known data range but was not measured.
Interpolation is the process of obtaining a value from a graph or table that is located between major
points given, or between data points plotted. It is a method of estimating values between known data
points. There are many ways to perform interpolation but this module describes the most basic way
which is linear interpolation.
Figure 3-9 shows a sample of data point that can be interpolated between known data points in a
distance versus time graph
𝑑𝑖𝑠𝑡𝑎𝑛𝑐𝑒 (𝑚𝑒𝑡𝑒𝑟)

Known data
points

Interpolated data
point

𝑡𝑖𝑚𝑒 (𝑠𝑒𝑐𝑜𝑛𝑑𝑠)
Figure 3-9: Interpolated data point between two known points – Source: Christian Kevin Villanueva, Graphed using
https:2//www.desmos.com/calculator

The concept of ratio and proportion plays an important role in estimating the value between two
𝑎 𝑐
points. Recall that = if and only if 𝑎𝑑 = 𝑏𝑐.
𝑏 𝑑

Example 3-68

From the table below, determine the missing righting arms (𝐺𝑍’𝑠) corresponding to the given angle
of heels.

Table 3-2: Table of angle of heel vs. GZ for Example 3-68 – Source: Ester Dinawanao

Angle of Heel, 𝜽
(𝑑𝑒𝑔𝑟𝑒𝑒𝑠) 5 10 12 15 20 25 30 35 40

Righting Arm, 𝑮𝒁
0.10 0.21 0.26 - 0.49 - 0.71 - 0.91
(𝑚𝑒𝑡𝑒𝑟)

Confidential to United Marine Training Center, Inc. Page 3-54


Pre Sea Review Materials <course code>
Version Number
Deck
5 November 2018

Solution
Identify the missing values in the table and label them appropriately
Angle of Heel, 𝜽
(𝑑𝑒𝑔𝑟𝑒𝑒𝑠) 5 10 12 15 20 25 30 35 40

Righting Arm, 𝑮𝒁
0.10 0.21 0.26 𝐺𝑍15 0.49 𝐺𝑍25 0.71 𝐺𝑍35 0.91
(𝑚𝑒𝑡𝑒𝑟)

For 𝐺𝑍15, extract the values before and after 𝐺𝑍15 and subtract them accordingly

𝑁1 = 20 − 12

𝐷1 = 15 − 12

Angle of Heel, 𝜽
(𝑑𝑒𝑔𝑟𝑒𝑒𝑠) 12 15 20

Righting Arm, 𝑮𝒁
0.26 𝐺𝑍15 0.49
(𝑚𝑒𝑡𝑒𝑟)

𝐷2 = 𝐺𝑍15 − 0.26

𝑁2 = 0.49 − 0.26

𝑵𝟏 𝑵𝟐 By ratio and proportion, the enteral equation for linear


=
𝑫𝟏 𝑫𝟐 interpolation

20 − 12 0.49 − 0.26 Substitute the explicit values from Table 3-2


=
15 − 12 𝐺𝑍15 − 0.26

8 0.23 Simplify the left side and right side of the equation
=
3 𝐺𝑍15 − 0.26 separately

8(𝐺𝑍15 − 0.26) = 3(0.23) Perform cross multiplication

8𝐺𝑍15 − 2.08 = 0.69 Simplify by distribution and multiplication

8𝐺𝑍15 = 0.69 + 2.08 Transpose −2.08 to the left side making it +2.08

8𝐺𝑍15 = 2.77 Simplify and divide both side by 8


8𝐺𝑍15 2.77
=
8 8
8𝐺𝑍15 2.77
=
8 8

𝑮𝒁𝟏𝟓 = 𝟑. 𝟒𝟔𝟑 𝒎𝒆𝒕𝒆𝒓𝒔 Answer

Confidential to United Marine Training Center, Inc. Page 3-55


Pre Sea Review Materials <course code>
Version Number
Deck
5 November 2018

For 𝐺𝑍25, extract the values before and after 𝐺𝑍25 and subtract them accordingly

𝑁1 = 30 −20

𝐷1 = 25 − 20

Angle of Heel, 𝜽
(𝑑𝑒𝑔𝑟𝑒𝑒𝑠) 20 25 30

Righting Arm, 𝑮𝒁
0.49 𝐺𝑍25 0.71
(𝑚𝑒𝑡𝑒𝑟)

𝐷2 = 𝐺𝑍25 − 0.49

𝑁2 = 0.71 − 0.49

𝑵𝟏 𝑵𝟐 By ratio and proportion, the general equation for


=
𝑫𝟏 𝑫𝟐 linear interpolation

30 − 20 0.71 − 0.49 Substitute the explicit values from Table 3-2


=
25 − 20 𝐺𝑍25 − 0.49

10 0.22 Simplify the left side and right side of the equation
=
5 𝐺𝑍25 − 0.49 separately

10(𝐺𝑍25 − 0.49) = 5(0.22) Perform cross multiplication

10𝐺𝑍25 − 4.9 = 1.1 Simplify by distribution and multiplication

10𝐺𝑍25 = 1.1 + 4.09 Transpose −4.9 to the left side making it +4.9

10𝐺𝑍25 = 6 Simplify and divide both side by 10


10𝐺𝑍25 6
=
10 10

𝑮𝒁𝟐𝟓 = 𝟎. 𝟔 𝒎𝒆𝒕𝒆𝒓𝒔 Answer

Confidential to United Marine Training Center, Inc. Page 3-56


Pre Sea Review Materials <course code>
Version Number
Deck
5 November 2018

For 𝐺𝑍25, extract the values before and after 𝐺𝑍15 and subtract them accordingly

𝑁1 = 40 − 30

𝐷1 = 35 − 30

Angle of Heel, 𝜽
(𝑑𝑒𝑔𝑟𝑒𝑒𝑠) 30 35 40
Righting Arm, 𝑮𝒁
0.71 𝐺𝑍35 0.91
(𝑚𝑒𝑡𝑒𝑟)

𝐷2 = 𝐺𝑍35 − 0.71

𝑁2 = 0.91 − 0.79

𝑵𝟏 𝑵𝟐 By ratio and proportion, the general equation for


=
𝑫𝟏 𝑫𝟐 linear interpolation

40 − 30 0.91 − 0.71 Substitute the explicit values from Table 2-2


=
35 − 30 𝐺𝑍35 − 0.71

10 0.22 Simplify the left side and right side of the equation
=
5 𝐺𝑍35 − 0.71 separately

10(𝐺𝑍35 − 0.71) = 5(0.20) Perform cross multiplication

10𝐺𝑍35 − 7.1 = 1 Simplify by distribution and multiplication

10𝐺𝑍35 = 1 + 7.1 Transpose −7.1 to the left side making it +7.1

10𝐺𝑍35 = 8.1 Simplify and divide both side by 10


10𝐺𝑍35 8.1
=
10 10

𝑮𝒁𝟑𝟓 = 𝟎. 𝟖𝟏 𝒎𝒆𝒕𝒆𝒓𝒔 Answer

Confidential to United Marine Training Center, Inc. Page 3-57


Pre Sea Review Materials <course code>
Version Number
Deck
5 November 2018

Example 3-69

Extracted values from the Traverse Table are shown Table 3-3. Find the difference of latitude (𝐷𝑙𝑎𝑡)
and departure (𝐷𝑒𝑝) at a distance of 430.6 NM.

Table 3-3: Traverse Table for Example 3-69 – Source: Ester Dinawanao

Distance, 𝒅𝒊𝒔𝒕 Departure, 𝑫𝒆𝒑 Difference of latitude, 𝑫𝒍𝒂𝒕


(𝑁𝑎𝑢𝑡𝑖𝑐𝑎𝑙 𝑚𝑖𝑙𝑒𝑠) (𝑁𝑎𝑢𝑡𝑖𝑐𝑎𝑙 𝑚𝑖𝑙𝑒𝑠) (𝑁𝑎𝑢𝑡𝑖𝑐𝑎𝑙 𝑚𝑖𝑙𝑒𝑠)
430 188.5 386.5
431 188.9 387.4

Solution
Create an additional entry between the distance of 430 NM and 431 NM and mark it as 430.6
NM. Label the missing values appropriately

Distance, 𝒅𝒊𝒔𝒕 Departure, 𝑫𝒆𝒑 Difference of latitude, 𝑫𝒍𝒂𝒕


(𝑁𝑎𝑢𝑡𝑖𝑐𝑎𝑙 𝑚𝑖𝑙𝑒𝑠) (𝑁𝑎𝑢𝑡𝑖𝑐𝑎𝑙 𝑚𝑖𝑙𝑒𝑠) (𝑁𝑎𝑢𝑡𝑖𝑐𝑎𝑙 𝑚𝑖𝑙𝑒𝑠)
430 188.5 386.5
430.6 𝐷𝑒𝑝 𝐷𝑙𝑎𝑡
431 188.9 387.4

For 𝐷𝑒𝑝, extract the values before and after 𝐷𝑒𝑝 and subtract them accordingly

Distance, 𝒅𝒊𝒔𝒕 Departure, 𝑫𝒆𝒑


(𝑁𝑎𝑢𝑡𝑖𝑐𝑎𝑙 𝑚𝑖𝑙𝑒𝑠) (𝑁𝑎𝑢𝑡𝑖𝑐𝑎𝑙 𝑚𝑖𝑙𝑒𝑠)
430 188.5
𝑁1 = 430 − 430.6 𝑁2 = 188.5 − 𝐷𝑒𝑝
430.6 𝐷𝑒𝑝
431 188.9
𝐷1 = 430 − 431 𝐷2 = 188.5 − 188.9

𝑵𝟏 𝑵𝟐 By ratio and proportion, the general equation for


= linear interpolation
𝑫𝟏 𝑫𝟐

430 − 430.6 188.5 − 𝐷𝑒𝑝 Substitute the explicit values from Table 3-3
=
430 − 431 188.5 − 188.9

−0.6 188.5 − 𝐷𝑒𝑝 Simplify the left side and right side of the equation
=
−1 −0.4 separately

(−0.4)(−0.6) = (−1)(188.5 − 𝐷𝑒𝑝) Perform cross multiplication

0.24 = −188.5 + 𝐷𝑒𝑝 Simplify by distributing and multiplication


Note that negative (-) multiplied to a negative (-)
number is positive (+)

0.24 + 188.5 = 𝐷𝑒𝑝 Transpose −188.5 to the right side making it +188.5

188.74 = 𝐷𝑒𝑝

𝑫𝒆𝒑 = 𝟏𝟖𝟖. 𝟕𝟒 𝑵𝑴 Answer

Confidential to United Marine Training Center, Inc. Page 3-58


Pre Sea Review Materials <course code>
Version Number
Deck
5 November 2018

For 𝐷𝑙𝑎𝑡, extract the values before and after 𝐷𝑙𝑎𝑡 and subtract them accordingly

Distance, 𝒅𝒊𝒔𝒕 Difference of


(𝑁𝑎𝑢𝑡𝑖𝑐𝑎𝑙 𝑚𝑖𝑙𝑒𝑠) latitude, 𝑫𝒍𝒂𝒕
(𝑁𝑎𝑢𝑡𝑖𝑐𝑎𝑙 𝑚𝑖𝑙𝑒𝑠)
430 386.5
𝑁1 = 430 − 430.6 𝑁2 = 386.5 − 𝐷𝑙𝑎𝑡
430.6 𝐷𝑙𝑎𝑡
431 387.4
𝑫𝟏 = 𝟒𝟑𝟎 − 𝟒𝟑𝟏 𝐷2 = 386.5 − 387.4

𝑵𝟏 𝑵𝟐 By ratio and proportion, the general equation for


=
𝑫𝟏 𝑫𝟐 linear interpolation

430 − 430.6 386.5 − 𝐷𝑙𝑎𝑡 Substitute the explicit values from Table 3-3
=
430 − 431 386.5 − 387.4

−0.6 386.5 − 𝐷𝑙𝑎𝑡 Simplify the left side and right side of the equation
= separately
−1 −0.9

(−0.6)(−0.9) = (−1)(386.5 − 𝐷𝑒𝑝) Perform cross multiplication

0.54 = −386.5 + 𝐷𝑒𝑝 Simplify by distributing and multiplication

0.54 + 386.5 = 𝐷𝑒𝑝 Transpose −386.5 to the right side making it +386.5

387.04 = 𝐷𝑒𝑝

𝑫𝒆𝒑 = 𝟑𝟖𝟕. 𝟎𝟒 𝑵𝑴 Answer

Confidential to United Marine Training Center, Inc. Page 3-59


Pre Sea Review Materials <course code>
Version Number
Deck
5 November 2018

Supplementary Exercises
Solve the following problems
Answer
1] Find 𝐷𝑒𝑝 and 𝐷𝑙𝑎𝑡 corresponding to a course of 26.250 using the
sample Traverse Table
Course 𝑫𝒍𝒂𝒕 𝑫𝒆𝒑 𝐷𝑙𝑎𝑡 = 386.1
(𝑛𝑎𝑢𝑡𝑖𝑐𝑎𝑙 𝑚𝑖𝑙𝑒𝑠) (𝑛𝑎𝑢𝑡𝑖𝑐𝑎𝑙 𝑚𝑖𝑙𝑒𝑠)
𝐷𝑒𝑝 = 190.4
26° 387.0 188.7
27° 383.6 195.5

2] Find the volume (𝑉) corresponding to a sounding of 4.13 m. Refer to


the values extracted from the fuel oil sounding table
Sounding Volume, 𝑽 122.926 𝑚3
(𝑚𝑒𝑡𝑒𝑟) (𝑐𝑢𝑏𝑖𝑐 𝑚𝑒𝑡𝑒𝑟 𝑜𝑟 𝑚3 )
4.10 120.91
4.15 124.27

Confidential to United Marine Training Center, Inc. Page 3-60


Pre Sea Review Materials <course code>
Version Number
Deck
5 November 2018

Plane Trigonometry and its Maritime Application

Trigonometry (from Greek words, “trigonon” meaning triangle and “metron” meaning measure) is a
branch of mathematics that studies triangles and the relationships between the lengths of their sides
and the angles between those sides.
The concepts and principles of trigonometry helps in finding inaccessible heights and distances, and
are used a lot in science, navigation, engineering, video games, and more.
Plane Trigonometry deals with the relations between the angles and sides of triangles of which the
three vertices are located on the surface of a plane and the sides are straight lines.

Definition of Angles

Since many functions in trigonometry are defined in terms of the measures of an angle, it is important
to first understand the notion of an angle.

Angle

𝜃
𝐴
𝑂 Initial Side

Figure 3-10: Angle between two rays – Source: Christian Kevin Villanueva

Based from the definition of geometry, an angle is formed by the intersection of the two rays. The
point of intersection is called the vertex. A ray is a part of a line that has one endpoint and extends
infinitely in one direction.
In Figure 3-10, 𝑂𝐵 ⃑⃑⃑⃑⃑ and 𝑂𝐴
⃑⃑⃑⃑⃑ are rays and 𝑂 is the vertex. Ray 𝑂𝐴 ⃑⃑⃑⃑⃑ is called the initial side (or the side
where the angle is referenced from), while ray 𝑂𝐵 ⃑⃑⃑⃑⃑ is called the terminal side (or the side after the
initial side is rotated).
The angle shown in Figure 3-10 can be named as:
 ∠𝑶 (using the vertex)
 ∠𝑨𝑶𝑩 or ∠𝑩𝑶𝑨 (using 3-letter notation)
 𝜽 (naming the angle itself using Greek letters)

Confidential to United Marine Training Center, Inc. Page 3-61


Pre Sea Review Materials <course code>
Version Number
Deck
5 November 2018

Angle in Standard Position


Figure 3-11 shows two angles in standard position. An angle is in standard position when its vertex is
at the origin of the coordinate system, and its initial side lies along the positive x-axis.

𝑦 𝑦

𝜃 𝑥 Initial Side 𝑥
𝑂 𝑂 𝜃
Initial Side

Figure 3-11: Angles in standard position – Source: Christian Kevin Villanueva

Angles and Rotation


Figure 3-12a illustrates a positive angle which is generated in a counter-clockwise rotation and Figure
3-12b illustrates a negative angle which is generated in a clockwise direction.
The most common unit used to measure angles is the degree abbreviated by the ° symbol. There are
other units used for measuring angle (which is discussed on the next sections) but for the purpose of
defining an angle, degree is used. Note that a complete rotation is equivalent to 360°.

𝑦 𝑦

𝜃 = 160°
𝑥 𝑥

𝜃 = −200°

a b
Figure 3-12: Positive and negative angles – Source: Christian Kevin Villanueva

Reference Angle
For each angle drawn in standard position, there is a related angle known as a reference angle. This
is the positive acute angle formed by the terminal side and either the negative or positive x-axis. In
Figure 3-13, the reference angle of 1150 is 650 . An acute angle is always less than 90°.

𝜃 = 115°
𝜑 = 65°
𝑥

Figure 3-13: Reference angle – Source: Christian Kevin Villanueva

Confidential to United Marine Training Center, Inc. Page 3-62


Pre Sea Review Materials <course code>
Version Number
Deck
5 November 2018

Example 3-70

Determine the reference angle of the following angles in standard position


a. 135°
b. 205°
c. 300°

Solution
Reference angle for a. 135°, Let 𝜃 be the given angle and 𝜑 be its reference angle
𝑦

𝜃 = 135°
𝜑
𝑥

𝝋 = 𝟏𝟖𝟎° − 𝜽 The terminal side of angle 135° is in the 2nd quadrant, to obtain its
reference angle 𝜑, subtract it from 180°

𝜑 = 180° − 135° Substitute the given values

𝝋 = 𝟒𝟓° The reference angle of 135° is 45°

Reference angle for b. 205°, Let 𝜃 be the given angle and 𝜑 be its reference angle
𝑦

𝜃 = 205°
𝑥
𝜑

𝝋 = 𝜽 − 𝟏𝟖𝟎° The terminal side of angle 205° is in the 3rd quadrant, to obtain its
reference angle 𝜑, subtract it by 180°

𝜑 = 205° − 180° Substitute the given values

𝝋 = 𝟐𝟓° The reference angle of 205° is 25°

Confidential to United Marine Training Center, Inc. Page 3-63


Pre Sea Review Materials <course code>
Version Number
Deck
5 November 2018

Reference angle for c. 300°, Let 𝜃 be the given angle and 𝜑 be its reference angle
𝑦

𝜃 = 300°
𝑥
𝜑

𝝋 = 𝟑𝟔𝟎° − 𝜽 The terminal side of angle 300° is in the 4th quadrant, to obtain its
reference angle 𝜑, subtract it from 360°

𝜑 = 360° − 300° Substitute the given values

𝝋 = 𝟔𝟎° The reference angle of 300° is 60°

Figure 3-14 summarizes how to determine the reference angle 𝜑 depending in which quadrant the
terminal side of angle 𝜃 lies

𝑦 𝑦

𝜑 𝜃
𝜃 𝜑 𝑥 𝑥

Quadrant I Quadrant II
𝜃=𝜑 𝜑 = 180° − 𝜃

𝑦 𝑦

𝜃 𝜃
𝑥 𝑥
𝜑 𝜑

Quadrant III Quadrant IV


𝜑 = 180° − 𝜃 𝜑 = 360° − 𝜃
Figure 3-14: Summary of solution for reference angle in each quadrant – Source: Christian Kevin Villanueva

The concept of reference angle is important in trigonometry as it turns out that angles that have the
same reference angles always have the same trigonometric function values (but the sign may vary).
For example, the reference angle for 150° is 30° and it turns out thatsin 150° = 0.5 and sin 30° = 0.5.
Further discussion on the trigonometric functions is discussed on the succeeding sections.

Confidential to United Marine Training Center, Inc. Page 3-64


Pre Sea Review Materials <course code>
Version Number
Deck
5 November 2018

Supplementary Exercises
Obtain the reference angle of the following angles
Answer
1] 28°
28°

2] 150°
30°

3] 225°
45°

4] 330°
30°

5] 170°
10°

6] 412°
52°

Confidential to United Marine Training Center, Inc. Page 3-65


Pre Sea Review Materials <course code>
Version Number
Deck
5 November 2018

Units of Angle Measurement

Review of Geometry of Circle

Since angles are best described using arcs, circles, revolution and rotation, it is helpful to be familiar
with the important terms regarding the geometry of a circle
 Circle. A simple shape consisting of those points in a plane that are of the same distant
from a given point – which is its center.

Distance is
always equal
from the center Center Random points in
to any point in a circle
the circle

Figure 3-15: Definition of a circle – Source: Christian Kevin Villanueva

 Radius. Radius is the distance from the center of a circle to any point on it. It is half of
𝑑
the diameter. In symbol, 𝑟 = 2 where 𝑟 is the radius and 𝑑 is the diameter of the circle

 Diameter. Diameter is the longest distance from one end of a circle to the other. It is
twice the radius. In symbol, 𝑑 = 2𝑟.
 Circumference. Circumference is the distance around the circle .It is obtained by
using the formula, 𝐶 = 2𝜋𝑟 or 𝐶 = 𝜋𝑑 where 𝐶 is the circumference. Circumference is
also known as the perimeter of the circle

Circumference, 𝐶

Radius, 𝑟 Center

Diameter, 𝑑

Figure 3-16: Radius, diameter and circumference of a circle – Source: Christian Kevin Villanueva

Confidential to United Marine Training Center, Inc. Page 3-66


Pre Sea Review Materials <course code>
Version Number
Deck
5 November 2018

 Central angle. The angle subtended at the center of a circle by an arc between those
two points. It is also known as the arc's angular distance. The symbol for the central
angle is usually denoted by the capital Greek letter phi (∅)
 Arc. Arc is a curved line that is part of the circumference of a circle. The length of an
arc 𝑠 is the central angle in radians ∅ multiplied to the radius 𝑟 if the circle. In symbol,
𝑠 = 𝑟∅.
 Minor arc. Minor arc is part of the circle in the interior of the central angle with measure
less than 1800 .
 Major Arc. Minor arc is part of the circle in the exterior of the central angle.

Minor Arc

Central
Angle, ∅

Arcs, 𝑠

Major Arc
Figure 3-17: Central angle, minor arc and major arc of a circle – Source: Christian Kevin Villanueva

Methods of Measuring Angles

There are 3 methods of measuring angles namely:


 Revolution system
 Sexagesimal system
 Circular System

Revolution System
A revolution is the measure of an angle formed when the initial side rotates all the way around its
vertex until it reaches its initial position. A revolution can be abbreviated as "rev". It uses 1 revolution
or a fraction of revolution as its unit. It is commonly used in engineering. One way to visualize a
revolution is to imagine a second hand of clock starting from the twelve o’clock mark and returning to
the twelve o’clock mark after 60 seconds or one minute. The distance travelled by any point on the
wheel is equal to one revolution. Figure 3-18 shows an initial side (green) and a terminal side (red) of
an angle that display how a revolution, a half revolution and a quarter revolution is formed.
Other synonymous names to rev or revolution are: full circle, turn or full turn and rotation or rot.

Confidential to United Marine Training Center, Inc. Page 3-67


Pre Sea Review Materials <course code>
Version Number
Deck
5 November 2018

1 revolution
½ revolution ¼ revolution

90°
360° 180°

Figure 3-18: Revolution as an angular measurement – Source: Christian Kevin Villanueva

Sexagesimals System
The Sexagesimals system or the base 60 system is a system most frequently used in surveying,
navigation and astronomy. Its basic unit is the degree. The degree symbol is a small circle (°) used
which is the same symbol used for temperatures, latitude and longitude, and angles of a circle .The
circumference of a circle is divided into 360 equal arcs.
A degree was used by the Babylonians as early as 1,000 B.C. At that time, they used a number
system based on the number 60, so it was natural for mathematicians of the day to divide the angles
of an equilateral triangle into 60 individual units. These units became known as degrees. Since six
equilateral triangles can be evenly arranged in a circle, the number of degrees in one revolution
became 6 × 60 = 360 (Refer to Figure 3-19). The unit of degrees was subdivided into 60 smaller
units called minutes and in turn, these minutes were subdivided into 60 smaller units called seconds.

Figure 3-19: Six equilateral triangles arranged inside a circle – Source: Christian Kevin Villanueva

To visualize a degree, imagine a circular pizza cut into 8 equal parts by slicing it through the center.
The curved end of each slice of pizza is comparable to an arc which is subtended by an angle equal
to one-eighth of a revolution. Now imagine a circle cut into 360 equal parts. Similarly, the edge of each
1
part of the cut circle forms an arc that is subtended by an angle equal to 360 of a revolution. An angular
1
measurement of of a revolution is equivalent to 1°.
360

Confidential to United Marine Training Center, Inc. Page 3-68


Pre Sea Review Materials <course code>
Version Number
Deck
5 November 2018

1
𝑟𝑒𝑣𝑜𝑙𝑢𝑡𝑖𝑜𝑛
8

1 𝑑𝑒𝑔𝑟𝑒𝑒 or 1°

Figure 3-20: Analogy of degree measurement to pizza slice – Source: Christian Kevin Villanueva

The degree is subdivided into 60 𝑚𝑖𝑛𝑢𝑡𝑒𝑠 (60′) and each minute is further subdivided into
60 𝑠𝑒𝑐𝑜𝑛𝑑𝑠 (60"); thus, 𝟏𝟎 = 𝟔𝟎′ and 𝟏′ = 𝟔𝟎" .

Circular System
This is another system of angular measurement in which the basic unit is the radian. Measuring
angles in radian are very useful in trigonometry and calculus since it expresses the magnitude of an
angle as a real number.

End point of arc


𝑠

𝑟
Central Angle
𝜃 = 1 𝑟𝑎𝑑

𝑟 End point of arc

Figure 3-21: Radian measure – Source: Christian Kevin Villanueva

Recall that a central angle (𝜃) of a circle is an angle with a vertex at the center of a circle. An
intercepted arc (𝑠) is the portion of the circle with endpoints on the sides of the central angle and
remaining points within the interior of the angle.

Confidential to United Marine Training Center, Inc. Page 3-69


Pre Sea Review Materials <course code>
Version Number
Deck
5 November 2018

When a central angle (𝜃) intercepts an arc (𝑠) that has the same length as a radius (𝑟) of the circle,
the measure of the angle is said to be one radian (or 1 rad).
Radian measure (𝜃) is used to express the magnitude of an angle as a real number. It is the ratio of
𝒔
the length of the arc (𝑠) and the radius (𝑟). In in symbol 𝜃 = 𝒓 . If 𝑠 = 𝑟 , then 𝜃 = 1 𝑟𝑎𝑑 as shown in
Figure 3-21.

𝐶
Central Angle
𝜃 ≈ 6.2832 𝑟𝑎𝑑
𝜃 = 2𝜋 𝑟𝑎𝑑

Figure 3-22: Measure of central angle in radian for one revolution – Source: Christian Kevin Villanueva

If the radius (𝑟) on Figure 3-22 is rotated through the entire circumference, then length of the arc is
equal to the circumference of the circle, 𝑠 = 𝐶
Since,
𝒔 By the definition of a radian as the ratio between the arc length
𝜃=
𝒓 𝑠 and radius 𝑟

𝐶 Substitute:
2𝜋 =
𝑟
𝜃 = 2𝜋 and 𝑠 = 𝐶

2𝜋𝑟 = 𝐶 Perform cross multiplication

𝑪 = 𝟐𝝅𝒓 Formula for circumference of a circle


From this formula, the circumference of a circle can also be
defined as an arc subtended by an angle equal to 2𝜋 multiplied
to the radius 𝑟 of that circle.

𝑪 = 𝝅𝒅S Another formula for the circumference of a circle


Note that the diameter 𝑑 is twice the radius 𝑟
𝑑 = 2𝑟

Confidential to United Marine Training Center, Inc. Page 3-70


Pre Sea Review Materials <course code>
Version Number
Deck
5 November 2018

Angle Conversions – Degrees and Radians


Radians and degrees are both units of measurements used to quantify the amount of rotation
between two lines, segment, plane or rays. For example, a minute hand originally pointing at the
twelve o’clock mark will move or rotate after 5 minutes to the one o’clock mark. The distance travelled
or more specifically, the angular distance travelled from twelve to one can be measured in degrees or
radians.
For degrees, a complete rotation is equivalent to 360° and for radians, it is 6.2832 rad or exactly 2𝜋

Figure 3-23: Degree-radian conversion chart - Source: https://commons.wikimedia.org/wiki/File:Degree-


Radian_Conversion.svg, Edited by: Christian Kevin Villanueva

The radian and the degree are the dominant units used to measure angles; thus, converting from
radian to degree or vice versa is a basic requirement in reading, understanding and interpreting
publications and plotting. An easy way to remember the conversion factor between degree and radian
is the angular distance of straight line. Considering the minute hand travelling from the 12 o’clock
mark to the 6 o’clock mark, an angular distance of 180° is completed which can also be interpreted as
half of a rotation. In radians, a half rotation is considered to be 3.1416 radian or π (pi) to be exact. If
half a rotation is equal to both 180° and π radian, then 1800 = 𝜋 𝑟𝑎𝑑. From this equivalence, degrees
to radian or radian to degrees can be converted. Refer to Table 3-4 below for the derivation.

𝜋 𝑟𝑎𝑑 = 180° Start here where the number of radians and degrees in half
revolution or a semicircle are equal.

𝜋 𝑟𝑎𝑑 180° Divide both sides of the equation by 𝜋


=
𝜋 𝜋
𝟏𝟖𝟎° Conversion factor of radian to degree
𝟏 𝒓𝒂𝒅 = = 𝟓𝟕. 𝟐𝟗𝟔°
𝝅

Confidential to United Marine Training Center, Inc. Page 3-71


Pre Sea Review Materials <course code>
Version Number
Deck
5 November 2018

180° = 𝜋 𝑟𝑎𝑑 Start here

180° 𝜋 𝑟𝑎𝑑 Divide both sides of the equation by 180


=
180 180
𝝅 𝒓𝒂𝒅 Conversion factor of degree to radian
𝟏° = = 𝟎. 𝟎𝟏𝟕𝟒𝟓
𝟏𝟖𝟎

Table 3-4: Derivation of the conversion factors radian to degree and vice –versa- Source: Ester Dinawanao

To mathematically express the conversion procedure,


𝝅 Converting from Degree to Radian
𝜽𝒓𝒂𝒅 = 𝜽𝒅𝒆𝒈 ∙ ( )
𝟏𝟖𝟎°
Where:
or
θrad = The angle in radian measure
𝜽𝒓𝒂𝒅 = 𝜽𝒅𝒆𝒈 ∙ (𝟎. 𝟎𝟏𝟕𝟒𝟓) θdeg = The given angle in degree measure
π = The constant pi which is approximately
equal to 3.1416
𝟏𝟖𝟎° Converting from Radian to Radian
𝜽𝒅𝒆𝒈 = 𝜽𝒓𝒂𝒅 ∙ ( )
𝝅
Where:
or θdeg = The angle in degree measure
𝜽𝒅𝒆𝒈 = 𝜽𝒓𝒂𝒅 ∙ (𝟓𝟕. 𝟐𝟗𝟔°) θrad = The given angle in radian measure
π = The constant pi which is approximately
equal to 3.1416

Example 3-71

Convert the following

a. 1° 𝑡𝑜 𝑟𝑎𝑑𝑖𝑎𝑛

𝜋 𝑟𝑎𝑑 Multiply to the conversion factor


1° ∙
180°

𝜋 𝑟𝑎𝑑 Perform cancelation of units


1° ∙
180°

𝟎. 𝟎𝟏𝟕𝟒𝟓 … 𝒓𝒂𝒅 Answer

b. 1 𝑟𝑎𝑑 𝑡𝑜 𝑑𝑒𝑔𝑟𝑒𝑒

180 ° Multiply to the conversion factor


1 𝑟𝑎𝑑 ∙
𝜋 𝑟𝑎𝑑
180 ° Perform cancelation of units
1 𝑟𝑎𝑑 ∙
𝜋 𝑟𝑎𝑑

𝟓𝟕. 𝟐𝟗𝟓𝟕 … ° Answer

Confidential to United Marine Training Center, Inc. Page 3-72


Pre Sea Review Materials <course code>
Version Number
Deck
5 November 2018

c. 𝜋
𝑟𝑎𝑑 𝑡𝑜 𝑑𝑒𝑔𝑟𝑒𝑒
4
𝜋 180 ° Multiply to the conversion factor
𝑟𝑎𝑑 ∙
4 𝜋 𝑟𝑎𝑑

𝜋 180 ° Perform cancelation of units and factors


𝑟𝑎𝑑 ∙
4 𝜋 𝑟𝑎𝑑

𝟒𝟓° Answer

d. 3𝜋
𝑟𝑎𝑑 𝑡𝑜 𝑑𝑒𝑔𝑟𝑒𝑒
7
3𝜋 180 ° Multiply to the conversion factor
𝑟𝑎𝑑 ∙
7 𝜋 𝑟𝑎𝑑
3𝜋 180 ° Perform cancelation of units and factors
𝑟𝑎𝑑 ∙
7 𝜋 𝑟𝑎𝑑

𝟕𝟕. 𝟏𝟒𝟐𝟖 … ° Answer

e. 60° 𝑡𝑜 𝑟𝑎𝑑𝑖𝑎𝑛

𝜋 𝑟𝑎𝑑 Multiply to the conversion factor


60° ∙
180°

𝜋 𝑟𝑎𝑑 Perform cancelation of units


60° ∙
180°
𝝅 Answer in fraction
𝒓𝒂𝒅
𝟑

𝟏. 𝟎𝟒𝟕𝟏𝟗 … 𝒓𝒂𝒅 Answer in decimal

f. 5𝜋
𝑟𝑎𝑑 𝑡𝑜 𝑑𝑒𝑔𝑟𝑒𝑒
4
5𝜋 180 ° Multiply to the conversion factor
𝑟𝑎𝑑 ∙
4 𝜋 𝑟𝑎𝑑

5π 180 ° Perform cancelation of units and factors


𝑟𝑎𝑑 ∙
4 𝜋 𝑟𝑎𝑑

𝟐𝟐𝟓° Answer

g. 270° 𝑡𝑜 𝑟𝑎𝑑𝑖𝑎𝑛

𝜋 𝑟𝑎𝑑 Multiply to the conversion factor


270° ∙
180°

Confidential to United Marine Training Center, Inc. Page 3-73


Pre Sea Review Materials <course code>
Version Number
Deck
5 November 2018

𝜋 𝑟𝑎𝑑 Perform cancelation of units


270° ∙
180°

𝟒. 𝟕𝟏𝟐 … 𝒓𝒂𝒅 Answer

Example 3-72

What is the radian measure of a central angle ∅ opposite an arc of 24 meters in a circle of radius
6 meters?
Solution
𝒔 Formula based in the definition of a central angle
∅=
𝒓

Substitute the given into the formula

24 𝑚 Simplify.
∅=
6𝑚

∅=𝟒 The central angle measures 4 radians or 4 rad.


An angular measurement with no unit is understood to be in radian.

Table 3-5 summarizes the conversions between the revolution system, sexagesimal system and
circular system methods of measuring angles for some of the most common angles

Revolution System Sexagesimal System Circular System


(𝑅𝑒𝑣𝑜𝑙𝑢𝑡𝑖𝑜𝑛, 𝑟𝑒𝑣) (𝐷𝑒𝑔𝑟𝑒𝑒, °) (𝑅𝑎𝑑𝑖𝑎𝑛, 𝑟𝑎𝑑)
1 𝜋
𝑟𝑒𝑣 1° 𝑟𝑎𝑑 𝑜𝑟 0.01745 …
360 180
1 𝜋
𝑟𝑒𝑣 90° 𝑟𝑎𝑑 𝑜𝑟 1.5707 …
4 2
1
𝑟𝑒𝑣 180° 𝜋 𝑟𝑎𝑑 𝑜𝑟 3.1416 …
2
3 3𝜋
𝑟𝑒𝑣 270° 𝑟𝑎𝑑 𝑜𝑟 4.7123 …
4 2
1 𝑟𝑒𝑣 360° 2𝜋 𝑟𝑎𝑑 𝑜𝑟 6.2831 …

Table 3-5: Conversion table of common angles – Source: Ester Dinawanao

Confidential to United Marine Training Center, Inc. Page 3-74


Pre Sea Review Materials <course code>
Version Number
Deck
5 November 2018

Supplementary Exercises
Solve the following problems
Answer
1] Convert the following to degrees.
−11𝜋
a. 3
a. −660°
9𝜋 b. 720°
b. 2 c. 810°
c. 4𝜋 d. 45°
1
d. 8
𝑟𝑒𝑣
2] 7𝜋
Convert the following to radian. a. − 6
a. −210° 4𝜋
b. 5
b. 505°
101𝜋
2 c. 𝑜𝑟 8.814
c. 5
𝑟𝑒𝑣 36

d. 2 𝑟𝑒𝑣 d. 20.106
3] A circle has a radius of 4 cm. Find the length of the arc 6.76 𝑐𝑚
intercepted by a central angle of 240°.
4] Determine the diameter and circumference of a circle if an arc 𝑑 = 10.44 𝑐𝑚
of length 4.75 cm subtends an angle of 0.91 radians. 𝐶 = 32.80 𝑐𝑚
5] A pendulum of length 1.2 m swings through an angle of 12° in
a single swing. Calculate the length of arc traced by the 0.25 𝑚
pendulum bob.

Confidential to United Marine Training Center, Inc. Page 3-75


Pre Sea Review Materials <course code>
Version Number
Deck
5 November 2018

Plane Triangle Trigonometry

On the following discussion about plane triangles, although there are many types of plane triangles
categorized by their angles or sides, the two types of triangle to be given emphasis are only the right
plane triangles and the oblique plane triangles

Right Triangle
A right-angled triangle or simply right triangle is a triangle in which one internal angle is a right
angle (90°).
In Figure 3-24, ∠𝑪 is the right angle denoted by a small square. The side opposite the right angle is
called the hypotenuse which is the longest side. The perpendicular sides 𝐴𝐶 ̅̅̅̅ and 𝐵𝐶
̅̅̅̅ are called legs
of a right triangle. The angles of a triangle are denoted by capital letters such 𝐴, 𝐵, and 𝐶 while the
sides opposite the corresponding angles are represented by small letters such as 𝑎, 𝑏, and 𝑐. The
relation between the sides and angles of a right triangle is the basis for trigonometry.

Leg, 𝑎

𝑨 𝑪
Leg, 𝑏

Figure 3-24: A right triangle – Source: Christian Kevin Villanueva

Pythagorean Theorem
One of the best known mathematical formulas is the Pythagorean Theorem, which provides the
relationship between the sides in a right triangle. It is named after Pythagoras of Samos (c.570 – c.
495 BC), a Greek philosopher and mathematician.
In a right angled triangle, the square of the hypotenuse is equal to the sum of the squares of the other
two sides. Using Figure 3-24, if 𝑐 is hypotenuse and 𝑎, 𝑏 are the legs of the 𝛥𝐴𝐵𝐶, then 𝑐 2 = 𝑎2 + 𝑏 2.
This is known as the Pythagorean Theorem.

Confidential to United Marine Training Center, Inc. Page 3-76


Pre Sea Review Materials <course code>
Version Number
Deck
5 November 2018

Example 3-73

Solve for the missing side of the triangle below.

12

Figure 3-25: Reference figure for Example 3-73 – Source: Christian Kevin Villanueva

Solution

𝒄𝟐 = 𝒂𝟐 + 𝒃𝟐 Pythagorean theorem

𝑐 2 = 122 + 52 Substitute the given values


Let:
𝑎 = 12; 𝑏=5

𝑐 2 = 144 + 25 Simplify

𝑐 2 = 169 Addition

√𝑐 2 = √169 Obtain the square root of both side to eliminate the exponent
of 𝑐

𝒄 = 𝟏𝟑 Answer

Example 3-74

Prove that the lengths 8, 15 and 17 are lengths of the sides of a right triangle.
Solution

𝒄𝟐 = 𝒂𝟐 + 𝒃𝟐 Use the Pythagorean theorem to prove the problem. Equation


must be satisfied to conclude that the given sides are sides of
a right triangle

172 = 82 + 152 Substitute the given values


Let:
𝑐 = 17 since the longest side must be the hypotenuse
𝑎 = 8; 𝑏 = 15 (the other two sides are the legs)
Simplify each sides separately.

289 = 64 + 225 Simplify

𝟐𝟖𝟗 = 𝟐𝟖𝟗 ✔ Answer


The Pythagorean Theorem is satisfied; thus, the three sides
are sides of a right triangle

Confidential to United Marine Training Center, Inc. Page 3-77


Pre Sea Review Materials <course code>
Version Number
Deck
5 November 2018

Example 3-75

Solve the missing side of the triangle of Figure 3-26

Figure 3-26: Reference figure for Example 3-75 – Source: Christian Kevin Villanueva

Solution

𝒄𝟐 = 𝒂𝟐 + 𝒃𝟐 Pythagorean theorem

152 = 92 + 𝑏 2 Substitute the given values


Let:
𝑐 = 15 since the longest side is opposite the right angle
𝑎=9

225 = 81 + 𝑏 2 Simplify

225 − 81 = 81 − 81 + 𝑏 2 Subtract both sides by 81

144 = 𝑏 2 Simplify

√144 = √𝑏 2 Obtain the square root of both side

12 = 𝑏

𝒃 = 𝟏𝟐 Answer
The Pythagorean Theorem is satisfied; thus, the three sides
are sides of a right triangle

Confidential to United Marine Training Center, Inc. Page 3-78


Pre Sea Review Materials <course code>
Version Number
Deck
5 November 2018

Example 3-76

Find the value of 𝑥 in the Figure 3-27.

𝑥 2

Figure 3-27: Reference figure for Example 3-76 – Source: Christian Kevin Villanueva

Solution
Prior solving for 𝑥, name the side common to both triangles as 𝑦

5
𝑦

𝑥 2

72 = 22 + 𝑦 2 Use Pythagorean theorem to solve for 𝑦

49 = 4 + 𝑦 2 Take the squares

49 − 4 = 𝑦 2 Transpose 4 to the left side making its sign (−) and simplify.
45 = 𝑦 2 Note that 𝑦 is the hypotenuse of the triangle with legs of 5 and 𝑥.

52 + 𝑥 2 = 𝑦 2 Apply Pythagorean Theorem and plug in the value of 𝑦 2 .


52 + 𝑥 2 = 45

𝑥 2 = 45 − 25 Isolate 𝑥 2 by transposing 52 𝑜𝑟 25 to the other side.


𝑥 2 = 20

√𝑥 2 = √20 Simplify by extracting the square roots on both sides of the


equation.
𝒙 = 𝟐√𝟓
Note: √20 = √4 ∙ 5 = √4 ∙ √5 = 2√5
𝒙 = 𝟒. 𝟒𝟕

Confidential to United Marine Training Center, Inc. Page 3-79


Pre Sea Review Materials <course code>
Version Number
Deck
5 November 2018

Special Right Triangles


Figure 3-28 shows two types of special right triangles. A right triangle with acute angles of
45° and 45° and a right triangle with acute angles of 30°and 60°

30°

√3 · 𝑥

𝑥 45°

45° 60°

𝑥 𝑥

(a) (b)

Figure 3-28: (a) A 45°- 45°- 90° special triangle and (b) a 30°- 60° -90° special triangle – Source: Christian Kevin Villanueva

In special right triangles, the hypotenuse or the legs of the right triangle can be determined easily
even without using the Pythagorean theorem and trigonometric functions.

Isosceles Right Triangle


 If 𝑥 represents the leg as shown in Figure 3-28(a), the hypotenuse is √2 times the leg or
√2 ∙ 𝑥.

 Similarly, the leg is the hypotenuse divided by the √2. In symbol, 𝑥 = where ℎ is the
√2
hypothenuse and 𝑥 is the leg.
 For instance, if 𝑥 is 10 in Figure 3-28(a), then the hypotenuse is 10√2.

Right Scalene Triangle


 The side opposite the 30° is called the shorter leg (𝑥) while the side opposite the 60° is
called the longer leg.
 Longer leg is √3 times the shorter leg. In symbol, 𝐿𝑜𝑛𝑔𝑒𝑟 𝑙𝑒𝑔 = √3 ∙ 𝑥.
 Hypotenuse is twice the shorter leg. In symbol, ℎ𝑦𝑝𝑜𝑡𝑒𝑛𝑢𝑠𝑒 = 2𝑥.
2 1
 Shorter leg (𝑥) is half the hypotenuse ℎ = 𝑥 . In symbol, 𝑥 = 2 ∙ ℎ𝑦𝑝𝑜𝑡𝑒𝑛𝑢𝑠𝑒.

Confidential to United Marine Training Center, Inc. Page 3-80


Pre Sea Review Materials <course code>
Version Number
Deck
5 November 2018

Example 3-77

In Figure 3-29, determine the value of 𝑥 and 𝑦.


𝑥

15

Figure 3-29: Reference figure for Example 3-77 – Source: Christian Kevin Villanueva

Solution

𝐿𝑜𝑛𝑔𝑒𝑟 𝑙𝑒𝑔 = √3 ∙ 𝑥 Let 𝑥 be the shorter leg and 𝑦 be the hypotenuse

15 = 𝑥√3 Given:
𝐿𝑜𝑛𝑔𝑒𝑟 𝑙𝑒𝑔 = 15
𝑥√3 = 15
Apply the special relationship between longer leg and shorter
leg, “longer leg is √3 times the shorter leg”
By symmetric property

𝑥√3 = 15 Divide both sides by √3.


𝑥√3 15
=
√3 √3

15 √3 Rationalize the denominator by multiplying the numerator and


𝑥= ∙ denominator by √3.
√3 √3
Note: √3 ∙ √3 = √9 = 3

15√3 Simplify
𝑥=
3
𝒙 = 𝟓√𝟑
𝒙 = 𝟖. 𝟔𝟔

𝑦 = 2𝑥 To solve for 𝑦, apply the special relationship between the


hypotenuse (𝑦) and the shorter leg (𝑥), “hypotenuse is 2 times
𝑦 = 2(5√3) the shorter leg"

𝒚 = 𝟏𝟎√𝟑 Simplify
𝒚 = 𝟏𝟕. 𝟑𝟐𝟏

Confidential to United Marine Training Center, Inc. Page 3-81


Pre Sea Review Materials <course code>
Version Number
Deck
5 November 2018

Example 3-78

The area of 300 − 600 − 900 triangle is 150 m2. Find the lengths of the 3 sides.
Solution
Draw the figure and label according to the features of a 300 − 600 − 900 triangle,
where 𝑥 is the shorter leg (opposite 30°).

𝑥 60°

30°

√3 · 𝑥

𝟏 Formula for Area of Triangle


𝑨= 𝒃𝒉
𝟐
Where:
A = Area of the triangle
b = Base of the triangle
h = Height of the triangle
1 Let:
150 = (√3 · 𝑥) (𝑥)
2 𝐴 = 150 m2
𝑏 = √3 · 𝑥 (longer leg)
ℎ = 𝑥 (shorter leg)

300 = 𝑥 2 (√3) Cross multiply

300 𝑥 2 (√3) Divide both sides by √3


=
√3 √3

173.205 = 𝑥 2 Extract the square roots on both sides.


13.16 = 𝑥 Take the positive root because 𝑥 represents the
length of the shorter leg.

𝒙 = 𝟏𝟑. 𝟏𝟔 𝒎 Shorter Leg

𝑥√3 = 13.16√3 Plug in the value of 𝑥 to determine the longer leg 𝑥√3

𝒙√𝟑 = 𝟐𝟐. 𝟕𝟗 Longer Leg

2𝑥 = 2(13.16) Plug in the value of 𝑥 to determine the


hypotenuse 2𝑥.
𝟐𝒙 = 𝟐𝟔. 𝟑𝟐

Confidential to United Marine Training Center, Inc. Page 3-82


Pre Sea Review Materials <course code>
Version Number
Deck
5 November 2018

Supplementary Exercises
Solve the following problems
Answer
1] Solve the right triangle ACB, where 𝐶 = 90°, 𝑎 = 21 𝑐𝑚 , and 𝑐 = 75 𝑐𝑚
𝑏 = 72 𝑐𝑚. 𝐴 = 16.26°
𝐵 = 3.74°
2] What is the height (ℎ) of the triangle? All measurements are in
cm.


ℎ = 5√2 𝑐𝑚 𝑜𝑟 7.07𝑐𝑚

2 5

3] Find the side of an equilateral triangle whose altitude is 14 cm. ℎ = 16.17 𝑐𝑚


4] Find the value of 𝑥.

45° 30° 𝑥 = 8.66

60°

5] Find the area of a square whose diagonal is 40 meters. 𝐴 = 1 800 𝑚2


6] In a 45° − 45° − 90° right triangle, each equal legs measure
15√2 𝑐𝑚 𝑜𝑟 21.21 𝑐𝑚
15 cm, how long is the hypotenuse.
7] Find the value of 𝑥 in fraction form.

32
𝑥=
9

Confidential to United Marine Training Center, Inc. Page 3-83


Pre Sea Review Materials <course code>
Version Number
Deck
5 November 2018

8] Find the values of 𝑥 and 𝑦.

𝑥 𝑥 = 10
20√3
45° 30° 𝑦= 𝑜𝑟 𝑦 = 11.55
3
60°

9] Find 𝑚 on the figure below.

𝑚 = 5√2 𝑜𝑟 𝑚 = 7.07

30° 45°

10] A six-meter-long ladder leans against a building. If the ladder


makes an angle of 60° with the ground, how far up the wall
5.20 𝑚
does the ladder reach? How far from the wall is the base of the
ladder?

Confidential to United Marine Training Center, Inc. Page 3-84


Pre Sea Review Materials <course code>
Version Number
Deck
5 November 2018

Trigonometric Functions

Leg, 𝑎

𝑨 Leg, 𝑏 𝑪

Figure 3-30: Right triangle with labelled angles – Source: Christian Kevin Villanueva

Figure 3-30 shows a right triangle 𝛥𝐴𝐵𝐶 with right angle at 𝐶. The acute angles are at 𝐴 and 𝐵 or
using Greek letters, the acute angles are represented 𝛼 and 𝛽 respectively. The sides are
represented by small letters 𝑎, 𝑏 and 𝑐.

Relative to acute angle 𝐴 or 𝛼 , the opposite side is 𝑎 and the adjacent side is 𝑐.
Relative to acute angle 𝐵 or 𝛽 , the opposite side is 𝑏 and the adjacent side is 𝑎.
For any right triangle, there are six possible ratios that can be formed from the lengths of the sides
𝑎 𝑏 𝑎 𝑐 𝑐 𝑏
𝑎, 𝑏, and 𝑐, namely; 𝑐 , 𝑐 , 𝑏 , 𝑎 , 𝑏 and 𝑎. These ratios are called trigonometric functions since they
depend on the acute angle of a right triangle. They are very important in technical subjects like
science, engineering, architecture, and navigation.
Trigonometric Functions (also known as trigonometric ratios) relate the angles in a right triangle to the
ratios of the sides. Table 3-6 shows the names, abbreviations and mathematical expressions of the
six basic trigonometric functions
Function Name Abbreviation Mathematical Expression Verbal Expression
for the Trigonometric
Function of an Angle
Sine sin sin 𝛼 Sine of angle alpha (α)
Cosine cos cos 𝛼 Cosine of angle alpha (α)
Tangent tan tan 𝛼 Tangent of angle alpha (α)
Cosecant csc csc 𝛼 Cosecant of angle alpha (α)
Secant sec sec 𝛼 Secant of angle alpha (α)
Cotangent cot cot 𝛼 Cotangent of angle alpha (α)
Table 3-6: Table of common trigonometric functions – Source: Ester Dinawanao

Confidential to United Marine Training Center, Inc. Page 3-85


Pre Sea Review Materials <course code>
Version Number
Deck
5 November 2018

Considering angle alpha (𝛼) of Figure 3-31,


𝑩

Leg, 𝑎
Opposite

𝑨 Leg, 𝑏 𝑪
Adjacent

Figure 3-31: Right triangle with reference angle alpha (𝛼) – Source: Christian Kevin Villanueva

The definition of the three basic trigonometric functions are listed in Table 3-7
General Definition of the Trigonometric Definition of the Trigonometric functions
Function of angle 𝜶 of the Triangle from Figure 3-31
𝑜𝑝𝑝𝑜𝑠𝑖𝑡𝑒 𝑎
sin 𝛼 = sin 𝛼 =
ℎ𝑦𝑝𝑜𝑡𝑒𝑛𝑢𝑠𝑒 𝑐
𝑎𝑑𝑗𝑎𝑐𝑒𝑛𝑡 𝑏
cos 𝛼 = cos 𝛼 =
ℎ𝑦𝑝𝑜𝑡𝑒𝑛𝑢𝑠𝑒 𝑐
𝑜𝑝𝑝𝑜𝑠𝑖𝑡𝑒 𝑎
tan 𝛼 = tan 𝛼 =
𝑎𝑑𝑗𝑎𝑐𝑒𝑛𝑡 𝑏
Table 3-7: The definition of the three basic trigonometric function – Source: Ester Dinawanao

The pneumonic SOH CAH TOA can be used to easily remember these definitions.

Reciprocal Functions
Often it is useful to use the reciprocal ratios, depending on the problem. (The reciprocal of a fraction is
determined by replacing the numerator with the denominator and replacing the denominator with the
numerator). Table 3-8 shows the reciprocal functions of the three basic trigonometric function of angle
𝛼.
Functions Reciprocal (Abbreviation) Reciprocal Definition of the
Functions Identities Reciprocal Function

1 ℎ𝑦𝑝𝑜𝑡𝑒𝑛𝑢𝑠𝑒
Sine Cosecant (csc) csc 𝛼 = csc 𝛼 =
sin 𝛼 𝑜𝑝𝑝𝑜𝑠𝑖𝑡𝑒
1 ℎ𝑦𝑝𝑜𝑡𝑒𝑛𝑢𝑠𝑒
Cosine Secant (sec) sec 𝛼 = sec 𝛼 =
cos 𝛼 𝑎𝑑𝑗𝑎𝑐𝑒𝑛𝑡
1 𝑎𝑑𝑗𝑎𝑐𝑒𝑛𝑡
Tangent Cotangent (cot) cot 𝛼 = cot 𝛼 =
cot 𝛼 𝑜𝑝𝑝𝑜𝑠𝑖𝑡𝑒
Table 3-8: The definition of the reciprocal of the three basic trigonometric function – Source: Ester Dinawanao

The pneumonic CHO SHA CAO can be used to easily remember these definitions.

Confidential to United Marine Training Center, Inc. Page 3-86


Pre Sea Review Materials <course code>
Version Number
Deck
5 November 2018

Note: The expressions 𝑐𝑠𝑐 𝛼 and 𝑠𝑖𝑛−1 𝛼 are not the same. The cosecant of 𝛼 is the one divided by the
1
value of 𝑠𝑖𝑛 𝛼 which is mathematically written as 𝑐𝑠𝑐 𝛼 = 𝑠𝑖𝑛 𝛼. On the other hand, the inverse sine or
arcsine of 𝛼, written as 𝑠𝑖𝑛−1 𝛼, obtains the value of an angle. Don’t use your 𝑠𝑖𝑛−1 button to find csc θ
when using a calculator
.

Example 3-79

13
If 𝐴 is an acute angle of a right triangle 𝛥𝐴𝐵𝐶 and sec 𝐴 = , find the other five (5)
5
trigonometric functions of angle 𝐴.

Solution
Define the secant function of angle 𝐴, solve for all the sides and sketch the right
triangle

ℎ𝑦𝑝𝑜𝑡𝑒𝑛𝑢𝑠𝑒 Definition of secant of an angle. In the pneumonic


sec 𝐴 = CHO SHA CAO, SHA stands for secant of 𝐴 is the
𝑎𝑑𝑗𝑎𝑐𝑒𝑛𝑡
hypotenuse divided by the adjacent side.

13 Compare the given ratio and extract the given


sec 𝐴 = sides
5

𝒉𝒚𝒑𝒐𝒕𝒆𝒏𝒖𝒔𝒆 = 𝒄 = 𝟏𝟑 The hypotenuse and adjacent side


𝒂𝒅𝒋𝒂𝒄𝒆𝒏𝒕 𝒔𝒊𝒅𝒆 = 𝒃 = 𝟓

𝑐 2 = 𝑎2 + 𝑏 2 Use the Pythagorean Theorem to solve for the


opposite side. Let:
𝑐 = 13 (hypotenuse)
𝑏 = 5 (leg 𝑏, adjacent side of angle 𝐴)

132 = 𝑎2 + 52 Substitute the given values

169 = 𝑎2 + 25 Simplify

169 − 25 = 25 − 25 + 𝑎2 Subtract both sides by 15

144 = 𝑎2

√𝑎2 = √144 Rearrange and extract the square root of both


sides
𝒂 = 𝟏𝟐
Let a, the opposite side of angle 𝐴
𝒐𝒑𝒑𝒐𝒔𝒊𝒕𝒆 𝒔𝒊𝒅𝒆 = 𝟏𝟐

Confidential to United Marine Training Center, Inc. Page 3-87


Pre Sea Review Materials <course code>
Version Number
Deck
5 November 2018

Hypotenuse
𝑐 = 13 Opposite Side
𝑎 = 12

𝑨 𝑪

Adjacent Side
𝑏=5

Use the triangle to define the other five trigonometric functions

𝟏𝟐 𝑜𝑝𝑝𝑜𝑠𝑖𝑡𝑒
𝒔𝒊𝒏 𝑨 = sin 𝐴 =
𝟏𝟑 ℎ𝑦𝑝𝑜𝑡𝑒𝑛𝑢𝑠𝑒

𝟓 𝑎𝑑𝑗𝑎𝑐𝑒𝑛𝑡
𝒄𝒐𝒔 𝑨 = cos 𝐴 =
𝟏𝟑 ℎ𝑦𝑝𝑜𝑡𝑒𝑛𝑢𝑠𝑒

𝟏𝟐 𝑜𝑝𝑝𝑜𝑠𝑖𝑡𝑒
𝒕𝒂𝒏 𝑨 = tan 𝐴 =
𝟓 𝑎𝑑𝑗𝑎𝑐𝑒𝑛𝑡

𝟏𝟑 ℎ𝑦𝑝𝑜𝑡𝑒𝑛𝑢𝑠𝑒
𝒄𝒔𝒄 𝑨 = csc 𝐴 =
𝟏𝟐 𝑜𝑝𝑝𝑜𝑠𝑖𝑡𝑒

𝟓 𝑎𝑑𝑗𝑎𝑐𝑒𝑛𝑡
𝒄𝒐𝒕 𝑨 = cot 𝐴 =
𝟏𝟐 𝑜𝑝𝑝𝑜𝑠𝑖𝑡𝑒

Example 3-80

Find the values of the six trigonometric functions for angle 𝜃 in standard position if a point with
coordinates (−6, 8) lies on its terminal side.
Solution
Sketch the graph of the given angle and coordinates. Recall the definition of angle
in standard position. Label the missing parts
 Let 𝛽 be the reference angle of 𝜃. Note that The trigonometric functions of θ
are the same as β except the signs (possible to have a negative signs).
 Let 𝑟 be the distance from the origin which is also the hypotenuse of the
triangle formed

Confidential to United Marine Training Center, Inc. Page 3-88


Pre Sea Review Materials <course code>
Version Number
Deck
5 November 2018

𝑦 − 𝑎𝑥𝑖𝑠 𝑦 − 𝑎𝑥𝑖𝑠
(−6, 8) (−6, 8)

𝜃 𝑦=8 𝜃
𝛽

(0, 0) 𝑥 − 𝑎𝑥𝑖𝑠 (0, 0) 𝑥 − 𝑎𝑥𝑖𝑠

𝑥 = −6

𝑟2 = 𝑥2 + 𝑦 2 Use the Pythagorean theorem to solve for 𝑟

𝑟 2 = (−6)2 + 82 Substitute the given values


Since the terminal side of the angle lies on the second quadrant,
𝑥 must be negative

𝒓 = 𝟏𝟎 Simplify
𝒉𝒚𝒑𝒐𝒕𝒆𝒏𝒖𝒔𝒆 = 𝟏𝟎 𝑟 is the hypotenuse

𝒐𝒑𝒑𝒐𝒔𝒊𝒕𝒆 = 𝟖 Using the sketch, identify the parts of the adjacent side and opposite
side relative to angle 𝛽
𝑨𝒅𝒋𝒂𝒄𝒆𝒏𝒕 = −𝟔

Use the triangle to define the other six trigonometric functions

8 4 𝑦 𝑜𝑝𝑝𝑜𝑠𝑖𝑡𝑒
𝑠𝑖𝑛 𝜃 = ; 𝑠𝑖𝑛 𝜃 = sin 𝛽 = =
10 5 𝑟 ℎ𝑦𝑝𝑜𝑡𝑒𝑛𝑢𝑠𝑒

−6 −3 𝑥 𝑎𝑑𝑗𝑎𝑐𝑒𝑛𝑡
𝑐𝑜𝑠 𝜃 = ; 𝑐𝑜𝑠 𝜃 = ; cos 𝛽 = =
10 5 𝑟 ℎ𝑦𝑝𝑜𝑡𝑒𝑛𝑢𝑠𝑒

8 4 𝑦 𝑜𝑝𝑝𝑜𝑠𝑖𝑡𝑒
𝑡𝑎𝑛 𝜃 = ; 𝑡𝑎𝑛 𝜃 = tan 𝛽 = =
−6 −3 𝑥 𝑎𝑑𝑗𝑎𝑐𝑒𝑛𝑡

10 5 𝑟 ℎ𝑦𝑝𝑜𝑡𝑒𝑛𝑢𝑠𝑒
𝑐𝑠𝑐 𝜃 = ; 𝑐𝑠𝑐 𝜃 = csc 𝛽 = =
8 4 𝑦 𝑜𝑝𝑝𝑜𝑠𝑖𝑡𝑒

10 5 𝑟 ℎ𝑦𝑝𝑜𝑡𝑒𝑛𝑢𝑠𝑒
𝑠𝑒𝑐 𝜃 = ; 𝑠𝑒𝑐 𝜃 = sec 𝛽 = =
−6 −3 𝑥 𝑎𝑑𝑗𝑎𝑐𝑒𝑛𝑡

−6 −3 𝑥 𝑎𝑑𝑗𝑎𝑐𝑒𝑛𝑡
𝑐𝑜𝑡 𝜃 = ; 𝑐𝑜𝑡 𝜃 = cot 𝛽 = =
8 4 𝑟 𝑜𝑝𝑝𝑜𝑠𝑖𝑡𝑒

Confidential to United Marine Training Center, Inc. Page 3-89


Pre Sea Review Materials <course code>
Version Number
Deck
5 November 2018

Example 3-81

Find the value of 𝜃 in Figure 3-32

6 𝑐𝑚

10 𝑐𝑚

Figure 3-32: Reference figure for Example 3-81 – Source: Christian Kevin Villanueva

Solution

6 𝑐𝑚 Relative to 𝜃, the known sides are the opposite side


𝑡𝑎𝑛 𝜃 = (6 𝑐𝑚) and the adjacent side (10 𝑐𝑚).Thus, tan 𝜃 is the
10 𝑐𝑚
appropriate function to be used.

3 Reduce the fraction to lowest term or get its decimal


𝑡𝑎𝑛 𝜃 = equivalent by dividing the numerator by the denominator.
5
𝑡𝑎𝑛 𝜃 = 0.6 The unit , cm , is cancelled .

𝑡𝑎𝑛−1(𝑡𝑎𝑛 𝜃) = 𝑡𝑎𝑛−1(0.6) Find the value of 𝜃 using inverse tangent function.


𝜃 = 𝑡𝑎𝑛−1(0.6)

𝜽 = 𝟑𝟎. 𝟗𝟔𝟒𝟎 Use a scientific calculator to solve for the inverse tangent
of 0.6

Example 3-82

Find the value of 𝑎 in Figure 3-33.

𝑎 𝑐𝑚

45°30′

Figure 3-33: Reference figure for Example 3-82 – Source: Christian Kevin Villanueva

Solution

𝑜𝑝𝑝𝑜𝑠𝑖𝑡𝑒 Let 𝜃 = 450 30′


𝑠𝑖𝑛𝜃 =
ℎ𝑦𝑝𝑜𝑡𝑒𝑛𝑢𝑠𝑒 Relative to the angle given, the given sides are
the hypotenuse and the opposite side a
𝑎 Substitute the given into the equation
𝑠𝑖𝑛 450 30′ =
7

Confidential to United Marine Training Center, Inc. Page 3-90


Pre Sea Review Materials <course code>
Version Number
Deck
5 November 2018

ℎ𝑦𝑝𝑜𝑡𝑒𝑛𝑢𝑠𝑒 = 7𝑐𝑚

𝑎 = 7 𝑠𝑖𝑛 450 30′ Cross multiply to find side 𝑎.

𝒂 = 𝟒. 𝟗𝟗 𝒄𝒎 Use scientific calculator to get the answer.

Cofunctions
Consider the triangle of Figure 3-34

𝜃
𝑎

Figure 3-34: Reference right triangle for cofunctions – Source: Christian Kevin Villanueva

Using the rations of the sides, the six trigonometric functions of both angles 𝛼 and 𝜃 are given by
Table 3-9

Angle alpha Angle theta


𝜶 𝜽
𝑎 𝑏
sin 𝛼 = sin 𝜃 =
𝑐 𝑐
𝑏 𝑎
cos 𝛼 = cos 𝜃 =
𝑐 𝑐
𝑎 𝑏
tan 𝛼 = tan 𝜃 =
𝑏 𝑎
𝑐 𝑐
csc 𝛼 = csc 𝜃 =
𝑎 𝑏
𝑐 𝑐
sec 𝛼 = sec 𝜃 =
𝑏 𝑎
𝑏 𝑎
cot 𝛼 = cot 𝜃 =
𝑎 𝑏

Table 3-9: Trigonometric functions of 𝛼 and 𝜃 of triangle of Figure 3-34 – Source: Ester Dinawanao

Confidential to United Marine Training Center, Inc. Page 3-91


Pre Sea Review Materials <course code>
Version Number
Deck
5 November 2018

It is observed that there are trigonometric functions of 𝛼 and 𝜃 that are equal such as sin 𝛼 and cos 𝜃
𝑎
which is equal to 𝑐 . The other pairs are given by Table 3-10

Ratio of Angle alpha Angle theta


sides 𝜶 𝜽
𝑎
sin 𝛼 cos 𝜃
𝑐
𝑏
cos 𝛼 sin 𝜃
𝑐
𝑎
tan 𝛼 cot 𝜃
𝑏
𝑐
csc 𝛼 sec 𝜃
𝑎
𝑐
sec 𝛼 csc 𝜃
𝑏
𝑏
cot α tan 𝜃
𝑎
Table 3-10: Cofunction pair – Source: Ester Dinawanao

The functions in each pair of Table 3-10 are referred to as cofunctions. Notice the prefix 𝑐𝑜 in cosine,
cotangent and cosecant functions. These means that cosine is the cofunctions of sine and these
cofunctions are based on the relationship of complementary angles. Recall that, if the sum of two
positive angles is 900 , then the angles are said to be complementary.
In the right triangle of Figure 3-34, 𝜃 and 𝛼 are complementary since 𝛼 + 𝜃 = 90°.

𝜃 = 90° − 𝛼
𝛼 = 90° − 𝜃

Co-function Theorem
Any trigonometric function of an angle is equal to the co-function of the complement of the angle.
Sine and cosine are cofunctions which means that the sine of an angle is the same as the cosine of
the complement of that angle.
If 𝛼 + 𝜃 = 900 , then the following relationship in Table 2-10 follows

Cofunction Identities
sin 𝜃 = cos(90° − θ) = cos 𝛼
cos 𝜃 = sin(90° − θ) = sin 𝛼
tan 𝜃 = cot(90° − θ) = cot 𝛼
csc 𝜃 = sec(90° − θ) = sec 𝛼
sec 𝜃 = csc(90° − θ) = csc 𝛼
cot 𝜃 = tan(90° − θ) = tan 𝛼

Table 3-11: Cofunction Identities – Source: Ester Dinawanao

Confidential to United Marine Training Center, Inc. Page 3-92


Pre Sea Review Materials <course code>
Version Number
Deck
5 November 2018

Example 3-83

Express each of the following quantities as a function of the complementary angle.


a. sin 38°
b. cot 65° 17′
c. cos 45° 45′
d. csc 78° 4′ 20"
Solution

a. sin 38° Apply the cofunction theorem


sin 38° = cos(90° − 38°) 𝑠𝑖𝑛𝜃 = 𝑐𝑜𝑠(900 − 𝜃) = 𝑐𝑜𝑠𝛼
𝐜𝐨𝐬 𝟓𝟐°

b. cot 65° 17′ Apply the cofunction theorem


cot 65° 17′ = tan(90° − 65° 17′) 𝑐𝑜𝑡𝜃 = 𝑡𝑎𝑛(900 − 𝜃) = 𝑡𝑎𝑛𝛼
𝐭𝐚𝐧 𝟐𝟒° 𝟒𝟑′

c. cos 45° 45′ Apply the cofunction theorem


cos 45° 45′ = sin(90° − 45° 45′) 𝑐𝑜𝑠𝜃 = 𝑠𝑖𝑛(900 − 𝜃) = 𝑠𝑖𝑛𝛼
𝐬𝐢𝐧 𝟒𝟓° 𝟒𝟓′

d. csc 78° 4′ 20" Apply the cofunction theorem


sec 38° = sec(90° − 78° 4′ 20") 𝑐𝑠𝑐𝜃 = 𝑠𝑒𝑐(900 − 𝜃) = 𝑠𝑒𝑐𝛼
𝐬𝐞𝐜 𝟏𝟏°𝟓𝟓′𝟒𝟎"

Example 3-84

If 3𝑥 and 6𝑥 are acute angles and sin 3𝑥 = cos 6𝑥, find the value of 𝑥.

Solution

3𝑥 + 6𝑥 = 90° Since sin 3𝑥 = cos 6𝑥 , then by co-function


theorem, 𝑠𝑖𝑛𝜃 = cos(90° − 𝜃) where
𝜃 + (900 − 𝜃) = 900
6𝑥 is the complement of 3𝑥

9𝑥 = 90° Combine like terms

9𝑥 90° Divide both sides of the equation by 9


=
9 9

𝑥 = 10° The value of x is 100 .

𝑠𝑖𝑛 3𝑥 = 𝑐𝑜𝑠 6𝑥 Verify and check


𝑠𝑖𝑛 30° = 𝑐𝑜𝑠 60° 3𝑥 = 3(10°) = 30°
1 1 6𝑥 = 6(10°) = 60°
= ✔
2 2

Confidential to United Marine Training Center, Inc. Page 3-93


Pre Sea Review Materials <course code>
Version Number
Deck
5 November 2018

Supplementary Exercises
Solve the following problems
Answer
1] 24 7
If 𝜃 is an acute angle of a right triangle and cot 𝜃 = , find the tan 𝜃 =
7 24
other five (5) trigonometric functions of 𝜃. 24
cos 𝜃 =
25
25
csc 𝜃 =
7
25
sec 𝜃 =
24
7
sin 𝜃 =
25

2] −16
Find the values of the six trigonometric functions of angle, 𝜃 in sin 𝜃 =
standard position, if 𝑷(𝟏𝟐, −𝟏𝟔) lies on the terminal side of 𝜃 . 20
12
cos 𝜃 =
20
−16
tan 𝜃 =
12
12
cot 𝜃 =
−16
20
csc 𝜃 =
−16
20
sec 𝜃 =
12
3] Find the value of 𝑎 in the given diagram. 4.99 𝑐𝑚

𝑎 𝑐𝑚

45° 30′

4] Express each of the following quantities as a function of the a. cos 44° 21′
complementary angle. b. sin 71° 13′
c. cot 55° 7′ 32"
a. sin 45° 39′
d. csc 26°30′ 43"
b. cos 18° 47′
c. tan 34° 52′ 28"
d. sec 63° 29′ 17 "
5] Find x if the functions of positive acute angles are involved in a. 𝑥 = 10°
each of the following equations. b. 𝑥 = 9°
a. tan 5𝑥 = cot 4𝑥 c. 𝑥 = 4°
b. cos 6𝑥 = sin(2𝑥 + 18°)
c. 𝑐. sec(30° + 10𝑥) = csc 5𝑥

Confidential to United Marine Training Center, Inc. Page 3-94


Pre Sea Review Materials <course code>
Version Number
Deck
5 November 2018

Circular Functions
The circle whose radius is 1 unit and the center is the origin of a rectangular coordinate system is
called the unit circle, refer to Figure 3-35.

𝑦 − 𝑎𝑥𝑖𝑠
1
(𝑥, 𝑦)

−1 𝑥 1 𝑥 − 𝑎𝑥𝑖𝑠

−1

Figure 3-35: Unit Circle – Source: Christian Kevin Villanueva

If (𝑥, 𝑦) are the coordinates of a point on the circle, then a right triangle is formed with legs
represented by these coordinates and the length of the hypotenuse is 1 unit (radius of the unit circle).
By Pythagorean Theorem, 𝑥 2 + 𝑦 2 = 1 . This is the equation of the unit circle.
Let 𝜃 be an angle in standard position and (𝑥, 𝑦) is a point on the terminal side of 𝜃 that lies in the unit
circle (Refer to Figure 3-36). Using standard trigonometric ratios, the sine and cosine of 𝜃 as the
coordinates of a point is defined as
𝑦
sin 𝜃 = ; sin 𝜃 = 𝑦
1
𝑥
cos 𝜃 = ; cos 𝜃 = 𝑥
1

𝑦 − 𝑎𝑥𝑖𝑠
1
(𝑥, 𝑦)

𝑦
𝜃
𝑥 − 𝑎𝑥𝑖𝑠
−1 𝑥 1

−1

Figure 3-36: Unit circle with angle 𝜃 – Source: Christian Kevin Villanueva

Now it can be seen that the y-coordinate of the point (𝑥, 𝑦) is always equal to the sine of the angle,
and the x-coordinate of this point is always equal to the cosine of the angle. In symbol,

𝑃(𝑥, 𝑦) ≈ 𝑃(cos 𝜃 , sin 𝜃)

Confidential to United Marine Training Center, Inc. Page 3-95


Pre Sea Review Materials <course code>
Version Number
Deck
5 November 2018

Trigonometric functions are also called circular functions. This is because the two
fundamental trigonometric functions –the sine and the cosine – are defined as the coordinates of a
point 𝑃 travelling around on the unit circle of radius 1. The other circular functions (the tangent,
cotangent, secant, and cosecant) can be defined in terms of the sine and cosine.

𝐬𝐢𝐧 𝜽 = 𝒚 These are the basic identities defined in terms of the unit circle of Figure
3-36
𝐜𝐨𝐬 𝜽 = 𝒙

𝑦 Using the definition of the tangent and cotangent functions


tan 𝜃 =
𝑥
𝑥
cot 𝜃 =
𝑦

sin 𝜃 Substituting the basic identities in the previous equations


tan 𝜃 =
cos 𝜃 𝑦 = sin 𝜃
cos 𝜃
cot 𝜃 = 𝑥 = cos 𝜃
sin 𝜃

𝐬𝐢𝐧 𝜽 These are the ratio identities


𝐭𝐚𝐧 𝜽 =
𝐜𝐨𝐬 𝜽
𝐜𝐨𝐬 𝜽
𝐜𝐨𝐭 𝜽 =
𝐬𝐢𝐧 𝜽
1 Using the definition of the cosecant and secant functions
csc 𝜃 =
𝑦
1
sec 𝜃 =
𝑥
1 Substituting the basic identities in the previous equations
csc 𝜃 =
sin 𝜃 𝑦 = sin 𝜃
1
sec 𝜃 = 𝑥 = cos 𝜃
cos 𝜃
𝟏 These are the reciprocal identities
𝐜𝐬𝐜 𝜽 =
𝐬𝐢𝐧 𝜽
𝟏
𝐬𝐞𝐜 𝜽 =
𝐜𝐨𝐬 𝜽

Confidential to United Marine Training Center, Inc. Page 3-96


Pre Sea Review Materials <course code>
Version Number
Deck
5 November 2018

The Coordinates of the Points in the Unit Circle

𝑦 − 𝑎𝑥𝑖𝑠

1
sin 𝜃
𝑃(𝑥, 𝑦) = (cos 𝜃 , sin 𝜃) ; tan 𝜃 =
cos 𝜃

𝑥 1 𝑥 − 𝑎𝑥𝑖𝑠

Figure 3-37: The coordinates of the points in the unit circle – Source: Christian Kevin Villanueva

The coordinates of Point 𝑃 can be expressed in terms of the angle. As the angle 𝜃 rotates counter-
clockwise from the positive x-axis, the values of the 𝑥 and 𝑦 coordinates changes.

 Angle theta at 0, 𝜃 = 0°
𝑦 − 𝑎𝑥𝑖𝑠

1
(𝑥, 𝑦) = (cos 𝜃 , sin 𝜃)
(𝑥, 𝑦) = (cos 0° , sin 0°)
(𝑥, 𝑦) = (1, 0)

sin 𝜃 0
tan 𝜃 = ; tan 𝜃 =
cos 𝜃 , 1
𝜃 = 0° tan 𝜃 = 0 (No Slope)
1 𝑥 − 𝑎𝑥𝑖𝑠
𝑥

 Angle theta at 0, 𝜃 = 30°


𝑦 − 𝑎𝑥𝑖𝑠
(𝑥, 𝑦) = (cos 𝜃 , sin 𝜃)
1 (𝑥, 𝑦) = (cos 30° , sin 30°)
√3 1
(𝑥, 𝑦) = ( , ) 𝑜𝑟 (1.2247, 0.5)
2 2

sin 𝜃 1ൗ 1 2
tan 𝜃 = ; tan 𝜃 = 2 ; tan 𝜃 = ∙
𝑦 cos 𝜃 , √3ൗ 2 √3
2
𝜃 = 30° 1 1 √3 √3
tan 𝜃 = ; tan 𝜃 = ∙ ; tan 𝜃 =
1 𝑥 − 𝑎𝑥𝑖𝑠 √3 √3 √3 3
𝑥

Confidential to United Marine Training Center, Inc. Page 3-97


Pre Sea Review Materials <course code>
Version Number
Deck
5 November 2018

 Angle theta at 0, 𝜃 = 45°


𝑦 − 𝑎𝑥𝑖𝑠
(𝑥, 𝑦) = (cos 𝜃 , sin 𝜃)
1 (𝑥, 𝑦) = (cos 45° , sin 30°)
√2 √2
(𝑥, 𝑦) = ( , ) 𝑜𝑟 (0.7071, 0.7071)
2 2

sin 𝜃 √2ൗ
𝑦 tan 𝜃 = ; tan 𝜃 = 2
cos 𝜃 , √2ൗ
2
𝜃 = 45° tan 𝜃 = 1
1 𝑥 − 𝑎𝑥𝑖𝑠
𝑥

 Angle theta at 0, 𝜃 = 60°


𝑦 − 𝑎𝑥𝑖𝑠
(𝑥, 𝑦) = (cos 𝜃 , sin 𝜃)
1 (𝑥, 𝑦) = (cos 60° , sin 60°)
1 √3
(𝑥, 𝑦) = ( , ) 𝑜𝑟 (0.5, 1.2247)
2 2

sin 𝜃 √3ൗ
2 ; √3 2
𝑦 tan 𝜃 = ; tan 𝜃 = tan 𝜃 = ∙
cos 𝜃 , 1ൗ 2 1
2
tan 𝜃 = √3
𝜃 = 60° 1 𝑥 − 𝑎𝑥𝑖𝑠
𝑥

 Angle theta at 0, 𝜃 = 90°

𝑦 − 𝑎𝑥𝑖𝑠
(𝑥, 𝑦) = (cos 𝜃 , sin 𝜃)
1 (𝑥, 𝑦) = (cos 90° , sin 90°)
(𝑥, 𝑦) = (0, 1)

sin 𝜃 1
tan 𝜃 = ; tan 𝜃 =
cos 𝜃 , 0
𝑦 tan 𝜃 𝑖𝑠 𝑢𝑛𝑑𝑒𝑓𝑖𝑛𝑒𝑑

𝜃 = 90°

1 𝑥 − 𝑎𝑥𝑖𝑠

Figure 3-38: Coordinates in a unit circle – Source: Christian Kevin Villanueva

Confidential to United Marine Training Center, Inc. Page 3-98


Pre Sea Review Materials <course code>
Version Number
Deck
5 November 2018

Figure 3-39 shows a summary of the circular functions of the whole circle where the angle 𝜃 is in both
degrees and radians. The angles shown are just multiples of the special angles 300 , 450 and 600 . The
coordinates are similar to the factors that relate the sides of the special triangles. The coordinates of
the points are numerically the same except their signs as they depend on the quadrants where they
lie.

Figure 3-39: Unit circle with angles and point coordinates equivalent - Source:
https://commons.wikimedia.org/wiki/File:Unit_circle_angles_color.svg

Signs of Trigonometric Functions


The signs of the trigonometric functions follow the signs of 𝑥 and 𝑦 coordinates in each quadrant.
Refer to Figure 3-40 for the signs of 𝑥 and 𝑦 depending on which quadrant they are located.

𝑦 − 𝑎𝑥𝑖𝑠

2nd Quadrant 1st Quadrant


(−𝑥, 𝑦) (𝑥, 𝑦)

(0, 0) Origin

𝑥 − 𝑎𝑥𝑖𝑠

rd th
3 Quadrant 4 Quadrant
(−𝑥, − 𝑦) (𝑥, − 𝑦)

Figure 3-40: Signs of the 𝑥 and 𝑦 coordinates in the rectangular coordinate system – Source: Christian Kevin Villanueva

Confidential to United Marine Training Center, Inc. Page 3-99


Pre Sea Review Materials <course code>
Version Number
Deck
5 November 2018

Recall that 𝑥 = cos 𝜃, 𝑦 = sin 𝜃 and tan 𝜃 = 𝑦/𝑥, thus the cosine function follows the sign of 𝑥, the sine
function follows the sign of 𝑦 and the tangent function is positive if 𝑥 and 𝑦 are both positive or both
negative and its negative if 𝑥 and 𝑦 are of different signs.

To summarize:
Quadrant I: All functions are positive. Any point in the first quadrant is represented by 𝑷(𝒙, 𝒚) =
𝑷(𝐜𝐨𝐬𝜽, 𝐬𝐢𝐧𝜽).
Quadrant II: Sine and its reciprocal (cosecant) are positive. Any point in the second quadrant is
represented by 𝑷(−𝒙, 𝒚) = 𝑷(−𝐜𝐨𝐬𝜽, 𝐬𝐢𝐧𝜽). Since sin 𝜃 = 𝑦 , sine is positive in the 2nd
quadrant. The reciprocal of a positive number is positive, thus cosecant which is the
reciprocal of sine is also positive.
Quadrant III: Tangent and its reciprocal (cotangent) are positive. Any point in the third quadrant is
𝒚 −𝒚 𝒔𝐢𝐧𝜽 −𝐬𝐢𝐧𝜽
represented by 𝑷(−𝒙, −𝒚) = 𝑷(−𝐜𝐨𝐬𝜽, −𝐬𝐢𝐧𝜽). Since 𝐭𝐚𝐧𝜽 = = = = , tangent
𝒙 −𝒙 𝐜𝐨𝐬𝜽 −𝐜𝐨𝐬𝜽
is positive when 𝑥 and 𝑦 are both positive or both negative.
Quadrant IV: Cosine and its reciprocal (secant) are positive. Any point in the fourth quadrant is
represented by 𝑷(𝒙, −𝒚) = 𝑷(𝒄𝒐𝒔𝜽, −𝒔𝒊𝒏𝜽). Since 𝑐𝑜𝑠𝜃 = 𝑥, cosine is positive in the 4th
quadrant.

Another way to summarize and easily remember the signs of the trigonometric functions is by using
the pneumonic of Figure 3-41. These signs are important when we are finding an angle from a given
ratio.
Let A stand for all trigonometric functions. S stand for sine, T stand for tangent, and C stand for
cosine. These single letters correspond in which quadrant sine, cosine, and tangent are positive.
Create a phrase starting with these letters to remember them easily such as - “All Seafarers
Take ColRegs.”

𝑦 − 𝑎𝑥𝑖𝑠

Sine and its All


reciprocal 2nd Quadrant 1st Quadrant trigonometric
function are Seafarers All functions are
positive positive
(0, 0) Origin

𝑥 − 𝑎𝑥𝑖𝑠

Tangent and Cosine and its


its reciprocal 3rd Quadrant 4th Quadrant reciprocal
function are Take ColRegs function are
positive positive

Figure 3-41: Pneumonic for the signs of trigonometric functions – Source: Ester Dinawanao

Confidential to United Marine Training Center, Inc. Page 3-100


Pre Sea Review Materials <course code>
Version Number
Deck
5 November 2018

Example 3-85

Determine the signs of the following trigonometric functions of the following angles without
simplifying them using a scientific calculator
a. sin 50°
b. cos 150°
c. tan 215°
d. csc 300°
e. sec(−45°)
f. cot(−120°)
g. sin 150°

Solution

a. 50° 𝑖𝑠 𝑖𝑛 𝑡ℎ𝑒 1𝑠𝑡 𝑄𝑢𝑎𝑑𝑟𝑎𝑛𝑡 If the angle terminates in the 1st quadrant, all
trigonometric functions are positive
𝐬𝐢𝐧 𝟓𝟎° 𝒊𝒔 𝒑𝒐𝒔𝒊𝒕𝒊𝒗𝒆

b. 150° 𝑖𝑠 𝑖𝑛 𝑡ℎ𝑒 2𝑛𝑑 𝑄𝑢𝑎𝑑𝑟𝑎𝑛𝑡 If the angle terminates in the 2nd quadrant, only sine
and cosecant are positive; else, it is negative
𝐜𝐨𝐬 𝟏𝟓𝟎° 𝒊𝒔 𝒏𝒆𝒈𝒂𝒕𝒊𝒗𝒆

c. 215° 𝑖𝑠 𝑖𝑛 𝑡ℎ𝑒 3𝑟𝑑 𝑄𝑢𝑎𝑑𝑟𝑎𝑛𝑡 If the angle terminates in the 3rd quadrant, tangent
and cotangent are positive
𝐭𝐚𝐧 𝟐𝟏𝟓° 𝒊𝒔 𝒑𝒐𝒔𝒊𝒕𝒊𝒗𝒆

d. 300° 𝑖𝑠 𝑖𝑛 𝑡ℎ𝑒 4𝑡ℎ 𝑄𝑢𝑎𝑑𝑟𝑎𝑛𝑡 If the angle terminates in the 4th quadrant, only
cosine and secant are positive; else, it is negative
𝐜𝐬𝐜 𝟑𝟎𝟎° 𝒊𝒔 𝒏𝒆𝒈𝒂𝒕𝒊𝒗𝒆

e. −45° 𝑖𝑠 𝑖𝑛 𝑡ℎ𝑒 4𝑡ℎ 𝑄𝑢𝑎𝑑𝑟𝑎𝑛𝑡 Angle is measured clockwise from the positive x-axis
𝐬𝐞𝐜(−𝟒𝟓°) 𝒊𝒔 𝒑𝒐𝒔𝒊𝒕𝒊𝒗𝒆 If the angle terminates in the 4th quadrant, cosine and
secant are positive

f. −120° 𝑖𝑠 𝑖𝑛 𝑡ℎ𝑒 3𝑟𝑑 𝑄𝑢𝑎𝑑𝑟𝑎𝑛𝑡 Angle is measured clockwise from the positive x-axis
𝐜𝐨𝐭(𝟏𝟐𝟎°) 𝒊𝒔 𝒑𝒐𝒔𝒊𝒕𝒊𝒗𝒆 If the angle terminates in the 3rd quadrant, tangent
and cotangent are positive

g. 150° 𝑖𝑠 𝑖𝑛 𝑡ℎ𝑒 2𝑛𝑑 𝑄𝑢𝑎𝑑𝑟𝑎𝑛𝑡 If the angle terminates in the 2nd quadrant, sine and
cosecant are positive
𝐬𝐢𝐧 𝟏𝟓𝟎° 𝒊𝒔 𝒑𝒐𝒔𝒊𝒕𝒊𝒗𝒆

Oblique Triangles

An oblique triangle is a triangle that doesn’t have a right angle. It can also be classified as an acute
triangle or an obtuse triangle. Figure 3-42 shows the two classifications of oblique triangle.
𝐶

𝑏 𝐶
𝑎
𝑏 𝑎

𝐴 𝑐 𝐵 𝐴 𝑐 𝐵
(a) (b)
Figure 3-42: (a) Acute triangle and (b) obtuse triangle – Source: Christian Kevin Villanueva

Confidential to United Marine Training Center, Inc. Page 3-101


Pre Sea Review Materials <course code>
Version Number
Deck
5 November 2018

To solve oblique triangles, the laws of sine and cosine can be used. There are four different potential
scenarios when solving for the sides and angles of an oblique triangle.

Law of Sine
The Law of Sines is the relationship between the sides and angles of non-right triangles. It states that
the ratio of the length of a side of a triangle to the sine of the angle opposite that side is the same for
all sides and angles in a given triangle. It is mathematically represented as
𝒂 𝒃 𝒄 Formula for the Law of Sines (Plane Triangles)
= =
𝐬𝐢𝐧 𝑨 𝐬𝐢𝐧 𝑩 𝐬𝐢𝐧 𝑪
Where:
a, b & c = Sides of the triangle
A, B & C = Angles opposite of 𝑎, 𝑏 and 𝑐 respectively

In solving the angles, it also helpful to recall that the sum of the angles inside any right triangle is
always 180°

Cases to be considered in using Law of Sine


 Case 1. Two angles and one side are given (SAA or AAS)

𝑏 𝑎

𝐴 𝑐 𝐵

Figure 3-43: Two angles and one side case – Source: Christian Kevin Villanueva

𝒂 𝒃 𝒂 𝐬𝐢𝐧 𝑩
= ; 𝒃=
𝐬𝐢𝐧 𝑨 𝐬𝐢𝐧 𝑩 𝐬𝐢𝐧 𝑨

𝒂 𝒄 𝒂 𝐬𝐢𝐧 𝑪
= ; 𝒄=
𝐬𝐢𝐧 𝑨 𝐬𝐢𝐧 𝑪 𝐬𝐢𝐧 𝑨

Example 3-86

Solve all the missing sides and angles of the triangle with the following data:
𝐵 = 45°, 𝐶 = 105° and 𝑎 = 6.
Solution
Illustrate and label the triangle with the given parts

𝑏 𝑎 = 6𝑐𝑚
𝐶 = 105°
𝐴 𝐵 = 45°
𝑐

Confidential to United Marine Training Center, Inc. Page 3-102


Pre Sea Review Materials <course code>
Version Number
Deck
5 November 2018

Solve for the missing angle,

𝐴 + 𝐵 + 𝐶 = 180° The sum of the angles inside any plane triangle is


180°

𝐴 + 45° + 105° = 180° Substitute the given values

𝐴 = 180° − 45° − 105° Subtract both sides by 45° and 105°

𝑨 = 𝟑𝟎° Angle 𝐴

Solve the missing sides using the sine law,

𝑎 𝑏 Since 𝑎 , 𝐴, 𝑎𝑛𝑑 𝐵 are known, then 𝑏 can be


= solved using the sine law.
𝑠𝑖𝑛 𝐴 𝑠𝑖𝑛 𝐵

6 𝑏 Substitute the given into the equation


=
𝑠𝑖𝑛 30° 𝑠𝑖𝑛 45°

6 𝑠𝑖𝑛 45° Perform formula transformation


𝑏=
𝑠𝑖𝑛 30°

𝒃 = 𝟖. 𝟒𝟗 Use scientific calculator to evaluate

𝑎 𝑐 Since 𝑎 , 𝐴, 𝑎𝑛𝑑 𝐶 are known, then 𝑐 can be


=
𝑠𝑖𝑛 𝐴 𝑠𝑖𝑛 𝐶 solved using the sine law

6 𝑐 Substitute the given into the equation


=
𝑠𝑖𝑛 30° 𝑠𝑖𝑛 105°
6 𝑠𝑖𝑛 105° Formula transformation
𝑐=
𝑠𝑖𝑛 30°

𝒄 = 𝟏𝟏. 𝟓𝟗 Use scientific calculator to evaluate


Therefore, the missing parts are:
𝑨 = 𝟑𝟎°,
𝒃 = 𝟖. 𝟒𝟗,
𝒄 = 𝟏𝟏. 𝟓𝟗 .

Confidential to United Marine Training Center, Inc. Page 3-103


Pre Sea Review Materials <course code>
Version Number
Deck
5 November 2018

 Case 2. Two sides and an angle opposite one of these sides are given (SSA)
𝐶
𝑏
𝑎

𝐴 𝑐 𝐵

Figure 3-44: Two sides and an angle opposite either sides – Source: Christian Kevin Villanueva

𝒂 𝒃
=
𝐬𝐢𝐧 𝑨 𝐬𝐢𝐧 𝑩

Case 2.1. If 𝐬𝐢𝐧 𝑩 > 𝟏, then there is no solution


Case 2.2. If 𝐬𝐢𝐧 𝑩 = 𝟏, then there is one solution (A right triangle)
Case 2.3. If 𝐬𝐢𝐧 𝑩 < 𝟏, then there are two solutions

Example 3-87

Solve the triangle with the following data: 𝐴 = 30°, 𝑎 = 3 𝑚, 𝑏 = 8 𝑚.


Solution
Sketch the triangle approximately to scale
𝐶

𝑎=3 𝑏=8

𝐴 = 30°

Solve for sin 𝐵,

𝑎 𝑏 Two sides and an angle opposite one of the


= given sides are given, thus apply the law of sine.
𝑠𝑖𝑛 𝐴 𝑠𝑖𝑛 𝐵
3 8 Substitute the given into the equation
=
𝑠𝑖𝑛 30° 𝑠𝑖𝑛 𝐵
8 𝑠𝑖𝑛 30° Formula transformation
𝑠𝑖𝑛 𝐵 =
3
4
𝑠𝑖𝑛 𝐵 =
3
4 Since the sine of an angle can never be greater
𝑠𝑖𝑛 𝐵 > 1 𝑜𝑟 >1 than 1, the problem has no solution.
3

4 No triangle can be formed.


𝐵 = 𝑠𝑖𝑛−1 ( )
3 No solution

Confidential to United Marine Training Center, Inc. Page 3-104


Pre Sea Review Materials <course code>
Version Number
Deck
5 November 2018

Example 3-88

Solve the triangle with the following data: 𝐴 = 30°, 𝑎 = 3 𝑚, 𝑏 = 6 𝑚.


Solution
Sketch the triangle approximately to scale
𝐶

𝑏=8
𝑎=6

𝐵 𝐴 = 30°

Solve for sin 𝐵 and solve determine the solution,

𝑎 𝑏 Two sides and an angle opposite one of the


= given sides are given , thus apply the law of
𝑠𝑖𝑛 𝐴 𝑠𝑖𝑛 𝐵
sine.

3 6 Substitute the given into the equation


=
𝑠𝑖𝑛 30° 𝑠𝑖𝑛 𝐵
6 𝑠𝑖𝑛 30° Formula transformation
𝑠𝑖𝑛 𝐵 =
3
3
𝑠𝑖𝑛 𝐵 =
3
3 Since sin 𝐵 = 1 , the triangle has one solution.
𝑠𝑖𝑛 𝐵 = 1 𝑜𝑟 =1
3

𝐵 = 𝑠𝑖𝑛−1 (1) Solve for the value of B by using inverse sine


function.
𝐵 = 90°

𝐴 + 𝐵 + 𝐶 = 180° The sum of the 3 angles is 1800 , since two


angles are known, angle C can be solved.
30° + 90° + 𝐶 = 180°
𝐶 = 180° − 90° − 30°
𝐶 = 60°

𝑐 Since 𝐵 = 900, the triangle is a right triangle,


cos 30° =
6 thus cosine function can be used to solve for
side c.
Note; the triangle is a special triangle,30° −
60° − 90° , thus c can be obtained by multiplying
6 by √3.

𝑐 = 6 cos 30° Formula transformation and evaluation using


scientific calculator.
𝒄 = 𝟑√𝟑
Therefore, the missing parts are:
𝒄 = 𝟓. 𝟏𝟗𝟔
𝑩 = 𝟗𝟎°,
𝑪 = 𝟔𝟎°,
𝒄 = 𝟓. 𝟏𝟗𝟔

Confidential to United Marine Training Center, Inc. Page 3-105


Pre Sea Review Materials <course code>
Version Number
Deck
5 November 2018

Example 3-89

Solve the triangle with the following data: 𝐴 = 60°, 𝑎 = 8 𝑚, 𝑏 = 4 𝑚.


Solution
Sketch the triangle approximately to scale

𝑎=8 𝑏=4 𝑎=8 𝑏=4

𝐴 = 60° 𝐴 = 60°

Same length of side A but two possible


measures of side B and angle B

Solve for sin 𝐵 and determine the possible solutions,

𝑎 𝑏 Two sides and an angle opposite one of the


= given sides are given , thus apply the law of
𝑠𝑖𝑛 𝐴 𝑠𝑖𝑛 𝐵
sine.

8 4 Substitute the given into the equation


=
𝑠𝑖𝑛 60° 𝑠𝑖𝑛 𝐵
4 𝑠𝑖𝑛 60° Formula transformation
𝑠𝑖𝑛 𝐵 =
8
𝑠𝑖𝑛 𝐵 = 0.433

𝑠𝑖𝑛 𝐵 < 1 𝑜𝑟 0.433 < 1 Since sin 𝐵 < 1 , the triangle has one or two
solutions.

𝐵 = 𝑠𝑖𝑛−1 (0.433) Solve for the value of 𝐵 by using inverse sine


function.
𝑩 = 𝟐𝟓. 𝟔𝟔°
Note:
𝑩 = 𝟏𝟓𝟒. 𝟑𝟒°
Due to the definition of reference angle,
sin 25.66° = 0.433
sin 154.34° = 0.433
sin 25.66° = sin 154.34° = 0.433

Confidential to United Marine Training Center, Inc. Page 3-106


Pre Sea Review Materials <course code>
Version Number
Deck
5 November 2018

Using 25.66°,
𝐴 + 𝐵 + 𝐶 = 180° The sum of the 3 angles is 1800 , since two
angles are known, angle C can be solved.
60° + 25.66° + 𝐶 = 180°
𝐶 = 180° − 60° − 25.66°
𝐶 = 94.34°

𝑎 𝑐 Use sine law to solve for side 𝑐 since angle 𝐶 is


=
𝑠𝑖𝑛 𝐴 𝑠𝑖𝑛 𝐶 known already.

8 𝑐 Substitute the given into the equation


=
𝑠𝑖𝑛 60° 𝑠𝑖𝑛 94.34°
8 𝑠𝑖𝑛 94.34° Formula transformation
𝑐=
𝑠𝑖𝑛 60°

𝒄 = 𝟗. 𝟐𝟏 Simplify using scientific calculator


Therefore, the missing parts in one solution are:
𝑩 = 𝟐𝟓. 𝟔𝟔°,
𝑪 = 𝟗𝟒. 𝟑𝟒°,
𝒄 = 𝟗. 𝟐𝟏 𝒎.

Using 154.34°,

𝐴 + 𝐵 + 𝐶 = 180° This is not possible; therefore, there is only one


solution
60° + 154.34° + 𝐶 = 180°
Therefore the missing parts are 𝐵 = 25.660 ,
𝐶 = 180° − 60° − 154.34°
𝐶 = 94.340 , 𝑐 = 9.21 𝑚.
𝐶 = −34.34°

Confidential to United Marine Training Center, Inc. Page 3-107


Pre Sea Review Materials <course code>
Version Number
Deck
5 November 2018

Law of Cosines
The law of cosines relates the three sides of the triangle to one of the angles. It states that the square
of the length of any side is equal to the sum of the squares of the lengths of the other two sides minus
twice the product of those side lengths times the cosine of the angle between them. Mathematically
represented as

𝒄𝟐 = 𝒂𝟐 + 𝒃𝟐 − 𝟐𝒂𝒃 𝒄𝒐𝒔 𝑪 Formula for the Law of Cosines


𝒃𝟐 = 𝒂𝟐 + 𝒄𝟐 − 𝟐𝒂𝒄 𝒄𝒐𝒔 𝑩 Where:
𝟐 𝟐
𝒂 = 𝒃 + 𝒄 − 𝟐𝒃𝒄 𝐜𝐨𝐬 𝑨 a, b & c = Sides of the triangle
A, B & C = Angles opposite of 𝑎, 𝑏 and 𝑐 are
respectively

Cases to be considered in using Law of Cosine:

 Case 3. Two sides and an included angle are given (SAS)


𝐴

𝑏
𝑐

𝐶 𝑎 𝐵

Figure 3-45: Two sides and an included angle – Source: Christian Kevin Villanueva

𝑐 2 = 𝑎2 + 𝑏 2 − 2𝑎𝑏 𝑐𝑜𝑠 𝐶
𝑐 𝑠𝑖𝑛 𝐵 𝑐 𝑠𝑖𝑛 𝐵
𝑠𝑖𝑛 𝐵 = ; 𝐵 = 𝑠𝑖𝑛−1
𝑏 𝑏
𝐴 = 180° − 𝐵 − 𝐶

Example 3-90

Solve the triangle with the following data: 𝐶 = 30°, 𝑎 = 10 𝑚, 𝑏 = 7 𝑚.


Solution
Sketch the triangle approximately to scale,

𝑏=7 𝑐

𝐶 = 30°
𝐶 𝑎 = 10 𝐵

Confidential to United Marine Training Center, Inc. Page 3-108


Pre Sea Review Materials <course code>
Version Number
Deck
5 November 2018

Solve for the missing parts using both cosine law and sine law

𝑐 2 = 𝑎2 + 𝑏 2 − 2𝑎𝑏 𝑐𝑜𝑠 𝐶 Two sides and an included angle are given, thus
apply the law of cosine.

𝑐 = √𝑎2 + 𝑏 2 − 2𝑎𝑏 𝑐𝑜𝑠 𝐶 Extract the square root on both sides of the
equation

𝑐 = √102 + 72 − 2(10)(7)(𝑐𝑜𝑠 30°) Substitute the given into the equation

𝑐 = √27.75644 Evaluate using calculator to solve for the value of


𝑐.
𝑐 = 5.27

𝑏 𝑐 Use sine law to solve for angle 𝐵 since


=
𝑠𝑖𝑛 𝐵 𝑠𝑖𝑛 𝐶 𝑏, 𝑐 and 𝐶 are known.

7 5.27 Substitute the given in the formula


=
𝑠𝑖𝑛 𝐵 𝑠𝑖𝑛 30°

7 𝑠𝑖𝑛 30° Formula transformation and simplification


𝑠𝑖𝑛 𝐵 =
5.27
𝑠𝑖𝑛 𝐵 = 0.66414

𝐵 = 𝑠𝑖𝑛−1 (0.66414) Solve for the value of 𝐵 by using inverse sine


function.
𝐵 = 41.62°

𝐴 = 180° − 𝐶 − 𝐵 The sum of the 3 angles is 1800 , since two


angles are known, angle 𝐵 can be solved.
𝐴 = 180° − 30° − 41.62°
Therefore, the missing parts are:
𝐴 = 108.38°
𝑩 = 𝟒𝟏. 𝟔𝟐°,
𝑨 = 𝟏𝟎𝟖. 𝟑𝟖°,
𝒄 = 𝟓. 𝟐𝟕 𝒎.

Confidential to United Marine Training Center, Inc. Page 3-109


Pre Sea Review Materials <course code>
Version Number
Deck
5 November 2018

 Case 4. Three sides are given (SSS)


𝐶

𝑏 𝑎

𝐴 𝑐 𝐵

Figure 3-46: Three sides are given – Source: Christian Kevin Villanueva

𝑏 2 + 𝑐 2 − 𝑎2
𝑐𝑜𝑠 𝐴 =
2𝑏𝑐
𝑎2 + 𝑐 2 − 𝑏 2
𝑐𝑜𝑠 𝐵 =
2𝑎𝑐
𝑎2 + 𝑏 2 − 𝑐 2
𝑐𝑜𝑠 𝐶 =
2𝑎𝑏

Example 3-91

Solve the triangle with the following data: 𝑎 = 15 𝑚, 𝑏 = 22 𝑚 and 𝑐 = 17 𝑚.


Solution
Sketch the triangle approximately to scale,

𝑏 = 22
𝑎 = 15

𝐴 𝐵
𝑐 = 17

Solve for the missing angles

𝑎2 + 𝑏 2 − 𝑐 2 Since all the sides are known, apply the law of


𝑐𝑜𝑠 𝐶 = cosine to solve for the angles.
2𝑎𝑏
The formula for solving the angle is obtained by
applying formula transformation on:
𝑐 2 = 𝑎2 + 𝑏 2 − 2𝑎𝑏 ∙ cos 𝐶

Confidential to United Marine Training Center, Inc. Page 3-110


Pre Sea Review Materials <course code>
Version Number
Deck
5 November 2018

152 + 222 − 172 Substitute the given into the equation


𝑐𝑜𝑠 𝐶 =
2(15)(22)

420 Simplify the numerator and denominator


𝑐𝑜𝑠 𝐶 = separately , then divide.
660
𝑐𝑜𝑠 𝐶 = 0.6363636

𝐶 = 𝑐𝑜𝑠 −1(0.6363636) Apply definition of inverse cosine function

𝑪 = 𝟓𝟎. 𝟒𝟖° Angle 𝐶

𝑏 2 + 𝑐 2 − 𝑎2 Formula transformation of the cosine law to solve


𝑐𝑜𝑠 𝐴 = for angle 𝐴
2𝑏𝑐

222 + 172 − 152 Substitute the given into the equation


𝑐𝑜𝑠 𝐴 =
2(22)(17)

548 Simplify the numerator and denominator


𝑐𝑜𝑠 𝐴 = separately, then divide
748
𝑐𝑜𝑠 𝐴 = 0.732620

𝐴 = 𝑐𝑜𝑠 −1(0.732620) Apply definition of inverse cosine function

𝑨 = 𝟒𝟐. 𝟖𝟗° Angle 𝐴

𝐵 = 180° − 𝐴 − 𝐶 The sum of the interior angles in a plane triangle


is always 180°

𝐵 = 180° − 42.89° − 50.48° Substitute the solved values

𝑩 = 𝟖𝟔. 𝟔𝟑° Angle 𝐵

Confidential to United Marine Training Center, Inc. Page 3-111


Pre Sea Review Materials <course code>
Version Number
Deck
5 November 2018

Supplementary Exercises
Solve for the missing parts of the following oblique triangles using the appropriate method.
Answer
1] 𝐴 = 60°; 𝑎 = 12.49;
𝑏 = 14; 𝐵 = 76.10°;
𝑐 = 10 𝐶 = 43.90°
2] 𝑎 = 5; 𝐴 = 44.42°;
𝑏 = 6; 𝐵 = 57.13°;
𝑐=7 𝐶 = 78.45°
3] 𝐶 = 105°; 𝐴 = 40°;
𝐵 = 35°; 𝑎 = 7.85;
𝑏=7 𝑐 = 11.79
4] 𝐶 = 63°; 𝐴 = 67.41°;
𝑐 = 5.5; 𝐵 = 49.59°;
𝑏 = 4.7 𝑎 = 5.7
5] 𝐴 = 42°; 𝐶 = 63°;
𝐵 = 75°; 𝑎 = 16.52;
𝑐 = 22 𝑏 = 23.85

Confidential to United Marine Training Center, Inc. Page 3-112


Pre Sea Review Materials <course code>
Version Number
Deck
5 November 2018

Applications of Plane Trigonometry

Today, trigonometry is widely used in physics, astronomy, engineering, navigation, surveying, and
various fields of mathematics and other disciplines. In this section we will see some of the ways in
which trigonometry can be applied in the maritime professions such as finding the distance and
course made good, the bearing of the ship from the lighthouse or from other objects, the departure
and difference of latitude in plane sailing, parallel sailing, and Mercator sailing.

Angle of Elevation and Angle of Depression


Trigonometry also helps in finding the height and distance of objects as well as distance between
points without actually measuring these height and distances, like for example the height of the cliff,
the distance of the ship from the base of the cliff or from other ships etc. By measuring suitable angles
it is possible to calculate the height and distance.

Figure 3-47 shows the two common terms which are used to calculate height and distance.
 Angle of Elevation
 Angle of Depression

Observer A
Horizontal
Angle of Depression

Angle of Elevation
Horizontal

Observer B

Figure 3-47: Angle of depression and angle of elevation - – Source: Christian Kevin Villanueva

Note that the angle of elevation and the angle of depression are congruent by alternate-interior angles
concept in geometry.

Observer’s Eye Angle of Depression Object being observed


Horizontal
Cliff Cliff

Object being observed Horizontal

Angle of Elevation Observer’s Eye

(a) (b)

Figure 3-48: (a) Angle of depression and (b) Angle of elevation – Source: Christian Kevin Villanueva

Confidential to United Marine Training Center, Inc. Page 3-113


Pre Sea Review Materials <course code>
Version Number
Deck
5 November 2018

Let’s take a look at the figure of Figure 3-48 to understand the concepts of angle of elevation and
angle of depression
The angle of elevation is the angle between the horizontal line (eye level) and the line from the
object to the observer's eye (the line of sight) if the object is above the level of the observer.
The angle of depression is the angle between the horizontal (eye level) and the observer's line of
sight if the object is below the level of the observer.

Example 3-92

From the top of a vertical cliff 40 𝑚 high, the angle of depression of a ship at sea is 28° What is the
distance (to two decimal places) of the ship from the foot of the cliff?

Solution
Draw a sketch of the situation and label the figure.
Let 𝑑 represents the distance of the ship from the foot of the cliff
Let 𝛼 is a pair of alternate interior angle of 28°

Horizontal

Cliff 𝛼 = 28°

40 𝑚 40 𝑚

𝛼 = 28°
Horizontal
90°
𝑑

Solve for the distance 𝑑 using the appropriate trigonometric function

𝒐𝒑𝒑𝒐𝒔𝒊𝒕𝒆 Consider the right triangle with an acute angle 𝛼 = 28°, the given
𝐭𝐚𝐧𝛉 = part is the opposite side and the unknown part is the adjacent
𝒂𝒅𝒋𝒂𝒄𝒆𝒏𝒕
side. Thus, apply the tangent function.
40 𝑚 Plug in the given
tan 28° =
𝑑

𝑑 ∙ (tan 28° ) = 40 𝑚 Cross multiply

𝑑(tan 28°) 40 𝑚 Divide both sides of the equation by tan 28° to find d.
=
tan 28° tan 28°
40 𝑚
𝑑=
tan 28°

Use scientific calculator to get the value of d. (correct to two


𝒅 = 𝟕𝟓. 𝟐𝟑 𝒎 decimal places).
Therefore, the distance of the ship from the foot of the cliff is
75.23 𝑚.

Confidential to United Marine Training Center, Inc. Page 3-114


Pre Sea Review Materials <course code>
Version Number
Deck
5 November 2018

Example 3-93

From a ship at sea, the angle of elevations of the top and bottom of a vertical lighthouse standing on
the edge of a vertical cliff are 31° and 26°, respectively. If the lighthouse is 25.0 𝑚 high, calculate the
height of the cliff.
Solution
Draw a sketch of the situation and label the figure.
Let ℎ represents the height of the cliff
Let 𝑥 represents the distance of the ship (𝑆) to the base of the cliff (𝑂)

𝐿
25 m

31° 26°
Horizontal 𝑂 𝑆
𝑥

Solution and calculation for 𝑥

Using right triangle 𝛥𝑆𝑂𝐶, There are two right triangles namely
∆𝑆𝑂𝐶 and ∆𝑆𝑂𝐿 .With respect to the

𝑡𝑎𝑛 26° = → 𝑒𝑞. 1 given angles, we have the ratio of
𝑥 opposite side to adjacent side, thus apply
tangent function,
Using right triangle 𝛥𝑆𝑂𝐿, 𝑜𝑝𝑝𝑜𝑠𝑖𝑡𝑒
tan 𝜃 =
ℎ + 25 𝑎𝑑𝑗𝑎𝑐𝑒𝑛𝑡
𝑡𝑎𝑛 31° = → 𝑒𝑞. 2
𝑥

From eq.1 Solve for h in equations 1 and 2 and


label them equations 3 and 4
ℎ = 𝑥 𝑡𝑎𝑛 26° → 𝑒𝑞. 3 respectively.

From eq. 2
ℎ + 25 = 𝑥 𝑡𝑎𝑛 310
ℎ = 𝑥 𝑡𝑎𝑛 310 − 25 → 𝑒𝑞. 4

ℎ=ℎ By transitive property which states that


𝑥 𝑡𝑎𝑛 26° = 𝑥 𝑡𝑎𝑛 31° − 25 𝑖𝑓 𝑎 = 𝑏, and 𝑏 = 𝑐, then 𝑎 = 𝑐.
Since ℎ = 𝑥 tan 26° and
ℎ = 𝑥 tan 31° − 25 then
𝑥 tan 26° = 𝑥 tan 31° − 25

Confidential to United Marine Training Center, Inc. Page 3-115


Pre Sea Review Materials <course code>
Version Number
Deck
5 November 2018

25 = 𝑥 𝑡𝑎𝑛 31° − 𝑥 𝑡𝑎𝑛 26° Regrouping the terms

25 = 𝑥 (𝑡𝑎𝑛 31° − 𝑡𝑎𝑛 26°) Factor out 𝑥 on the right side of the
equation

25 𝑥 (𝑡𝑎𝑛 31° − 𝑡𝑎𝑛 26°) Isolate 𝑥 by dividing both sides of the


= equation by the common factor,
(𝑡𝑎𝑛 31° − 𝑡𝑎𝑛 26°) (𝑡𝑎𝑛 31° − 𝑡𝑎𝑛 26°)
tan 31° − tan 26°
25 Simplest form of the equation.
𝑥=
𝑡𝑎𝑛 31° − 𝑡𝑎𝑛 26° Use scientific calculator to solve the
value of 𝑥.
25 Get the values of tan 31° and tan 26°
𝑥=
0.60086 − 0.48773 separately and then subtract the values,
25 then divide 25 by the difference..
𝑥=
0.11313
𝑥 = 220.98
ℎ = 𝑥 𝑡𝑎𝑛 260 → 𝑒𝑞. 3 Using equation 3 solve for height by
plugging in the value of 𝑥.
ℎ = 220.98 𝑡𝑎𝑛 26°
ℎ = 107.78
ℎ = 𝑥 𝑡𝑎𝑛 31° − 25 → 𝑒𝑞. 4 Equation 4 can also be used to solve for
the height.
ℎ = 220.98 𝑡𝑎𝑛 31° − 25
ℎ = 132.78 − 25
Therefore, the height of the cliff is
𝒉 = 𝟏𝟎𝟕. 𝟕𝟖 𝟏𝟎𝟕. 𝟕𝟖 𝒎.

Example 3-94

From a ship, the angle of elevation of a point 𝐵 at the top of a cliff is 21°13’. After the ship sailed
2,500 𝑚 directly towards 𝐵, its angle of elevation is found to be 47°17’. Find the height of the cliff.
Solution
Draw a sketch of the situation and label the figure
Let:
𝑆 be the position of the ship on the first observation
𝐶 be the position of the ship on the 2nd observation
𝐴 be the point at the base of the cliff
𝐵 be the point on top of the cliff (object being observed)
̅̅̅̅
𝐴𝐵 be the height of the cliff

47° 17′ 21° 13′


Horizontal 𝐴 𝐶 𝑆
2 500 𝑚

Confidential to United Marine Training Center, Inc. Page 3-116


Pre Sea Review Materials <course code>
Version Number
Deck
5 November 2018

Solution and calculation for the height of the cliff

∠𝑆𝐶𝐵 = 180° − 47°17′ Definition of linear pair.


∠𝑆𝐶𝐵 = 132° 43′ Two angles form linear pair if they are adjacent and
supplementary.

∠𝑆𝐵𝐶 = 180° − ∠𝐵𝑆𝐶 − ∠𝑆𝐶𝐵 Using triangle ∆𝑺𝑪𝑩 , solve for ∠𝑺𝑩𝑪.
′ ′
∠𝑆𝐵𝐶 = 180° − 21°13 − 132°43 The sum of the 3 angles in any triangle is equal to
1800 .
∠𝑆𝐵𝐶 = 26°4′
Substitute the known angles
2500 𝐵𝐶 Solve for 𝐵𝐶 in ∆𝑺𝑪𝑩 (𝑎𝑛 𝑜𝑏𝑙𝑖𝑞𝑢𝑒 𝑡𝑟𝑖𝑎𝑛𝑔𝑙𝑒) using

= sine law
𝑠𝑖𝑛 26°4 𝑠𝑖𝑛 21°13′

2 500 𝑠 𝑠𝑖𝑛 21°13′ Formula transformation and evaluation using


𝐵𝐶 = scientific calculator
𝑠𝑖𝑛 26°4′

𝐵𝐶 = 2 058.96 𝑚
𝐴𝐵 In right .∆𝐵𝐴𝐶 , solve for the height of the cliff ℎ =
𝑠𝑖𝑛 ∠𝐵𝐶𝐴 = ̅̅̅̅
𝐴𝐵 by using sine function.
𝐵𝐶
𝐴𝐵 Substitute the given values
𝑠𝑖𝑛 470 17′ =
2 058.96
𝐴𝐵 = 2 058.96 𝑠𝑖𝑛 47°17′ Formula transformation and evaluation using
scientific calculator.
𝑨𝑩 = 𝟏 𝟓𝟏𝟐. 𝟕𝟓 𝒎
Therefore, the height of the cliff is 1 512.75 𝑚.

Example 3-95

As observed from the top of a 75 𝑚 high lighthouse from the sea level, the angles of depression of
two ships are 30° and 45°. If one ship is exactly behind the other on the same side of the lighthouse,
find the distance between the two ships.

Solution
Draw a sketch of the situation and label the figure.
Let C be the point of the ship closer to the light house
Let D be the point of the ship farther to the light house
̅̅̅̅ = 𝑦 and 𝐵𝐶
Let 𝐵𝐷 ̅̅̅̅ = 𝑥 be the distances between the ships and the light house.
Let Point A be the point on the top of the lighthouse 𝐴𝐵

Confidential to United Marine Training Center, Inc. Page 3-117


Pre Sea Review Materials <course code>
Version Number
Deck
5 November 2018

30° 45°

75 𝑚

Horizontal 𝐵 𝐶 𝐷
𝑥

̅̅̅̅
Solution and calculation for 𝐶𝐷

Using right triangle 𝛥𝐴𝐵𝐶 There are two special right triangles namely ∆𝐴𝐵𝐶 or
75 45° − 45° − 90°and ∆𝐴𝐵𝐷 or 30° − 60° − 90° .
𝑡𝑎𝑛 45° = → 𝑒𝑞. 1 Since ∆𝐴𝐵𝐶 is a special right triangle, 𝐵𝐶 = 𝑥 can be
𝑥
determined immediately even without using
75 trigonometric function.
𝑥=
𝑡𝑎𝑛 45°
Recall that legs in a 45° − 45° − 90° triangle are
75
𝑥= = 75 𝑚 equal so 𝑩𝑪 = 𝑨𝑩 = 𝒙 = 𝟕𝟓 𝒎
1
𝐵𝐶 = 𝐴𝐵 = 𝑥 = 75 𝑚

Using right triangle 𝛥𝐴𝐵𝐷 ∆𝐴𝐵𝐷 is 30° − 60° − 90° with shorter leg, 𝐴𝐵 = 75 𝑚
75 𝐵𝐷 = 𝑦 = 𝑙𝑜𝑛𝑔𝑒𝑟 𝑙𝑒𝑔
𝑡𝑎𝑛 30° = → 𝑒𝑞. 2
𝑦
75
𝑦= From this relationship,
𝑡𝑎𝑛 30°
𝑦 = 75√3 = 129.90 𝑙𝑜𝑛𝑔𝑒𝑟 𝑙𝑒𝑔 = 𝑠ℎ𝑜𝑟𝑡𝑒𝑟 𝑙𝑒𝑔 √3
𝐵𝐷 = 𝑦 = 75√3

𝐶𝐷 = 𝑦 − 𝑥 Since the distance between two ships is represented


by 𝐶𝐷 = 𝑦 − 𝑥.
𝐶𝐷 = 129.90 − 75
Plug in the values of 𝑥 and 𝑦.

𝑪𝑫 = 𝟓𝟒. 𝟗𝟎 𝒎 The distance between the two ships which are on the
same side of the lighthouse is 54.90 𝑚.

Confidential to United Marine Training Center, Inc. Page 3-118


Pre Sea Review Materials <course code>
Version Number
Deck
5 November 2018

Supplementary Exercises
Solve the following problems
Answer
1] The angle of elevation of the top of a lighthouse from a ship 50 m
from it, is the complement of the angle of elevation of the same
74.16 𝑚
lighthouse when the ship is 110 𝑚 from it. Find the height of the
lighthouse.
2] The angle of elevation of a ship’s masthead 80 𝑚 above the sea is
observed from a boat to be 14.5° above the horizontal plane of the
301.60 𝑚
observer’s eye. The observer’s eye is 2 𝑚 above the sea. How far is
he from the ship?
3] From the top of a vertical cliff 130 𝑚 high the angle of depression of
a buoy at sea is 18°. What is the distance of the buoy from the foot 400.10 𝑚
of the cliff?
4] From the top of a 25 𝑚 lighthouse, an operator sees a capsized
boat and determines an angle of depression of 7° to the boat. A
patrol boat is also spotted at an angle of depression of 5°
a. 82.14 𝑚
a. If the two boats are on the same side of the lighthouse, how
far apart are the two boats? b. 489.36 𝑚
b. If the boats are on opposite sides of the lighthouse, how far
apart are the two boats?
5] As observed from the top of a 75 𝑚 high lighthouse from the sea
level, the angle of depression of two ships are 30° and45°. If one
54.90 𝑚
ship is exactly behind the other on the same side of the lighthouse,
find the distance between the two ships.

Confidential to United Marine Training Center, Inc. Page 3-119


Pre Sea Review Materials <course code>
Version Number
Deck
5 November 2018

Directions and Bearings

Direction
The direction to a point is stated as the number of degrees east or west of north or south.
There are two commonly used systems for indicating direction:
 Quadrantal Notation
 Three Figure Notation

Quadrantal Notation
The angles are measured from the North to East or West and South to East or West, 0° to 90°. The
angles are measured clockwise from the North 000° to 360°. To illustrate these notations, refer to
Figure 3-49 below:

North

B
60° 30°

O East
West
D 80°
70° C

South
Figure 3-49: Direction and bearing – Source: Christian Kevin Villanueva

Examples Quadrantal Notation Three Figure Notation

16. the direction of A from O 𝑵 𝟑𝟎° 𝑬 𝟎𝟑𝟎°


“ means 30º east of north”
17. the direction of B from O 𝑵 𝟔𝟎° 𝑾 𝟑𝟎𝟎°
“ means 60º west of north”
18. the direction of C from O 𝑺 𝟕𝟎° 𝑬 𝟏𝟏𝟎°
“ means 30º east of south”
19. the direction of D from O 𝑺 𝟖𝟎° 𝑾 𝟐𝟔𝟎°
“ means 30º west of south”
Table 3-12: Quadrantal and three-figure notation – Source: Ester Dinawanao

Confidential to United Marine Training Center, Inc. Page 3-120


Pre Sea Review Materials <course code>
Version Number
Deck
5 November 2018

Bearing
A bearing is used to indicate the direction. It is commonly used to navigate the sea (ship) or air
(plane). Examples of bearing are:
 The direction of an object (lighthouse) from a ship’s heading.
 The direction of the ship from the true North.
A three-digit notation is used to represent the bearing of one-point relative to another point.
In Figure 3-50, the bearing of 𝐴 from 𝐵 is 065° while the bearing of 𝐵 from 𝐴 is 245°.

North

North
West A East

245°

065°
B
West East
South

South

Figure 3-50: Bearing – Source: Christian Kevin Villanueva

Confidential to United Marine Training Center, Inc. Page 3-121


Pre Sea Review Materials <course code>
Version Number
Deck
5 November 2018

Types of Bearing
Figure 3-51 shows two types of bearing which are the true bearing and the relative bearing.
315° 000° 045°
True North

135°T

270° 090°

045° relative

225° 180° 135°


Figure 3-51: Relative and true bearing – Source: Christian Kevin Villanueva

True Bearing
 The true bearing to a point is the angle measured in degrees in a clockwise direction from
the True North (000°).
 It is the most common type of bearing and being referred to as simply bearing.
 It is reported using 3-digit notation followed by (𝑇).
Examples:
1. In Figure 3-49, the true bearing of point B is 𝟑𝟎𝟎° 𝑻.
2. In Figure 3-50, the true bearing of point B from point A is 𝟐𝟒𝟓° 𝑻 while the true bearing of
point A from point B is 𝟎𝟔𝟓° 𝑻.
3. In Figure 3-51, the true bearing of the red flag is 𝟏𝟑𝟓°𝑻.

Relative Bearing
 It is an angle measured in degrees in a clockwise direction from the ship’s heading or from
own ship’s centerline.
 It is reported using 3-digit notation followed by the word “relative”.
Example:
1. In Figure 3-51, the relative bearing of the red flag is 045° 𝑟𝑒𝑙𝑎𝑡𝑖𝑣𝑒.

Note: Heading is the direction in which the bow of the ship is pointing, expressed in an angular
distance from North. Course angle refers to the direction an object is actually moving.

Confidential to United Marine Training Center, Inc. Page 3-122


Pre Sea Review Materials <course code>
Version Number
Deck
5 November 2018

Reciprocal Bearing
 Reciprocal bearing is a bearing differing by 180°, or measured in the opposite direction from
a given bearing.
 Figure 3-52 illustrates reciprocal bearings or back bearings. The true bearings,
060° 𝑇 and 240° 𝑇 are in opposite directions to each other and 180° apart.

Figure 3-52: Reciprocal bearing – Source: Christian Kevin Villanueva

 In Figure 3-53 below the reciprocal bearing of 300° is 150° because if you extend the line in
the opposite direction of 300° , it is 330° − 180° = 150°

Figure 3-53: Reciprocal Bearing of 330° – Source: Christian Kevin Villanueva

Confidential to United Marine Training Center, Inc. Page 3-123


Pre Sea Review Materials <course code>
Version Number
Deck
5 November 2018

Conventional Bearing
 The conventional bearing of point is the number of degrees (clockwise or counter clockwise)
from the north-south line.
 It is reported using the quadrant notation. If the position of a point is exactly west or east,
then it can be reported as “due East” or “due West” respectively.
 The numbers in the conventional bearing format can be in decimal or degrees, minutes and
seconds with “N” or “S” as the prefix and “E” and “W” as the suffix.
 Examples are N 73°56’ W and S 33.25° E which stands for 73°56’ West of North and 33.25°
East of South respectively.
 Figure 3-54 shows how a position is reported using true bearing and conventional bearing.

North North

West East West East

South South

True Bearing: 051° T True Bearing: 125° T


Conventional Conventional
N 51° E S 55° E
Bearing: Bearing:

North North

West East West East

South South

True Bearing: 221° T True Bearing: 316° T


Conventional Conventional
S 41° W N 44° W
Bearing: Bearing:

Figure 3-54: True bearing (blue arrow) and conventional bearing (green arrow) - Source: Christian Kevin Villanueva

Confidential to United Marine Training Center, Inc. Page 3-124


Pre Sea Review Materials <course code>
Version Number
Deck
5 November 2018

Example 3-96

A ship sails from A to B on a bearing of 120° 𝑇. On what bearing will it have to sail to return from 𝐵
to 𝐴?
Solution
Sketch the graph of the situation, from true north of ship 𝐴, measure an angle of
1200 in a clockwise direction to ship 𝐵.

North

120°
West East
𝐴

North
South

West East
𝐵

South

Extend the line from A to B, then rotate through 180° to head in the opposite

North

120°
West East
𝐴
North

South
120°
West East
𝐵
180°

300°
South

𝟏𝟐𝟎° + 𝟏𝟖𝟎° = 𝟑𝟎𝟎° From the diagram, the bearing (represented by the
violet arrow) from 𝐵 to 𝐴 is 300° 𝑇.
.

Confidential to United Marine Training Center, Inc. Page 3-125


Pre Sea Review Materials <course code>
Version Number
Deck
5 November 2018

Example 3-97

𝐴, 𝐵 and 𝐶 are three ships. The bearing of 𝐴 from 𝐵 is 045° 𝑇. The bearing of 𝐶 from 𝐴 is 135° 𝑇. If
𝐴𝐵 = 8 𝑁𝑀 and 𝐴𝐶 = 6 𝑁𝑀, what is the bearing of ship 𝐵 from ship 𝐶?

Solution:
Draw a sketch of the situation and label the figure.
Let 𝑌 be the alternate interior angle of 180° − 135° = 45°
Let 𝑋 represent the bearing of ship 𝐵 from ship 𝐶.

135° N
N 𝐴
8 𝑁𝑀 6 𝑁𝑀
𝑌
45°
𝐶 𝑋

Solution for the bearing of ship B from ship C

𝐴 = 135° − 45° = 900 Since 𝐴 = 90°, then it is a right triangle.

8 With respect to acute angle 𝐶 of the right triangle


tan 𝐶 =
6 𝛥𝐵𝐴𝐶, the given sides are opposite and adjacent,
thus tangent function is used.

8 Apply the definition of inverse function to determine


𝐶 = 𝑡𝑎𝑛−1 ( )
6 the angle given the ratio of opposite side and
adjacent side.
𝐶 = 53.130
Use scientific calculator to solve for 𝐶.

𝑌 + 𝑋 + 𝐶 = 360° Notice from the diagram, that the sum of 3 angles,


𝐶, 𝑋 and 𝑌 is 1 full circle (1 𝑟𝑒𝑣 = 360°) .

𝑋 = 360° − 𝑌 − 𝐶 Formula transformation and inserting grouping


𝑋 = 360° − (𝑌 + 𝐶) symbol preceded by (-) sign.

𝑋 = 360° − (45° + 53.13°) Substitute the values of angles, 𝑌 = 45° and


𝑋 = 261.87°
𝐶 = 53.13°

𝑿 = 𝟐𝟔𝟐° 𝑻 The true bearing is reported as 3 digits followed by


T. Therefore, the bearing of ship 𝐵 from ship 𝐶 is
262° 𝑇.

Confidential to United Marine Training Center, Inc. Page 3-126


Pre Sea Review Materials <course code>
Version Number
Deck
5 November 2018

Example 3-98

Two ships, 𝑅 and 𝑄, left a port, 𝑃, at the same time. After two hours, 𝑅 had travelled 8 𝑁𝑀 on a
bearing 0750 𝑇 from port and 𝑄 had travelled 10 𝑁𝑀 on a bearing 305° 𝑇 from port. How far apart
are the two ships after two hours?
Solution:
Draw a sketch of the situation and label the figure.

Let 𝑃 be the port


Let 𝑄 and 𝑅 be the ships
̅̅̅̅ = 𝑥 be the distance between the two ships after two hours
Let 𝑄𝑅

Given:
𝑁𝑃𝑅 = 75° is the bearing from 𝑃 to 𝑅
Reflex angle 𝑁𝑃𝑄 = 305° is the bearing from 𝑃 to 𝑄
Acute angle 𝑁𝑃𝑄 = 360° − 305° = 55°

𝑄
𝑥

10 𝑁𝑀 𝑅
55°
75°
8 𝑁𝑀
𝑃
305°

Solution for 𝑥

𝑅𝑃𝑄 = 75° + 55° Since 𝑅𝑃𝑄 = 𝑁𝑃𝑅 + 𝑁𝑃𝑄


𝑅𝑃𝑄 = 130°
RPQ is the included angle between 𝑃𝑅 and 𝑃𝑄.

𝑥 2 = 82 + 102 − 2(8)(10) cos 130° Law of cosine will be used to solve for 𝑥 because the
situation satisfies the condition, SAS (side angle
𝑥 2 = 266.85 side).
Simplify using scientific calculator

𝒙 = 𝟏𝟔. 𝟑𝟑 Extract the square root and take the positive root
only because 𝑥 represents the distance
Therefore, the distance between the two ships, 𝑃
and 𝑄, after two hours is 𝟏𝟔. 𝟑𝟑 𝑵𝑴.

Confidential to United Marine Training Center, Inc. Page 3-127


Pre Sea Review Materials <course code>
Version Number
Deck
5 November 2018

Example 3-99

A ship is sailing due north. At a certain point the bearing of a lighthouse 12.5 𝑁𝑀 distant is
𝑁 39° 𝐸. Later on, the captain notices that the bearing of the lighthouse has become 𝑆 54° 𝐸.
a. How far did the ship travel between the two observations of the lighthouse?
b. Determine the bearing of the first observation point from the lighthouse.

Solution
Draw a sketch of the situation and label the figure.

Let 𝐿 be the lighthouse


Let 𝑁 and M be the first and second points of observation respectively
̅̅̅̅̅ = 𝑑 be the distance between the two observations of the lighthouse
Let 𝑁𝑀
Let 𝛽 be the bearing of the initial position of the ship from the lighthouse

Given:
𝑀𝑁𝐿 = 39° = 𝑁 39° 𝐸 is the bearing of the lighthouse (𝐿) from the first observation
(𝑁)

𝑁𝑀𝐿 = 54° = 𝑆 54° 𝐸 is the bearing of the lighthouse (𝐿) from the second
observation (𝑀).

̅̅̅̅
𝑁𝐿 = 12.5 𝑁𝑀 the distance of the lighthouse (𝐿) from the first observation (𝑁).

𝑀
54°
𝐿
𝜃
𝛽
𝑑

39°

Solution for the distance and bearing

𝜃 = 𝑀𝐿𝑁 = 180° − 39° − 54° = 87° The sum of the three angles in a triangle is 180°

12.5 𝑑 Law of sine will be used to solve for 𝑑 because


=
𝑠𝑖𝑛 54° 𝑠𝑖𝑛 87° the situation satisfies the condition, SAA( Two
angles and a side) .

12.5 ∙ 𝑠𝑖𝑛 87° Formula transformation


𝑑=
𝑠𝑖𝑛 54°

Confidential to United Marine Training Center, Inc. Page 3-128


Pre Sea Review Materials <course code>
Version Number
Deck
5 November 2018

𝒅 = 𝟏𝟓. 𝟒𝟑 Simplify.
Therefore, the distance between the two
observations 𝑀 and 𝑁, is 𝟏𝟓. 𝟒𝟑 𝑵𝑴.

𝛽 = 180° + 39° By alternate interior angles. (Refer to the figure)


𝜷 = 𝟐𝟏𝟗𝟎 𝑻 N

𝑀
54°
𝐿
𝜃
𝛽
𝑑
39°
39°

The bearing of the first observation point from


the lighthouse is 219° 𝑇.

Confidential to United Marine Training Center, Inc. Page 3-129


Pre Sea Review Materials <course code>
Version Number
Deck
5 November 2018

Supplementary Exercises
Solve the following problems
Answer
1] What is the back bearing or reciprocal bearing to each of
the bearings below? a. 225°𝑇
a. 045° b. 020°𝑇
b. 200° c. 000°𝑇 or North
c. 180°
2] The diagram shows the positions of two ships, 𝐴 and 𝐵.
What is the bearing:
a. from ship 𝐵 to ship 𝐴?
b. from ship 𝐴 to ship 𝐵?
North

a. 300° 𝑇
West East
𝐴 b. 120° 𝑇

South
60°

West East
𝐵

South

3] A ship leaves port at noon and heads due west at


20 𝑘𝑛𝑜𝑡𝑠, or 20 𝑛𝑎𝑢𝑡𝑖𝑐𝑎𝑙 𝑚𝑖𝑙𝑒𝑠 (𝑁𝑀) 𝑝𝑒𝑟 ℎ𝑜𝑢𝑟. At 1400𝐻, 57.50 𝑁𝑀 (distance)
the ship changes course to 𝑁 54° 𝑊 . Find the ship’s 282° 𝑇 (bearing)
bearing and distance from the port of departure at 1500𝐻.
4] A ship sailing due east in the North Atlantic has been
warned to change course to avoid icebergs. The captain
turns and sails on a bearing of 062° 𝑇, then changes
course again to a bearing of 115° 𝑇 until the ship reaches
its original course (see figure). How much farther did the
ship have to travel to avoid the icebergs?
N

N 115° 5.84 𝑚𝑖𝑙𝑒𝑠

65°

50 𝑚𝑖𝑙𝑒𝑠

Confidential to United Marine Training Center, Inc. Page 3-130


Pre Sea Review Materials <course code>
Version Number
Deck
5 November 2018

5] A ship leaves a pier on a bearing of 𝑆 62° 𝐸 and travels


for 75 𝑁𝑀. It then turns around and continues on a
083° 𝑇
bearing 𝑁 28° 𝐸 for 53 𝑁𝑀. Determine the bearing of the
ship from the pier?
6] From the berth, the vessel sailed Northerly
for 24 𝑛𝑎𝑢𝑡𝑖𝑐𝑎𝑙 𝑚𝑖𝑙𝑒𝑠 and then Easterly for 048° 𝑇
27 𝑛𝑎𝑢𝑡𝑖𝑐𝑎𝑙 𝑚𝑖𝑙𝑒𝑠. What is its bearing from the berth?
7] Two ships left the same port at the same time, one going
in the direction 𝑁 70° 𝐸 and the other sailing East. The
first ship travelled at 30 𝑁𝑀. After 30 𝑚𝑖𝑛𝑢𝑡𝑒𝑠, the second
ship was observed to be directly south of the first ship. 28.20 𝑘𝑛𝑜𝑡𝑠
Find the speed of the second ship.
8] A ship leaves its home port and sails on a bearing of
𝑆 61° 50′ 𝐸. Another ship leaves the same port at the
same time and sails on a bearing of 𝑁 28°10′ 𝐸. If the first 91.21 𝑁𝑀
ship sails at 14 𝑘𝑛𝑜𝑡𝑠 and the second sails at 18 𝑘𝑛𝑜𝑡𝑠,
find the distance between the two ships after 4 ℎ𝑜𝑢𝑟𝑠.
9] The length of the vertical post of a jib crane is 15 𝑚. The
angle between jib and post is 35° 30’ and between 22.97 𝑚 (length of jib)
topping lift wire and post the angle is 105° 30’. Calculate 3.84 𝑚 (topping lift wire)
the length of the jib and topping lift wire.
10] Two ships leave the same port at the same time. Ship A
sails on a course of 125° 𝑇 at 18 𝑘𝑛𝑜𝑡𝑠 while Ship 𝐵 sails 100.56 𝑁𝑀 (distance)
on a course of 230° 𝑇 at 24 𝑘𝑛𝑜𝑡𝑠. Find, after 3 hours, 261° 𝑇 (bearing)
the distance and bearing of ship 𝐵 from ship 𝐴.
11] 𝐴, 𝐵 and 𝐶 are three ships. The bearing of 𝐶 from 𝐴 is
035° 𝑇. The bearing of 𝐵 from 𝐴 is 346° 𝑇. The distance of
𝐶 from 𝐴 is 340 𝑚. The distance of 𝐵 from 𝐴 is 160 𝑁𝑀. a. 590
a. Find the magnitude of ∠𝐵𝐴𝐶. b. 291.83 𝑁𝑀
b. Use the cosine rule to find the distance of 𝐵 to 𝐶.
12] A ship leaves port A and steams 15 𝑁𝑀 due east. It then
turns and steams for 22 𝑁𝑀 due north. a. 034° 𝑇
a. What is the bearing of the ship from port 𝐴? b. 214° 𝑇
b. What is the bearing of port 𝐴 from the ship?
13] A ship travels from 𝑃 to 𝑄 on a course of 050° 𝑇 for
35 𝑁𝑀 and then from 𝑄 to 𝑅 on a course of 020° 𝑇 for 33.65 𝑁𝑀
20 𝑁𝑀. How far east of 𝑃 is 𝑅?
14] Starting at point 𝐴, a ship sails 18.5 𝑁𝑀 on a bearing of
189° 𝑇, then turns and sails 47.8 𝑁𝑀 on a bearing of 39.22 𝑁𝑀
317°𝑇. Find the distance of the ship from point 𝐴.
15] A boat sails from a certain port in the direction 𝑁 30° 𝑊.
After the boat has sailed 20 𝑁𝑀, how far is it west of the 10 𝑁𝑀
port?

Confidential to United Marine Training Center, Inc. Page 3-131


Pre Sea Review Materials <course code>
Version Number
Deck
5 November 2018

Sailings
Sailing is the term used in the maritime world to find the course and distance between two positions
on the earth’s surface by mathematical calculation.

Terrestrial Sphere
The globe can be mathematically modelled as a sphere; thus, mathematical calculations can be used
to solve for the lengths of the segment on the sphere which is equivalent to the distance travelled
around the globe.
Figure 3-55 shows the names of the arcs, angles, circles and points if they are to represent the
navigational terms.

Difference of Longitude, 𝑫𝒍𝒐


The angle between meridians PAP’ and PBP’ Point P North Pole

Arc ab, 𝒂𝒃 Axis of the Earth’s Rotation


α°
The Departure
Line P Circle L Parallel of Latitude
Small Circle
𝑫𝑳𝑶
Latitude, Lat or Φ
an angular distance from the a α°
equator to the parallel of b
latitude Φ° Latitude, Lat or Φ
The latitude measurement
Earth’s Radius as seen on the globe
Φ° O
Earth’s Radius α° Earth’s Center

A
B 0° Equator
Arc AB, 𝑨𝑩
Earth’s Radius
An arc of the Circle E
The Departure at the Equator Circle E
𝑫𝑳𝑶
The DLO in arc min or
nautical miles Equator
Great Circle
Arc PAP’

Meridians Arc PBP’

Point P’ South Pole

Figure 3-55: Terrestrial sphere – Source: Christian Kevin Villanueva

Lines of Latitude
 known as parallels and used to measure latitude
 run in an east-west direction
 measure distance north or south from the equator
 cross the prime meridian at right angles
 get shorter towards the pole

Confidential to United Marine Training Center, Inc. Page 3-132


Pre Sea Review Materials <course code>
Version Number
Deck
5 November 2018

Figure 3-56: Lines of latitude – Source: https://kids.britannica.com/students/assembly/view/54583

Latitude (𝑳𝒂𝒕)
It is an angular distance from the equator, measured northward or southward through 90 ⁰ (N or S)
North Pole

Latitude 46° N

Earth’s Center
Equator
46° N

Figure 3-57: Latitude as angular measurement – Source: Christian Kevin Villanueva

Difference of Latitude (𝑫𝒍𝒂𝒕)


It is the numerical difference of latitudes between two positions. In symbol, 𝐷𝑙𝑎𝑡 = 𝐿𝑎𝑡2 − 𝐿𝑎𝑡1.

𝐷𝑙𝑎𝑡 = 𝐿𝑎𝑡2 − 𝐿𝑎𝑡1 General Formula for Difference of Latitude


Where:
Dlat = Difference of Latitude.
Note:
1. If the result of the formula is positive, then the
direction of 𝐷𝑙𝑎𝑡 is North; else if the result is
negative, then the direction is South.
Lat1 = Latitude of departure position. Make the sign
positive if it is located in North; else make the sign
negative if it is located South.
Lat2 = Latitude of arrival position. Make the sign positive if
it is located in North; else make the sign negative if
it is located South.

Confidential to United Marine Training Center, Inc. Page 3-133


Pre Sea Review Materials <course code>
Version Number
Deck
5 November 2018

Lines of Longitude
 Known as meridians and used to measure longitude
 Run in a north–south direction
 Measure distance east or west of the prime meridian
 Halves of great circles
 Lie in planes that pass through the Earth’s axis

Figure 3-58: Lines of longitude – Source: https://kids.britannica.com/students/assembly/view/54584

Longitude
It is an arc of a parallel or the angle at the pole, between the prime meridian and the meridian of a
point measured eastward or westward from the prime meridian through 180° E or W.

Difference of Longitude (𝑫𝒍𝒐)


It is the numerical difference of the longitudes between two positions. In symbol,

𝑫𝒍𝒐 = 𝑳𝒐𝒏𝒈𝟐 − 𝑳𝒐𝒏𝒈𝟏 General Formula for Difference of Longitude


Where:
Dlo = Difference of longitude.
Note:
1. If the absolute value of the result is less than
180° and
is positive, then the direction of 𝐷𝑙𝑜 is East; else if
the sign is negative, then the direction is West.
2. If the absolute value of the result from step 1 is
more than 180°, then add 360° (if negative) or
subtract 360° (if positive). If the result is positive,
then the direction of DLO is East ; else if the
result is negative, then the direction is West.
Long1 = The longitude of the departure position. Make the
sign positive if it is located in East; else make the
sign negative if it is located West.
Long2 = The longitude of the arrival position. Make the
sign positive if it is located in East; else make the
sign negative if it is located West.

Confidential to United Marine Training Center, Inc. Page 3-134


Pre Sea Review Materials <course code>
Version Number
Deck
5 November 2018

Departure (𝑫𝒆𝒑)
It is the east-west distance, in nautical miles, between two meridians along a parallel of latitude.

Distance (𝑫𝒊𝒔𝒕)
It is the length of a line joining two places on the surface of the earth, expressed in nautical miles.

Course (𝑪𝟎 )
It is the angle that the intended path of the ship makes with a fixed reference object (typically true
north).

Plane Sailing

Plane Sailing is a method used to solve problems involving a single course and distance, difference
of latitude, and departure between two positions in which the Earth is regarded as a plane surface.
For a short distance (less than 600 miles) between two positions, the curvature of the earth is
negligible.
This method assumes that the meridian through the point of departure, the parallel through the point
of arrival, and the course line form a 90° − 𝒕𝒓𝒊𝒂𝒏𝒈𝒍𝒆 in a plane, known as the "plane sailing triangle"
as shown in Figure 3-59 below. Hence, right plane trigonometry can be used to derive the plane
sailing formulas.

North

𝐷𝑒𝑝 𝑃2

𝐷𝑙𝑎𝑡 𝐶𝑜

West East
𝑃1

South

Figure 3-59: Plane sailing – Source: Ester Dinawanao

In Figure 3-59 points 𝑷𝟏 and 𝑷𝟐 , represent the departure and arrival positions respectively. The legs
are labelled 𝑫𝒍𝒂𝒕 and 𝑫𝒆𝒑 and the hypotenuse is the distance between the two positions. The
course angle is the angle formed between the North-South line and the distance. In navigation,
course angle refers to the direction an object is actually moving.

Confidential to United Marine Training Center, Inc. Page 3-135


Pre Sea Review Materials <course code>
Version Number
Deck
5 November 2018

Plane sailing formulas derived from the plane sailing triangle are presented below. Note that
trigonometric functions and Pythagorean theorem of a right triangle are the basis for the following
formulas. All formulas make use of the same variables namely:

𝐶𝑜 = Course angle
𝐷𝑒𝑝 = Departure
𝐷𝑙𝑎𝑡 = Difference of latitude
𝐷𝑖𝑠𝑡 = Distance

𝐷𝑒𝑝 Formulas using the Sine Function for Plane Sailing


sin 𝐶𝑜 =
𝐷𝑖𝑠𝑡

𝐷𝑒𝑝 = 𝐷𝑖𝑠𝑡 ∙ sin 𝐶𝑜

𝐷𝑒𝑝
𝐷𝑖𝑠𝑡 =
sin 𝐶𝑜
𝐷𝑙𝑎𝑡 Formulas using the Cosine Function for Plane Sailing
cos 𝐶𝑜 =
𝐷𝑖𝑠𝑡

𝐷𝑙𝑎𝑡 = 𝐷𝑖𝑠𝑡 ∙ cos 𝐶𝑜

𝐷𝑙𝑎𝑡
𝐷𝑖𝑠𝑡 =
cos 𝐶𝑜
𝐷𝑒𝑝 Formulas using the Tangent Function for Plane Sailing
tan 𝐶𝑜 =
𝐷𝑙𝑎𝑡

𝐷𝑒𝑝 = 𝐷𝑙𝑎𝑡 ∙ tan 𝐶𝑜

𝐷𝑒𝑝
𝐷𝑙𝑎𝑡 =
tan 𝐶𝑜

𝐷𝑖𝑠𝑡 2 = 𝐷𝑙𝑎𝑡 2 + 𝐷𝑒𝑝2 Formulas using the Pythagorean Theorem for Plane Sailing

𝐷𝑒𝑝2 = 𝐷𝑖𝑠𝑡 2 − 𝐷𝑙𝑎𝑡 2

𝐷𝑙𝑎𝑡 2 = 𝐷𝑖𝑠𝑡 2 − 𝐷𝑒𝑝2

Confidential to United Marine Training Center, Inc. Page 3-136


Pre Sea Review Materials <course code>
Version Number
Deck
5 November 2018

Figure 3-60 shows projected Plane Sailing Triangle from the globe given 𝑷𝟏 (32°46’S, 178°00’E) and
𝑷𝟐 (32°00’S, 179°50’W).

180° 00’ (IDL)


NORTH

179° 50’ W

179° 00’ W
179° 00’ E
178° 00’ E
179°W

International Date Line (IDL)

178° E

0° (Equator) 32°00’ S
Departure

32°00’ S Dlat 𝐶𝑜

33°00’ S 32°46’ S

33°00’ S

SOUTH
*Not drawn to scale

Figure 3-60: Projected plane sailing triangle from globe – Source: Christian Kevin Villanueva

Figure 3-61 shows the four possible orientations of the Plane Sailing Triangle. It should be noted that
the course, 𝐶𝑜, is always referenced from the North or South line (never on the East or West line).

NORTH

Departure

Departure
Dlat

Dlat

𝐶𝑜 𝐶𝑜
Distance
Distance

Distance
𝐶𝑜 𝐶𝑜
Dlat

Dlat

Distance

Departure
Departure

SOUTH

Figure 3-61: Orientations of the plane sailing triangles – Source: Christian Kevin Villanueva

Confidential to United Marine Training Center, Inc. Page 3-137


Pre Sea Review Materials <course code>
Version Number
Deck
5 November 2018

Example 3-100

Find the course and distance made good by a ship if she made a difference of latitude of 431
nautical miles south and a departure of 132 nautical miles west

Solution
Illustrate the plane sailing triangle and label the given parts.

North

West East

𝐶𝑜 𝐷𝑙𝑎𝑡

𝐷𝑒𝑝

South

Solution for the distance and true course

𝐷𝑖𝑠𝑡 2 = (𝐷. 𝐿𝑎𝑡)2 + 𝐷𝑒𝑝2 By Pythagorean theorem ℎ𝑦𝑝2 = 𝑙𝑒𝑔12 + 𝑙𝑒𝑔22

𝐷𝑖𝑠𝑡 2 = 4312 + 1322 By substitution

𝐷𝑖𝑠𝑡 = √185 761 + 17 424 Simplification

𝐷𝑖𝑠𝑡 = √204 048


𝑫𝒊𝒔𝒕 = 𝟒𝟓𝟏. 𝟕𝟐 𝑵𝑴

𝐷𝑒𝑝 𝑜𝑝𝑝𝑜𝑠𝑖𝑡𝑒
Use 𝑡𝑎𝑛𝜃 = 𝑎𝑑𝑗𝑎𝑐𝑒𝑛𝑡 to solve for the course
𝑡𝑎𝑛 𝐶𝑜 =
𝐷𝑙𝑎𝑡
angle 𝐶𝑜.

132 By substitution
𝑡𝑎𝑛𝐶𝑜 =
431

𝐶𝑜 = 𝑡𝑎𝑛−1 (0.306264501) Apply the definition of inverse tangent function


of both sides
𝐶𝑜 = 170 02′

𝑻. 𝑪𝒐 = 𝟏𝟕𝟎 𝟎𝟐′ + 𝟏𝟖𝟎𝟎 The plane sailing triangle lies in the 3rd
quadrant (S - W).
𝑻. 𝑪𝒐 = 𝟏𝟗𝟕𝟎 𝑻

The distance and course made good by the ship a ship are 𝟒𝟓𝟏. 𝟕𝟐 𝑵𝑴 and 𝟏𝟗𝟕𝟎 𝑻
respectively.

Confidential to United Marine Training Center, Inc. Page 3-138


Pre Sea Review Materials <course code>
Version Number
Deck
5 November 2018

Example 3-101

A ship sails for 3 hours from A at a speed of 18 knots on course 3070 𝑇.


a. Find the departure and difference in latitude for the final position B.
b. If the latitude of A is 2738’N, find the latitude of B.

Solution
Illustrate the plane sailing triangle and label the
North given parts

𝐷𝑒𝑝
Since 𝑡 = 3 ℎ𝑜𝑢𝑟𝑠 and 𝑠 = 18 𝑘𝑛𝑜𝑡𝑠 are given ,
𝐵 then the distance can be calculated by the
distance formula, 𝑑 = 𝑠𝑡
Note:
𝐷𝑙𝑎𝑡
𝐶𝑜 1𝑁𝑀
1 𝑘𝑛𝑜𝑡 =
ℎ𝑟
West East The unit hour is cancelled.
𝐴

South

𝑛𝑚
𝑑 = 𝑠𝑡 = (18) (3 ℎ𝑟) = 54𝑁𝑀
ℎ𝑟

𝐶𝑜 = 3600 − 3070 Get the course angle by subtracting the true


course from 3600 since the plane sailing
𝐶𝑜 = 530
triangle lies in the 4ht quadrant and used this
value to calculate the required parts.
Note:
Course angle is always less than 900 𝐶𝑜 < 900 )
because all the trigonometric functions in the
first quadrant are positive.

𝐷𝐸𝑃 𝑜𝑝𝑝𝑜𝑠𝑖𝑡𝑒
By sine function , 𝑠𝑖𝑛𝜃 = ℎ𝑦𝑝𝑜𝑡𝑒𝑛𝑢𝑠𝑒
𝑠𝑖𝑛𝐶𝑜 =
𝐷𝐼𝑆𝑇
𝐷𝐸𝑃 By substitution
𝑠𝑖𝑛 𝐶𝑜 =
54

𝐷𝐸𝑃 = 54𝑁𝑀 𝑠𝑖𝑛530 By formula transformation

𝑫𝑬𝑷 = 𝟒𝟑. 𝟏𝟑 𝑵𝑴 Simplification

𝐷𝑙𝑎𝑡 By cosine function , 𝑐𝑜𝑠𝜃 =


𝑎𝑑𝑎𝑐𝑒𝑛𝑡
𝑐𝑜𝑠𝐶𝑜 = ℎ𝑦𝑝𝑜𝑡𝑒𝑛𝑢𝑠𝑒
𝐷𝐼𝑆𝑇
𝐷𝑙𝑎𝑡 By substitution
𝑐𝑜𝑠 530 =
54

𝐷𝑙𝑎𝑡 = 54𝑁𝑀 𝑐𝑜𝑠 530 By formula transformation

𝑫𝒍𝒂𝒕 = 𝟑𝟐. 𝟒𝟗𝟖𝟎𝟏𝟏𝟐𝟓 𝑵𝑴 Simplification

Confidential to United Marine Training Center, Inc. Page 3-139


Pre Sea Review Materials <course code>
Version Number
Deck
5 November 2018

𝟏𝟎 The unit for DLAT is degrees, thus convert it


𝑫𝒍𝒂𝒕 = 𝟑𝟐. 𝟒𝟗𝟖𝟎𝟏𝟏𝟐𝟓 𝑵𝑴 ( ) using the conversion factor ,10 = 60𝑁𝑀.
𝟔𝟎 𝑵𝑴
𝑫𝒍𝒂𝒕 = 𝟎𝟎𝟎 𝟑𝟐. 𝟓′ 𝑵 Affix the direction North(N) based on the
orientation of the plane sailing triangle.

𝑳𝒂𝒕 𝒐𝒇 𝑩 = 𝑳𝒂𝒕 𝒐𝒇 𝑨 + 𝑫𝒍𝒂𝒕 By formula transformation;


𝐷𝑙𝑎𝑡 = 𝐿𝑎𝑡 𝑜𝑓 𝐵 − 𝐿𝑎𝑡 𝑜𝑓 𝐴

𝑳𝒂𝒕 𝒐𝒇 𝑩 = 𝟐𝟕𝟎 𝟑𝟖′ 𝑵 + 𝟎𝟎𝟎 𝟑𝟐. 𝟓′ 𝑵 By substitution and simplification


𝑳𝒂𝒕 𝒐𝒇 𝑩 = 𝟐𝟖𝟎 𝟏𝟎. 𝟓′ 𝑵

The departure and difference in latitude are 𝟒𝟑. 𝟏𝟑 𝑵𝑴 and 𝟎𝟎𝟎 𝟑𝟐. 𝟓′ 𝑵 respectively.
The latitude of B is 𝟐𝟖𝟎 𝟏𝟎. 𝟓′ 𝑵 if the latitude of A is 2738’N.

Confidential to United Marine Training Center, Inc. Page 3-140


Pre Sea Review Materials <course code>
Version Number
Deck
5 November 2018

Supplementary Exercises
Solve the following the problems
Answer
1] Find the 𝑫𝒍𝒂𝒕 and 𝑫𝒆𝒑 made good from A to B if the course 𝐷𝑙𝑎𝑡 = 01°57′ 𝑆
is 124° 𝑇 and the distance runs is 210 𝑁𝑀. 𝐷𝑒𝑝 = 174.10 𝑁𝑀

2] A vessel steamed 450 𝑁𝑀 on a course of 295° 𝑇. The


departure point was 35°30’ 𝑁. What latitude will the vessel 38° 40′ 𝑁
arrive at?
Find the 𝑫𝒍𝒂𝒕 and 𝑫𝒆𝒑 made good from “A” to “B” if the 𝐷𝑙𝑎𝑡 = 2° 12′ 𝑁
3]
course is 055° 𝑇 and the distance is 230 𝑛𝑎𝑢𝑡𝑖𝑐𝑎𝑙 𝑚𝑖𝑙𝑒𝑠. 𝐷𝑒𝑝 = 188.40 𝑁𝑀

4] Find the distance travelled and course steered by a ship 𝑇/𝐶𝑜 = 234° 𝑇
that has moved 95 𝑁𝑀 to the south and 130 𝑁𝑀 to the west
of its initial position. 𝐷𝑖𝑠𝑡 = 161.01 𝑁𝑀

5] A ship has steamed 136 𝑁𝑀 north and 203 𝑁𝑀 west. Find 304° 𝑇
the true course.

Confidential to United Marine Training Center, Inc. Page 3-141


Pre Sea Review Materials <course code>
Version Number
Deck
5 November 2018

Parallel Sailing

Parallel Sailing is a method used to find the distance between two positions on the same latitude.
Consider Figure 3-62 which is a mathematical model of the earth. If the globe is viewed at point P
(North pole) , two concentric circles are seen 𝐿𝐿′ 𝑎𝑛𝑑 𝑄𝑄′ which are the parallel of latitude and the
equator respectively as shown in Figure 3-63. Concentric circles are circles with a common center.

Figure 3-62: Terrestrial Sphere – Source: The Sailings by Capt. Udo Moeller

Figure 3-63: Concentric circles – Source: The Sailings by Capt. Udo Moeller

Confidential to United Marine Training Center, Inc. Page 3-142


Pre Sea Review Materials <course code>
Version Number
Deck
5 November 2018

Below is the derivation of the parallel sailing formulas using Figure 3-62 and Figure 3-63.
𝑎𝑟𝑐 𝑎𝑏 = 𝐷𝑒𝑝𝑎𝑟𝑡𝑢𝑟𝑒 Given
𝑎𝑟𝑐 𝐴𝐵 = 𝐷𝑖𝑓𝑓𝑒𝑟𝑒𝑛𝑐𝑒 𝑜𝑓 𝐿𝑜𝑛𝑔𝑖𝑡𝑢𝑑𝑒
𝑐𝑏 = 𝑐𝑎 𝑟𝑎𝑑𝑖𝑖 𝑜𝑓 𝑡ℎ𝑒 𝑝𝑎𝑟𝑎𝑙𝑙𝑒𝑙 𝑜𝑓 𝑙𝑎𝑡𝑖𝑡𝑢𝑑𝑒
𝐶𝐵 = 𝐶𝐴 𝑟𝑎𝑑𝑖𝑖 𝑜𝑓 𝑡ℎ𝑒 𝑒𝑞𝑢𝑎𝑡𝑜𝑟

𝑎𝑟𝑐 𝑎𝑏 𝑐𝑏 Since arcs of concentric circles subtended by


= → 𝑒𝑞. 1 the same angle are proportional to their radii.
𝑎𝑟𝑐 𝐴𝐵 𝐶𝐵
Note:
AB and ab are arcs of concentric circles if
viewed from the pole, their centers C and c
being on the earth’s axis.

𝑎𝑟𝑐 𝑎𝑏 𝐷𝑒𝑝 By definition of Departure (Dep) and Difference


= → 𝑒𝑞. 2 of Longitude (Dlo)
𝑎𝑟𝑐 𝐴𝐵 𝐷𝑙𝑜

𝑐𝑏 𝐷𝑒𝑝 By transitive property which states that if a = b


= → 𝑒𝑞. 3 and b = c, then b = c.
𝐶𝐵 𝐷𝑙𝑜

𝑐𝑏 𝐷𝑒𝑝 By substitution since 𝐶𝐵 = 𝐶𝑏 ( both radii of


= → 𝑒𝑞. 4 the same sphere)
𝐶𝑏 𝐷𝑙𝑜
Refer to Figure 3-62 and 3-63.

Looking at the globe from the side , ∆ 𝐶𝑐𝑏 is


observed to be a plane right triangle .
From the figure,
< 𝐵𝐶𝑏 = 𝐿𝑎𝑡 by definition of latitude
< 𝐶𝑏𝑐 =< 𝐵𝐶𝑏 = 𝑙𝑎𝑡 by alternate interior
angles

𝑐𝑏 Since ∆ 𝐶𝑐𝑏 is a plane right triangle with radius


= 𝑐𝑜𝑠 𝐿𝑎𝑡 → 𝑒𝑞. 5 𝑐𝑏 and radius 𝐶𝑏 as the opposite side of
𝐶𝑏
(latitude) and hypotenuse respectively.

Since By transitive property


𝑐𝑏 𝐷𝑒𝑝
= → 𝑒𝑞. 4
𝐶𝑏 𝐷𝑙𝑜
and
𝑐𝑏
= 𝑐𝑜𝑠 𝐿𝑎𝑡 → 𝑒𝑞. 5
𝐶𝑏
then
𝐷𝑒𝑝
= 𝑐𝑜𝑠 𝐿𝑎𝑡 𝑒𝑞. 6
𝐷𝑙𝑜

Confidential to United Marine Training Center, Inc. Page 3-143


Pre Sea Review Materials <course code>
Version Number
Deck
5 November 2018

𝐷𝑒𝑝 = 𝐷𝑙𝑜 ∗ 𝑐𝑜𝑠 𝐿𝑎𝑡 𝑒𝑞. 7 By formula transformation


𝐷𝑒𝑝
𝐷𝑙𝑜 = 𝑒𝑞. 8
𝑐𝑜𝑠 𝐿𝑎𝑡

The following are the parallel sailing formulas:


𝐷𝑒𝑝
= 𝑐𝑜𝑠 𝐿𝑎𝑡
𝐷𝑙𝑜
𝐷𝑒𝑝
𝐷𝑙𝑜 =
𝑐𝑜𝑠 𝐿𝑎𝑡
𝐷𝑒𝑝 = 𝐷𝐿𝑜 𝑐𝑜𝑠 𝐿𝑎𝑡

Example 3-102

How far must a ship sail due West on the parallel of 30° to change her longitude 2°?
Solution
Illustrate the problem based on the given;
𝐿𝑎𝑡𝑖𝑡𝑢𝑑𝑒 = 30° 𝑁 𝑜𝑟 𝑆
𝐷𝐿𝑂 = 2° 𝑊

Required: 𝐷𝑒𝑝𝑎𝑟𝑡𝑢𝑟𝑒 (𝐷𝑒𝑝) =?

Dep = Dlo ∙ cos 𝐿𝑎𝑡 Since latitude and 𝐷𝑙𝑜 are given and departure
is required
60 𝑛𝑚 Substitute the given values to the formula and
Dep = 20 ( ) ∙ cos 300
10 convert 𝐷𝑙𝑜 to nautical mile using the
conversion factor, 10 = 60 𝑛𝑚.

𝐃𝐞𝐩 = 𝟏𝟎𝟑. 𝟗𝟐 𝐧𝐦 Simplification

Confidential to United Marine Training Center, Inc. Page 3-144


Pre Sea Review Materials <course code>
Version Number
Deck
5 November 2018

Example 3-103

On a certain parallel of latitude, the distance between two meridians is 150 NM. On the equator, the
distance between the same two meridians is 235 NM. Find the latitude of the parallel of latitude.
Solution
Illustrate the problem based on the given;
𝐷𝑒𝑝 at a certain parallel of latitude = 150 𝑁𝑀
𝐷𝑙𝑜 at the equator = 235 𝑁𝑀

Required:
Latitude at a certain parallel of latitude =?

𝐷𝑒𝑝 Since 𝐷𝑒𝑝𝑡 and 𝐷𝑙𝑜 are given and Latitude is


cos 𝐿𝑎𝑡 =
𝐷𝑙𝑜 required
150 𝑁𝑀 Substitute the given values to the formula
cos 𝐿𝑎𝑡 =
235 𝑁𝑀 Cancel common unit
−1
𝐿𝑎𝑡 = cos (0.638297872) By inverse cosine function
𝑳𝒂𝒕 = 𝟓𝟎𝟎 𝟐𝟎′ 𝑵 𝒐𝒓 𝑺 Simplification

Example 3-104

Find the distance between initial positions 50N 163 W and 50 N 179 E.
Solution
Illustrate the problem based on the given
positions:
50𝑁 163 𝑊
50 𝑁 179 𝐸

Required: 𝐷𝑒𝑝𝑎𝑟𝑡𝑢𝑟𝑒 (𝐷𝑒𝑝) = ?

𝐷𝑙𝑜 = 𝐿𝑜𝑛𝑔2 − 𝐿𝑜𝑛𝑔1 Formula for difference of longitudes

𝐷𝑙𝑜 = 179°𝐸 − 163°𝑊 Substitute the given longitudes to the formula

𝐷𝑙𝑜 = 179° − (−163°) Apply the signs for East (+) and West(-)

𝐷𝑙𝑜 = 342° 𝐸 Simplification

Confidential to United Marine Training Center, Inc. Page 3-145


Pre Sea Review Materials <course code>
Version Number
Deck
5 November 2018

𝐷𝑙𝑜 = 342°− 3600 = −180 Since 𝐷𝑙𝑜 is greater than 1800 , subtract 360°
from it to make use of the shorter distance in
𝑫𝒍𝒐 = 𝟏𝟖° 𝑾
the Great Circle

𝐷𝑒𝑝 = 𝐷𝑙𝑜 ∙ cos 𝐿𝑎𝑡 Use parallel sailing formula to calculate the
departure since the positions have the same
latitudes

60 𝑁𝑀 Substitute the given values to the formula


𝐷𝑒𝑝 = (18°) ( ) cos 50°

𝑫𝒆𝒑 = 𝟔𝟗𝟒. 𝟐𝟏 𝑵𝑴 Simplification

Confidential to United Marine Training Center, Inc. Page 3-146


Pre Sea Review Materials <course code>
Version Number
Deck
5 November 2018

Supplementary Exercises
Solve the following problems
Answer
1] By sailing due east for a distance of 245 𝑁𝑀, a vessel
change her longitude by 7° 25’. What latitude was the 56° 36′ 𝑁 𝑜𝑟 𝑆
vessel in?
2] A vessel in initial position 60° 𝑁 steams due east for
480 𝑁𝑀
240 𝑁𝑀. Calculate the difference in longitude.

3] A vessel steams 090° 𝑇 from position


(41° 20′ 𝑆, 35° 25′ 𝑊). Find the arrival position if she 28° 53.5′ 𝑊
steams for 294 𝑁𝑀.

4] In what latitude will a difference of longitude of 3°40′


56° 56.7′ 𝑁 𝑜𝑟 𝑆
correspond to a departure of 120 𝑁𝑀.

5] Find the distance to steam between the two positions:


(51° 20′ 𝑁, 48° 30′ 𝑊) 387.4 𝑁𝑀
(51° 20′ 𝑁, 38° 10′ 𝑊)

Confidential to United Marine Training Center, Inc. Page 3-147


Pre Sea Review Materials <course code>
Version Number
Deck
5 November 2018

Mercator Sailing

The disadvantage of Plane Sailing is not considering the Globe’s curvature; thus, the distance and
course calculated using this method contains errors that are tolerable only up to 600 nautical miles.
Thus, for distances more than 600 NM, another method known as Mercator Sailing, is used.
The Mercator Sailing is the most accurate method employed in marine navigation to handle
calculations related to Rhumb Line sailing.
It is used to find the course and distance between two positions that are in different latitudes but
instead of using the actual Difference of Latitude in arc minutes or nautical miles as measured from
the Globe, it makes use of the Difference of Meridional Parts (DMP). A Meridional Part, MP, is
described as the adjusted length of the latitude after it was projected from the globe to a two-
dimensional plane. Figure 3-64 shows the length of the latitude in arc min or actual nautical miles
compared to its equivalent Meridional Part.

Figure 3-64: Projection of Points A and B to Points D and E – Source: Christian Kevin Villanueva

Arc AB and Arc BC both subtends a central angle of 30° which means that the actual nautical miles
between A and B and B and C are the same. After the projection of these points to a rolled plane
(cylinder) via a Tangent Line, the length of Arc AB is stretched to the length of segment ED and the
length of Arc BC is stretched to the length of segment EC. The projection creates a distorting effect
which is more evident as the arc length moves farther from the equator (The green islands on the
globe are of the same land area but the island farther to the equator seems to be larger after the
projection). The distortion or the stretch length is called the Meridional Part.
The Meridional Parts for any latitude is the increased length of the meridians on a Mercator chart
measured from the equator to the parallel and expressed in minutes of the Longitude scale. Since this
Meridional Part is already projected in the rolled plane (cylinder), the use of plane trigonometry to
calculate the distance and course becomes more accurate.
The amount of stretch or the Meridional Part of a certain latitude is given by
𝐿𝑎𝑡 Formula Name for Meridional Part
𝑀𝑃 = 7915.704 log(tan ( ) + 45) − 23.3 sin(𝐿𝑎𝑡)
2
Where:
MP = The Meridional Part
Lat = The latitude of the given
point

Confidential to United Marine Training Center, Inc. Page 3-148


Pre Sea Review Materials <course code>
Version Number
Deck
5 November 2018

Difference Of Meridional Parts (𝑫𝑴𝑷)


The difference of meridional parts (𝐷𝑀𝑃) between any two latitudes is the length of a meridian on a
Mercator Chart between the two parallels expressed in units of the longitude scale. It is calculated in
the same way as the Difference in Latitude. It is the difference from the second Meridional Part (MP2)
to the first Meridional Part (MP1) where North is considered as a positive value and South as a
negative value. Mathematically calculates as

𝐷𝑀𝑃 = 𝑀𝑃2 − 𝑀𝑃1 Formula Name for Difference of Meridional Parts


Where:
DMP = Difference of Meridional Parts
MP1 = Meridional Part of Departure Point
MP2 = Meridional Part of Arrival Point

The 𝑀𝑃 and 𝐷𝑀𝑃 are considered North or going North if positive (+) and South or going South if
negative (-).
By inspection, the 𝐷𝑀𝑃 can also be identified as the difference or sum of the Meridional Parts
depending on which hemisphere they lie.
Figure 3-65 shows the cases in which the Meridional Parts are added or subtracted.
Considering A as the departure point and B as the arrival point, (a) shows that the distance between
them is the difference between the 𝑀𝑃1 and 𝑀𝑃2 and is going to the South direction while (b) shows
that their distance apart is 𝑀𝑃1 plus 𝑀𝑃2 and is going to the North direction.

Figure 3-65: Meridional parts - Source: Christian-Kevin Villanueva

The calculation of the 𝐷𝑀𝑃 is one step to completing the right triangle model for Mercator Sailing. This
right triangle, known as the 𝐷𝑀𝑃 Triangle, has its legs as the 𝐷𝑀𝑃 and 𝐷𝐿𝑂𝑁𝑀 .
Figure 3-66 shows how the 𝐷𝐿𝑂𝑁𝑀 is projected to complete the 𝐷𝑀𝑃 triangle. The length of 𝐷𝐿𝑂𝑁𝑀
after being projected is kept constant on whatever parallel of latitude.

Confidential to United Marine Training Center, Inc. Page 3-149


Pre Sea Review Materials <course code>
Version Number
Deck
5 November 2018

Figure 3-66: The DMP Triangle after projecting the DMP and DLO NM from the globe

Figure 3-67 represents the complete Mercator Sailing Triangle. By applying right triangle trigonometry
principles, the course and distance between the two positions can be determined.

Figure 3-67: Mercator Sailing Triangle – Source: Christian-Kevin Villanueva

As you can see, the figure can be broken down into two right triangles as shown in Figure 3-68.

Confidential to United Marine Training Center, Inc. Page 3-150


Pre Sea Review Materials <course code>
Version Number
Deck
5 November 2018

Figure 3-68: Breakdown of the Mercator sailing triangle - Source: Christian-Kevin Villanueva

To find the course, consider the right triangles that involves the 𝐷𝑀𝑃 and 𝐷𝐿𝑂𝑁𝑀 in Figure 3-68 as the
legs since these are one measured to scale.
𝐷𝑙𝑜𝑁𝑀 Formula for the Course using Mercator Sailing
tan 𝐶𝑜 =
𝐷𝑀𝑃
Where:
Co = Course Angle
Note:
1. The course angle is named depending on the
orientation of the DMP triangle
DloNM = The Difference of longitude converted to nautical
miles
DMP = Difference of meridional parts

To find the distance, consider the right triangle in Figure 3-68 that involves the difference of latitude
converted in nautical miles 𝐷𝑙𝑎𝑡𝑁𝑀
𝐷𝑙𝑎𝑡𝑁𝑀 Formula for the Distance using Mercator Sailing
𝐷𝑖𝑠𝑡 =
cos 𝐶𝑜
Where:
Dist = Distance
DlatNM = Difference of latitude converted to nautical miles
Co = Course Angle

Confidential to United Marine Training Center, Inc. Page 3-151


Pre Sea Review Materials <course code>
Version Number
Deck
5 November 2018

Example 3-105

Solve the distance and the course using Mercator Sailing for the following departure and arrival
positions.
Departure Position: (32°46’S, 178°00’E) and Arrival Position: (32°00’S, 179°50’W)

Solution
𝐷𝑙𝑜 = 𝐿𝑜𝑛𝑔2 − 𝐿𝑜𝑛𝑔1 The mathematical formula for 𝐷𝑙𝑜

𝐷𝑙𝑜 = 1790 50′ 𝑊 − 1780 00′ 𝐸 Substitute the values of the


longitudes

𝐷𝑙𝑜 = (−1790 50′ ) − (1780 00′ ) Apply sign conventions:


West(-) East (+)

𝐷𝑙𝑜 = (−1790 50′ ) + (−1780 00′ ) Apply the rule of subtraction of


𝐷𝑙𝑜 = −357 50 0 ′ integers;
Change the sign of the subtrahend
and proceed to addition

𝐷𝑙𝑜 = −3570 50′ + 3600 Add 3600 since the result is more
𝐷𝑙𝑜 = 20 10′ than 1800 and to be able to get the
smaller arc between the two positions

𝐷𝑙𝑜 = 20 10′ 𝐸 By sign convention ; East (+)

60 𝑁𝑀 1𝑁𝑀 Convert the Dlo to NM using the


𝐷𝑙𝑜 = 20 ( ) + 10′ ( ′ )
10 1 conversion factors,
𝐷𝑙𝑜 = 120 𝑁𝑀 + 10 𝑁𝑀 10 = 60 𝑁𝑀 1′ = 1 𝑁𝑀
𝐷𝑙𝑜 = 130 𝑁𝑀

𝐷𝑙𝑎𝑡 = 𝐿𝑎𝑡2 − 𝐿𝑎𝑡1 The mathematical formula for DLAT

𝐷𝑙𝑎𝑡 = 320 00′ 𝑆 − 320 46′ 𝑆 Substitute the values of the latitudes

𝐷𝑙𝑎𝑡 = (−320 00′ ) − (−320 46′ ) Apply sign conventions:


South (-) North (+)

𝐷𝑙𝑎𝑡 = (−320 00′ ) + (320 46′ ) Apply the rule of subtraction of


0
𝐷𝑙𝑎𝑡 = 00 46 ′ integers;
Change the sign of the subtrahend
and proceed to addition

𝐷𝑙𝑎𝑡 = 000 46′ 𝑁 By sign convention ; North (+)

1𝑁𝑀 Convert the DLAT to NM using the


𝐷𝑙𝑎𝑡 = 000 + 46′ ( )
1′ conversion factor, 𝟏′ = 𝟏 𝑵𝑴
𝐷𝑙𝑎𝑡 = 46 𝑁𝑀

Confidential to United Marine Training Center, Inc. Page 3-152


Pre Sea Review Materials <course code>
Version Number
Deck
5 November 2018

The corresponding DMP triangle


since the directions for DLAT is North
NORTH and the DLO is east.

DLONM

DMP
D. Lat.
𝐶𝑜

SOUTH

𝑙𝑎𝑡 Formula for Meridional Part


𝑀𝑃 = 7915.704 𝑙𝑜𝑔(𝑡𝑎𝑛 + 45) − 23.3 𝑠𝑖𝑛 𝑙𝑎𝑡
2

32°46′ Substitute the Departure Latitude to


𝑀𝑃1 = 7915.704 𝑙𝑜𝑔(𝑡𝑎𝑛 + 45) − 23.3 𝑠𝑖𝑛 32°46′
2 determine its equivalent Meridional
Part (MP)

𝑀𝑃1 = 2,070.245 𝑁𝑀 𝑆𝑜𝑢𝑡ℎ Simplification


This is the equivalent meridional part
of the latitude 32°46.
The direction of MP1 follows the
direction of departure latitude

32° Determine the equivalent Meridional


𝑀𝑃2 = 7915.704 𝑙𝑜𝑔(𝑡𝑎𝑛 + 45) − 23.3 𝑠𝑖𝑛 32°
2 Part (MP) of the Arrival Latitude

𝑀𝑃2 = 2,016.037 𝑁𝑀 𝑆𝑜𝑢𝑡ℎ Simplification


This is the equivalent meridional part
of the latitude 32°.
The direction of 𝑀𝑃2 follows the
direction of arrival latitude

𝐷𝑀𝑃 = 𝑀𝑃2 − 𝑀𝑃1 General formula for the Difference of


Meridional Part
DMP = MP of Arrival minus MP of
Departure

Substitute MP1 and MP2 and apply


𝐷𝑀𝑃 = 2,016.037 𝑁𝑀 𝑆𝑜𝑢𝑡ℎ − 2,070.245 𝑁𝑀 𝑆𝑜𝑢𝑡ℎ sign convention of South (-)
𝐷𝑀𝑃 = −2,016.037 𝑁𝑀 − (−2,070.245 𝑁𝑀)

Confidential to United Marine Training Center, Inc. Page 3-153


Pre Sea Review Materials <course code>
Version Number
Deck
5 November 2018

𝐷𝑀𝑃 = (−2,016.037 𝑁𝑀 ) + (2,070.245 𝑁𝑀) By subtraction of integer


𝐷𝑀𝑃 = 54.208 𝑁𝑀

𝐷𝑀𝑃 = 54.208 𝑁𝑀 𝑁 Simplify and applying the sign


convention North (+)

𝐷𝐿𝑂𝑁𝑀 Formula to solve the course angle


𝑡𝑎𝑛 𝐶𝑜 =
𝐷𝑀𝑃

130𝑁𝑀 Substitute DLO and DMP


𝑡𝑎𝑛 𝐶𝑜 =
54.208 𝑁𝑀

𝐶𝑜 = 𝑡𝑎𝑛 −1 2.398170012 Simplification


0 ′
𝐶𝑜 = 67 22

𝐶𝑜 = 𝑁 670 22′ 𝐸 By considering the orientation of the


𝐶𝑜 = 067 𝑇 0 Mercator Sailing Triangle and
reporting using quadrantal notation
and 3 digit notation

𝐷. 𝐿𝑎𝑡.𝑁𝑀 Formula for distance using the


𝐷𝑖𝑠𝑡 =
𝑐𝑜𝑠 𝐶𝑜 Mercator Sailing Triangle

46 𝑁𝑀 Substitute the Course angle and


𝐷𝑖𝑠𝑡 =
𝑐𝑜𝑠 670 22′ DLAT to solve the distance

𝐷𝑖𝑠𝑡 = 119.53 𝑁𝑀 Simplification

The distance and course for the given positions are 𝟏𝟏𝟗. 𝟓𝟑 𝑵𝑴 and 𝟎𝟔𝟕𝟎 𝑻 using the
Mercator Sailing.

Example 3-106

A vessel at position (18°54’N, 054°E) heads for destination at position (13°12’N, 073°E). Determine
the true course and distance by Mercator Sailing.
Solution
Given: Required:
Departure Position (18°54’N, 054°E) True Course
Arrival Position (13°12’N, 073°E) Distance

𝐷𝑙𝑎𝑡 = 𝐿𝑎𝑡2 − 𝐿𝑎𝑡1 Solve for DLAT using the mathematical


0 ′ 0 ′
formula
𝐷𝑙𝑎𝑡 = 13 12 𝑁 − 18 54 𝑁
Substitute the latitudes

𝐷𝑙𝑎𝑡 = 130 12′ − 180 54′ Apply sign conventions and rule of
0 ′ 0 ′
subtraction of integer.
𝐷𝑙𝑎𝑡 = 13 12 + (−18 54 )
South (-) North (+)
𝐷𝑙𝑜 = −50 42′

𝐷𝑙𝑎𝑡 = 050 42′ 𝑆 Note: Sign Convention South (-)

Confidential to United Marine Training Center, Inc. Page 3-154


Pre Sea Review Materials <course code>
Version Number
Deck
5 November 2018

The sign of the DLAT is negative which


means that the ship is going South.

60 𝑁𝑀 1 𝑁𝑀 Convert the DLAT to NM using the


𝐷𝑙𝑎𝑡 = 050 ( ) + 42′ ( ′ )
1 0 1 conversion factors,
𝐷𝑙𝑎𝑡 = 300 𝑁𝑀 + 42 𝑁𝑀 10 = 60 𝑁𝑀
𝐷𝑙𝑎𝑡 = 342 𝑁𝑀 𝑆 1′ = 1 𝑁𝑀

𝐷𝑙𝑜 = 𝐿𝑜𝑛𝑔2 − 𝐿𝑜𝑛𝑔1 Solve for 𝐷𝑙𝑜 using the mathematical


formula

𝐷𝑙𝑜 = 0730 𝐸 − 0540 𝐸 Substitute the longitudes

𝐷𝑙𝑜 = 0730 − 0540 = 0190 Apply sign conventions and rule of


0
subtraction of integer
𝐷𝑙𝑜 = 019 𝐸
West (-) East (+)
The sign of the DLO is positive which
means that the ship is going East .

60 𝑁𝑀 Conversion to nautical miles


𝐷𝑙𝑜 = 190 ( )
10
𝑫𝒍𝒐 = 𝟏𝟏𝟒𝟎 𝑵𝑴 𝑬

Since the directions for 𝐷𝑙𝑎𝑡 is South and


the 𝐷𝑙𝑜 is east
NORTH

𝐶𝑜
DMP

𝐷𝑒𝑝
D. Lat.

DLONM
SOUTH

𝑙𝑎𝑡 Formula for Meridional Part


𝑀𝑃 = 7915.704 log(tan + 45) − 23.3 sin 𝑙𝑎𝑡
2

18°54′ Substitution of Departure Latitude


𝑀𝑃1 = 7915.704 log(tan + 45) − 23.3 sin 18°54′
2

𝑀𝑃1 = 1147.6 𝑁𝑀 𝑁 Simplification


This is the equivalent meridional part of the
latitude 18°54′ 𝑁.
The direction of 𝑀𝑃1 follows the direction of
departure latitude.

13°12′ Substitution of Arrival Latitude


𝑀𝑃2 = 7915.704 log(tan + 45) − 23.3 sin 13°12′
2

𝑀𝑃2 = 793.8 𝑁𝑀 𝑁 Simplification

Confidential to United Marine Training Center, Inc. Page 3-155


Pre Sea Review Materials <course code>
Version Number
Deck
5 November 2018

This is the equivalent meridional part of the


latitude 13°12′ 𝑁.
The direction of 𝑀𝑃2 follows the direction of
arrival latitude.

General formula for the Difference of


Meridional Part (Meridional Part of Arrival
𝐷𝑀𝑃 = 𝑀𝑃2 − 𝑀1 minus Meridional Part of Departure)

𝐷𝑀𝑃 = 793.8𝑁𝑀 𝑁 − 1147.6𝑁𝑀 𝑁 Substitute MP1 and MP2 and


𝐷𝑀𝑃 = (793.8𝑁𝑀) − (1147.6𝑁𝑀) apply sign convention of
North (+)

𝐷𝑀𝑃 = 793.8𝑁𝑀 + (−1147.6𝑁𝑀) By subtraction of integer


𝐷𝑀𝑃 = −353.8𝑁𝑀

𝐷𝑀𝑃 = 353.8𝑁𝑀 𝑆 Simplify and applying the sign convention


South (-)
Note: DMP has same direction with the 𝐷𝑙𝑎𝑡

𝐷𝐿𝑂𝑁𝑀 Formula to solve the course angle


tan 𝐶𝑜 =
𝐷𝑀𝑃

1140𝑁𝑀 Substitute 𝐷𝑙𝑜 and 𝐷𝑀𝑃


tan 𝐶𝑜 =
353.8 𝑁𝑀

𝐶𝑜 = tan−1(3.222159412) Simplification
𝐶𝑜 = 720 46′

𝐶𝑜 = 𝑆 720 46′ 𝐸 By considering the orientation of the


Mercator Sailing Triangle and reporting
𝐶𝑜 = 1070 𝑇 using quadrantal notation and 3 digit
notation

𝐷. 𝐿𝑎𝑡.𝑁𝑀 Formula for distance using the Mercator


𝐷𝑖𝑠𝑡 =
cos 𝐶𝑜 Sailing Triangle

342 Substitute the Course angle and 𝐷𝑙𝑎𝑡 to


𝐷𝑖𝑠𝑡 =
𝑐𝑜𝑠720 46′ solve the distance

𝐷𝑖𝑠𝑡 = 1154.38 𝑁𝑀 Simplification

The distance and course for the given positions are 𝟏𝟏𝟓𝟒. 𝟑𝟖 NM and 𝟏𝟎𝟕° 𝑻 using the
Mercator Sailing.

Confidential to United Marine Training Center, Inc. Page 3-156


Pre Sea Review Materials <course code>
Version Number
Deck
5 November 2018

Supplementary Exercises
Solve the following problems
Answer
1] A vessel at 𝑃1 (18° 54′ 𝑁, 73°𝐸) heads for a destination at 𝑇/𝐶0 = 253° 𝑇
𝑃2 (13°12′ 𝑁, 54° 𝐸). Determine the true course and distance
by Mercator sailing. 𝑑𝑖𝑠𝑡 = 1153.9 𝑁𝑀

2] By Mercator sailing, find the true course and distance 𝑇/𝐶0 = 205° 𝑇
between the positions 𝑃1 (49°10′ 𝑁, 12° 30′ 𝑊) and
𝑃2 (25°15′ 𝑁, 26°50′ 𝑊). 𝑑𝑖𝑠𝑡 = 1586.7 𝑁𝑀

3] Determine the true course and distance between 𝑇/𝐶0 = 106° 𝑇


𝐴(20° 24′ 𝑆, 57° 26′ 𝐸) and 𝐵 (34° 10′ 𝑆, 112° 28′ 𝐸) by
Mercator Sailing. 𝑑𝑖𝑠𝑡 = 3053.4 𝑁𝑀

4] Using Mercator sailing, find the course and distance


between: 𝑇/𝐶0 = 224° 𝑇
𝑃1 (42° 45′ 𝑁, 38° 20′ 𝑊) 𝑑𝑖𝑠𝑡 = 1268.13 𝑁𝑀
𝑃2 (27° 30′ 𝑁, 56° 15′ 𝑊)

5] By Mercator sailing, find the course and distance between:


𝑇/𝐶0 = 074° 𝑇
𝑃1 (47° 14.3′ 𝑆, 167° 26′ 𝐸)
𝑑𝑖𝑠𝑡 = 702 𝑁𝑀
𝑃2 (43° 55′ 𝑆, 176° 34′ 𝑊)

Confidential to United Marine Training Center, Inc. Page 3-157


Pre Sea Review Materials <course code>
Version Number
Deck
5 November 2018

Spherical Trigonometry and its Maritime


Application

Spherical Trigonometry is the science of trigonometry applied to the triangles marked on the surface
of a sphere and the relationships between the sides and the angles. The development of this subject
lead to improvements in the art of navigation, geographic map making, the positions of sunrise and
sunset, and improvements to the sundial.

The Sphere
A sphere is the set of all points in three-dimensional space that are equidistant from a fixed point
called center. Figure 3-69 show the basic parts and properties of a sphere.

Figure 3-69: Parts of the sphere - Source: https://www.mathalino.com/reviewer/solid-mensuration-solid-geometry/sphere and


http://www.vikdhillon.staff.shef.ac.uk/teaching/phy105/celsphere/phy105_earth.html

 Radius is the distance from the center to any point of the sphere. It is half of the diameter of
the sphere.
 Diameter is the distance between two points of a sphere passing through its center.
 Every section in the sphere made by a cutting plane is a circle. If the cutting plane passes
through the center of the sphere, the section made is a great circle; otherwise the section is
a small circle.
 Great Circle is the largest circle on the surface of a sphere whose plane passes through its
center. It has the largest area. The circumference of a sphere is the circumference of a great
circle. Every great circle divides the sphere into two equal parts called hemispheres. All
meridians and the equator are great circles.
 A small circle is one whose plane does not pass through the center of the sphere. A
parallel of latitude is a small circle.
 The spherical distance between two endpoints on a sphere is the length of the shorter arc
of the great circle containing the points.
 Axis of a Great Circle is the diameter of the sphere perpendicular to the plane of that
circle.
 The ends of the axis of the circle of a sphere are called poles. The pole of a great circle is
a point, on the surface of the sphere, which is equidistant, and hence ninety degrees away
from, all parts of the circumference of the great circle.

Confidential to United Marine Training Center, Inc. Page 3-158


Pre Sea Review Materials <course code>
Version Number
Deck
5 November 2018

Spherical Triangle

Figure 3-70: Formation of the spherical triangle – Source: https://www.kisspng.com/png-spherical-trigonometry-solution-of-


triangles-spher-1024715/

 A spherical triangle is a triangle formed on the surface of a sphere by the intersection of the
arcs of three great circles, for example, ∆𝑪𝑩𝑨.
 The sides of a spherical triangles are curved and not a straight line. Thus, the sides like the
angles, are expressed in degrees, minutes and seconds of arc.
 The length of side of a spherical triangle is defined as the angle subtended by that side at
the center of the sphere.
 Spherical angle is formed by the intersection of two great-circle arcs and the point of
intersection of the arcs is the vertex of the spherical angle. In Figure 3-70, the angles are A
or , B or , and C while the sides are represented by 𝒂𝒓𝒄 𝑨𝑪, 𝒂𝒓𝒄 𝑨𝑩, 𝒂𝒏𝒅 𝑨𝒓𝒄 𝑩𝑪 or by
small letters .
 In any spherical triangle, the greater angle is opposite to the greater side, and the converse
is also true, i.e., the greater side is opposite to the greater angle

Limitations on Sides and Angles of a Spherical Triangle

Figure 3-71: Sides and angles of a spherical triangle – Source: http://1.bp.blogspot.com/-


REiyDx5332U/UYlh9G6oRNI/AAAAAAAATZ4/WfIL20n-bdY/s1600/picture2.jpg

Confidential to United Marine Training Center, Inc. Page 3-159


Pre Sea Review Materials <course code>
Version Number
Deck
5 November 2018

Limitations on Sides
 Each side of a spherical triangle must be less than 1800 . In symbol,
𝒂 < 𝟏𝟖𝟎𝟎 , 𝒃 < 𝟏𝟖𝟎𝟎 , 𝒂𝒏𝒅 𝒄 < 𝟏𝟖𝟎𝟎
 The sum of the three sides must lie between 00 to 3600 . In symbol,
𝟎𝟎 < (𝒂 + 𝒃 + 𝒄) < 𝟑𝟔𝟎𝟎
 The sum of any two sides of spherical triangle is greater than the third side. In symbol,
(𝒂 + 𝒃) > 𝒄 , (𝒂 + 𝒄) > 𝒃 , (𝒃 + 𝒄) > 𝒂

Limitations on Angles
 Each angle of a spherical triangle must be less than1800 . In symbol, 𝑨 < 𝟏𝟖𝟎𝟎 , 𝑩 <
𝟏𝟖𝟎𝟎 , 𝒂𝒏𝒅 𝑪 < 𝟏𝟖𝟎𝟎
 The sum of the three angles must lie between 1800 to 5400 . In symbol, 𝟏𝟖𝟎𝟎 <
(𝑨 + 𝑩 + 𝑪) < 𝟓𝟒𝟎𝟎
 The sum of two angles of a spherical triangle is less than 180◦ plus the third angle. In
symbol, (𝐴 + 𝐵) < 1800 + 𝐶 , (𝐴 + 𝐶) < 1800 + 𝐵 , (𝐵 + 𝐶) < 1800 + 𝐴

Example 3-107

Is the spherical triangle with side 𝑎 = 1600 , 𝑏 = 1000 , 𝑎𝑛𝑑 𝑐 = 900 possible to construct or not?
Why?
Solution
𝑎 = 1600 , 𝑏 = 1000 , 𝑎𝑛𝑑 𝑐 = 900 Since the given parts are sides of the spherical
triangle, consult the limitations on sides

𝑎 < 1800 ? = 1600 < 1800 ? Yes Limitation on sides (a) has been satisfied since
each side of a spherical triangle is less
𝑏 < 1800 ? = 1000 < 1800 ? Yes
than1800 .
𝑐 < 1800 ? = 900 < 1800 ? Yes

𝟎𝟎 < (𝒂 + 𝒃 + 𝒄) < 𝟑𝟔𝟎𝟎 The sum of the three sides, 3500 lies between
00 and 3600 .Thus, it satisfies limitation on sides
00 < 1600 + 1000 + 900 < 3600 ?
(b).
00 < 𝟑𝟓𝟎𝟎 < 3600 ? Yes

(𝑎 + 𝑏) > 𝑐 ? It satisfies the limitation on sides (c) which


0 0 0 states that “the sum of any two sides of
(160 + 100 ) > 90 ? spherical triangle is greater than the third side”.
2600 > 900 ? Yes

Since all the limitation on sides are fully


(𝑎 + 𝑐) > 𝑏? satisfied, then, it is possible to construct such
(1600 + 900 ) > 1000 ? spherical triangle with sides
2500 > 1000 ? Yes 𝑎 = 1600
𝑏 = 1000 , 𝑎𝑛𝑑
(𝑏 + 𝑐) > 𝑎? 𝑐 = 900 .
(1000 + 900 ) > 1600 ?
1900 > 1600 ? Yes

Confidential to United Marine Training Center, Inc. Page 3-160


Pre Sea Review Materials <course code>
Version Number
Deck
5 November 2018

Example 3-108

Is it possible to construct a spherical triangle ABC with the following angles? 𝐴 = 500 , 𝐵 =
850 , and 𝐶 = 1300
Solution
𝐴 = 500 , 𝐵 = 850 , 𝑎𝑛𝑑 𝐶 = 1300 ? Since the given parts are angles of the spherical
triangle, use the limitations on angles.

𝐴 < 1800 ? = 500 < 1800 ? Yes Limitation on angles (a) has been satisfied since
each angle of a spherical triangle is less
𝐵 < 1800 ? = 850 < 1800 ? Yes
than 1800 .
𝐶 < 1800 ? = 1300 < 1800 ? No

𝟏𝟖𝟎𝟎 < (𝑨 + 𝑩 + 𝑪) < 𝟓𝟒𝟎𝟎 The sum of the three angles, 2650 lies between
1800 and 5400 .
1800 < (500 + 850 + 1300 ) < 5400 ?
It satisfies the limitations on angles (b).
1800 < 𝟐𝟔𝟓𝟎 < 5400 ? Yes

(𝐴 + 𝐵) < 1800 + 𝐶 ? It satisfies the third limitation on angles which


states that “the sum of two angles of a spherical
(500 + 850 ) < 1800 + 1300 ?
triangle is less than 1800 plus the third angle”.
1350 < 3100 ? Yes

Since all limitations on angles are fully satisfied,


(𝐴 + 𝐶) < 1800 + 𝐵 ? therefore, it is possible to construct the given
spherical triangle.
(500 + 1300 ) < 1800 + 850 ?
1800 < 2650 ? Yes

(𝐵 + 𝐶) < 1800 + 𝐴 ?
850 + 1300 ) < 1800 + 500 ?
2150 < 2300 ? Yes

Confidential to United Marine Training Center, Inc. Page 3-161


Pre Sea Review Materials <course code>
Version Number
Deck
5 November 2018

Example 3-109

The parts given below are those of a spherical triangle ABC, state whether the spherical triangle is
possible or not, and give your reason(s).
𝒂 = 𝟏𝟔𝟏𝟎 , 𝒃 = 𝟏𝟖𝟑𝟎 , 𝒂𝒏𝒅 𝒄 = 𝟏𝟑𝟗𝟎

Solution
𝒂 = 𝟏𝟔𝟏𝟎 , 𝒃 = 𝟏𝟖𝟑𝟎 , 𝒂𝒏𝒅 𝒄 = 𝟏𝟑𝟗𝟎 Since the given parts are sides of the spherical
triangle, refer to the limitations on sides.

𝑎 < 1800 ? = 1610 < 1800 ? Yes Limitation on sides (a) is not fully satisfied since
𝒃 > 𝟏𝟖𝟎𝟎 .
𝑐 < 1800 ? = 1390 < 1800 ? Yes
𝑏 < 1800 ? = 1830 < 1800 ? No
𝟎𝟎 < (𝒂 + 𝒃 + 𝒄) < 𝟑𝟔𝟎𝟎 ? Limitation on sides (b) is not satisfied since the
sum of the three sides, 𝟒𝟖𝟑𝟎 is greater than
00 < 1610 + 1830 + 1390 ) < 3600 ?
3600 .
00 < 𝟒𝟖𝟑𝟎 < 3600 ? No
(𝑎 + 𝑏) > 𝑐 ? It satisfies limitation on sides which states that
“the sum of any two sides of spherical triangle is
(1610 + 1830 ) > 1390 ? greater than the third side”
3440 > 1390 ? Yes

It is not possible to construct a spherical triangle


(𝑎 + 𝑐) > 𝑏? with sides
𝒂 = 𝟏𝟔𝟏𝟎 , 𝒃 = 𝟏𝟖𝟑𝟎 , 𝒂𝒏𝒅 𝒄 = 𝟏𝟑𝟗𝟎 since
(1610 + 1390 ) > 1830 ? one side is greater than 1800 and the sum of
3000 > 1830 ? Yes the 3 sides is more than 3600 .

(𝑏 + 𝑐) > 𝑎?
(1830 + 1300 ) > 1610 ?
3130 > 1610 ? Yes

Confidential to United Marine Training Center, Inc. Page 3-162


Pre Sea Review Materials <course code>
Version Number
Deck
5 November 2018

Supplementary Exercises
Solve the following the problems
Answer
1] 0
𝐴 = 168 ,
𝐵 = 700 , 𝑎𝑛𝑑 Possible
𝐶 = 1400

2] 𝑎 = 1100 ,
𝑏 = 300 , 𝑎𝑛𝑑 Not Possible
𝑐 = 2100

3] 𝑎 = 980 ,
𝑏 = 880 , 𝑎𝑛𝑑 Possible
𝑐 = 1580

4] 𝐴 = 1400 ,
𝐵 = 1600 , 𝑎𝑛𝑑 Not Possible
𝐶 = 2400
5] 𝐴 = 1100 ,
𝐵 = 380 , 𝑎𝑛𝑑 Possible
𝐶 = 1730

Confidential to United Marine Training Center, Inc. Page 3-163


Pre Sea Review Materials <course code>
Version Number
Deck
5 November 2018

Right Spherical Triangle

A right spherical triangle is a spherical triangle that has one angle equal to 90° as shown in Figure
3-72.
𝐵

a
𝑐
90°
𝐶
b
𝐴
Figure 3-72: Right spherical triangle - Source: Christian Kevin Villanueva

Spherical triangle can have one or two or three 90° interior angle. Spherical triangle is said to be right
if only one of its included angle is equal to 90°. Triangles with more than one 90° angle are oblique.
Any right spherical triangle can be solved by following the Napier’s rules. The rules are aided with the
Napier’s circle.
Figure 3-73 is the Napier’s circle corresponding to the right spherical triangle on Figure 3-72. It is
divided into 5 parts as one angle (right angle C) is already known. The sides and angle of the triangle
are written in consecutive order (not including the right angle), and complimentary angles are taken for
quantities opposite the right angle.

Napier’s Circle

Figure 3-73: Napier's circle of right spherical triangle of Figure 3-72 - Source: Christian Kevin Villanueva

𝐴 = 𝑐𝑜 − 𝐴 = 90° − 𝐴

𝐵 = 𝑐𝑜 − 𝐵 = 90° − 𝐵

𝑐 = 𝑐𝑜 − 𝑐 = 90° − 𝑐

Confidential to United Marine Training Center, Inc. Page 3-164


Pre Sea Review Materials <course code>
Version Number
Deck
5 November 2018

Parts of a Napier’s Circle


 Middle Part
It can be any part of the Napier’s circle.
 Adjacent Parts
These are the parts next to the middle part, having a common boundary with the middle part.
For instance:
 If 𝑎 is the middle part, then 𝑏 𝑎𝑛𝑑 𝐵 are the adjacent parts.

 If 𝑐 is the middle part , then 𝐴 𝑎𝑛𝑑 𝐵 are the adjacent parts.

 Opposite Parts
The parts in which has no common boundary with the middle part
For instance:
 If 𝑎 is the middle part , then 𝐴 𝑎𝑛𝑑 𝑐 are the opposite parts.
 If 𝑐 is the middle part , then 𝑎 𝑎𝑛𝑑 𝑏 are the opposite parts

Napier’s Rules

1. The sine of any middle part is equal to the product of the cosines of its opposite parts
 To aid familiarization of the rule, an acronym (𝑺𝑰𝑵 − 𝑪𝑶𝑶𝑷 ) can be used.
If 𝒂 is the middle part, its opposite parts are 𝑨 𝒂𝒏𝒅 𝒄 , then by (𝑺𝑰𝑵 − 𝑪𝑶𝑶𝑷 ) rule
𝐬𝐢𝐧 𝒂 = 𝒄𝒐𝒔𝒄 𝒄𝒐𝒔𝑨
𝐬𝐢𝐧 𝒂 = 𝒄𝒐𝒔(𝟗𝟎𝟎 − 𝒄) 𝒄𝒐𝒔 (𝟗𝟎𝟎 − 𝑨)
𝐬𝐢𝐧 𝒂 = 𝐬𝐢𝐧 𝒄 𝒔𝒊𝒏 𝑨

2. The sine of any middle part is equal to the product of the tangents of its adjacent parts
 To aid familiarization of the rule, an acronym (𝑺𝑰𝑵 − 𝑻𝑨𝑨𝑫 ) can be used.
If 𝒂 is the middle part, its adjacent parts are 𝒃 𝒂𝒏𝒅 𝑩, then by (𝑺𝑰𝑵 − 𝑻𝑨𝑨𝑫 ) rule
𝐬𝐢𝐧 𝒂 = 𝐭𝐚𝐧 𝒃 𝒕𝒂𝒏 𝑩
𝐬𝐢𝐧 𝒂 = 𝐭𝐚𝐧 𝒃 𝒕𝒂𝒏 (𝟗𝟎𝟎 − 𝑩)
𝐬𝐢𝐧 𝒂 = 𝐭𝐚𝐧 𝒃 𝒄𝒐𝒕 𝑩

1
Note : 𝑐𝑜𝑡 𝐵 =
𝑡𝑎𝑛 𝐵
1 tan 𝑏
sin 𝑎 = tan𝐵 ( )=
tan 𝐵 tan 𝐵

Confidential to United Marine Training Center, Inc. Page 3-165


Pre Sea Review Materials <course code>
Version Number
Deck
5 November 2018

Rules for Quadrants


In solving the missing parts of a right spherical triangle, the rules for quadrants should also be
considered to verify if the values obtained using the Napier’s rules are reasonable or not.
 Rule 1: In a right spherical triangle, an oblique angle and its opposite side are always in the
same quadrant.
 Rule 2(a): If the hypotenuse of a right spherical triangle is less than 90°, the two oblique
angles (and therefore the two legs) are in the same quadrant.
 Rule 2(b): If the hypotenuse is greater than 90°, the two oblique angles (and therefore the
two legs) are in different quadrants.
 Rule 3: When the two given parts of a right spherical triangle are a leg and its opposite
angle, there are two solutions.
Note: Since any side and any angle of the right spherical triangle is less than 180 °, only quadrants 1
and 2 will be considered.

Example 3-110

Solve for the spherical triangle whose parts are 𝐶 = 𝟗𝟎𝟎 , 𝒂 = 𝟏𝟏𝟔𝟎 , and 𝒃 = 𝟑𝟐𝟎 .

Solution:
Illustrate and label the triangle.

Required: A, B, and c

The Napier’s pie needed for this problem

sin 𝑏 = tan 𝑎 𝑡𝑎𝑛 𝐴 Referring to the Napier’s Circle, given side b can be
chosen as middle part so that it’s adjacent parts are
the unknown angle A and the other known side a
Apply 𝑺𝑰𝑵 − 𝑻𝑨𝑨𝑫

tan 𝑎 Note: Apply the concept of co-function


sin 𝑏 =
𝑡𝑎𝑛 𝐴 1
𝑡𝑎𝑛𝐴 = 𝑐𝑜𝑡𝐴 =
tan 𝐴
tan 𝑎 Formula transformation
𝑡𝑎𝑛𝐴 =
sin 𝑏

Confidential to United Marine Training Center, Inc. Page 3-166


Pre Sea Review Materials <course code>
Version Number
Deck
5 November 2018

𝑡𝑎𝑛 1160 Substitute the given parts into the equation


𝑡𝑎𝑛𝐴 =
𝑠𝑖𝑛 320

𝐴 = 𝑡𝑎𝑛−1(−3.869087199) Evaluate using inverse tangent function


0 ′ Based from rule 1 of quadrants, the value obtained
𝐴 = −75 30.5
for 𝐴 = −750 30.5′ is not reasonable since it lies in the
fourth quadrant.

𝐴 = −750 30.5′ + 1800 Since 𝑎 = 1160 , it lies in quadrant 2, then it follows


that 𝑨 must lie in quadrant 2 also.
𝑨 = 𝟏𝟎𝟒𝟎 𝟐𝟗. 𝟓′
To get an angle in the 2nd quadrant , add 1800 to A.

𝑠𝑖𝑛 𝑎 = 𝑡𝑎𝑛 𝑏 𝑡𝑎𝑛 𝐵 Referring to the Napier’s Circle, given side a can be
chosen as middle part so that it’s adjacent parts are
the unknown angle B and the other known side b
Apply SIN-TAAD

𝑡𝑎𝑛 𝑏 Note: Apply the concept of co-function


𝑠𝑖𝑛 𝑎 =
𝑡𝑎𝑛 𝐵 1
𝑡𝑎𝑛𝐵 = 𝑐𝑜𝑡𝐵 =
tan 𝐵

𝑡𝑎𝑛𝑏 Formula transformation


𝑡𝑎𝑛𝐵 =
𝑠𝑖𝑛 𝑎

𝑡𝑎𝑛 320 Substitute the given parts into the equation


𝑡𝑎𝑛𝐵 =
𝑠𝑖𝑛 1160

𝐵 = 𝑡𝑎𝑛−1(0.695230853) Since 𝑏 = 320, it lies in quadrant I, therefore its


opposite angle 𝐵 must lie in quadrant I also.
𝐵 = 𝟑𝟒𝟎 𝟒𝟖. 𝟓′
The answer is reasonable because b lies in 1st
quadrant, so, it follows that B lies also in the 1st
quadrant (Rule 1 of Quadrants).

𝑠𝑖𝑛𝑐 = 𝑐𝑜𝑠𝑎 𝑐𝑜𝑠 𝑏 The unknown side c can be chosen as the middle part
so that the given sides a and b are its opposite parts.
Apply SIN-COOP

𝑐𝑜𝑠 𝑐 = 𝑐𝑜𝑠𝑎 𝑐𝑜𝑠 𝑏 Apply the concept of co-function


Note: 𝑠𝑖𝑛𝑐 = cos 𝑐

𝑐𝑜𝑠 𝑐 = 𝑐𝑜𝑠1160 𝑐𝑜𝑠 320 Substitute the given into the equation

𝑐 = 𝑐𝑜𝑠 −1 0.371759816 Evaluate

𝒄 = 𝟏𝟏𝟏𝟎 𝟒𝟗′ Since the two given sides , 𝑎 𝑎𝑛𝑑 𝑏 lie in different
quadrants, it follows that c, the hypotenuse must
be greater than 𝟗𝟎𝟎 .
(Rule 2b of Quadrants).

Confidential to United Marine Training Center, Inc. Page 3-167


Pre Sea Review Materials <course code>
Version Number
Deck
5 November 2018

Example 3-111

Solve the right spherical triangle ABC, given C = 900, A  40 and c  130

Solution
Illustrate and label the triangle.

Required: a, b, and B

The Napier’s pie needed for this problem

𝑠𝑖𝑛 𝑎 = 𝑐𝑜𝑠 𝐴 𝑐𝑜𝑠 𝑐 Apply SIN-COOP with 𝑐̅and 𝐴̅ as the opposite


parts of 𝑎̅.

𝑠𝑖𝑛 𝑎 = 𝑠𝑖𝑛𝐴 𝑠𝑖𝑛𝑐 By means of co-function identity


𝑐𝑜𝑠𝐴 = 𝑠𝑖𝑛𝐴
𝑐𝑜𝑠𝑐 = 𝑠𝑖𝑛𝑐
𝑠𝑖𝑛 𝑎 = 𝑠𝑖𝑛 400 𝑠𝑖𝑛1300 Substitute the given values into the equation and
−1 evaluate
𝑎 = 𝑠𝑖𝑛 (0.492403876)
𝒂 = 𝟐𝟗𝟎 𝟑𝟎′ By Quadrant Rule (1), 𝐴 and 𝑎 must be on the
same quadrant, thus 𝐴 and 𝑎 have values that are
less than 90 or their values must lie in the first
quadrant.

𝑠𝑖𝑛𝐴 = 𝑡𝑎𝑛𝑐 𝑡𝑎𝑛 𝑏 To solve for the other parts, avoid using the
computed value, 𝑎 = 290 30′ . Always use the given
parts to avoid accumulation of mistakes if ever the
first value is incorrect.
To solve for b , consider 𝐴̅ as your middle part so
that 𝑐̅ and b are its adjacent, then apply SIN-
TAAD.
𝑡𝑎𝑛𝑏 By means of co-function identity
𝑐𝑜𝑠𝐴 =
𝑡𝑎𝑛𝑐
𝑠𝑖𝑛𝐴 = 𝑐𝑜𝑠𝐴
By means of co-function and reciprocal identities
1
𝑡𝑎𝑛𝑐 = 𝑐𝑜𝑡𝑐 =
𝑡𝑎𝑛𝑐

Confidential to United Marine Training Center, Inc. Page 3-168


Pre Sea Review Materials <course code>
Version Number
Deck
5 November 2018

𝑡𝑎𝑛𝑏 = 𝑐𝑜𝑠𝐴 𝑡𝑎𝑛𝑐 Formula transformation

𝑡𝑎𝑛𝑏 = 𝑐𝑜𝑠400 𝑡𝑎𝑛1300 Substitute the given parts into the equation and
−1 (0.912936217) evaluate
𝑏 = 𝑡𝑎𝑛
𝑏 = −420 24′
𝑏 = −420 24′ + 𝟏𝟖𝟎𝟎 Since the computed value is (-), just add 1800 to it
to get an angle that lies in the second quadrant.
𝒃 = 𝟏𝟑𝟕𝟎 𝟑𝟔′1
By Quadrant Rule (2b), a and b must be on
different quadrants because the hypotenuse, c , is
greater than 900 . Furthermore,
since 𝒂 is in quadrant 1 already , 𝑏 must lie in the
second quadrant.

𝑠𝑖𝑛𝑐 = 𝑡𝑎𝑛𝐴 𝑡𝑎𝑛𝐵 To solve for B ,consider 𝑐 as your middle part so


that 𝐴 and 𝐵 are its adjacent, then apply SIN-
TAAD.

1 By means of co-function identity


𝑐𝑜𝑠 𝑐 =
𝑡𝑎𝑛𝐴 𝑡𝑎𝑛𝐵 𝑠𝑖𝑛𝑐 = 𝑐𝑜𝑠𝑐

By means of co-function and reciprocal identities

1
𝑡𝑎𝑛𝐴 = 𝑐𝑜𝑡𝐴 =
𝑡𝑎𝑛𝐴

1
𝑡𝑎𝑛𝐵 = 𝑐𝑜𝑡𝐵 =
𝑡𝑎𝑛𝐵
1 Formula transformation
𝑡𝑎𝑛𝐵 =
𝑡𝑎𝑛𝐴 𝑐𝑜𝑠 𝑐
1 Substitute the given parts into the equation and
𝑡𝑎𝑛𝐵 = evaluate
𝑡𝑎𝑛40 𝑐𝑜𝑠 1300
0

𝐵 = 𝑡𝑎𝑛−1(−1.85403946)
𝐵 = −610 40′
𝐵 = −610 40′ + 1800 By Quadrant Rule (1),𝐵 and 𝑏 must be on the same
𝟎 ′ quadrant. Since the value of b is on Q2, it follows
𝑩 = 𝟏𝟏𝟖 𝟐𝟎
that B is also on Q2 as defined by Quadrant Rule
(1).

Thus, add 1800 to the computed value of B .

Confidential to United Marine Training Center, Inc. Page 3-169


Pre Sea Review Materials <course code>
Version Number
Deck
5 November 2018

Supplementary Exercises
Solve the following the problems
Answer
1] Solve the right spherical triangle ACB, given the following
parts: 𝑎 = 440 50′ ,
𝐶 = 900 , 𝑏 = 1180 09′ ,
𝐴 = 480 26′ , 𝑐 = 1090 33′
𝐵 = 1100 40′

2] Solve the right spherical triangle ACB, given the following


parts: 𝑎 = 580 04′ ,
𝐶 = 900 , 𝑏 = 810 47′ ,
𝐴 = 580 20′ , 𝐵 = 830 01′
𝑐 = 850 40′

3] Solve the right spherical triangle ACB, given the following


parts: 𝐴 = 620 59′ ,
𝐶 = 900 , 𝐵 = 1080 11′ ,
𝑎 = 610 26′ , 𝑐 = 990 38′
𝑏 = 1100 30′

Confidential to United Marine Training Center, Inc. Page 3-170


Pre Sea Review Materials <course code>
Version Number
Deck
5 November 2018

Oblique Spherical Triangle

A spherical triangle is oblique if no angle or side of the triangle is equal to 900.


Spherical triangles are said to be oblique if none of its included angle is 90° or two or three of its
included angles are 90°.
A spherical triangle can be solved by:
 The Law of Sines
 The Law of Cosine for Sides
 The Law of Cosine for Angles

The Law of Sines


 used when the given data contains a “pair”
 applicable for the following cases (SSA, AAS)
𝐬𝐢𝐧 𝒂 𝐬𝐢𝐧 𝒃 𝐬𝐢𝐧 𝒄 Formula for the Law of Sines (Spherical Triangles)
= =
𝐬𝐢𝐧 𝑨 𝐬𝐢𝐧 𝑩 𝐬𝐢𝐧 𝑪
Where:
a, b & c = Sides of the spherical triangle
A, B & C = Angles opposite of 𝑎, 𝑏 and 𝑐 respectively

One big disadvantage of the sine rule is due to the fact that 𝑠𝑖𝑛𝐴 = 𝑠𝑖𝑛(1800 − 𝐴) , i.e., both A and
(1800 − 𝐴) are possible candidates to the solution of our equation. Usually we overcome this
ambiguity by using:
 Physical setting of the problem.
 Greater side faces greater angle and hence (A − B) and (a − b) must be of the same sign.

Example 3-112

Consider triangle ABC in which 𝑏 = 260 21′ , 𝐵 = 520 22′ , 𝐴 = 1040 44′ . Find a.
Solution:
C Illustrate and label the triangle.

𝑏 = 26°21′ Required: a

𝐴 = 104°44′

c 𝐵 = 52°22′

𝑠𝑖𝑛 𝑎 𝑠𝑖𝑛𝑏 It is oblique spherical triangle since there’s no 900


= angle.
𝑠𝑖𝑛 𝐴 𝑠𝑖𝑛 𝐵
Use the law of sines since it satisfies AAS or
there’s a given pair of angle and side.

Confidential to United Marine Training Center, Inc. Page 3-171


Pre Sea Review Materials <course code>
Version Number
Deck
5 November 2018

𝑠𝑖𝑛𝑏 𝑠𝑖𝑛 𝐴 Formula transformation


𝑠𝑖𝑛 𝑎 =
𝑠𝑖𝑛 𝐵
𝑠𝑖𝑛 260 21′ ∗ 𝑠𝑖𝑛 1040 44′ Substitute the given values into the equation and
𝑠𝑖𝑛 𝑎 = evaluate
𝑠𝑖𝑛 520 22′
0.429259437
𝑠𝑖𝑛 𝑎 =
0.791934541
𝑎 = 𝑠𝑖𝑛−1(0.542039038)
𝒂 = 𝟑𝟐𝟎 𝟒𝟗′

𝒂 = 𝟏𝟖𝟎𝟎 − 𝟑𝟐𝟎 𝟒𝟗′ Since (𝐴 − 𝐵) = (1040 44′ − 520 22′ ) is positive,


and so (𝑎 − 𝑏) = (320 49′ − 260 21′ ) must also be
𝒂 = 𝟏𝟒𝟕𝟎 𝟏𝟏′
positive.
However, both values for a satisfy this
requirement; thus both are solutions of the data
as given.
Note:

𝒔𝒊𝒏 𝟑𝟐𝟎 𝟒𝟗′ = 𝒔𝒊𝒏 𝟏𝟒𝟕𝟎 𝟏𝟏′ = 𝟎. 𝟓𝟒𝟏𝟗𝟓𝟐𝟔𝟗𝟖

Since 𝑠𝑖𝑛 𝑎 > 0, whether a is in the first or 2nd


quadrant, there are two permissible values for a.
To get the required value, we must consider the
given values of A and B.

The Law of Cosines for Sides


 used when there are more sides given than an angle
 applicable for the following cases (SAS, SSS)

𝑐𝑜𝑠 𝑎 = 𝑐𝑜𝑠 𝑏 𝑐𝑜𝑠 𝑐 + 𝑠𝑖𝑛 𝑏 𝑠𝑖𝑛 𝑐 𝑐𝑜𝑠 𝐴 General formulas for solving the sides
𝑐𝑜𝑠 𝑏 = 𝑐𝑜𝑠 𝑎 𝑐𝑜𝑠 𝑐 + 𝑠𝑖𝑛 𝑎 𝑠𝑖𝑛 𝑐 𝑐𝑜𝑠 𝐵
𝑐𝑜𝑠 𝑐 = 𝑐𝑜𝑠 𝑎 𝑐𝑜𝑠 𝑏 + 𝑠𝑖𝑛 𝑎 𝑠𝑖𝑛 𝑏 𝑐𝑜𝑠 𝐶

𝑐𝑜𝑠 𝑎 − 𝑐𝑜𝑠 𝑏 𝑐𝑜𝑠 𝑐 Transformed formulas for solving the


𝑐𝑜𝑠 𝐴 = angles
𝑠𝑖𝑛 𝑏 𝑠𝑖𝑛 𝑐
𝑐𝑜𝑠 𝑏 − 𝑐𝑜𝑠𝑎 𝑐𝑜𝑠𝑐
𝑐𝑜𝑠 𝐵 =
𝑠𝑖𝑛 𝑎 𝑠𝑖𝑛 𝑐
𝑐𝑜𝑠 𝑐 − 𝑐𝑜𝑠 𝑎 𝑐𝑜𝑠 𝑏
𝑐𝑜𝑠 𝐶 =
𝑠𝑖𝑛 𝑎 𝑠𝑖𝑛 𝑏

Confidential to United Marine Training Center, Inc. Page 3-172


Pre Sea Review Materials <course code>
Version Number
Deck
5 November 2018

Example 3-113

In ∆PXZ , 𝑃 = 500 , 𝑧 = 700 45′ , 𝑥 = 620 10′ . Find p and Z (to the nearest degree and minute).

Solution

𝑃 = 50° Illustrate and label the triangle.

𝑥 = 62°10′
𝑧 = 70°45′ Required: p and Z

𝑍
𝑝
𝑋
𝑐𝑜𝑠 𝑝 = 𝑐𝑜𝑠 𝑥 𝑐𝑜𝑠 𝑧 + 𝑠𝑖𝑛 𝑥 𝑠𝑖𝑛 𝑧 𝑐𝑜𝑠 𝑃 It is oblique spherical triangle since
there’s no 900 angle.
Use the law of cosines to solve for
p since it satisfies SAS

𝑐𝑜𝑠 𝑝 = 𝑐𝑜𝑠 62° 10′ 𝑐𝑜𝑠 70°45′ + 𝑠𝑖𝑛 62°10′ 𝑠𝑖𝑛 70°45′ 𝑐𝑜𝑠 50° Substitute the given values into the
equation

𝑐𝑜𝑠 𝑝 = (0.153932954) + (0.536642088) Evaluate


−1
𝑝 = 𝑐𝑜𝑠 (𝟎. 𝟔𝟗𝟎𝟓𝟕𝟓𝟎𝟒𝟐)
𝒑 = 𝟒𝟔 𝟏𝟗′
𝟎

𝑐𝑜𝑠 𝑧 − 𝑐𝑜𝑠 𝑝 ∙ 𝑐𝑜𝑠 𝑥 Since all the sides are known , the
𝑐𝑜𝑠 𝑍 =
𝑠𝑖𝑛 𝑝 ∙ 𝑠𝑖𝑛 𝑥 law of cosine for sides can be
used to calculate for the value of
angle Z.
𝑐𝑜𝑠 70°45′ − 𝑐𝑜𝑠 46°19′ ∙ 𝑐𝑜𝑠 62°10′ Substitute the given values of the
𝑐𝑜𝑠 𝑍 =
𝑠𝑖𝑛 46°19′ ∙ 𝑠𝑖𝑛 62°10′ 3 sides into the equation to solve
for Z.
0.329690645 − 0.322475615
𝑐𝑜𝑠 𝑍 =
0.639504402
Evaluate
0.00721503
𝑐𝑜𝑠 𝑍 =
0.639504402
𝑍 = 𝑐𝑜𝑠 −1 0.011282221
𝒁 = 𝟖𝟗𝟎 𝟐𝟏′

Confidential to United Marine Training Center, Inc. Page 3-173


Pre Sea Review Materials <course code>
Version Number
Deck
5 November 2018

Example 3-114

Consider triangle ABC in which 𝑎 = 1250 38′ , 𝑐 = 730 24′ , 𝐵 = 1020 16′ . Find b. Write your
answer to the nearest degree and minute.

Solution

𝐵 = 102° 16′ Illustrate and label the


triangle.
Required: b, A, C

𝑐 = 73° 24′
𝑎 = 125° 38′

𝑐
𝐶

𝑐𝑜𝑠 𝑏 = 𝑐𝑜𝑠 𝑎 𝑐𝑜𝑠 𝑐 + 𝑠𝑖𝑛 𝑎 𝑠𝑖𝑛 𝑐 𝑐𝑜𝑠 𝐵 It is oblique spherical


triangle since there’s no
900 angle.
Use the law of cosines to
solve for b since it satisfies
SAS

𝑐𝑜𝑠 𝑏 = 𝑐𝑜𝑠 1250 38′ 𝑐𝑜𝑠 730 24′ + 𝑠𝑖𝑛 1250 38′ 𝑠𝑖𝑛 730 24′ 𝑐𝑜𝑠 1020 16′ Substitute the given values
into the equation

𝑐𝑜𝑠 𝑏 = (−0.166440875) + (−0.165484074) Evaluate


−1
𝑏 = 𝑐𝑜𝑠 (−𝟎. 𝟑𝟑𝟏𝟗𝟐𝟒𝟗𝟒𝟗)
𝒃 = 𝟏𝟎𝟗 𝟐𝟑′
𝟎

Note:
If you want to solve for the other angles, A and C, you can apply any of the following
 the law of sine since the opposite sides, a and c, are already known
sin 𝑎 𝑠𝑖𝑛𝑏 sin 𝑐 𝑠𝑖𝑛𝑏
sin 𝐴
= sin 𝐵 and sin 𝐶
= sin 𝐵

 the law of cosines;


cos 𝑎 − 𝑐𝑜𝑠 𝑏 𝑐𝑜𝑠 𝑐
𝑐𝑜𝑠 𝐴 =
𝑠𝑖𝑛 𝑏 𝑠𝑖𝑛 𝑐

cos 𝑐 − 𝑐𝑜𝑠 𝑎 𝑐𝑜𝑠 𝑏


𝑐𝑜𝑠 𝐶 =
𝑠𝑖𝑛 𝑎 𝑠𝑖𝑛 𝑏

Confidential to United Marine Training Center, Inc. Page 3-174


Pre Sea Review Materials <course code>
Version Number
Deck
5 November 2018

The Law of Cosines for Angles


 used when there are more angles given than side
 applicable for the following cases (ASA, AAA)

𝑐𝑜𝑠 𝐴 = −𝑐𝑜𝑠 𝐵 𝑐𝑜𝑠 𝐶 + 𝑠𝑖𝑛 𝐵 𝑠𝑖𝑛 𝐶 𝑐𝑜𝑠 𝑎 General formulas for solving the angles
𝑐𝑜𝑠 𝐵 = −𝑐𝑜𝑠 𝐴 𝑐𝑜𝑠 𝐶 + 𝑠𝑖𝑛 𝐴 𝑠𝑖𝑛 𝐶 𝑐𝑜𝑠 𝑏
𝑐𝑜𝑠 𝐶 = −𝑐𝑜𝑠 𝐴 𝑐𝑜𝑠 𝐵 + 𝑠𝑖𝑛 𝐴 𝑠𝑖𝑛 𝐵 𝑐𝑜𝑠 𝑐

𝑐𝑜𝑠 𝐴 + 𝑐𝑜𝑠 𝐵 𝑐𝑜𝑠 𝐶 Transformed formulas for solving the


𝑐𝑜𝑠 𝑎 =
𝑠𝑖𝑛 𝐵 𝑠𝑖𝑛 𝐶 sides
𝑐𝑜𝑠 𝐵 + 𝑐𝑜𝑠𝐴 𝑐𝑜𝑠𝐶
𝑐𝑜𝑠 𝑏 =
𝑠𝑖𝑛 𝐴 𝑠𝑖𝑛 𝐶
𝑐𝑜𝑠 𝐶 + 𝑐𝑜𝑠 𝐴 𝑐𝑜𝑠 𝐵
𝑐𝑜𝑠 𝑐 =
𝑠𝑖𝑛 𝐴 𝑠𝑖𝑛 𝐵

Example 3-115

Consider triangle ABC in which 𝐴 = 1210 13′ , 𝐵 = 800 41′ , 𝐶 = 880 51′ . Find c. Write your answer
to the nearest degree and minute.
Solution
𝐶 = 88°51′ Illustrate and label the triangle.
Required: c

𝑏
𝑎

𝐴 = 121°13′

𝑐
𝐵 = 80°41′

𝑐𝑜𝑠 𝐶 + 𝑐𝑜𝑠 𝐴 𝑐𝑜𝑠 𝐵 It is oblique spherical triangle since


𝑐𝑜𝑠 𝑐 =
𝑠𝑖𝑛 𝐴 𝑠𝑖𝑛 𝐵 there’s no 900 angle.
Use the law of cosines for angles to
solve for c since it satisfies AAA.

𝑐𝑜𝑠 880 51′ + 𝑐𝑜𝑠 1210 13′ 𝑐𝑜𝑠 800 41′ Substitute the given values into the
𝑐𝑜𝑠 𝑐 = equation
𝑠𝑖𝑛 1210 13′ 𝑠𝑖𝑛 800 41′

Confidential to United Marine Training Center, Inc. Page 3-175


Pre Sea Review Materials <course code>
Version Number
Deck
5 November 2018

0.020069938 + (−0.083904125) Evaluate


𝑐𝑜𝑠 𝑐 =
0.843932128
−0.063834187
𝑐𝑜𝑠 𝑐 =
0.843932128
−1 (−0.075639005)
𝑐 = 𝑐𝑜𝑠
𝒄 = 𝟗𝟒𝟎 𝟐𝟎′

Note:
To solve for the other sides, b and c, use the formulas below;
𝑐𝑜𝑠 𝐴 + 𝑐𝑜𝑠 𝐵 𝑐𝑜𝑠 𝐶
𝑐𝑜𝑠 𝑎 =
𝑠𝑖𝑛 𝐵 𝑠𝑖𝑛 𝐶

𝑐𝑜𝑠 𝐵 + 𝑐𝑜𝑠𝐴 𝑐𝑜𝑠𝐶


𝑐𝑜𝑠 𝑏 =
𝑠𝑖𝑛 𝐴 𝑠𝑖𝑛 𝐶

Follow the procedures above and the you will get the following answers:
𝒃 = 𝟕𝟗𝟎 𝟒𝟖′
𝒂 = 𝟏𝟐𝟏𝟎 𝟐𝟖′

Confidential to United Marine Training Center, Inc. Page 3-176


Pre Sea Review Materials <course code>
Version Number
Deck
5 November 2018

Supplementary Exercises
Solve the following problems. Express answers to the nearest degree and minute.
Answer

1] Solve the missing parts of triangle ABC, given the following sides: 𝐴 = 400 39′ ,
𝑎 = 𝟒𝟎𝟎 𝐵 = 720 14′
𝒃 = 𝟕𝟎𝟎 𝐶 = 500 56′
𝒄 = 𝟓𝟎𝟎

2] In a spherical triangle ABC, 𝑎 = 1150 34′


𝐴 = 1160 19′
𝐵 = 550 30′
𝐶 = 800 37′

Find the value of side 𝒂.

3] Solve the following oblique spherical triangle given that: 𝑎 = 630 49′ ,
𝑏 = 1560 12′ 𝐵 = 1540 04′
𝑐 = 1120 48′ 𝐶 = 870 27′
𝐴 = 760 32′

4] In the following oblique spherical triangle given that 𝐴 = 840 21′ 10"
𝐵 = 650 33′
𝑏 = 640 23′ 15"
𝑎 = 990 40′ 48"
Find angle 𝐴.

Confidential to United Marine Training Center, Inc. Page 3-177


Pre Sea Review Materials <course code>
Version Number
Deck
5 November 2018

Application of Spherical Trigonometry

One of the most important applications of spherical trigonometry is finding the distance between two
positions which do not lie on the same parallel of latitude on the earth’s surface.
A spherical triangle which has one of its vertices at the north or south pole is called a terrestrial
triangle. In the Figure 3-74, the terrestrial triangle ACB has its vertices at the north pole C and at two
points A and B on a great circle. In navigation, arc AB represents the projection of the path of a ship
upon the earth’s surface (known as great circle track).

Figure 3-74: Terrestrial sphere - Source: Christian Kevin Villanueva

Great Circle Sailing


Navigation along a great circle track is called great circle sailing. It is used for long ocean passages.
In here, the earth is considered a perfect spherical shape; therefore, the shortest distance between
two points on its surface is the arc of the great circle containing two points

Modelling the Terrestrial Triangle

Figure 3-75: Terrestrial triangle with labels on its parts - Source: Christian Kevin Villanueva

Confidential to United Marine Training Center, Inc. Page 3-178


Pre Sea Review Materials <course code>
Version Number
Deck
5 November 2018

Figure 3-75 shows the representation of the spherical triangle as terrestrial triangle. It is formed by the
intersection of three (3) Great Circles. The three Great Circles comprising the terrestrial triangle are:
The Great Circle containing the meridian of the point of departure; the Great Circle containing the
meridian of the point of arrival and the Great Circle containing the path of the vessel (also known as
the Great Circle Track). Below are the meanings of the labels for Figure 3-75.
 φA – Latitude A or latitude of departure, the angle between the equatorial plane and point of
departure whose vertex is the center of the Earth
 co-φA – the complement of latitude A (co latA), the angle between the pole and latitude A
whose vertex is the center of the Earth
 φB – Latitude B or latitude of arrival, the angle between the equatorial plane and point of
arrival whose vertex is the center of the Earth
 co-φB – the complement of B (co latB), the angle between the pole and latitude B whose
vertex is the center of the Earth
 GCD – Great Circle Distance, the shorter distance between two points on the Great Circle
Track
 DLO – Difference of Longitude, the smaller angle between the Great Circle plane passing
through the meridian of Point A and the Great Circle plane passing through the meridian of
Point B whose vertex is the axis of the Earth’s rotation
 IC – Initial Course, the angle between the Great Circle containing the Great Circle Track and
the Great Circle containing the meridian of the point of departure. It is the starting course of
the vessel.
 FC – Final Course, the angle between the Great Circle containing the Great Circle Track and
the Great Circle containing the meridian of the point of arrival. It is the course of the vessel
when it reaches its arrival position.

Figure 3-76 shows how the spherical triangle in Mathematics is labelled in Terrestrial Navigation. This
is important in deriving the formulas used in great circle sailing as the concepts and principles of
spherical trigonometry are the basis for the derivation.

Figure 3-76: Spherical Triangles with labels in Mathematics and Navigational Terms - Source: Christian Kevin Villanueva

In solving oblique spherical triangle, the law of sine, the law of cosine for sides and law of cosine for
angles are used. Let us use these methods to derive the formulas used in Great Circle Sailing like the
Great Circle Distance (GCD), Initial Course (I/Co), and Final Course (F/Co).

Confidential to United Marine Training Center, Inc. Page 3-179


Pre Sea Review Materials <course code>
Version Number
Deck
5 November 2018

Table 3-13 shows the derivation of the formulas used in Great Circle Sailing. For easy reference, two
columns are presented. On the left column, the oblique spherical triangle in Mathematics is shown
together with the equations, while on the right column, the terrestrial triangle with the navigational
terms is presented.

Oblique Spherical Triangle Terrestrial Triangle


(in Mathematics) (in Navigation)

Apply the Law of Cosine for Sides to calculate for side c or the Great Circle Distance
(GCD)
cos 𝐺𝐶𝐷 = cos 𝑐𝑜𝐿𝑎𝑡𝐵 cos 𝑐𝑜𝐿𝑎𝑡𝐴
+ sin 𝑐𝑜𝐿𝑎𝑡𝐵 sin 𝑐𝑜𝐿𝑎𝑡𝐴 cos 𝐷𝑙𝑜

Apply the concept of co-function to rename the equation above

Note: sin 𝜃 = cos(900 − 𝜃) = 𝑐𝑜𝑠𝛽


cos 𝑐𝑜𝐿𝑎𝑡𝐵 = sin 𝐿𝑎𝑡𝐵 𝑎𝑛𝑑 sin 𝑐𝑜𝐿𝑎𝑡𝐵 = cos 𝐿𝑎𝑡𝐵
cos 𝑐 = cos 𝑎 cos 𝑏 + sin 𝑎 sin 𝑏 cos 𝐶
The resulting equation is;

cos 𝐺𝐶𝐷 = sin 𝐿𝑎𝑡𝐵 sin 𝐿𝑎𝑡𝐴 + cos 𝐿𝑎𝑡𝐵 cos 𝐿𝑎𝑡𝐴 cos 𝐷𝑙𝑜

Rearranging the right side of the equation and obtaining the


inverse cosine of both sides gives

𝐺𝐶𝐷 = cos−1 (cos 𝐿𝑎𝑡𝐴 cos 𝐿𝑎𝑡𝐵 cos 𝐷𝑙𝑜 + sin 𝐿𝑎𝑡𝐴 sin 𝐿𝑎𝑡𝐵)

Multiply by 60 to convert from angular units (degree and


minutes) to nautical miles since 1′ = 1 𝑁𝑀 , 10 = 60 𝑁𝑀.

𝑮𝑪𝑫𝑵𝑴 = 𝐜𝐨𝐬 −𝟏(𝐜𝐨𝐬 𝑳𝒂𝒕𝑨 𝐜𝐨𝐬 𝑳𝒂𝒕𝑩 𝐜𝐨𝐬 𝑫𝑳𝑶


+ 𝐬𝐢𝐧 𝑳𝒂𝒕𝑨 𝐬𝐢𝐧 𝑳𝒂𝒕𝑩) (𝟔𝟎𝑵𝑴/𝟏𝟎 )

This is the formula for calculating the great circle


distance(GCD).

Note:

When crossing the equator, the formula becomes

Confidential to United Marine Training Center, Inc. Page 3-180


Pre Sea Review Materials <course code>
Version Number
Deck
5 November 2018

𝑮𝑪𝑫𝑵𝑴 = 𝐜𝐨𝐬 −𝟏(𝐜𝐨𝐬 𝑳𝒂𝒕𝑨 𝐜𝐨𝐬 𝑳𝒂𝒕𝑩 𝐜𝐨𝐬 𝑫𝑳𝑶


− 𝐬𝐢𝐧 𝑳𝒂𝒕𝑨 𝐬𝐢𝐧 𝑳𝒂𝒕𝑩) (𝟔𝟎𝑵𝑴/𝟏𝟎 )

Apply the Law of Sines to calculate for angle A or Initial Couse (I/Co)

sin 𝑎 sin 𝑐 sin 𝑐𝑜𝐿𝑎𝑡𝐵 sin 𝐺𝐶𝐷


= =
sin 𝐴 sin 𝐶 sin 𝐼/𝐶𝑜 sin 𝐷𝑙𝑜

By formula transformation and inverse sine of both equations above


sin 𝑎 sin 𝐶 sin 𝑐𝑜𝑙𝑎𝑡𝐵 sin 𝐷𝑙𝑜
sin 𝐴 = sin 𝐼/𝐶𝑜 =
sin 𝑐 sin 𝐺𝐶𝐷
sin 𝑎 sin 𝐶 sin 𝑐𝑜𝑙𝑎𝑡𝐵 sin 𝐷𝑙𝑜
𝐴 = 𝑠𝑖𝑛−1 ( ) 𝐼/𝐶𝑜 = 𝑠𝑖𝑛−1 ( )
sin 𝐺𝐶𝐷 sin 𝐺𝐶𝐷

Apply the concept of co function

𝐜𝐨𝐬 𝒍𝒂𝒕𝑩 𝐬𝐢𝐧 𝑫𝑳𝑶


𝐼/𝑪𝒐 = 𝒔𝒊𝒏−𝟏 ( )
𝐬𝐢𝐧 𝑮𝑪𝑫

This is the formula for I/Co using the concept of law of sine.

Apply the Law of Sines to calculate for angle B or Final Course ( F/Co)
sin 𝑏 sin 𝑐 sin 𝑐𝑜𝐿𝑎𝑡 𝐴 sin 𝐺𝐶𝐷
= =
sin 𝐵 sin 𝐶 sin 𝐹/𝐶𝑜 sin 𝐷𝑙𝑜

By formula transformation and inverse sine of both equations above


sin 𝑐𝑜𝑙𝑎𝑡𝐴 sin 𝐷𝑙𝑜
sin 𝐹/𝐶𝑜 =
sin 𝐺𝐶𝐷
sin 𝑏 sin 𝐶 sin 𝑐𝑜𝑙𝑎𝑡𝐴 sin 𝐷𝑙𝑜
sin 𝐵 = 𝐹/𝐶𝑜 = 𝑠𝑖𝑛−1 ( )
sin 𝑐 sin 𝐺𝐶𝐷

Apply the concept of co function


sin 𝑏 sin 𝐶
𝐵 = 𝑠𝑖𝑛−1 ( ) 𝐜𝐨𝐬 𝒍𝒂𝒕𝑨 𝐬𝐢𝐧 𝑫𝑳𝑶
sin 𝐺𝐶𝐷
𝐹/𝑪𝒐 = 𝒔𝒊𝒏−𝟏 ( )
𝐬𝐢𝐧 𝑮𝑪𝑫

This is the formula for F/Co using the concept of law of sine.

Apply the Law of Cosine for Sides to solve for angle A or Initial Course (I/Co)

cos 𝑎 = cos 𝑏 cos 𝑐 + sin 𝑏 sin 𝑐 cos 𝐴 cos 𝑐𝑜𝐿𝑎𝑡𝐵 = cos 𝑐𝑜𝐿𝑎𝑡𝐴 cos 𝐺𝐶𝐷 + sin 𝑐𝑜𝐿𝑎𝑡𝐴 sin 𝐺𝐶𝐷 cos 𝐼/𝐶𝑂

By formula transformation and inverse cosine of both equations above


cos 𝑎 − cos 𝑏 cos 𝑐 cos 𝑐𝑜𝐿𝑎𝑡𝐵 − cos 𝑐𝑜𝐿𝑎𝑡𝐴 cos 𝐺𝐶𝐷
cos A = cos 𝐼/𝐶𝑜 =
sin 𝑏 sin 𝑐 sin 𝑐𝑜𝐿𝑎𝑡𝐴 sin 𝐺𝐶𝐷
cos 𝑎 − cos 𝑏 cos 𝑐 cos 𝑐𝑜𝐿𝑎𝑡𝐵 − cos 𝑐𝑜𝐿𝑎𝑡𝐴 cos 𝐺𝐶𝐷
𝐴 = 𝑐𝑜𝑠 −1 ( ) 𝐼/𝐶𝑜 = 𝑐𝑜𝑠 −1 ( )
sin 𝑏 sin 𝑐 sin 𝑐𝑜𝐿𝑎𝑡𝐴 sin 𝐺𝐶𝐷

Apply the concept of co function

𝐬𝐢𝐧 𝑳𝒂𝒕𝑩 − 𝐬𝐢𝐧 𝑳𝒂𝒕𝑨 𝒄𝒐𝒔𝑮𝑪𝑫


𝑰/𝑪𝒐 = 𝒄𝒐𝒔−𝟏 ( )
𝐜𝐨𝐬 𝑳𝒂𝒕𝑨 𝐬𝐢𝐧 𝑮𝑪𝑫

Confidential to United Marine Training Center, Inc. Page 3-181


Pre Sea Review Materials <course code>
Version Number
Deck
5 November 2018

This is the formula for I/Co using the concept of law of cosine.

Apply the Law of Cosine for Sides to solve for angle B or Final Course (F/Co)

cos 𝑏 = cos 𝑎 cos 𝑐 + sin 𝑎 sin 𝑐 cos 𝐵 cos 𝑐𝑜𝐿𝑎𝑡 𝐴 = cos 𝑐𝑜 𝐿𝑎𝑡𝐵 cos 𝐺𝐶𝐷 + sin 𝑐𝑜𝐿𝑎𝑡𝐵 sin 𝐺𝐶𝐷 cos 𝐹/𝐶𝑂
By formula transformation and inverse cosine of both equations above
cos 𝑐𝑜𝐿𝑎𝑡𝐴 − cos 𝑐𝑜𝐿𝑎𝑡𝐵 cos 𝐺𝐶𝐷
cos 𝑏 − cos 𝑎 cos 𝑐 cos 𝐹/𝐶𝑜 =
sin 𝑐𝑜𝐿𝑎𝑡𝐵 sin 𝐺𝐶𝐷
cos B =
sin 𝑎 sin 𝑐
cos 𝑐𝑜𝐿𝑎𝑡𝐴 − cos 𝑐𝑜𝐿𝑎𝑡𝐵 cos 𝐺𝐶𝐷
𝐹/𝐶𝑜 = 𝑐𝑜𝑠 −1 ( )
cos 𝑏 − cos 𝑎 cos 𝑐 sin 𝑐𝑜𝐿𝑎𝑡𝐵 sin 𝐺𝐶𝐷
𝐵 = 𝑐𝑜𝑠 −1 ( )
sin 𝑎 sin 𝑐
Apply the concept of co function

𝐬𝐢𝐧 𝑳𝒂𝒕𝑨 − 𝐬𝐢𝐧 𝑳𝒂𝒕𝑩 𝒄𝒐𝒔𝑮𝑪𝑫


𝑭/𝑪𝒐 = 𝒄𝒐𝒔−𝟏 ( )
𝐜𝐨𝐬 𝑳𝒂𝒕𝑩 𝐬𝐢𝐧 𝑮𝑪𝑫

This is the formula for F/Co using the concept of law of cosine.

Table 3-13: Derivation of Formulas used in Great Circle Sailing Based on the Principles of Solving Oblique Spherical Triangles
- Source: Ester Dinawanao

Summary of The Derived Formulas

When not crossing the equator


𝑮𝑪𝑫𝑵𝑴 = 𝐜𝐨𝐬−𝟏(𝐜𝐨𝐬 𝑳𝒂𝒕𝑨 𝐜𝐨𝐬 𝑳𝒂𝒕𝑩 𝐜𝐨𝐬 𝑫𝑳𝑶 + 𝐬𝐢𝐧 𝑳𝒂𝒕𝑨 𝐬𝐢𝐧 𝑳𝒂𝒕𝑩) (𝟔𝟎𝑵𝑴/𝟏𝟎 )
GCD When crossing Equator
𝑮𝑪𝑫𝑵𝑴 = 𝐜𝐨𝐬−𝟏(𝐜𝐨𝐬 𝑳𝒂𝒕𝑨 𝐜𝐨𝐬 𝑳𝒂𝒕𝑩 𝐜𝐨𝐬 𝑫𝑳𝑶 − 𝐬𝐢𝐧 𝑳𝒂𝒕𝑨 𝐬𝐢𝐧 𝑳𝒂𝒕𝑩) (𝟔𝟎𝑵𝑴/𝟏𝟎 )

𝐜𝐨𝐬 𝒍𝒂𝒕𝑩 𝐬𝐢𝐧 𝑫𝑳𝑶


𝐼/𝑪𝒐 = 𝒔𝒊𝒏−𝟏 ( )
𝐬𝐢𝐧 𝑮𝑪𝑫
𝐬𝐢𝐧 𝑳𝒂𝒕𝑩 − 𝐬𝐢𝐧 𝑳𝒂𝒕𝑨 𝒄𝒐𝒔𝑮𝑪𝑫
𝑰/𝑪𝒐 = 𝒄𝒐𝒔−𝟏 ( )
𝐜𝐨𝐬 𝑳𝒂𝒕𝑨 𝐬𝐢𝐧 𝑮𝑪𝑫

I/Co Note:
 Make LatB (-) when crossing the equator
 Initial Course is labeled with prefix same as Lat A and suffix same as DLO

𝐜𝐨𝐬 𝒍𝒂𝒕𝑨 𝐬𝐢𝐧 𝑫𝑳𝑶


𝐹/𝑪𝒐 = 𝒔𝒊𝒏−𝟏 ( )
𝐬𝐢𝐧 𝑮𝑪𝑫
𝐬𝐢𝐧 𝑳𝒂𝒕𝑨 − 𝐬𝐢𝐧 𝑳𝒂𝒕𝑩 𝒄𝒐𝒔𝑮𝑪𝑫
𝑭/𝑪𝒐 = 𝒄𝒐𝒔−𝟏 ( )
𝐜𝐨𝐬 𝑳𝒂𝒕𝑩 𝐬𝐢𝐧 𝑮𝑪𝑫
F/Co
Note:
 Make Lat A (-) when crossing the equator
 Final course angle is labeled contrary to Lat B and same as DLO

Table 3-14: Summary of the Derived Formulas - Source: Ester Dinawanao

Confidential to United Marine Training Center, Inc. Page 3-182


Pre Sea Review Materials <course code>
Version Number
Deck
5 November 2018

Example 3-116

Your vessel departs from Yokohama from position A (35°27’N, 139°39’ E) bound for San Francisco,
CA at position B (37°48.5’ N, 122°24’W). By Great Circle Sailing, determine the great circle distance,
the initial and final courses.

Solution (using the general formula for spherical triangles),


Illustrate and label the oblique spherical triangle where

 𝑏 = 𝑐𝑜𝐿𝑎𝑡𝐴
 𝑎 = 𝑐𝑜𝐿𝑎𝑡𝐵
 𝑐 = 𝐺𝐶𝐷
 𝐴 = 𝐼𝑛𝑖𝑡𝑖𝑎𝑙 𝐶𝑜𝑢𝑟𝑠𝑒
 𝐵 = 𝐹𝑖𝑛𝑎𝑙 𝐶𝑜𝑢𝑟𝑠𝑒
 𝐶 = 𝐷𝑙𝑜

𝑎
𝐷𝑙𝑜
𝑐𝑜 𝑙𝑎𝑡𝐵
𝑏
𝑐𝑜 𝑙𝑎𝑡𝐴 𝐹/𝐶0
𝐼/𝐶0

𝑐 𝐵
𝐴 𝐺𝐶𝐷

𝑙𝑎𝑡𝐵

𝑙𝑎𝑡𝐴

Confidential to United Marine Training Center, Inc. Page 3-183


Pre Sea Review Materials <course code>
Version Number
Deck
5 November 2018

Solution for 𝐺𝐶𝐷, 𝐼/𝐶𝑜 and 𝐹/𝐶𝑜,

𝐷𝑙𝑜 = 𝑙𝑜𝑛𝑔2 − 𝑙𝑜𝑛𝑔1 Solve for DLO


𝐷𝑙𝑜 = −1220 24′ − (1390 39′ ) Note: West (-) and East is (+)
𝑙𝑜𝑛𝑔2 = 1220 24′ 𝑊
= − 1220 24′
𝑙𝑜𝑛𝑔1 = 1390 39′ 𝐸 = + 1390 39′

𝐷𝑙𝑜 = −1220 24′ + (−1390 39′ ) Apply the rule for subtraction of
0
𝐷𝑙𝑜 = −262 03 ′ integers.
Change the sign of the
subtrahend from (1390 39′ ) to
(−1390 39′ ) and proceed to
addition.
Since the integers have like
signs, add their absolute values
(the magnitude of a real number
without regard to its sign) and
copy the sign of the bigger
absolute value.

𝐷𝑙𝑜 = −2620 03′ + 3600 = 970 57′ Add 3600 since the absolute
value of the sum is more than
1800 .
DLO must be the smallest arc
possible between the meridians
of the two positions

𝑫𝒍𝒐 = 𝟗𝟕𝟎 𝟓𝟕′ 𝑬 Since the result is (+), it means


that the direction of DLO is East
(E).

𝑐𝑜𝑠 𝐺𝐶𝐷 = 𝑐𝑜𝑠 𝑐𝑜𝐿𝑎𝑡𝐵 𝑐𝑜𝑠 𝑐𝑜𝐿𝑎𝑡𝐴 Use the law of cosine for sides to
+ 𝑠𝑖𝑛𝑐𝑜𝐿𝑎𝑡𝐵 𝑠𝑖𝑛 𝑐𝑜𝐿𝑎𝑡𝐴 𝑐𝑜𝑠𝐷𝑙𝑜 solve for side c or the GCD.
cos 𝑐 = cos 𝑎 cos 𝑏
+ sin 𝑎 sin 𝑏 cos 𝐶

𝑐𝑜𝐿𝑎𝑡𝐴 = 900 − 𝐿𝑎𝑡𝐴 Based on the formula, we need to


0 0
𝑐𝑜𝐿𝑎𝑡𝐴 = 90 − 35 27 = 54 33 ′ 0 ′ have coLat A and coLat B.
𝒄𝒐𝑳𝑨𝒕𝑨 = 𝟓𝟒𝟎 𝟑𝟑′ 𝑵 coLat A means the complement
of the latitude A
𝑐𝑜𝐿𝑎𝑡𝐵 = 900 − 𝐿𝑎𝑡𝐵
coLat B is the complement of the
𝑐𝑜𝐿𝑎𝑡 𝐵 = 900 − 370 48.5′ = 520 11.5′
Latitude B
𝒄𝒐𝑳𝒂𝒕𝑩 = 𝟓𝟐𝟎𝟏𝟏. 𝟓′ 𝑵
𝑐𝑜𝑠 𝐺𝐶𝐷 = 𝑐𝑜𝑠 𝑐𝑜𝐿𝑎𝑡𝐵 𝑐𝑜𝑠 𝑐𝑜𝐿𝑎𝑡𝐴 Substitute the following values in
+ 𝑠𝑖𝑛𝑐𝑜𝐿𝑎𝑡𝐵 𝑠𝑖𝑛 𝑐𝑜𝐿𝑎𝑡𝐴 𝑐𝑜𝑠𝐷𝑙𝑜 the formula.
𝑐𝑜𝑠 𝐺𝐶𝐷 = 𝑐𝑜𝑠 52 11.5′ 𝑐𝑜𝑠 540 33′
0 𝑐𝑜𝐿𝐴𝑡𝐴 = 540 33′ 𝑁
+ 𝑠𝑖𝑛520 11.5′ 𝑠𝑖𝑛 540 33′ 𝑐𝑜𝑠970 57′ 𝑐𝑜𝐿𝑎𝑡𝐵 = 520 11.5′ 𝑁
𝐷𝑙𝑜 = 970 57′ 𝐸

Confidential to United Marine Training Center, Inc. Page 3-184


Pre Sea Review Materials <course code>
Version Number
Deck
5 November 2018

𝑐𝑜𝑠 𝐺𝐶𝐷 = 0.355548013 + (−0.089016284) Evaluate using scientific


𝑐𝑜𝑠𝐺𝐶𝐷 = 0.266531728 calculator

𝐺𝐶𝐷 = 𝑐𝑜𝑠 −1(0.266531728) Apply inverse cosine function


0 0 ′
𝐺𝐶𝐷 = 74.54201177 = 74 32 31.24"
𝟔𝟎𝑵𝑴 Multiply by the conversion factor
𝑮𝑪𝑫 = (𝟕𝟒𝟎 𝟑𝟐′ 𝟑𝟏. 𝟐𝟒") ( )
𝟏𝟎 10 = 60𝑁𝑀 to express the
𝑮𝑪𝑫 = 𝟒𝟒𝟕𝟐. 𝟓𝟐 𝑵𝑴 answer in nautical miles
The shortest distance between
Yokohama and San Francisco,
CA is 𝟒𝟒𝟕𝟐. 𝟓𝟐 𝑵𝑴.

𝑐𝑜𝑠 𝑐𝑜𝐿𝑎𝑡𝐵 − 𝑐𝑜𝑠 𝑐𝑜𝐿𝑎𝑡𝐴 𝑐𝑜𝑠 𝐺𝐶𝐷 Use the law of cosine for sides to
𝐼/𝐶𝑜 = 𝑐𝑜𝑠 −1 ( )
𝑠𝑖𝑛 𝑐𝑜𝐿𝑎𝑡𝐴 𝑠𝑖𝑛 𝐺𝐶𝐷 calculate the initial course (A).
cos 𝑎
= cos 𝑏 cos 𝑐 + sin 𝑏 sin 𝑐 cos 𝐴
cos 𝑎 − cos 𝑏 cos 𝑐
𝐴 = 𝑐𝑜𝑠 −1 ( )
sin 𝑏 sin 𝑐

𝐼/𝐶𝑜 Substitute the following values in


0 ′ 0 ′ 0 ′
𝑐𝑜𝑠 52 11.5 − 𝑐𝑜𝑠 54 33 𝑐𝑜𝑠 74 32 31.24" the formula.
= 𝑐𝑜𝑠 −1 ( )
𝑠𝑖𝑛 540 33′ 𝑠𝑖𝑛 740 32′ 31.24" 𝑐𝑜𝐿𝐴𝑡𝐴 = 540 33′ 𝑁
𝑐𝑜𝐿𝑎𝑡𝐵 = 520 11.5′ 𝑁
𝐺𝐶𝐷 = 740 32′ 31.24"
0.61302197 − 0.154586352 Evaluate using scientific
𝐼/𝐶𝑜 = 𝑐𝑜𝑠 −1 ( )
0.785153947 calculator and apply GEMDAS
Rule.
0.458435618
𝐼/𝐶𝑜 = 𝑐𝑜𝑠 −1 ( )
0.785153947

𝐼/𝐶𝑜 = 𝑐𝑜𝑠 −1(0.583879912)

𝐼/𝐶𝑜 = 540 16′ 33.96"

𝑰/𝑪𝒐 = 𝑵 𝟓𝟒𝟎 𝟏𝟕′ 𝑬 The great circle calculations of


initial course result in quadrantal
notation as cardinal compass.
Corrected quadrant must be
named in order to avoid mistakes
when converting into three-figure
notation (0°-360°).
Initial Course is labelled with prefix
same as Lat A and suffix same as
Dlo.

Confidential to United Marine Training Center, Inc. Page 3-185


Pre Sea Review Materials <course code>
Version Number
Deck
5 November 2018

𝑰/𝑪𝒐 = 𝟎𝟓𝟒𝟎 𝑻 Since it lies in the first quadrant,


the true initial course is the same
as the course angle but to be
reported using the 3 digit notation

𝑐𝑜𝑠 𝑐𝑜𝐿𝑎𝑡𝐴 − 𝑐𝑜𝑠 𝑐𝑜𝐿𝑎𝑡𝐵 𝑐𝑜𝑠 𝐺𝐶𝐷 Use the law of cosine for sides to
𝐹/𝐶𝑜 = 𝑐𝑜𝑠 −1 ( )
𝑠𝑖𝑛 𝑐𝑜𝐿𝑎𝑡𝐵 𝑠𝑖𝑛 𝐺𝐶𝐷 calculate the final course (B).
cos 𝑏
= cos 𝑎 cos 𝑐 + sin 𝑎 sin 𝑐 cos 𝐵
cos 𝑏 − cos 𝑎 cos 𝑐
𝐵 = 𝑐𝑜𝑠 −1 ( )
sin 𝑎 sin 𝑐

𝐹/𝐶𝑜 Substitute the following values in


𝑐𝑜𝑠 540 33′ − 𝑐𝑜𝑠 520 11.5′ 𝑐𝑜𝑠 740 32′ 31.24" the formula.
= 𝑐𝑜𝑠 −1 ( )
𝑠𝑖𝑛 520 11.5′ 𝑠𝑖𝑛 740 32′ 31.24" 𝑐𝑜𝐿𝐴𝑡𝐴 = 540 33′ 𝑁
𝑐𝑜𝐿𝑎𝑡𝐵 = 520 11.5′ 𝑁
𝐺𝐶𝐷 = 740 32′ 31.24"

0.579992284 − 0.163389812 Evaluate using scientific


𝐹/𝐶𝑜 = 𝑐𝑜𝑠 −1 ( )
0.76148613 calculator and apply GEMDAS
0.416602472 Rule.
𝐹/𝐶𝑜 = 𝑐𝑜𝑠 −1 ( )
0.76148613

𝐹/𝐶𝑜 = 𝑐𝑜𝑠 −1(0.547091346)


𝐹/𝐶𝑜 = 560 49′ 56.3"
The great circle calculations of
𝟎
𝑭/𝑪𝒐 = 𝑺 𝟓 𝟔 𝟓𝟎 𝑬 ′ final course result in quadrantal
notation as cardinal compass.
Corrected quadrant must be
named in order to avoid mistakes
when converting into three-figure
notation (0°-360°).
Final course angle is labelled
contrary to Lat B and same as
DLO.

Since it lies in the second


𝑭/𝑪𝒐 = 𝟏𝟐𝟑 𝑻 𝟎 quadrant, the true course is
obtained by subtracting the
calculated value from 1800 .

Using mathematical approach, the following answers are obtained:


𝑮𝑪𝑫 = 𝟒𝟒𝟕𝟐. 𝟓𝟐 𝑵𝑴 𝑰/𝑪𝒐 = 𝟎𝟓𝟒𝟎 𝑻 𝑭/𝑪𝒐 = 𝟏𝟐𝟑𝟎 𝑻

Confidential to United Marine Training Center, Inc. Page 3-186


Pre Sea Review Materials <course code>
Version Number
Deck
5 November 2018

Alternative Solution (using the derived formulas for Great Circle Sailing)
Illustrate and label the terrestrial triangle.

𝑎
𝐷𝑙𝑜
𝑐𝑜 𝑙𝑎𝑡𝐵
𝑏
𝑐𝑜 𝑙𝑎𝑡𝐴 𝐹/𝐶0
𝐼/𝐶0

𝑐 𝐵
𝐴 𝐺𝐶𝐷

𝑙𝑎𝑡𝐵

𝑙𝑎𝑡𝐴

Solution for 𝐺𝐶𝐷, 𝐼/𝐶𝑜 and 𝐹/𝐶𝑜

𝐷𝑙𝑜 = 1220 24′ + 1390 39′ = 2620 03′ Solve for DLO using the convention known
by seafarers.
Note:
𝐷𝑙𝑜 = 3600 −2620 03′ = 970 57′
The reason why the convention work is the
𝐷𝑙𝑜 = 970 57′ 𝐸
mathematical approach presented above.
If longitudes have same names, subtract
but if they different names, add.
Subtract 3600 if the result is more than
1800 and affix the sign of longitude 1.

𝐺𝐶𝐷 = 𝑐𝑜𝑠 −1(𝑐𝑜𝑠 𝐿𝑎𝑡𝐴 𝑐𝑜𝑠 𝐿𝑎𝑡𝐵 𝑐𝑜𝑠 𝐷𝑙𝑜 Use the derived formula for calculating the
GCD.
+ 𝑠𝑖𝑛 𝐿𝑎𝑡𝐴 𝑠𝑖𝑛 𝐿𝑎𝑡𝐵)

𝐺𝐶𝐷 = 𝑐𝑜𝑠 −1(𝑐𝑜𝑠 350 27′ 𝑐𝑜𝑠 370 48. 5′ 𝑐𝑜𝑠 970 57′ From the given positions we know that
0 ′ 0 ′)
+ 𝑠𝑖𝑛 35 27 𝑠𝑖𝑛 37 48. 5 𝐿𝑎𝑡 𝐴 = 350 27′ and 𝐿𝑎𝑡 𝐵 = 370 48. 5′ ,
Substitute these values to the formula
together with the value of 𝐷𝑙𝑜 = 970 57′ 𝐸.

Confidential to United Marine Training Center, Inc. Page 3-187


Pre Sea Review Materials <course code>
Version Number
Deck
5 November 2018

𝑐𝑜𝑠 𝐺𝐶𝐷 = 0.355548013 + (−0.089016284) Use scientific calculator to evaluate and


apply inverse cosine function.
𝑐𝑜𝑠𝐺𝐶𝐷𝑁𝑀 = (0.266531728)
𝐺𝐶𝐷𝑁𝑀 = 𝑐𝑜𝑠 −1(0.266531728)
𝐺𝐶𝐷 = 74.542011770 = 740 32′ 31.24"
𝟔𝟎𝑵𝑴 Multiply by the conversion factor 10 =
𝑮𝑪𝑫 = (𝟕𝟒𝟎 𝟑𝟐′ 𝟑𝟏. 𝟐𝟒") ( )
𝟏𝟎 60𝑁𝑀 to express the answer in nautical
miles.
𝑮𝑪𝑫 = 𝟒𝟒𝟕𝟐. 𝟓𝟐 𝑵𝑴
𝑠𝑖𝑛 𝐿𝑎𝑡𝐵 − 𝑠𝑖𝑛 𝐿𝑎𝑡𝐴 𝑐𝑜𝑠𝐺𝐶𝐷 Solve for initial course using the formula
𝐼/𝐶𝑜 = 𝑐𝑜𝑠 −1 ( ) derived from the law of cosine for sides as
𝑐𝑜𝑠 𝐿𝑎𝑡𝐴 𝑠𝑖𝑛 𝐺𝐶𝐷
it is more accurate than the formula
derived from the law of sine. The more
complicated the formula is ,the more
accurate.

𝐼/𝐶𝑜 Substitute the values of the departure and


0 ′ 0 ′ 0 ′ arrival latitudes and the GCD in angular
𝑠𝑖𝑛 37 48. 5 − 𝑠𝑖𝑛 35 27 𝑐𝑜𝑠74 32 31.24"
= 𝑐𝑜𝑠 −1 ( ) units
𝑐𝑜𝑠 350 27′ 𝑠𝑖𝑛 740 32′ 31.24"
𝐿𝑎𝑡 𝐴 = 350 27′ and 𝐿𝑎𝑡 𝐵 = 370 4𝟖. 𝟓′ ,
𝑮𝑪𝑫 = 𝟕𝟒𝟎𝟑𝟐′ 𝟑𝟏. 𝟐𝟒"
0.61302197 − 0.154586352 Evaluate using scientific calculator and
𝐼/𝐶𝑜 = 𝑐𝑜𝑠 −1 ( ) follow the GEMDAS Rule.
0.785153947

0.458435618
𝐼/𝐶𝑜 = 𝑐𝑜𝑠 −1 ( )
0.785153947

𝐼/𝐶𝑜 = 𝑐𝑜𝑠 −1(0.583879912)

𝐼/𝐶𝑜 = 540 16′ 33.96"


𝑰/𝑪𝒐 = 𝑵 𝟓𝟒𝟎 𝟏𝟕′ 𝑬 Apply the naming of the I/Co.
Initial Course is labelled with prefix same
as Lat A and suffix same as DLO

𝑰/𝑪𝒐 = 𝟎𝟓𝟒𝟎 𝑻 Since it lies in the first quadrant, the true


course is the same as the course angle
but 3 digit notation must be used.

𝑠𝑖𝑛 𝐿𝑎𝑡𝐴 − 𝑠𝑖𝑛 𝐿𝑎𝑡𝐵 𝑐𝑜𝑠𝐺𝐶𝐷 Solve for final course using the formula
𝐹/𝐶𝑜 = 𝑐𝑜𝑠 −1 ( )
𝑐𝑜𝑠 𝐿𝑎𝑡𝐵 𝑠𝑖𝑛 𝐺𝐶𝐷 derived from the law of cosine for sides.

Substitute the values of the departure and


arrival latitudes and the GCD in angular
𝐹/𝐶𝑜
units
𝑠𝑖𝑛 350 27′ − 𝑠𝑖𝑛 370 48.5′ 𝑐𝑜𝑠740 32′ 31.24"
= 𝑐𝑜𝑠 −1 ( ) 𝑳𝒂𝒕 𝑨 = 𝟑𝟓𝟎 𝟐𝟕′ and 𝑳𝒂𝒕 𝑩 = 𝟑𝟕𝟎 𝟒𝟖. 𝟓′ ,
𝑐𝑜𝑠 370 48.5′ 𝑠𝑖𝑛 740 32′ 31.24"
𝑮𝑪𝑫 = 𝟕𝟒𝟎𝟑𝟐′ 𝟑𝟏. 𝟐𝟒"

0.579992284 − 0.163389812
𝐹/𝐶𝑜 = 𝑐𝑜𝑠 −1 ( )
0.76148613 Evaluate using scientific calculator and
follow the GEMDAS Rule.
0.416602472
𝐹/𝐶𝑜 = 𝑐𝑜𝑠 −1 ( )
0.76148613

Confidential to United Marine Training Center, Inc. Page 3-188


Pre Sea Review Materials <course code>
Version Number
Deck
5 November 2018

𝐹/𝐶𝑜 = 𝑐𝑜𝑠 −1(0.547091346)

𝐹/𝐶𝑜 = 560 49′ 56.3"

Apply the naming of the F/Co.


𝐹/𝐶𝑜 = 𝑆 5 60 50′ 𝐸 Final course angle is labelled contrary to
Lat B and same as Dlo.

Since it lies in the second quadrant, the


𝟎 true course is obtained by subtracting the
𝑭/𝑪𝒐 = 𝟏𝟐𝟑 𝑻 calculated value from 1800 .

The answers to the problem are:


𝑮𝑪𝑫 = 𝟒𝟒𝟕𝟐. 𝟓𝟐 𝑵𝑴 𝑰/𝑪𝒐 = 𝟎𝟓𝟒𝟎 𝑻 𝑭/𝑪𝒐 = 𝟏𝟐𝟑𝟎 𝑻

Example 3-117

Find the shortest distance when sailing from 𝑨 (𝟑𝟑𝟎 𝟐𝟐′ 𝑺, 𝟏𝟏𝟑𝟎 𝟎𝟖′ 𝑬) to 𝑩(𝟏𝟎𝟎 𝟓𝟏′ 𝑺, 𝟎𝟒𝟗𝟎 𝟏𝟔′ 𝑬).

Solution (using the general formula for spherical triangles),


Illustrate and label the terrestrial triangle

𝐹/𝐶0

𝐵
𝐺𝐶𝐷
𝐹/𝐶0 𝐴

𝐼/𝐶0 𝑏
𝑎 𝑐𝑜 𝑙𝑎𝑡𝐴
𝑐𝑜 𝑙𝑎𝑡𝐵
𝐷𝑙𝑜

Calculation for 𝐺𝐶𝐷

𝐷𝑙𝑜 = 𝑙𝑜𝑛𝑔2 − 𝑙𝑜𝑛𝑔1 Solve for DLO


𝐷𝑙𝑜 = 0490 16′ − (1130 08′ ) Note: East is (+)
0 ′ 0 ′
𝐷𝑙𝑜 = 049 16 + (−113 08 ) Apply subtraction of integers.
𝐷𝑙𝑜 = −0630 52′ Change the sign of the subtrahend from
𝟎 ′ (1130 08′ )to (−1130 08′ )) and proceed to
𝑫𝑳𝑶 = 𝟎𝟔𝟑 𝟓𝟐 𝑾 addition.

Confidential to United Marine Training Center, Inc. Page 3-189


Pre Sea Review Materials <course code>
Version Number
Deck
5 November 2018

Since the integers have unlike signs,


subtract their absolute values (the magnitude
of a real number without regard to its sign)
and copy the sign of the bigger absolute
value.

𝑐𝑜𝑠 𝐺𝐶𝐷 = 𝑐𝑜𝑠 𝑐𝑜𝐿𝑎𝑡𝐵 𝑐𝑜𝑠 𝑐𝑜𝐿𝑎𝑡𝐴 Use the law of cosine for sides to solve for
side c or the GCD.
+ 𝑠𝑖𝑛𝑐𝑜𝐿𝑎𝑡𝐵 𝑠𝑖𝑛 𝑐𝑜𝐿𝑎𝑡𝐴 𝑐𝑜𝑠𝐷𝑙𝑜
𝑐𝑜𝐿𝑎𝑡𝐴 = 900 − 𝐿𝑎𝑡𝐴 Based on the formula, we need to have
coLat A and coLat B I addition to DLO
= 900
− 330 22′ = 𝟓𝟔𝟎 𝟑𝟖′ 𝑺
𝑐𝑜𝐿𝑎𝑡𝐵 = 900 − 𝐿𝑎𝑡𝐵 = 900 − 100 51′
= 𝟕𝟗𝟎 𝟗′ 𝑺
𝑐𝑜𝑠 𝐺𝐶𝐷 = 𝑐𝑜𝑠 790 9′ 𝑐𝑜𝑠 560 38′ Substitute the following values: 𝒄𝒐𝑳𝑨𝒕𝑨 =
𝟓𝟔𝟎 𝟑𝟖′
+ 𝑠𝑖𝑛790 9′ 𝑠𝑖𝑛 560 38′ 𝑐𝑜𝑠 630 52′
𝒄𝒐𝑳𝒂𝒕𝑩 = 𝟕𝟗𝟎 𝟗′
𝑫𝑳𝑶 = 𝟎𝟔𝟑𝟎 𝟓𝟐′

𝑐𝑜𝑠𝐺𝐶𝐷 = 0.464813485 Evaluate and apply inverse cosine function


−1 (0.464813485)
𝐺𝐶𝐷 = 𝑐𝑜𝑠
𝐺𝐶𝐷 = 62.301848160 = 620 18′ 6.65"
60𝑁𝑀 Multiply by the conversion factor , 10 =
𝐺𝐶𝐷 = 620 18′ 6.65" ( )
10 60 𝑁𝑀to express the GCD in nautical miles.

𝑮𝑪𝑫 = 𝟑𝟕𝟑𝟖. 𝟏𝟏 𝑵𝑴 The shortest distance from position A to


position B is 3738.11 NM.

Alternative Solution (using the derived formulas for Great Circle Sailing)
Illustrate and label the terrestrial triangle

𝐹/𝐶0

𝐵
𝐺𝐶𝐷
𝐹/𝐶0 𝐴

𝐼/𝐶0 𝑏
𝑎 𝑐𝑜 𝑙𝑎𝑡𝐴
𝑐𝑜 𝑙𝑎𝑡𝐵
𝐷𝑙𝑜

Confidential to United Marine Training Center, Inc. Page 3-190


Pre Sea Review Materials <course code>
Version Number
Deck
5 November 2018

Calculation for GCD

𝐷𝑙𝑜 = 1130 08′ − 0490 16′ = 𝟎𝟔𝟑𝟎 𝟓𝟐′ 𝑾 Since the longitudes have the same names
(both South), subtract and analyze the
drawing / given positions to get the direction
of DLO

𝐺𝐶𝐷 = 𝑐𝑜𝑠 −1 (𝑐𝑜𝑠 𝐿𝑎𝑡𝐴 𝑐𝑜𝑠 𝐿𝑎𝑡𝐵 𝑐𝑜𝑠 𝐷𝑙𝑜 Use the derived formula for GCD
+ 𝑠𝑖𝑛 𝐿𝑎𝑡𝐴 𝑠𝑖𝑛 𝐿𝑎𝑡𝐵)
𝐺𝐶𝐷 Substitute these values to the formula
= 𝑐𝑜𝑠 −1(𝑐𝑜𝑠 330 22′ 𝑐𝑜𝑠 100 51′ 𝑐𝑜𝑠 630 52′ 𝐿𝑎𝑡 𝐴 = 330 22′ , 𝐿𝑎𝑡 𝐵 = 100 51′ , 𝐷𝑙𝑜 =
630 52′
+ 𝑠𝑖𝑛 330 22′ 𝑠𝑖𝑛 100 51′ )
𝐺𝐶𝐷 = 𝑐𝑜𝑠 −1 (0.464813485)
𝐺𝐶𝐷 = 62.301848160 = 620 18′ 6.65" Evaluate
60𝑁𝑀 Multiply by the conversion factor , 10 =
𝐺𝐶𝐷 = 620 18′ 6.65" ( )
10 60 𝑁𝑀to express the GCD in nautical miles.
𝑮𝑪𝑫 = 𝟑𝟕𝟑𝟖. 𝟏𝟏 𝑵𝑴 The shortest distance from position A to
position B is 3738.11 NM.

Confidential to United Marine Training Center, Inc. Page 3-191


Pre Sea Review Materials <course code>
Version Number
Deck
5 November 2018

Supplementary Exercises
Solve the following the problems
Answer

1] Determine the shortest distance (GCD) between 𝐷𝑙𝑜 = 1610 52′ 𝐸


𝑀𝑎𝑛𝑖𝑙𝑎(140 36′ 𝑁, 1210 05′ 𝐸) and 𝑮𝑪𝑫 = 𝟕𝟒𝟑𝟐. 𝟔𝟓 𝑵𝑴
𝑊𝑎𝑠ℎ𝑖𝑛𝑔𝑡𝑜𝑛(380 54′ 𝑁, 0770 03′ 𝑊).

2] Find the distance and initial course of great circle 𝐷𝑙𝑜 = 800 𝐸
sailing from 𝐴(170 00′ 𝑆, 1700 00′ 𝐸) to 𝑮𝑪𝑫 = 𝟓𝟐𝟒𝟕. 𝟐 𝑵𝑴
𝐵(220 00′ 𝑁, 1100 00′ 𝑊)
𝐼/𝐶𝑜 = 𝑆113.90 𝐸 = 𝟎𝟔𝟔𝟎 𝑻

3] A vessel sails on a great circle from 𝐷𝑙𝑜 = 350 𝐸


𝐴(400 00′ 𝑁, 0500 00′ 𝑊) to 𝐵(430 00′ 𝑁, 0150 00′ 𝑊). Find 𝐺𝐶𝐷 = 1571.6 𝑁𝑀
the initial course.
𝐼/𝐶𝑜 = 𝑁 71.870 𝐸 = 𝟎𝟕𝟐𝟎 𝑻

4] Given the following positions below, find the Great 𝐷𝑙𝑜 = 480 58′ 𝑊
Circle Distance, initial course and final course: 𝑮𝑪𝑫 = 𝟏𝟔𝟗𝟔. 𝟓 𝑵𝑴
𝐷𝑒𝑝𝑎𝑟𝑡𝑢𝑟𝑒 𝑝𝑜𝑠𝑖𝑡𝑖𝑜𝑛 = 𝑃1 (560 20′ 𝑁, 0080 12′ 𝑊) 𝐼/𝐶𝑜 = 𝑁 77.40 𝑊 = 𝟐𝟖𝟑𝟎 𝑻
𝐴𝑟𝑟𝑖𝑣𝑎𝑙 𝑝𝑜𝑠𝑖𝑡𝑖𝑜𝑛 = 𝑃2 (520 12′ 𝑁, 0570 10′ 𝑊) 𝐹/𝐶𝑜 = 𝑆 66.00 𝑊 = 𝟐𝟒𝟐𝟎 𝑻

5] Find the great circle distance and the initial course on 𝐷𝑙𝑜 = 320 25′ 𝑊
a voyage from Vancouver to Hawaii between the 𝑮𝑪𝑫 = 𝟐𝟐𝟒𝟕. 𝟓 𝑵𝑴
positions (480 20′ 𝑁, 1250 00′ 𝑊) and
(210 15′ 𝑁, 1570 25′ 𝑊). 𝐼/𝐶𝑜 = 𝑁 1240 46′ 𝑊 = 𝟐𝟑𝟓𝟎 𝑻

6] Two places A and B on the earth have the following 𝐷𝑙𝑜 = 400 𝐸
latitudes and longitudes: 𝐺𝐶𝐷 = 3244.08 𝑁𝑀
𝐴(400 𝑁, 0180 𝐸) and 𝐵(000 𝑁, 0580 𝐸). Find the angle 𝑰/𝑪𝒐 = 𝟏𝟐𝟕𝟎 𝑻
of departure from A to B of the great circle route.

Confidential to United Marine Training Center, Inc. Page 3-192


Pre Sea Review Materials <course code>
Version Number
Deck
5 November 2018

Section 4
Physics

Measurement

Measurement
Measurement is a complete numerical description of an attribute that is composed of a magnitude
and a unit. It is a process of detecting an unknown physical quantity by using standard quantity (KC,
2018).For example, a ruler is used to find the length of a notebook. The result is 15.5 cm.
The process of measurement was done where:
 The unknown physical quantity is the length of the notebook
 The standard quantity is the ruler
 The magnitude is 15.5
 The unit of the length of the notebook is cm (centimeter)

System of Measurement
System of measurement is a collection of units of measurement and rules relating them to each
other (Aaravjindal, 2018). There are two main systems: International system (SI) and English
system.

1. International System (SI – Système International)


 A system of measuring based on the meter (length), kilogram (mass), and second (time)
 Decimal-based; uses a series of prefixes for the multiple of a basic unit

Base Quantity Unit Symbol

Length meter m
Mass kilogram kg
Time second s
Electric Current ampere A
Temperature kelvin K
Amount of Substance mole mol
Luminous Intensity candela cd
Table 4-1: SI base units - - Source: Karizsa Denise D. Salvador

Confidential to United Marine Training Center, Inc. Page 4-1


Pre Sea Review Materials <course code>
Version Number
Deck
5 November 2018

Derived
Unit Symbol Equivalent SI Unit
Quantity

Frequency hertz Hz s-1


Force newton N kg∙m∙s-2
Pressure Pascal Pa N/m2
Energy joule J N∙m
Power watt W J/s
Electric
coulomb C A∙s
Charge
Electric
volt V W/A
Potential
Electric
ohm V/A
Resistance
Table 4-2: SI derived units - - Source: Karizsa Denise D. Salvador

Factor Prefix Symbol Numerical Value

1012 tera T 1 000 000 000 000


109 giga G 1 000 000 000
106 mega M 1 000 000
103 kilo k 1 000
102 hecto h 100
101 deca da 10
10−1 deci d 0.1
10−2 centi c 0.01
10−3 milli m 0.001
10−6 micro 0.000 001
1012 tera T 1 000 000 000 000
Table 4-3: SI prefixes - - Source: Karizsa Denise D. Salvador

2. English System
 System of measuring based on the foot (length), pound (mass), and second (time)

Base Quantity Unit Symbol

Length meter m
Mass kilogram kg
Time second s
Electric Current ampere A
Temperature kelvin K
Amount of
mole mol
Substance
Luminous
candela cd
Intensity
Table 4-4: English units - Source: Karizsa Denise D. Salvador

Confidential to United Marine Training Center, Inc. Page 4-2


Pre Sea Review Materials <course code>
Version Number
Deck
5 November 2018

Unit Consistency and Conversion


Equations are used to express relationships among physical quantities that are represented by
algebraic symbols. Each algebraic symbol denotes both a number and a unit (Young, Freedman, &
Ford, 2012). An equation must always be dimensionally consistent – units of the same quantities
must be the same.

Length

1 m = 100 cm = 39.97 in = 3.281 ft.


1 cm = 10 mm = 0.3937 in
1 km = 1000 m = 0.6214 mi
1 NM = 6076 ft. = 1.152 mi = 1.852 km
1 in = 2.54 cm
1 mi = 5280 ft. = 1.609 km
1 ft. = 12 in = 0.3048 m = 30.48 cm

Pressure

1 atm = 1.013 x 105 Pa


= 1.013 bar
= 760 mmHg

Volume

1 tsp = 5 mL
1 tbsp. = 15 mL
1 fl oz. = 30 mL
1 cup = 0.24 L
1 pint = 0.47 L
1 gal = 3.8 L
1 ft3 = 0.028 m3

Mass

1 oz. = 28 g
1 lb. = 0.45 kg
1 T = 1000 kg
1 kg = 0.0685 slug

Confidential to United Marine Training Center, Inc. Page 4-3


Pre Sea Review Materials <course code>
Version Number
Deck
5 November 2018

Speed

1 knot = 1 NM/hr
= 1.852 km/hr
= 0.514 m/s

Force

1N = 0.2248 lb.
= 105 dyne
Table 4-5: Conversion Factors - Source: Karizsa Denise D. Salvador

Example 4-1

The fastest ship in service today is Cunard’s Queen Mary 2. Designed as an ocean liner, she
needs speed in order to handle the variable conditions of the North Atlantic. Her official top speed
is 28 knots but the ship has done close to 30 knots. Express her top speed in m/s.
Solution

From the table of conversion factors, 𝑚


1 𝑘𝑛𝑜𝑡 = 0.514
𝑠
𝑚
Therefore, 0.514
28 𝑘𝑛𝑜𝑡𝑠 ( 1 𝑘𝑛𝑜𝑡𝑠 )
𝑚
= 14.39
𝑠

1
Note: The unit 𝑘𝑛𝑜𝑡 cancels knots and the product has a
𝑚
unit of .
𝑠

Alternate Solution
From the table of conversion factors, 1 𝑛𝑚 = 1852 𝑚
1 ℎ𝑟 = 3600 𝑠
𝑛𝑚
1 𝑘𝑛𝑜𝑡 = 1
ℎ𝑟
𝑛𝑚
So, 28 𝑘𝑛𝑜𝑡𝑠 = 28 ℎ𝑟

Therefore, 𝑛𝑚 1852 𝑚 1 ℎ𝑟
28 ( )( )
ℎ𝑟 1 𝑛𝑚 3600 𝑠
𝑚
= 14.4
𝑠

1 1
Note: The unit cancels NM and unit hr cancels .
𝑛𝑚 ℎ𝑟
𝑚
The product has a unit of 𝑠 .

Confidential to United Marine Training Center, Inc. Page 4-4


Pre Sea Review Materials <course code>
Version Number
Deck
5 November 2018

Example 4-2

The biggest port in the world based on cargo throughput is the Port of Shanghai which covers an
area of 3,619 km2. What is its area in square miles?
Solution
From the table of conversion factors, 1 𝑚𝑖 = 1.609 𝑘𝑚
2
Therefore, 1 𝑚𝑖
3619 𝑘𝑚2 ( )
1.609 𝑘𝑚
1 𝑚𝑖 2
= 3619 𝑘𝑚2 ( )
2.588881 𝑘𝑚2
= 1397.90 𝑚𝑖 2

Note: The conversion factor is squared to be able to


1
form the unit and eventually cancelling
𝑘𝑚 2

𝑘𝑚2. The product has a unit of 𝑚𝑖 2 .

Confidential to United Marine Training Center, Inc. Page 4-5


Pre Sea Review Materials <course code>
Version Number
Deck
5 November 2018

Supplementary Exercises
Solve the following problems
Answer
1] Convert 4,100 millimeters to kilometers. 0.0041 km

2] Express 5 feet 4 inches in centimeters. 162.56 cm

𝑛𝑚 𝑚 𝑚
3] Change 1.9 to 2. 2.72 x 10-4
ℎ𝑟2 𝑠 𝑠2

Confidential to United Marine Training Center, Inc. Page 4-6


Pre Sea Review Materials <course code>
Version Number
Deck
5 November 2018

Vector Addition

Introduction of a variety of quantities that are used to describe the physical world is involved in the
study of motion. All these quantities can be divided into two categories – scalars and vectors. A
variety of mathematical operations can be performed with and upon vectors. One such operation is
the addition of vectors.

Scalar Quantity
 A quantity that is fully described by magnitude alone
 Examples of scalar quantities include distance, speed, mass, and time
 Scalar addition is adding the magnitudes arithmetically
Example: Mass A is 55 kg and Mass B is 77 kg. Find the total mass.
Answer: Total Mass = 55 kg + 77 kg = 132 kg

Vector Quantity
 A quantity that is fully described by both magnitude and direction
 Examples of vector quantities include displacement, velocity, weight, and force

Resultant Vector
The resultant vector is the sum of added two or more vectors. It is a single vector that has the same
effect as all the others put together.

Vector Addition (One-dimension)


 Two or more vectors can be added to determine the resultant vector
 Rules for summing vectors are the following:

Note: Same principle goes to vertical direction

Confidential to United Marine Training Center, Inc. Page 4-7


Pre Sea Review Materials <course code>
Version Number
Deck
5 November 2018

Kinematics

Kinematics is a branch of classical mechanics concerned with describing the motion of a body or
system of bodies using words, diagrams, numbers, graphs and equations without consideration of
the forces involved – causes and effects of motions.

Distance
It is a scalar quantity that refers to “how much ground an object has covered” during its motion

Displacement
It is a vector quantity that refers to “how far out of place an object is”; it is the object’s overall change
in position

Figure 4-1: Distance and displacement - Source: www.marinetraffic.com

Speed
It is a scalar quantity that refers to “how fast an object is moving”; Rate of which an object covers
distance
 Instantaneous Speed is the speed at any given instant time
 Average Speed is the average of all instantaneous speeds and is calculated using the
total distance and time interval ratio
𝒅 Formula for Average Speed
𝒗=
𝒕
Where:
v = Average speed
d = Total displacement
t = Time interval

Confidential to United Marine Training Center, Inc. Page 4-8


Pre Sea Review Materials <course code>
Version Number
Deck
5 November 2018

Velocity
It is a vector quantity which describes the rate at which the position changes
 Instantaneous Velocity is the velocity at any given instant time
 Average Velocity is the average of all instantaneous velocities. It is calculated using the
total displacement and time interval ratio

⃑𝒅⃑ Formula for Average Velocity


⃑⃑ =
𝒗
𝒕 Where:
𝑣⃑ = Average velocity
𝑑⃑ = Total displacement covered
t = Time interval

Acceleration
It is a vector quantity that refers to how quickly velocity changes. Velocity of an object can change
by changing its speed, by changing its direction, or by changing both speed and direction
The formula for getting acceleration is

∆𝒗 ⃑⃑ Formula for Acceleration


⃑⃑ =
𝒂
𝒕 Where:
⃑⃑⃑⃑⃑
𝒗𝒇 − 𝒗 ⃑⃑⃑⃑𝒊 𝑎⃑ = Acceleration
⃑⃑ =
𝒂
𝒕 ∆𝑣⃑ = Change in velocity
⃑𝑣⃑⃑𝑖 = Initial velocity
𝑣
⃑⃑⃑⃑⃑𝑓 = Final velocity
t = Time interval

Example 4-3

A vehicle’s odometer reads 22,722 km at the start of a trip and 23,010 at the end. The trip took 5.5
hours. What was the vehicle’s average speed in kilometers per hour? In meters per second?

Solution
Using the formula for average speed, 𝑑
𝑣=
𝑡
Where, 𝑑 = 23010 𝑘𝑚 − 22722 𝑘𝑚
𝑑 = 288 𝑘𝑚
And, 𝑡 = 5.5 ℎ𝑟

Hence, 288 𝑘𝑚
𝑣=
5.5 ℎ𝑟
𝑘𝑚
𝑣 = 52.36
ℎ𝑟
Converting the answer to meters per 𝑘𝑚 1000 𝑚 1 ℎ𝑟
𝑣 = (52.36 )( )( )
ℎ𝑟 1 𝑘𝑚 3600 𝑠
second,
𝑚
𝑣 = 14.54
𝑠

Confidential to United Marine Training Center, Inc. Page 4-9


Pre Sea Review Materials <course code>
Version Number
Deck
5 November 2018

Example 4-4

A runner makes one lap around 250-m track in a time of 32 s. What were the runner’s average
speed and average velocity?
Solution
Average speed
Using the formula for average speed, 𝑑
𝑣=
𝑡
Where, 𝑑 = 250 𝑚
And, 𝑡 = 32 𝑠

Hence, 250 𝑚
𝑣=
32 𝑠
𝑚
𝑣 = 7.81
𝑠

Average velocity
Using the formula for average velocity, 𝑑⃑
𝑣⃑ =
𝑡
Because the run ended at the starting 𝑑⃑ = 0
point, the displacement from starting
point to end point is zero

And, 𝑡 = 32 𝑠
Hence, 0𝑚
𝑣⃑ =
32 𝑠
𝑣⃑ = 0

Example 4-5

A tanker vessel accelerates from rest to a velocity of 9 knots along the positive x-axis in 7.5 min.
What is the acceleration of the vessel?
Solution
Using the formula for acceleration, 𝑣
⃑⃑⃑⃑⃑𝑓 − ⃑𝑣⃑⃑𝑖
𝑎⃑ =
𝑡
Where, ⃑𝑣⃑⃑𝑖 = 0
𝑛𝑚𝑖
𝑣
⃑⃑⃑⃑⃑𝑓 = 9 𝑘𝑛 𝑜𝑟
ℎ𝑟
𝑡 = 7.5 𝑚𝑖𝑛
Convert time from min to hr for dimensional 1 ℎ𝑟
𝑡 = (7.5 𝑚𝑖𝑛) ( )
60 𝑚𝑖𝑛
consistency
= 0.125 ℎ𝑟
So, 𝑛𝑚𝑖
9 ℎ𝑟 − 0
𝑎⃑ =
0.125 ℎ𝑟
𝑛𝑚𝑖
𝑎⃑ = 72
ℎ𝑟 2

Confidential to United Marine Training Center, Inc. Page 4-10


Pre Sea Review Materials <course code>
Version Number
Deck
5 November 2018

Supplementary Exercises
Solve the following problems
Answer
1] A container ship slows down from a velocity of 15 knots to rest in 𝑚
𝑚 −7.15 𝑥 10−3 2
18 minutes. What is the acceleration of the ship in 𝑠2? 𝑠

2] Starting with a velocity of 5 knots along positive x-axis, a vessel 15 𝑘𝑡𝑠


𝑛𝑚𝑖
accelerates for 10 minutes with an acceleration of 60 ℎ𝑟2 . What
is its velocity at the end of the period of 10 minutes?

3] How long, in hours, does it take to accelerate a tugboat from a 0.12 ℎ𝑟


velocity of 10 knots to 19 knots along negative x-axis at an
𝑛𝑚𝑖
acceleration of −78 2 ?
ℎ𝑟

Confidential to United Marine Training Center, Inc. Page 4-11


Pre Sea Review Materials <course code>
Version Number
Deck
5 November 2018

Forces and Newton’s Laws

Force
 A vector quantity
 A push or a pull
 An interaction between two objects or between an object and its environment
𝑘𝑔 𝑚
 SI unit is 𝑠2
or N

Two Categories of Forces


1. Contact Forces – two interacting objects are perceived to be physically in contact
a. ⃑⃑𝒂𝒑𝒑 )
Applied Force (𝑭
It is a force that is applied to an object by a person or another object. (Types of Forces,
1996-2018)

𝐹⃑𝑎𝑝𝑝

(a)

𝐹⃑𝑎𝑝𝑝

(b)

Figure 4-2: Applied (a) pushing force and (b) pulling force - Source: Karizsa Denise D. Salvador

b. ⃑⃑𝑵 )
Normal Force (𝑭
It is the support force exerted upon an object that is in contact with another stable object
(Types of Forces, 1996-2018). When an object rests or pushes on a surface, the surface
exerts a push on it that is directed perpendicular to the surface. For example, if a 20’
container is resting upon the deck, then the deck surface is exerting an upward force upon
the container in order to support its weight.
For a completely horizontal surface:

𝐹⃑𝑁 = 𝐹⃑𝑔

Confidential to United Marine Training Center, Inc. Page 4-12


Pre Sea Review Materials <course code>
Version Number
Deck
5 November 2018

𝐹⃑𝑁

Figure 4-3: Normal force -- Source: Karizsa Denise D. Salvador

c. ⃑⃑𝒇 )
Friction Force (𝑭
It is the force exerted by a surface as an object moves across it or makes an effort to
move across it. Friction results from two surfaces being pressed together closely, causing
intermolecular attractive forces between molecules of different surfaces. As such, friction
depends upon the nature of the two surfaces and upon the degree to which they are
pressed together. The amount of friction force that a surface can exert upon an object can
be calculated using:

𝑭𝒇 =  𝑭𝑵 Formula for Friction Force


Where:
Ff = Friction force in Newton
µ = Coefficient of friction
FN = Normal force in Newton

Motion

𝐹⃑𝑓

Figure 4-4: Friction force - Source: Karizsa Denise D. Salvador

d. ⃑⃑𝒂𝒊𝒓 )
Air Resistance Force (𝑭
It is a special type of frictional force that acts upon objects as they travel through air. This
force will be frequently neglected due to its negligible magnitude (and due to the fact that
its value is mathematically difficult to predict).

e. ⃑⃑𝑻 )
Tension Force (𝑭
It is the force that is transmitted through a string, rope, cable or wire when it is pulled tight
by forces acting from opposite ends. The tension force is directed along the length of the
wire and pulls equally on the objects on the opposite ends of the wire (Types of Forces,
1996-2018).

Confidential to United Marine Training Center, Inc. Page 4-13


Pre Sea Review Materials <course code>
Version Number
Deck
5 November 2018

𝐹⃑𝑇

Figure 4-5: Tension force - Source: Karizsa Denise D. Salvador

f. ⃑⃑𝒔𝒑𝒓𝒊𝒏𝒈 )
Spring Force (𝑭
It is the force exerted by a compressed or stretched spring upon any object that is
attached to it. An object that compresses or stretches a spring is always acted upon by a
force that restores the spring to its rest or equilibrium position (Types of Forces, 1996-
2018).

2. Non-contact Forces – two interacting objects are not in physical contact with each other, yet
are able to exert a push or pull despite their physical separation (The Meaning of Force, 1996-
2018).
a. ⃑⃑𝒈 )
Gravitational Force (𝑭
It is the force with which the earth, moon, or other massively large object attracts another
object towards itself (Types of Forces, 1996-2018). It is the weight of an object as given by
the equation:

⃑𝑭⃑𝒈 = 𝒎𝒈 Formula for Gravitational Force


Where:
𝐹⃑𝑔 = Gravitational Force in Newton
m = Mass in kilogram
g = Acceleration due to gravity
= 9.8 𝑚/𝑠 2

𝐹⃑𝑔

Figure 4-6: Gravitational force - Source: Karizsa Denise D. Salvador

Note: Do not confuse weight with mass. Weight is the force of gravity acting upon an object. Mass
is the amount of matter that is contained by the object.

b. Electrical Force
The attractive or repulsive interaction between electrical charges

c. Magnetic Force
The attractive or repulsive interaction between electrically charged particles due to their
motion

Confidential to United Marine Training Center, Inc. Page 4-14


Pre Sea Review Materials <course code>
Version Number
Deck
5 November 2018

Net Force
The sum of all the forces acting upon an object; also called unbalanced force

Newton’s Laws of Motion

1. Law of Inertia
This law states that an object at rest stays at rest and an object in motion stays in motion with
constant velocity and zero acceleration unless acted upon by an unbalanced force.

Newton’s first law of motion predicts the behavior of objects for which all existing forces are
balanced (Newton's Second Law, 1996-2018). The tendency of a body at rest to remain at rest and
the tendency of a body to keep moving result from a property called inertia. Inertia is the resistance
an object has to a change in its state of motion and it varies directly with mass.

Figure 4-7: An object at rest will remain at rest – Source: https://fairplay.ihs.com/bulk/article/4293706/divergent-fortunes-


forecast-for-tanker-and-bulk-markets

Figure 4-8: Unless acted upon by an unbalanced force – Source:


https://hendersoninternational.wordpress.com/2014/12/21/improve-a-propeller-improve-a-ships-voyage/

Confidential to United Marine Training Center, Inc. Page 4-15


Pre Sea Review Materials <course code>
Version Number
Deck
5 November 2018

Figure 4-9: An object in motion will continue with constant speed and direction – Source: http://vansail.com/service/sea-
freight/

Figure 4-10: Unless acted upon by an unbalanced force – Source: http://www.pinoyseamanph.com/2017/11/marina-exam-


reviewer-if-ships-bow-has.html

2. Law of Acceleration
This law states that the acceleration of an object as produced by a net force is directly proportional
to the magnitude of the net force, in the same direction as the net force, and inversely proportional
to the mass of the object (Newton's Second Law, 1996-2018). This can be expressed in the below
equation:
𝑭𝒏𝒆𝒕 Newton’s Second Law (Law of Acceleration)
𝒂=
𝒎
Where:
𝑎 = Acceleration in 𝑚/𝑠 2
𝐹𝑛𝑒𝑡 = Net force in 𝑁𝑒𝑤𝑡𝑜𝑛 or 𝑘𝑔 ∙ 𝑚/𝑠 2
m = Mass on kilogram

𝑭𝒏𝒆𝒕 = 𝒎𝒂 = 𝜮𝑭 Transformed formula of the Law of Acceleration

Newton’s second law of motion pertains to the behavior of objects for which all existing forces
are not balanced.

Confidential to United Marine Training Center, Inc. Page 4-16


Pre Sea Review Materials <course code>
Version Number
Deck
5 November 2018

Acceleration

Net Force

Figure 4-11: Law of Acceleration - Source: Karizsa Denise D. Salvador

More
Acceleration

More
Net Force

Figure 4-12: Law of accelaration_2 - Source: Karizsa Denise D. Salvador

3. Law of Interaction
This law states that for every action, there is an equal and opposite reaction. The statement means
that in every interaction, there is a pair of forces acting on the two interacting objects (Newton's
Third Law, 1996-2018). The magnitude of the force on the first object is equal to the magnitude of
the force on the second object. The direction of the force on the first object is opposite to the
direction of the force on the second object. Forces always come in pairs – equal and opposite
action-reaction force pairs.

Action Reaction
Water Flow pushed backward Ship moves forward

Figure 4-13: Law of interaction - Source: Karizsa Denise D. Salvador

Free-Body Diagram (FBD)


 Diagram used to show the relative magnitude and direction of all forces acting upon an
object in a given situation (Drawing Free-Body Diagrams, 1996-2018)
 The size of the arrow reflects the magnitude of the force and its direction shows the
direction where the force is acting (Drawing Free-Body Diagrams, 1996-2018)
 Each force arrow is labelled to indicate the exact type of force
 Generally, the object is represented by a box and the force arrow is drawn from the center
of the box outward in the direction that the force is acting

Confidential to United Marine Training Center, Inc. Page 4-17


Pre Sea Review Materials <course code>
Version Number
Deck
5 November 2018

Figure 4-14: FBD - Source: Karizsa Denise D. Salvador

Example 4-6

A force is applied to the right to push an object in order to move it across the room with a
rightward acceleration. Consider frictional forces. Neglect air resistance. Draw the free-body
diagram.

Solution
There are four forces acting on the
object: 𝐹𝑁
𝐹𝑔 – Gravitational force or weight of the
object
𝐹𝑁 – Normal force or support force 𝐹𝑓 𝐹𝑎𝑝𝑝
exerted by the surface upon the
object
𝐹𝑓 – Friction force exerted by the surface 𝐹𝑔
opposite the direction of the
object’s motion
𝐹𝑎𝑝𝑝 – Applied force to the right

The motion of the object is rightward;


therefore, the magnitudes of the vertical
forces are the same (𝐹𝑁 and 𝐹𝑔 arrows
are drawn the same size depicting same
magnitudes) but acting opposite each
other’s direction (𝐹𝑁 and 𝐹𝑔 arrows are
pointing up and down, respectively). This
will result to no acceleration on the

Confidential to United Marine Training Center, Inc. Page 4-18


Pre Sea Review Materials <course code>
Version Number
Deck
5 November 2018

vertical direction due to zero net force.


𝐹𝑎𝑝𝑝 arrow is relatively longer than 𝐹𝑓
arrow indicating acceleration to the
right.

Example 4-7

A trainee rests a backpack upon his shoulder. The pack is suspended motionless by one strap
from one shoulder. Diagram the forces acting on the backpack.

Solution
There are two forces acting on the backpack:
𝐹𝑔 – Gravitational force or weight of the backpack
𝐹𝑇
𝐹𝑇 – Tension force directed along the strap

𝐹𝑔

Example 4-8

A horizontal force of 100 N is needed to pull a 50-kg cargo box across the deck floor at constant
speed. What is coefficient of friction between the deck floor and box?

Solution

Draw the free-body diagram


𝐹𝑁

Note: Only applied force is known that’s 𝐹𝑓 𝐹𝑎𝑝𝑝


why the arrows do not differ in
length in the FBD on the right. 𝐹𝑔

Because, 𝐹𝑓 =  𝐹𝑁
𝐹𝑓
And, 𝜇=
𝐹𝑁

Confidential to United Marine Training Center, Inc. Page 4-19


Pre Sea Review Materials <course code>
Version Number
Deck
5 November 2018

Then 𝐹𝑁 and 𝐹𝑓 must be solved first: 𝛴𝐹𝑦 = 𝑚𝑎𝑦


𝑚
𝛴𝐹𝑦 = (50 𝑘𝑔) (0 )
𝑠2
Because the box does not move in the
𝛴𝐹𝑦 = 0
vertical direction, 𝑎𝑦 = 0 and so

But, 𝛴𝐹𝑦 = 𝐹𝑁 + (−𝐹𝑔 )

And, 𝐹𝑔 = −𝑚𝑔
Note: negative sign for downward
direction
Hence, 0 = 𝐹𝑁 + (−𝑚𝑔)
𝐹𝑁 = 𝑚𝑔
𝑚
𝐹𝑁 = (50 𝑘𝑔) (9.8 )
𝑠2
𝐹𝑁 = 490 𝑁
Furthermore, because the box is moving 𝛴𝐹𝑥 = 𝑚𝑎𝑥
𝑚
horizontally at constant speed, 𝑎𝑥 = 0 𝛴𝐹𝑥 = (50 𝑘𝑔) (0 )
𝑠2
and so 𝛴𝐹𝑥 = 0

But, 𝛴𝐹𝑥 = 𝐹𝑎𝑝𝑝 + (−𝐹𝑓 )


Note: negative sign of 𝑭𝒇 indicates leftward
direction
And, 𝐹𝑎𝑝𝑝 = 100 𝑁

Hence, 0 = 140 𝑁 + (−𝐹𝑓 )


𝐹𝑓 = 140 𝑁
Therefore, 𝐹𝑓
𝜇=
𝐹𝑁
140 𝑁
𝜇=
490 𝑁
𝜇 = 0.29

Confidential to United Marine Training Center, Inc. Page 4-20


Pre Sea Review Materials <course code>
Version Number
Deck
5 November 2018

Example 4-9
𝑚
A 3.8-kg can of paint is to be given an upward acceleration of 0.2 𝑠2 by a rope pulling straight
upward on it. What must be the tension in the rope?

Solution

𝐹𝑇

Draw the free-body diagram

𝐹𝑔

Using, 𝛴𝐹𝑦 = 𝑚𝑎𝑦

And, 𝛴𝐹𝑦 = 𝐹𝑇 + (−𝑚𝑔)

Therefore, 𝐹𝑇 + (−𝑚𝑔) = 𝑚𝑎𝑦


Note: Acceleration is in upward direction
therefore 𝒂𝒚 is positive
Substituting the values, 𝑚 𝑚
𝐹𝑇 + [−(3.8 𝑘𝑔) (9.8 )] = (3.8 𝑘𝑔) (0.2 2 )
𝑠2 𝑠
Solving for 𝐹𝑇 𝐹𝑇 + (−37.24 𝑁) = 0.76 𝑁
𝐹𝑇 = 0.76 𝑁 + 37.24 𝑁
𝐹𝑇 = 38 𝑁

For checking, 𝑭𝑻 is larger than 𝑭𝒈 , as it must be if


the object is to accelerate upward.

Confidential to United Marine Training Center, Inc. Page 4-21


Pre Sea Review Materials <course code>
Version Number
Deck
5 November 2018

Supplementary Exercises
Solve the following problems
Answer
1] m
A 294852600-N tanker is to be given an acceleration of 0.005 s2 . How
1 474 263 𝑁
large an unbalanced force must act upon it?
2] A 90-kg cargo box is slid along the floor by a 500-N force. The
coefficient of friction between the box and the floor is 0.52. Find the 0.46 𝑚/𝑠 2
acceleration of the cargo box.
3] A 4.5-kg mass is suspended at the end of a cord of a crane. Find the a. 49.95 𝑁
𝑚
tension in the cord if the acceleration of the mass is (a) 1.3 𝑠2 up, (b) b. 38.25 𝑁
𝑚 𝑚
1.3 𝑠2 down, (c) 9.8 𝑠2 down during lifting and lowering. c. 0

Confidential to United Marine Training Center, Inc. Page 4-22


Pre Sea Review Materials <course code>
Version Number
Deck
5 November 2018

Work, Energy and Power

Work

 It is the effort exerted on something that will change its energy


 Two ingredients to work: force and displacement
 In order for a force to qualify as having done work on an object, there must be a
displacement and the force must cause or hinder the displacement.

Read the following statements and determine whether or not they represent examples of work:
 Doing push-ups
 A ship accelerates through water
 A power tool falls off a shelf and free falls to the floor
 Pushing on a wall of a room
 Carrying a bag on your shoulder while walking across the room

The first statement is an example of work. When you do push-ups, you do work against your weight.
The second statement is also an example of work. There is force (the ship’s propeller pushes the
water backwards which will result to a reaction force which is equal in magnitude but opposite in
direction) which causes the ship to be displaced through water. On the third statement, there is a
force (gravity) that acts on the power tool which causes it to be displaced in a downward direction;
therefore, this is an example of work. The last two statements are not examples of work. The wall is
not displaced on the fourth statement. For the last statement, there is a force (the bag was pulled up
towards the shoulder) and there is a displacement (the bag is moved horizontally across the room).
Yet the force does not cause the displacement; there must be a component of force in the direction
of displacement.
Work transfers energy from one place to another or one form to another. Work can change the
potential energy of an object, energy in a thermal system, or the electrical energy in an electrical
device.

Work Equation
Mathematically, work can be expressed by the following equation:

⃑⃑ ⃑𝒅⃑ 𝐜𝐨𝐬 𝜽
𝑾=𝑭 Formula for Work
Where:
𝑊 = Work in Joule
𝐹⃑ = Force in Newton
𝑑⃑ = Displacement in meter
𝜃 = Angle between the force and the displacement
in degrees

Note: In cases wherein, the angle between the force and displacement is zero, 𝒄𝒐𝒔𝜽 is omitted from
the formula.

Confidential to United Marine Training Center, Inc. Page 4-23


Pre Sea Review Materials <course code>
Version Number
Deck
5 November 2018

Consider following scenarios show the analysis of the effect of the angle to the work done:

1. A force acts leftward upon an object as it is displaced leftward. In this case, the force and
displacement are in the same direction. Thus, the angle is 0°.

Figure 4-15: Zero Angle - Source: Karizsa Denise D. Salvador

2. A force acts rightward upon an object that is displaced leftward. In this case, the force and
displacement are in opposite directions. Thus, the angle is 180°.

Figure 4-16: 180° angle - Source: Karizsa Denise D. Salvador

3. A force acts upward upon an object that is displaced rightward. In this case, the force and
displacement are perpendicular to each other. Thus, the angle is 90°.

Figure 4-17: 90° angle - Source: Karizsa Denise D. Salvador

Negative Work
Work will be negative if the force acts in the direction opposite the object’s motion, usually to slow it
down. The negative of negative work refers to the numerical value that resulted when values of
force, displacement and theta are substituted in the equation of work. Since the force vector is
directly opposite the displacement vector, theta is 180°. The cosine of 180° is -1. Negative value
results for the amount of work done upon the object (Definition and Mathematics of Work, 1996-
2018).

Confidential to United Marine Training Center, Inc. Page 4-24


Pre Sea Review Materials <course code>
Version Number
Deck
5 November 2018

Motion

Fluid Friction

Figure 4-18: Negative work - Source: Karizsa Denise D. Salvador

Mechanical Energy

When work is done to raise a heavy shipping container, the container acquires the ability to do work
on the object it hits when it falls. When work is done to wind a spring mechanism, the spring
acquires the ability to do work on various gears. Something has been acquired in each case that
enables the object to do work. This something that enables an object to do work is energy. Energy
appears in many forms but this chapter will focus on the two forms of mechanical energy: potential
energy and kinetic energy.

Potential Energy
Energy may be stored by an object by virtue of its position. The energy that is stored and held in
readiness is called potential energy (PE) because in the stored state, it has the potential for doing
work. A stretched or compressed spring has a potential for doing work to another object. The
chemical energy in fuels is potential energy. It is energy by virtue of position at the sub microscopic
level. This energy is available when the positions of electric charges within and between molecules
are altered. This is when a chemical change occurs (Hewitt, 2012).

Gravitational Potential Energy


Gravitational potential energy is the energy stored in an object as the result of its vertical position or
height. The energy is stored as the result of gravitational attraction of the Earth for the object. There
is a direct relation between gravitational potential energy and the mass of an object (Potential
Energy, 1996-2018). Objects which are more massive have greater gravitational potential energy.
Between the height where the object was brought and gravitational potential energy, there is also a
direct relation. The higher the object is elevated, the greater its gravitational potential energy. These
relationships can be expressed by the following equation:

𝑷𝑬𝒈 = 𝒎𝒉𝒈 Formula for Average Velocity


Where:
𝑃𝐸𝑔 = Gravitational potential energy in Joule
𝑚 = Mass in kilogram
ℎ = Height in meters
𝑔 = Acceleration due to gravity in 𝑚/𝑠 2
= 9.8 𝑚/𝑠 2

A zero height position must first be arbitrarily assigned to determine the gravitational potential
energy of object (Potential Energy, 1996-2018).

Confidential to United Marine Training Center, Inc. Page 4-25


Pre Sea Review Materials <course code>
Version Number
Deck
5 November 2018

To summarize, potential energy is the energy that is stored in an object due to its position relative to
some reference level. An object possesses gravitational potential energy if it is positioned at a
height relative to a reference point.
Consider Figure 4-19. I1f container K has the same mass with containers S and D, then container K
has greater gravitational potential energy. Containers D and S have no gravitational potential
energy.

D S
Reference Level

Figure 4-19: Gravitational potential energy – Source: https://jp.freepik.com/free-photos-vectors/vessel

Work and Gravitational Potential Energy


Work done against the gravitational force goes into a form of stored energy – gravitational potential
energy. If an object is lifted straight up, then the force needed to lift it up is equal to its weight. The
work done on the object then changes its potential energy. When the force does positive work, it
increases the gravitational potential energy of the object.

𝑃𝐸𝑔2 = 𝑚𝑔ℎ2

𝑃𝐸𝑔1 = 𝑚𝑔ℎ1

Figure 4-20: Change in potential energy - Source: Karizsa Denise D. Salvador

Work equals change in potential energy:

𝑾 = ∆𝑷𝑬
𝑾 = 𝑷𝑬𝒇 − 𝑷𝑬𝒊
𝑾 = 𝒎𝒈𝒉𝟐 − 𝒎𝒈𝒉𝟏

Confidential to United Marine Training Center, Inc. Page 4-26


Pre Sea Review Materials <course code>
Version Number
Deck
5 November 2018

Kinetic Energy
Kinetic energy is the energy that an object possesses due to its motion. If an object is moving, then
it is capable of doing work. It has energy in motion. If a ship is sailed, work is done on it to give it
speed. The moving ship can then collide with something and push it, doing work on what it hits. The
amount of kinetic energy that an object has depends upon two variables: the mass of the object and
the speed of the object. The equation used to represent kinetic energy is as follows:
𝟏 Formula for Kinetic Energy
𝑲𝑬 = ∙ 𝒎𝒗𝟐
𝟐
Where:
KE = Kinetic Energy in Joule
m = Mass in kilogram
v = Speed in 𝑚/𝑠

This equation discloses that the kinetic energy of an object is directly proportional to the square of
its speed. That means for a double increase in speed, the kinetic energy will increase by a factor of
four. There is also a direct relation between the mass of an object and its kinetic energy. Objects
which are more massive will have greater kinetic energy.

Work and Change in Kinetic Energy


The work-energy theorem states that the work done by the net force on a particle equals the change
in the particle’s kinetic energy (Work-Energy Theorem, 2013). In Figure 4-21, a net force does work
on the block. The kinetic energy of the block changes (increases for this particular diagram) as a
result of work done on it. This relationship is generalized in the work-energy theorem

Figure 4-21: Work-energy theorem - Source: Karizsa Denise D. Salvador

Work equals change in kinetic energy:

𝑾 = ∆𝑲𝑬
𝑾 = 𝑲𝑬𝒇 − 𝑲𝑬𝒊
𝟏 𝟏
𝑾= 𝒎𝒗𝒇 𝟐 − 𝒎𝒗𝒊 𝟐
𝟐 𝟐

The work in this equation is the net work – that is, the work based on the net force.

Note: It is important to remember that only the CHANGE IN ENERGY can be negative or zero
which the former indicates that a body loses energy and the latter means the energy of a body
remained unchanged. The ENERGY itself cannot be negative; instead, it is always positive.

Confidential to United Marine Training Center, Inc. Page 4-27


Pre Sea Review Materials <course code>
Version Number
Deck
5 November 2018

Power

The description of work says nothing about how long it takes to do the work. The same amount of
work is done when carrying something up a flight of stairs, whether we walk or run. Thus why are
we more tired after running upstairs in a few seconds than after walking upstairs after a few
minutes? To understand the difference, the measure of how fast the work is done should be
discussed – power.

Power is equal to the amount of work done per time it takes to do it (Hewitt, 2012). It is the rate at
which a certain amount of work is done. Power is calculated using the following equation:
𝑾 Formula for Power
𝑷=
𝒕
Where:
P = Power in watts
W = Work done in joules
t = Time the work is done

The standard metric unit of power is Watt (W). As implied by the equation of power, a unit of power
is equivalent to a unit of work divided by a unit of time. Hence, a Watt is equivalent to a
Joule/second (J/s). The horsepower is occasionally used to describe the power delivered by a
machine. One horsepower is equivalent to approximately 746 Watts (Power, 1996-2018).

Example 4-10

A horizontal force of 20 N pulls a box along a surface. How much work does it do in pulling the box
through a distance of 0.7 m?

Solution
Since the displacement is in the same 𝜃=0
direction as the force causing it, then,
And work equation is, 𝑊 = 𝐹⃑ 𝑑⃑ cos 𝜃
Therefore, 𝑊 = (20 𝑁)(0.7 𝑚)cos(0)
𝑊 = 14 𝐽

Example 4-11

How much work is done against gravity in lifting a 5-kg object through a distance of 0.45 m?

Solution
To lift a 5-kg object, an upward force 𝐹⃑ = 𝑚𝑔
equal in magnitude to its weight must 𝑚
𝐹⃑ = (5 𝑘𝑔) (9.8 )
𝑠2
be exerted on it,
𝐹⃑ = 49 𝑁
The work done by this force is, 𝑊 = 𝐹⃑ 𝑑⃑ cos 𝜃
Therefore, 𝑊 = (49 𝑁)(0.45 𝑚)cos(0)
𝑊 = 22.05 𝐽

Confidential to United Marine Training Center, Inc. Page 4-28


Pre Sea Review Materials <course code>
Version Number
Deck
5 November 2018

Example 4-12

A 20,000 kg shipping container is lifted by a crane to a height of 10 m from a reference level. What
is its potential energy?

Solution
Potential energy formula is, 𝑃𝐸𝑔 = 𝑚𝑔ℎ
Substituting the given values, 𝑚
𝑃𝐸𝑔 = (20,000 𝑘𝑔) (9.8 ) (10 𝑚)
𝑠2
Hence, 𝑃𝐸𝑔 = 1,960,000 𝐽

Example 4-13

A ship with displacement 40,000 T is sailing at a speed of 10 knots. Find the (a) ship’s kinetic
energy at 10 knots, (b) work required to stop the ship, and (c) the force needed to stop the ship
over a 1.8 NM distance.
Solution:
Ship’s kinetic energy at 10 knots
The formula for kinetic energy is, 1
𝐾𝐸 = 𝑚𝑣 2
2
To get a unit of joule for kinetic energy,
mass should be in kg and speed should
𝑚
be in 𝑠 .
1000 𝑘𝑔
𝑚 = (40,000 𝑇) ( ) = 40,000,000 𝑘𝑔
1𝑇
So,
And, 𝑛𝑚𝑖 1852 𝑚 1 ℎ𝑟 𝑚
𝑣 = (10 )( )( ) = 5.14
ℎ𝑟 1 𝑛𝑚𝑖 3600 𝑠 𝑠

Therefore, 1
𝐾𝐸 = (40,000,000 𝑘𝑔)(5.14)2
2
𝐾𝐸 = 528,392,000 𝐽

Work required to stop the ship


To be able to stop the ship, its speed 𝑚
𝑣𝑖 = 5.14
𝑚 𝑠
must go from 10 knots (5.14 ) to 0.
𝑠 𝑣𝑓 = 0
Changing the speed of the ship means 1 1
𝑊 = ∆𝐾𝐸 = 𝑚𝑣𝑓 2 − 𝑚𝑣𝑖 2
2 2
changing its kinetic energy, therefore,
work must be done,

Confidential to United Marine Training Center, Inc. Page 4-29


Pre Sea Review Materials <course code>
Version Number
Deck
5 November 2018

Hence, 1
𝑊 = 𝑚(𝑣𝑓 2 − 𝑣𝑖 2 )
2
1
𝑊 = (40,000,000 𝑘𝑔)(02 − 5.142 )
2
𝑊 = −528,392,000 𝐽

Force needed to stop the ship over a 1.8 NM distance


Since work required to stop the ship is, 𝑊 = −528,392,000 𝐽

Over a distance of, 1852 𝑚


𝑑 = (1.8 𝑛𝑚𝑖) ( ) = 3333.6 𝑚
1 𝑛𝑚𝑖
And, 𝑊 = 𝐹⃑ 𝑑⃑
𝑊
𝐹⃑ =
𝑑⃑

Therefore, −528,392,000 𝐽
𝐹⃑ =
3333.6 𝑚
𝐹⃑ = −158,504.92 𝑁

Example 4-14

If a 20,000,000 kg ship will accelerate from rest to a speed of 5 knots in a time of 120 seconds,
what power must the engine produce?

Solution
The work done to accelerate the ship 1 1
𝑊 = 𝑚𝑣𝑓 2 − 𝑚𝑣𝑖 2
𝑚 2 2
from rest to 5 knots or 2.57 𝑠
is,
1 𝑚 2
𝑊 = (20,000,000 𝑘𝑔) (2.57 )
2 𝑠
1
− (40,000,000 𝑘𝑔)(0)2
2
𝑊 = 66,049,000 𝐽
The time taken for this work is, 𝑡 = 120 𝑠
Hence, 𝑊
𝑃=
𝑡
66,049,000 𝐽
𝑃=
120 𝑠
𝑃 = 550,408.33 𝑊

Confidential to United Marine Training Center, Inc. Page 4-30


Pre Sea Review Materials <course code>
Version Number
Deck
5 November 2018

Supplementary Exercises
Solve the following problems
Answer
1] In unloading grain from the hold of a ship, an elevator lifts the gain 308.72 W
through 14 m. Grain is discharged at the top of the elevator at a rate
of 2.2 kg each second, and the discharge speed of each grain
𝑚
particle is 2.5 𝑠 . Find the minimum power rating for a motor that can
elevate grain in this manner.
2] A vessel with a displacement of 19, 840 𝑇 accelerates from 15 𝑘𝑛𝑜𝑡𝑠 14,578.87 m
to 18 𝑘𝑛𝑜𝑡𝑠 over a period of 30 𝑚𝑖𝑛𝑢𝑡𝑒𝑠. If its main engine does
2.48 𝑥 108 𝐽 of work on the vessel, how far did the vessel reach?
3] A supertanker does not come with brakes. Using engines alone, it (a) −1.39 𝑥 106 N
takes a loaded supertanker 13 𝑘𝑚 to stop. A typical vessel of this (b) −12, 046,666 W
class has a gross mass of about 150, 000 𝑇 and a cruising speed
of 50 𝑘𝑚/ℎ. If it takes 25 𝑚𝑖𝑛𝑢𝑡𝑒𝑠 for the vessel to stop, determine
(a) the average stopping force applied to the ship (b) the average
power dissipated while stopping it.
4] A pump discharges 700 𝑙𝑖𝑡𝑒𝑟𝑠 of fuel oil into a tank 25 𝑚 above the Yes; 152,635 KJ
pump’s intake. Is there a change in gravitational potential energy of
fuel oil at the said height? If yes, by how much? (One cubic
centimeter of fuel oil has a mass of 0.89 g and one liter is
equivalent to 1, 000 𝑐𝑚3 ).
5] An 18.5-hp overhead crane is used to lift a load at a rate of 3.5
𝑚
. 24,141.83 kg
𝑚𝑖𝑛
How much load can it raise at this lifting speed?
⃑⃑⃑⃑
(Hint: Use 𝐹 𝑔 = 𝑚𝑔
⃑)

Fluids

Fluids are divided into liquids and gases. A liquid is hard to compress and it changes its shape
according to the shape of its container with an upper free surface. Gas on the other hand is easy to
compress, and fully expands to fill its container. In general, liquids are called incompressible fluids
and gases compressible fluids. (Alshara, 2015).
Fluid mechanics has two branches: fluid statics and fluid dynamics. Fluid statics studies fluids at
rest. It embraces the study of the conditions under which fluids are at rest in stable equilibrium. Fluid
dynamics deals with fluid flow. Liquids and gases are in motion in contrast with fluid statics.

Density
The masses of the atoms and the spacing between them determine the density of materials (Hewitt,
2012). It is a measure of the compactness of matter, of how much mass occupies a given space. It
is the amount of mass per unit volume. Mathematically expressed as:
𝒎 Formula for Density
𝝆=
𝑽
Where:
𝜌 = Density
𝑚 = Mass
𝑉 = Volume

Confidential to United Marine Training Center, Inc. Page 4-31


Pre Sea Review Materials <course code>
Version Number
Deck
5 November 2018

Density is a property of material; it doesn’t matter how much of the material is involved. Density is
𝑘𝑔
usually expressed in metric unit, which is kilograms per cubic meter ( 3). For example, water has a
𝑚
𝑘𝑔
mass density of 1000 𝑚3. So the mass of a cubic meter of fresh water is 1000 kg. Equivalently, the
mass of a cubic centimeter of fresh water is 1 g (Hewitt, 2012).

Pressure in a Fluid
A static fluid exerts a force on any surface with which it comes in contact and the direction of the
force is perpendicular to the surface. Suppose a block is placed in a container filled with water as
shown in Figure 4-22. Pressure exerted by the fluid is the same in every direction in a fluid at a
given depth.

Figure 4-22: Force acting on a submerged object – Source:


http://www.atmo.arizona.edu/students/courselinks/fall16/atmo336s2/lectures/sec1/Fluid_Pressure.html

Pressure is defined as the perpendicular force applied over the surface area. Mathematically
expressed as:
𝑭 Formula for Pressure in Fluid (given force and area)
𝑷=
𝑨
Where:
𝑃 = Pressure in Pascal (Pa) or 𝑚2
𝑁

𝐹 = Force applied on the surface in Newton or N


𝐴 = Area of the surface in 𝑚2

𝑁
The SI unit of pressure is the Pascal, 1 𝑃𝑎 = 1 2 , this means that 1 Pascal is equivalent to 1
𝑚
Newton of force applied over a 1 𝑚2 surface area.
Another commonly used unit of pressure is the atmosphere (atm). One atm is the average air
pressure at sea level. Sometimes pound per square inch (psi) is also used to express pressure. The
conversion factor among Pascal, atmosphere and psi is: 1 𝑎𝑡𝑚 = 1.013 𝑥 105 𝑃𝑎 = 14.7 𝑝𝑠𝑖

A general relationship between the pressure at any point in a fluid at rest and the elevation at any
point can be derived. As shown in the Figure 4-23, the liquid inside the container is applying a force
at the bottom of the container that is equal to its weight (𝐹⃑𝑔 = 𝑚𝑔). The liquid also has a volume
equal to its surface area multiplied by the depth or height (𝑉 = 𝐴ℎ).

Confidential to United Marine Training Center, Inc. Page 4-32


Pre Sea Review Materials <course code>
Version Number
Deck
5 November 2018

Figure 4-23: Forces acting on a submerged object - Source:


http://www.atmo.arizona.edu/students/courselinks/fall16/atmo336s2/lectures/sec1/Fluid_Pressure.html

The derivation for the pressure variation with depth in a static fluid is as follows:
Use the equation for pressure, 𝐹
𝑃=
𝐴
Substitute force for the weight of the fluid 𝑚𝑔
𝑃=
𝐴
⃑⃑𝑔 = 𝑚𝑔,
𝐹

But 𝑚 = 𝜌𝑉 based on the density equation, 𝜌𝑉𝑔


𝑃=
𝐴
And volume of the liquid is given by 𝑉 = 𝐴ℎ, 𝜌𝐴ℎ𝑔
𝑃=
𝐴
Cancel out common factor 𝐴, 𝜌𝐴ℎ𝑔
𝑃=
𝐴
Thus, 𝑃 = 𝜌𝑔ℎ
Where: P = pressure (in Pa)
𝑘𝑔
𝜌 = density of the fluid (in 𝑚3)

g = acceleration due to gravity


𝑚
= 9.8 𝑠2

h = height/depth of the fluid

This equation is true as long as the density is the same throughout the fluid and as well as the
acceleration due to gravity is applied. This means that the pressure applied by the fluid is only
dependent on the height of the fluid. As the depth of the fluid increases, the pressure it applies also
increases. Suppose there is a container that is filled with water and poked three holes on its sides
as shown in the Figure 4-24. Notice that the strongest water jet is at the hole near the bottom part
and the weakest is the one near the top. Since pressure at bottom is greatest then water from the
hole will reach farthest distance among the three.

Confidential to United Marine Training Center, Inc. Page 4-33


Pre Sea Review Materials <course code>
Version Number
Deck
5 November 2018

Figure 4-24: Pressure applied by the fluid at the bottom of the container – Source: https://www.school-for-
champions.com/experiments/fluids_pressure_depth_relationship.htm#.W3TaNugzaUk

This verifies that fluid pressure increases as depth increases. This is also the reason why one feels
greater pressure as one dives deeper in the water while swimming. Note that the pressure is the
same at any two points at the same level in the fluid. The shape of the container does not matter.

The equation above only refers to the amount of pressure applied by the liquid. Supposing that
there is a container with fluid and is open to the atmosphere. The pressure exerted at any point in
the fluid does not only comprise by the pressure applied by the fluid itself but also the pressure
applied by the surrounding air or the atmosphere. This total pressure exerted at any point in the fluid
is called the absolute pressure and mathematically expressed as:

𝑷𝒂𝒃𝒔 = 𝑷𝟎 + 𝝆𝒈𝒉 Formula for Pressure in Fluid (given fluid density and
height)
Where:
𝑃𝑎𝑏𝑠 = Absolute pressure in Pascal
𝑃0 = Atmospheric pressure in Pascal
= 1.013 𝑥 105 Pa (at sea level)
𝜌 = Density of fluid in 𝑘𝑔/𝑚3
g = Acceleration due to gravity 𝑚/𝑠 2
= 9.8 𝑚/𝑠 2
ℎ = Depth or height of the fluid in meter

The excess pressure above the atmospheric pressure is usually called the gauge pressure.
Mathematically expressed as:

𝑷𝒈 = 𝑷𝒂𝒃𝒔 − 𝑷𝟎 Formula for Gauge Pressure


Where:
𝑃𝑔 = Gauge pressure in Pascal
𝑃𝑎𝑏𝑠 = Absolute pressure in Pascal
𝑃0 = Atmospheric pressure in Pascal
= 1.013 𝑥 105 Pa

Confidential to United Marine Training Center, Inc. Page 4-34


Pre Sea Review Materials <course code>
Version Number
Deck
5 November 2018

Pascal’s Principle
One of the most important facts about fluid pressure is that the pressure applied to an enclosed
incompressible fluid at rest is transmitted undiminished to any point in the fluid and to the
walls of the container. This was discovered by Blaise Pascal in the seventeenth century and is
called as the Pascal’s Principle.

Figure 4-25: Enclosed fluid in a pipe with two pistons - Source: http://pascalteam.hu/en_pascal_law.php

Suppose you have an enclosed fluid in a pipe with two pistons of different cross-sectional areas as
shown in Figure 4-25. A force 𝐹1 applied on a smaller piston with cross-sectional area 𝐴1 will cause
a pressure 𝑃1 applied on the fluid. According to Pascal’s Principle, the pressure applied by the
smaller piston on the fluid will transfer undiminished throughout the fluid and the walls of the pipe. In
return, this same amount of pressure will be applied by the fluid on the larger piston. Thus, the force
𝐹2 applied by the fluid over the cross-sectional area 𝐴2 will produce a pressure 𝑃2 that is equal to
pressure 𝑃1 . Mathematically expressed as:

𝑷𝟏 = 𝑷𝟐 Pascal’s Principle
Substitute the formula Where:
for pressure: 𝑃 = 𝐴
𝐹 𝐹1 = Force applied on first piston in Newton
𝐹2 = Force applied on second piston in Newton
𝑭𝟏 𝑭𝟐 𝐴1 = Cross-sectional area of first piston in 𝑚2
=
𝑨𝟏 𝑨𝟐 𝐴2 = Cross-sectional area of second piston in 𝑚2

A hydraulic lift is one of the basic applications of the Pascal’s Principle. Figure 4-25 shows a
schematic diagram of a hydraulic lift. By applying a small amount of force 𝐹1 on smaller piston with
cross-sectional area 𝐴1 , the load placed on the larger piston with cross-sectional area 𝐴2 will be
lifted by force 𝐹2. Since the pressure is equal then the force and the cross-sectional area are directly
proportional to each other. This implies that small cross-sectional area requires less force whereas
greater cross-sectional area requires greater force to move. Therefore, a hydraulic lift is a force
multiplying device with a multiplication factor equal to the ratio of the areas of the two pistons.

Since a force is applied on the piston and the piston moves a certain distance, then work (𝑊 = 𝐹𝑑)
is done on each piston. The work done by moving the small piston is equal to the work done by the
large piston in raising the load. The work done by the pistons are equal even if the forces are not
equal because the distances that will be moved by both pistons are not the same. According to the
equation for work, when work is constant, the force and distance are inversely proportional to each
other. This means that small force moves greater distance and vice versa. Thus, small piston with
small force moves greater distance while large piston with great force moves less distance.

Confidential to United Marine Training Center, Inc. Page 4-35


Pre Sea Review Materials <course code>
Version Number
Deck
5 November 2018

The advantage of the smaller force applied to the small piston is balanced by a greater distance it
must be moved. The small piston has to move a long distance 𝑙1 while the large piston moves a
short distance 𝑙2 . Since the fluid is incompressible, the volume of fluid displaced by each piston is
the same. Therefore,

Since fluid is incompressible, 𝑉1 = 𝑉2


Substituting volume equation 𝑉 = 𝐴𝑙 𝐴1 𝑙1 = 𝐴2 𝑙2
The equation can be written as, 𝐴1 𝑙2
=
𝐴2 𝑙1

This system can be thought of as a simple machine (lever) since the force is being multiplied. Its
mechanical advantage is mathematically expressed as:
𝒍 𝟏 𝑨𝟐 Formula for Mechanical Advantage, 𝑴. 𝑨.
𝑴. 𝑨. = =
𝒍 𝟐 𝑨𝟏
Where:
𝐴1 = area of first piston in 𝑚2
𝐴2 = area of second piston in 𝑚2
𝑙1 = distance moved by first piston in meter
𝑙2 = distance moved by second piston in meter

Since Mechanical advantage tells the ratio of the output force and the input force then it has no
units. Usually, it is being round down to its nearest whole number because one cannot claim that the
mechanical advantage is greater than what it can do.

The Pascal’s Principle is the basic concept of how hydraulic system works. These hydraulic systems
are widely used on board for deck machinery and cargo equipment.

Bernoulli’s Principle
A fluid speed up when it goes from a wide to a narrow part of the pipe for continuous flow. This is
evident when water flowing from a hose speeds up when one squeezes the end of the hose to make
the stream narrower.

Figure 4-26: Fluid Flow - Source: https://www.slideshare.net/rexparis36/05-lecture-outline

Confidential to United Marine Training Center, Inc. Page 4-36


Pre Sea Review Materials <course code>
Version Number
Deck
5 November 2018

The motion of a fluid in steady flow follows imaginary streamlines, represented by thin lines in
Figure 4-26. Streamlines are the smooth paths of fluid. In narrower regions, the lines are closer
together. Daniel Bernoulli, an 18th-century Swiss scientist, studied the fluid flow in pipes (Hewitt,
2012). His discovery, a physical principle, called Bernoulli’s Principle, states that: “As the speed of
a moving fluid increases, the pressure within the fluid decreases”
Flow speed is greater and pressure within the fluid is less where streamlines of a fluid are closer
together.
Internal pressure is greater in slower-moving fluid in the wider region, as evidenced by the more
squeezed bubbles in Figure 4-27. The bubbles are bigger in the narrow part because internal
pressure there is less (Hewitt, 2012).

Figure 4-27: Internal Pressure - Source: https://slideplayer.com/slide/3621809/

Bernoulli’s principle applies to a smooth, steady flow (laminar flow) of constant density fluid. At
speeds above some critical point, however, the flow may become chaotic (turbulent flow). This
exerts friction on the fluid and dissipates some of its energy. Then, Bernoulli’s principle doesn’t
apply (Hewitt, 2012).

Figure 4-28: Laminar Flow vs Turbulent Flow – Source: https://www.cfdsupport.com/openfoam-training.html

Interaction between two vessels during overtaking in open waters involves Bernoulli’s principle. It is
also observed on bow cushion, bank suction, and squat effect.

Confidential to United Marine Training Center, Inc. Page 4-37


Pre Sea Review Materials <course code>
Version Number
Deck
5 November 2018

Example 4-15

Find the density of a substance if 0.071 kg of it occupies 70 x 10-6 𝑚3.

Solution
The density formula is, 𝑚
𝜌=
𝑉
Substituting the given values of mass 𝑚 = 0.071 𝑘𝑔
and volume, 𝑉 = 70 𝑥 10−6 𝑚3
Hence, 0.071 𝑘𝑔
𝜌=
70 𝑥 10−6 𝑚3
𝑘𝑔
𝑃 = 1014.29 3
𝑚

Example 4-16

A 100-kg metal cylinder 3 m long and with each end of area 25 x 10-4 m2 stands vertically on one
end. What pressure does the cylinder exert on the floor?

Solution
The formula for pressure is, 𝐹
𝑃=
𝐴
Since force is equal to the weight of the 𝐹 = 𝑚𝑔
cylinder, 𝑚
𝐹 = (100 𝑘𝑔) (9.8 )
𝑠2
𝐹 = 980 𝑁
And cross-sectional area of the cylinder 𝐴 = 25 𝑥 10−4 𝑚2
is,
Therefore, 980 𝑁
𝑃=
25 𝑥 10−4 𝑚2
𝑃 = 392,000 𝑃𝑎

Example 4-17

𝑘𝑔
An intermediate fuel oil (𝜌 = 996 3) stands 12 m deep in a storage tank whose top is open to the
𝑚
atmosphere. What are the (a) absolute pressure and (b) gauge pressure at the bottom of the
tank?

Solution
Absolute pressure
The formula for absolute pressure is, 𝑃𝑎𝑏𝑠 = 𝑃0 + 𝜌𝑔ℎ

Since the atmospheric pressure in Pa is, 𝑃0 = 1.013 𝑥 105 𝑃𝑎

Confidential to United Marine Training Center, Inc. Page 4-38


Pre Sea Review Materials <course code>
Version Number
Deck
5 November 2018

And liquid pressure is, 𝑃 = 𝜌𝑔ℎ


𝑘𝑔 𝑚
𝑃 = (996 3
) (9.8 2 ) (12 𝑚)
𝑚 𝑠
𝑃 = 117,129.6 𝑃𝑎

Therefore, 𝑃𝑎𝑏𝑠 = 1.013 𝑥 105 𝑃𝑎 + 117,129.6 𝑃𝑎


𝑃𝑎𝑏𝑠 = 218,429.6 𝑃𝑎
Gauge pressure
The formula for gauge pressure is, 𝑃𝑔 = 𝑃𝑎𝑏𝑠 − 𝑃0
Since the absolute pressure is, 𝑃𝑎𝑏𝑠 = 218,429.6 𝑃𝑎
And the atmospheric pressure in Pa is, 𝑃0 = 1.013 𝑥 105 𝑃𝑎
Therefore, 𝑃𝑔 = 218,429.6 𝑃𝑎 − 1.013 𝑥 105 𝑃𝑎
𝑃𝑔 = 117,129.6 𝑃𝑎

Example 4-18

A calibrated flask has a mass of 0.02 kg when empty, 0.095 kg when filled with water, and 0.058
kg when filled with denatured alcohol. Find the density of the alcohol.

Solution
𝑚
To find the density of the alcohol, its 𝜌=
𝑉
mass and volume should be known.

𝑚 = 0.058 𝑘𝑔 − 0.02 𝑘𝑔
Mass of the alcohol is the mass of the
𝑚 = 0.038 𝑘𝑔
flask filled with alcohol subtracted by the
mass of the flask when empty,
The volume of the alcohol is not given
but the volume of water can be readily
obtained.
Volume of water = Volume of alcohol
Since the water filled the flask, the
volume of water is equal to the volume
of the flask. Since the alcohol also filled
the flask, the volume of the flask and
alcohol are the same as well. Therefore,
the volume of the water is equal to the
volume of the alcohol.
𝑚
The volume of the water can be solved 𝜌= 𝑉
using the density formula,

Confidential to United Marine Training Center, Inc. Page 4-39


Pre Sea Review Materials <course code>
Version Number
Deck
5 November 2018

So, 𝑚
𝑉=
𝜌

Since mass of the water is the mass of 𝑚 = 0.095 𝑘𝑔 − 0.02 𝑘𝑔


the flask filled with water subtracted by 𝑚 = 0.075 𝑘𝑔
mass of the flask when empty,
𝑘𝑔
And density of the water is, 𝜌 = 1000 𝑚3

Hence, 0.075 𝑘𝑔
𝑉=
𝑘𝑔
1000 3
𝑚
𝑉 = 7.5 𝑥 10−5 𝑚3
0.038 𝑘𝑔
Therefore, density of alcohol is, 𝜌=
7.5 𝑥 10−5 𝑚 3
𝑘𝑔
𝜌 = 506.67
𝑚3

Example 4-19

A force of 98 N is applied to the input piston of a hydraulic system. The area of the input piston is
22 cm2, and that of the output piston is 70 cm 2. (a) How much force does the output piston exert?
(b) If the input piston moves 2 cm per stroke, how far does the output piston move?

Solution
Force exerted by the output piston
The formula to be used to get the value 𝐹1 𝐹2
=
𝐴1 𝐴2
of the force exerted on the output piston
(F2) is,
To get the formula for F2, transform the 𝐹1 𝐴2
𝐹2 =
𝐴1
formula shown previously,
Since, 𝐹1 = 98 𝑁
𝐴1 = 22 𝑐𝑚2
𝐴2 = 70 𝑐𝑚2
Therefore, (98 𝑁)(70 𝑐𝑚2 )
𝐹2 =
22 𝑐𝑚2
𝐹2 = 311.82 𝑁

Distance moved by the output piston


To get the formula for the distance that 𝐴1 𝑙2
=
𝐴2 𝑙1
the output piston has moved (𝑙2 ),
transform the formula,
𝐴1 𝑙1
𝑙2 =
𝐴2
To,

Confidential to United Marine Training Center, Inc. Page 4-40


Pre Sea Review Materials <course code>
Version Number
Deck
5 November 2018

Since, 𝐴1 = 22 𝑐𝑚2
𝐴2 = 70 𝑐𝑚2
𝑙1 = 2 𝑐𝑚
Therefore, (22 𝑐𝑚2 )(2 𝑐𝑚)
𝑙2 =
70 𝑐𝑚2
𝑙2 = 0.63 𝑐𝑚

Confidential to United Marine Training Center, Inc. Page 4-41


Pre Sea Review Materials <course code>
Version Number
Deck
5 November 2018

Supplementary Exercises
Solve the following the problems
Answer
1] What volume does 500 g of mercury occupy? The density 3.67 x 10-5 m3
𝑘𝑔
of mercury is 13,600 𝑚3.
𝑔
If the density of a diamond is 3.5 𝑐𝑚3, what would be the 1.8 g
2]
mass of a diamond whose volume is 0.5 𝑐𝑚3?
𝑘𝑔 882 Pa
3] A vertical test tube has 2.5 cm of oil (𝜌 = 800 3) floating
𝑚
on 7 cm of water. What is the pressure at the bottom of the
tube due to the liquid in it?
A container will break when the gauge pressure within it 59.62 m
4]
reaches 520 kPa. It is attached to the lower end of a vertical
pipe, with the pipe and the container filled with liquid (𝜌 =
𝑘𝑔
890 𝑚3). How long can the pipe be if the container is not to
break?
A hydraulic lift has a large piston 40 cm in diameter and a (a) 99.23 N
5]
small piston 3 cm in diameter. (a) What force is required on (b) 140,374.66 Pa
the small piston to lift a load of 1800 kg? (b) What is the
pressure increase due to the force in the confined liquid?
A cylinder is raised by applying a force of 150 N to a 148.06 kg
6]
hydraulic piston of area 0.01 m 2. If the cylinder is attached
to a piston of area 0.1 m2, how massive a load can the
cylinder raise? Assume the cylinder itself has a mass of 5
kg.

Confidential to United Marine Training Center, Inc. Page 4-42


Pre Sea Review Materials <course code>
Version Number
Deck
5 November 2018

Section 5
Terrestrial Navigation

Magnetic Compass

Why is a Magnetic Compass Required in the Age of


Electronic Navigation?

Despite the modern tendency to rely heavily on Electronic Navigational Aids (ENA), the magnetic
compass remains a primary navigation instrument on any vessel, and continues to operate
independently, in the not uncommon event of an electrical failure or electronics malfunction.

Users should be aware that ENA have limitations and have been known to provide erroneous
information. Reliable and accessible alternatives for back up and cross reference should always be
readily available.

Vessels are required to be equipped with a means of determining direction and heading, readable
from the steering position and independent of any power supply. A correctly installed and adjusted
magnetic compass, of a size and type suitable for the vessel, fulfills the requirement of SOLAS
Chapter V Safety of Navigation.

There is little doubt that Global Navigation Satellite Systems (GNSS), such as GPS, help to make
modern sea travel generally safer and, in many respects, easier than it used to be, particularly when
interfaced with A.I.S., radar and electronic chart display systems such as ECDIS. It is, however,
worth taking the following into consideration:
 GPS is currently the only fully operational GNSS. It is owned and controlled by the U.S.
Department of Defense and its commercial and recreational use is incidental to its primary,
military purpose.
 GNSS signals are vulnerable to loss and error, both intentional and unintentional.
Malicious jamming of GNSS is a very real threat. GPS signals can be terminated or
corrupted by the US military for security purposes.
 Commercial GPS operates on a Single frequency only. Military GPS receivers operate on
a dual frequency system which is more reliable and less vulnerable to error caused by
atmospheric conditions.
 GNSS signals are extremely vulnerable to solar activity such as solar flares. The sun is
currently entering a phase of intense solar flare activity which is due to last for several
years.
 Some areas of the world, particularly in the higher latitudes, have problematic or no
GNSS/GPS coverage.
 Other signal errors, such as multipath effect, occur locally when the signal to the antenna
is reflected off nearby objects, such as superstructure, masts and funnels.
 Entering the wrong antenna height into the receiver can cause significant errors. E.G. the
difference between the antenna of a large vessel in ballast and sea level.
 Entering the wrong datum can put the vessel's position miles from where it really is. Datum
used in GPS calculations is WGS84. In some areas of the world electronic chart coverage

Confidential to United Marine Training Center, Inc. Page 5-43


Pre Sea Review Materials <course code>
Version Number
Deck
5 November 2018

is by raster charts (scanned paper charts) alone. The datum of many raster charts is not
WGS84.
 When GPS shows a compass course, it is not showing the ship's heading, it is showing
the track of the vessel - where she has been in relation to her current position. With the
vessel stationary, GPS will not provide any directional information.

Most electronic compasses (GPS and gyro compasses are two exceptions) are not affected by
magnetic deviation. They are also reliant on a power supply. Electronic compasses used for marine
navigation, include:
 GPS Compass - comprising 2, or preferably 3, antennas aligned symmetrically fore and
aft, will show the ship's heading, in either true or magnetic form, and is normally accurate
to within +/- one degree on a steady heading. As with all satellite derived data, it is
vulnerable to signal error and reliant on a supply of electricity.
 Fluxgate Compass - uses a number of electrical coils wound on a magnetic core to
detect its alignment with the magnetic meridian. It will also detect any other magnetic
fields around it and is therefore as susceptible to deviation as the standard compass.
 Electro-Magnetic Resistors - used in some electronic compasses to measure the earth's
magnetic field. As the vessel changes direction or alignment with the magnetic meridian,
resistance increases or decreases and is interpreted as heading.
 Gyro Compass - usually fitted on larger vessels. It is set to point true north and does not
use the earth's magnetic field. It is normally accurate to +/- one or two degrees. Modern
fiber optic gyro compasses are continuously corrected by computers, which are updated
from GPS. It can take many hours for a gyro compass to operate correctly from the time it
is switched on, or switched back on, after a power outage.
 Laser and Atomic Compasses - still in early days of development for commercial marine
use but may be commonplace in the not too distant future.

In Summary - State of the Art Technology can be a great asset to the modern seafarer - when it
works properly. As we all know, it sometimes doesn't, and then things can very quickly turn pear
shaped. User error due to inadequate training, fatigue and "information overload" can also
contribute to inaccuracies and misinterpretation of data. Over reliance on electronic navigation aids
leads to complacency and sometimes to disaster.

In recent years, there have been numerous well documented occasions (and many not so well
documented) on which a sudden, unexpected loss of power or the undetected inaccuracy of
electronic instruments has rapidly developed into a serious crisis.

Very often, the ability and readiness to switch to old fashioned "manual" navigation, including the
use of a reliable magnetic compass (and looking out of the window!), has made the difference
between continuing the voyage safely and a major marine incident.

Magnetic Compass Location and Installation

On most large merchant vessels, the Standard compass is installed on the "Monkey Island", i.e.
above the wheelhouse. It is usually viewed from the helm via a viewing tube, similar to a periscope.
Often, electronic repeaters are installed so that compass headings can be viewed around the
wheelhouse. Being installed on the highest deck of the ship enables it to be used for taking bearings
and keeps it as far away from magnetic interference as possible.

Confidential to United Marine Training Center, Inc. Page 5-44


Pre Sea Review Materials <course code>
Version Number
Deck
5 November 2018

Figure 5-1: Magnetic Compass - Source: http://www.compassadjustment.com/

Smaller merchant vessels and warships often have a steering compass installed inside the wheel
house in front of the helm. In a fully enclosed steel wheelhouse a magnetic compass is bound to be
affected by a number of deviating magnetic fields and a certain amount of skill is required on the
part of the compass adjuster to compensate for these.

Ideally, the compass should be installed on the vessel's center line so that deviating magnetic
forces are mostly symmetrical around the compass. On certain vessels, such as aircraft carriers,
some fishing vessels and some modern container ships with a narrow superstructure section, the
compass is offset, and this can create interesting challenges for compass adjusters.

On small vessels the compass is usually located in front of the helm position. Care should be taken
to ensure the compass is installed far enough away from structural members, equipment and
instruments such as radios, speakers, engine rev counters (tachometers), etc., which can produce
strong magnetic fields. A few inches one way or the other can sometimes be the difference between
major and minor deviation.

It should be installed so it is easily readable from the helm and also accessible for adjusting. A great
many modern vessels, particularly luxury motor yachts, have not been designed with this in mind.
On one particular sleek, multi-million-dollar super yacht, it was found that, in order to access the
integral correctors of the flush fitting compass, either the console would need to be partially
demolished or the raked wheelhouse windscreen would have to be removed.

Ideally, the compass should be sited so that bearings of objects and other vessels may be taken.
This is not always practicable, particularly on smaller vessels, in which case other means of taking
bearings should be provided. It should not be forgotten that the compass is a valuable tool in
collision avoidance.
Some vessels have their compass installed in an overhead, deck head mounted position. A number
of manufacturers produce compasses which can be mounted in this fashion. This has an obvious
advantage in being easy to read close to eye level. In an "upside down" type, such as that pictured
above, it also means that air bubbles in the compass liquid are not such a problem. It is also away
from a lot of the deviating magnetic fields often found around a console mounted compass.

Suffice to say, all fastenings used to install the compass should be of non-ferrous, non-magnetic
material, e.g. bronze or marine grade stainless steel.

Confidential to United Marine Training Center, Inc. Page 5-45


Pre Sea Review Materials <course code>
Version Number
Deck
5 November 2018

Variation and Deviation

MAGNETIC VARIATION (or DECLINATION) is the difference between True North and Magnetic
North. It is due to:
1. The earth's magnetic field, which travels from South to North, not travelling in a straight line. In
some locations, variation can be in excess of 30 degrees. In some locations it is zero.

2. The Magnetic North and South Poles being located considerable distances from the
Geographic North and South Poles respectively. (The Magnetic North Pole is over 1,000 miles
from the Geographic North Pole and this distance is currently increasing by about 40 miles a
year).

Figure 5-2: Magnetic Chart – Source: http://www.compassadjustment.com/

The map shows the world's magnetic variation values.


Red lines indicate easterly variation.
Blue lines indicate westerly variation.
Green lines indicate zero variation.
The compass is said to be pointing magnetic north when it is perfectly aligned with the earth's
magnetic field - along the magnetic meridian. Therefore, the direction of magnetic north will vary
between zero degrees and in excess of 30 degrees to east or west of true north, depending on the
location.

COMPASS DEVIATION is the difference between magnetic north and the direction in which the
compass is pointing. Both variation and deviation are measured in degrees east (+) or west (-).
Easterly deviation should be added to the compass heading to give the magnetic heading and
westerly deviation should be subtracted.

Remember: ''ERROR EAST - COMPASS LEAST''


Similarly, easterly variation must be added to the magnetic heading to give the true heading and
westerly variation must be subtracted.

Causes of Deviation
All vessels have numerous magnetic fields. Some of these fields are permanently built into the
structure of the vessel and some are caused by the type of cargo carried, electronic instruments,
electrical appliances, position of machinery and equipment, etc.

Confidential to United Marine Training Center, Inc. Page 5-46


Pre Sea Review Materials <course code>
Version Number
Deck
5 November 2018

Some cargoes may affect the magnetic compass more than others. These magnetic fields can
combine to cause the compass needle to point away, or deviate, from magnetic north. The amount
of deviation can vary considerably from heading to heading as the vessel's magnetism is influenced
by the earth's own. The vessel's soft iron magnetism changes with the orientation and location of
the vessel and is also known as induced magnetism. Hard iron magnetism remains constant, is built
into the vessel and is also known as permanent magnetism. Equipment, machinery, cargo, etc.
can produce both permanent and induced magnetic fields.

The aim of the compass adjuster is to nullify the effect of the unwanted magnetic fields by placing
correctors (magnets and soft iron) adjacent to the compass. These create equal but opposing
magnetic fields, thus eliminating the deviating fields around the compass, enabling it to align
correctly. Each axis, vertical, longitudinal and athwart ships is treated separately.

Swinging the Compass, Swinging the Ship

Swinging the compass, or swinging the ship (as the operation is sometimes more accurately
called as the ship swings around the compass card which, ideally, remains pointing north), involves
taking the vessel to a suitable location in open water with plenty of room for maneuvering. With the
vessel steady on each of the eight primary compass points, existing compass headings or bearings
are compared with what we know the actual magnetic headings or bearings should be, the
difference being the deviation.

During the process, any magnetic fields, created by the ship's structure, equipment, etc., which
cause the compass to deviate are reduced or, if possible, eliminated, by creating equal but opposite
magnetic fields using compensating correctors. These are placed inside the compass binnacle or
adjacent to the compass:
 Magnets are aligned fore and aft and athwart ships to create horizontal magnetic fields to
compensate for the permanent horizontal components of the ship's magnetism.
 Soft iron correcting spheres or plates and the Flinders bar compensate for the induced
magnetism caused by the effect the earth's magnetic field has on the ship's magnetism.
 Heeling error magnets compensate for the vertical component of the ship's magnetism.

The timing and logistics of this operation are often governed by the tide, the weather and other
vessels in the vicinity. The time it takes to swing and adjust the compass is also influenced by the
condition and accessibility of the compass and correctors, the maneuverability of the vessel, the skill
of the helmsman and the complexity of, and reasons for, the deviating magnetic fields involved.

On successful completion of compass swing, a table recording any remaining residual deviation and
a statement as to the good working order of the compass will be issued. A current deviation card /
certificate of adjustment is a legal requirement on all sea going commercial vessels.

Figure 5-3: Shadow Pin – Source:https//safety4sea.com/how-to-maintain-and-adjust-magnetic-compasses/

Confidential to United Marine Training Center, Inc. Page 5-47


Pre Sea Review Materials <course code>
Version Number
Deck
5 November 2018

Deviation can be determined by a number of methods: the sun's azimuth or known bearings of
distant objects, such as a mountain peak or lighthouse are considered most accurate. In certain
circumstances, such as poor visibility, calibration is carried out by making comparisons with other
navigation instruments, such as a gyro or GPS compass.

Using other navigation instruments to find deviation is only satisfactory if the absolute accuracy of
these instruments has first been verified, or any known error is factored into the calculations. Most
professionals prefer something tangible, such as a fixed landmark, with a known position and
bearing to work with.

GPS compasses are normally accurate to within a degree or so with the vessel on a steady
heading but are often useless on a swinging vessel. All navigation instruments, whether portable or
fixed, including GPS compasses, should themselves be checked for error each time they are used
for calibrating a magnetic compass.

Figure 5-4:Signal Flag for Compass adjustment - Source: http://www.compassadjustment.com/

Why can't the compass be swung with the ship alongside?


It should be noted that the compass cannot be adjusted to any degree of verifiable accuracy without
swinging the ship. Deviation must be observed and required adjustments made with the ship's head
steady on numerous headings. This requires the vessel to be in open water, clear of other vessels
and away from possible sources of magnetic interference such as cranes, steel piles, reinforced
concrete jetties, etc.

Some preliminary adjustments, based on a detailed analysis of compass deviation history (if
available), may be made prior to sailing. Other adjustments, if made with the vessel alongside, will
be largely based on guess work and cannot be relied upon until the compass has been fully swung.
The compass adjuster must be on board the vessel for the compass swing!

A few compass adjusters will claim that, because of their "expertise", there is no need for them to go
to the trouble of swinging the ship. Large discrepancies between actual deviation and that
"predicted" by the adjuster, sometimes as much as 30 degrees, have been observed on compasses
which have been "expertly adjusted" without swinging. A valid deviation card cannot be issued until
the compass has been properly swung.

Confidential to United Marine Training Center, Inc. Page 5-48


Pre Sea Review Materials <course code>
Version Number
Deck
5 November 2018

Why Does a Compass Card Sometimes Tilt?

The earth's magnetic field travels from the Magnetic South Pole to the Magnetic North Pole.
For mathematical convenience it is divided into two major components: vertical and horizontal. The
closer to the poles: the stronger the vertical component and weaker the horizontal component. At
the magnetic equator the horizontal component is at its strongest and the vertical component is
zero.

Figure 5-5:Earth magnetic Field – Source: http://www.compassadjustment.com/

The angle of the magnetic field in relation to the horizontal is known as MAGNETIC INCLINATION
or MAGNETIC DIP.

In the south, the magnetic field comes up, out of the earth and in the north, it goes back down, into
the earth. As the compass needle will naturally align itself with the earth's magnetic field and is
integral with the card of a marine compass, the upwards or downwards magnetic force will cause
the compass card to tilt. The closer to the poles: the stronger the upwards or downwards force and
the greater the tilt. To counter this, the card of a boat compass usually has a small counter-weight
attached to enable it to sit level.

A compass specifically designed for Northern Hemisphere use will have a weight positioned to
counter the downward magnetic force. When this compass is brought to the Southern Hemisphere,
the combination of the weight and the upwards magnetic force will create an exaggerated tilt on the
card. Obviously, the same thing will happen to a Southern Hemisphere compass when it goes to the
north. Sometimes the tilt is so great the compass becomes inoperative.

Rebalancing the compass for the opposite hemisphere involves dismantling the compass and
moving the weight to the opposite side of the card and is not usually considered economically viable
or indeed practicable. Travelling between the higher latitudes of one hemisphere to the other,
carrying two inter-changeable compasses, one balanced for each hemisphere might be advisable.

Other reasons for compass card tilt:


 Heeling error magnets (if fitted) require adjustment
 Damaged card float chamber
 Damaged jewel pivot
 Card dislodged from pivot
 Low liquid / fluid level

Confidential to United Marine Training Center, Inc. Page 5-49


Pre Sea Review Materials <course code>
Version Number
Deck
5 November 2018

Why Engage a Professional, Qualified Compass


Adjuster?

Effective correction, or compensation, of the marine compass for any deviation error found during
the compass swing requires an understanding of the earth's and ship's magnetic fields and an ability
to differentiate between the permanent magnetism of the ship's hard iron and the induced
magnetism of the ship's soft iron.

It is necessary to recognize the effect the various magnetic fields have on the ship's compass and to
have a practical knowledge of the workings of the marine compass and its correctors. Simply
reducing or eliminating compass deviation on a vessel in one location can actually make it worse
when the vessel travels to another location, particularly when substantial changes in latitude are
involved.

Whilst amateur or DIY compass adjusting is not a completely outrageous concept on pleasure craft,
it has been known to transform a relatively simple problem into a fairly complex one, particularly on
steel vessels.

Most licensed compass adjusters are highly skilled technicians, professional seafarers and qualified
navigators who have undertaken rigorous and comprehensive training to meet national and
international standards.

National marine agencies specify that commercial vessels have their compass adjusted only by a
person qualified and authorized to do so. International standards for magnetic compasses and
compass adjusting are governed by the International Organization for Standardization (ISO) and the
International Maritime Organization (IMO) SOLAS 74 Convention.

Figure 5-6: Compass liquid - Source: http://www.compassadjustment.com/

From time to time an air bubble may appear in the damping liquid in the bowl of a marine compass.
This is often a result of leakage around the seals between the bowl and the diaphragm or the glass.
Sometimes it indicates damage to the bowl or diaphragm. A small bubble will not in itself affect the
performance of the compass but may partially obscure the compass card. A larger bubble can have
an adverse effect on performance.

Removing the bubble requires some patience as it is necessary to replace the air with liquid. Some
modern, cheaper compasses are sealed units and cannot be refilled. If the compass is refillable and
is leaking a lot of liquid, an attempt at repairing might be made before refilling. Often, particularly in
the case of small cheaper compasses, purchasing a new compass is found to be the most
economical option.

Confidential to United Marine Training Center, Inc. Page 5-50


Pre Sea Review Materials <course code>
Version Number
Deck
5 November 2018

Time to Buy a New Compass


Finding the correct liquid/fluid for the compass can be a problem. It can be one, or a mixture, of
several ingredients. Different manufacturers use different ingredients and some are not compatible
with others. Some are not compatible with the compass and can remove the paint and markings
from the compass card or cause other damage. Some are oil based, some are water/spirit based.

The safest option is to obtain the correct liquid from the manufacturer. Unfortunately, this can be
difficult. Some chandlers will stock "compass liquid" but the ingredients of this are often unknown. If
the required ingredients can be determined, it may be possible to obtain suitable liquid from local
sources, at a much cheaper rate.

To check compatibility, draw some existing liquid out of the bowl with a syringe and mix with a small
amount of the new liquid. It will often be immediately obvious if it is not compatible.

The following are some of the main types of compass liquid ingredients:
 Ethyl alcohol (ethanol) / distilled water
 Isopropanol (rubbing alcohol) / distilled water
 Kerosene (paraffin oil)
 Silicon oil

Legislative Requirements

Over a period of time and after certain events, the addition or removal of equipment or the carriage
of magnetic cargo such as iron ore, the vessel's magnetic fields may change, altering the residual
deviation of the compass. In some circumstances the changes can be quite dramatic.
New steel vessels will have their compass adjusted when first commissioned. It has been known for
a one or two year old vessel to record deviation as high as 30 to 40 degrees, as the residual
magnetic fields created during the building process gradually dissipate.

Navigators of sea going vessels are required to observe and record compass error daily whilst on
passage. These observations are important, not only for safe navigation, but also to assist the
compass adjuster in making an accurate analysis of the causes of deviation, should the compass
require adjustment.

National marine agencies specify that commercial vessels have their compass adjusted only by a
person qualified and authorized to do so. International standards for magnetic compasses and
compass adjusting are governed by the International Organization for Standardization (ISO) and the
International Maritime Organization (IMO) SOLAS 74 Convention and HSC Code (for high speed
craft).

SOLAS Chapter V

1. a properly adjusted standard magnetic compass, or other means, independent of any power
supply, to determine the ship's heading and display the reading at the main steering position.
2. a pelorus or compass bearing device, or other means, independent of any power supply, to
take bearings over an arc of the horizon of 360°".
ISO 25862 : 2009 (E): States that all SOLAS vessels should have their compass swung/adjusted
and a new deviation card issued at maximum two yearly intervals. When a new vessel is
commissioned, compass deviation on any heading should be no more than 3°. Thereafter, deviation
on any heading should be 5° or less.

Confidential to United Marine Training Center, Inc. Page 5-51


Pre Sea Review Materials <course code>
Version Number
Deck
5 November 2018

Vessels transiting the Panama Canal are required by the canal authorities to have had a valid
compass deviation card issued within the previous 12 months.

Many maritime authorities and organizations stipulate that the magnetic compass is to be swung
and adjusted annually. Prudent mariners and vessel operators will always ensure that the compass
is regularly checked and properly adjusted.

In Australia, any Non-SOLAS vessel operating under state survey (USL Code) is required to have
its magnetic compass examined and adjusted by an approved compass adjuster at maximum three
yearly intervals or maximum four yearly intervals if operating under the recently introduced NSCV.

In addition to regular routine checking of the compass for deviation, and adjustment for survey
compliance, all sea going vessels should have their compass inspected, swung and adjusted, and a
new deviation card issued, when any of the following apply:
 On a new vessel
 After periods of lay up
 When a new compass is installed
 When deviation exceeds 5 degrees on any heading
 After trauma, such as lightning strike, grounding, fire, etc.
 When compass performance is unsatisfactory or unreliable
 When a record of compass deviation has not been maintained
 After alterations & additions to vessel's structure & equipment
 After repairs involving welding, cutting, grinding, etc. which may affect the compass
 When electrical or magnetic equipment close to the compass is added, removed or altered
 When compass deviation does not appear to correspond with that shown on deviation
card
 When the validity period of the deviation card set by the national or state marine authority
is due to expire

Problems involving Compass Error


Example 5-1

Your ship is steering 201° PSC, variation for the area is 7°E, and deviation is 4°W. What is the
true course being made good?
Solutions

Variation 7° E
Deviation 4° W
C/ Error 3° E
C/ Course 201°
True Course 204°

C 201°
D (-) 4° W
M 197°
V (+) 7°E
T 204°

Confidential to United Marine Training Center, Inc. Page 5-52


Pre Sea Review Materials <course code>
Version Number
Deck
5 November 2018

Supplementary Exercises
Solve the following the problems
Answer
1] What is the compass course to steer to make good a true 353°
course 002° if the variation is 4°E and deviation is 5°E.?

2] You desire to make a good a true course of 359°. The 359° T


variation for the area is 2°E, and deviation is 3°W. What is
the true course to steer per standard compass to make
good the true course if an easterly wind produces a 1°
leeway?

3] The gyro error of the sun was found to be 1°E. Gyro course 1° E
was to be 091° and compass course was 095° PSC.
Variation on the area is 2°W and deviation is 3°E. Find the
deviation.

4] Given the following data, find the deviation. Dev. = NIL/ 0


G.B. = 310
G.E. =2W
S.C.B. = 319 (Standard Compass Brg)
Var. = 11 W

5] Given the following data, find the deviation. Dev. = 03 E


G.B. = 088
G.E. = 03E
S.C.B. = 079
Var. = 09E

6] Given the following data, find the deviation Dev. = 7 E


G.B. = 180
G.E. = 1E
S.C.B. = 185
Var. = 11W

7] Given the following data, find the deviation Dev. = 1 W


G.B. = 356
G.E. = 2W
S.C.B. =345
Var. = 10E

Confidential to United Marine Training Center, Inc. Page 5-53


Pre Sea Review Materials <course code>
Version Number
Deck
5 November 2018

Gyro Compass

Brief History

Mechanical gyroscopes are based on a principle discovered in the 19th century by Jean-Bernard-
Léon Foucault, a French physicist who gave the name gyroscope to a wheel, or rotor, mounted in
gimbal rings. The angular momentum of the spinning rotor caused it to maintain its attitude even
when the gimbal assembly was tilted.

Figure 5-7: Jean- Bernard-Leon Foucault – Source: https://de.wikipedia.org/wiki/L%C3%A9on_Foucault

During the 1850s Foucault conducted an experiment using such a rotor and demonstrated that the
spinning wheel maintained its original orientation in space regardless of the Earth’s rotation.
In order to demonstrate the rotation of the Earth without the complication of the dependence on
latitude, Foucault used a gyroscope in an 1852 experiment. The gyroscope's spinning rotor tracks
the stars directly. Its axis of rotation is observed to return to its original orientation with respect to the
earth after one day whatever the latitude, not being subject to the unbalanced Coriolis forces acting
on the pendulum as a result of its geometric asymmetry.
Foucault corroborated the observation by using a spinning top in a similar manner. He placed a
wheel, rotating at high-speed, in a supporting ring in such a way that the axis of the spinning wheel
could move independently of the ring. In fact, the supporting ring moved over the course of a day,
as it was connected to the surface of the rotating Earth. The axis of the wheel remained pointed in
its original direction, confirming that the Earth was rotating in a twenty-four-hour period.
This ability suggested a number of applications for the gyroscope as a direction indicator, and in
1908 the first workable gyrocompass was developed by the German inventor Hermann Anschütz-
Kaempfe for use in a submersible.
In 1909 the American inventor Elmer A. Sperry built the first automatic pilot using a gyroscope to
maintain an aircraft on course. The first automatic pilot for ships was installed in a Danish
passenger ship by a German company in 1916, and in that same year a gyroscope was used in the
design of the first artificial horizon for aircraft.

Figure 5-8: Elmer A. Sperry - Source: https://de.wikipedia.org/wiki/L%C3%A9on_Foucault

Meanwhile, in 1913, C. Plath (a Hamburg, Germany-based manufacturer of navigational equipment


including sextants and magnetic compasses) developed the first gyrocompass to be installed on a
commercial vessel. C. Plath sold many gyrocompasses to the Weems’ School for Navigation in
Annapolis, MD, and soon the founders of each organization formed an alliance and became Weems
& Plath.

Confidential to United Marine Training Center, Inc. Page 5-54


Pre Sea Review Materials <course code>
Version Number
Deck
5 November 2018

A Gyro compass is a form of gyroscope, used widely on ships employing an electrically powered,
fast-spinning gyroscope wheel and frictional forces among other factors utilizing the basic physical
laws, influences of gravity and the Earth’s rotation to find the true north.

Construction
Gyro compass has become one indispensable instrument in almost all merchant ships or naval
vessels for its ability to detect the direction of true north and not the magnetic north. It is comprised
of the following units:
 Master Compass: Discovers and maintains the true north reading with the help of
gyroscope.
 Repeater Compasses: Receive and indicate the true direction transmitted electrically
from the Master Compass.
 Course Recorder: Makes a continuous record of the maneuvering on a moving strip of
paper.
 Control Panel: Governs the electrical operation of the system and ascertains the running
condition by means of a suitable meter.
 Voltage Regulator: Maintains constant supply of the ship to the motor-generator.
 Alarm Unit: Indicates failure of the ship’s supply.
 Amplifier Panel: Controls the follow-up system.
 Motor Generator: Converts the ship’s DC supply to AC and energizes the Compass
equipment.

Gyro compasses are linked to the repeater compasses via one transmission system. The fast-
spinning rotor attached weighs from 1.25 pounds to 55 pounds. It is driven thousands of revolutions
per minute by another electric motor. However, the most essential part in a Gyro compass system is
the spinning wheel, which is known as the Gyroscope.
External magnetic fields which deflect normal compasses cannot affect Gyro compasses. When a
ship alters its course the independently driven framework called ‘Phantom’ moves with it, but the
rotor system continues to point northward. This lack of alignment enables it to send signal to the
driving motor, which moves the phantom step in with the rotor system again in a path where the
phantom may have crossed only a fraction of a degree or several degrees of the compass circle. As
soon as they are aligned, electrical impulses are sent by the phantom to the repeater compasses for
each degree it traverses.

Confidential to United Marine Training Center, Inc. Page 5-55


Pre Sea Review Materials <course code>
Version Number
Deck
5 November 2018

Figure 5-9: Construction of a Compass - Source: https://www.marineinsight.com/marine-navigation/gyro-compass-on-


ships-construction-working-and-usage/

The Gyroscope in the Gyrocompass is mounted in such a way so that it can move freely about three
mutually perpendicular axes and is controlled as to enable its axis of spin settled parallel with the
true meridian, influenced by the Earth’s rotation and gravity. The Gyro compass system applications
are based upon two fundamental characteristics, which are:
 Gyroscopic Inertia: The tendency of any revolving body to uphold its plane of rotation.
 Precession: A property that causes the gyroscope to move, when a couple is applied. But
instead of moving in the direction of the couple, it moves at right angles to the axis of the
applied couple and also the spinning wheel.
These two properties and the utilization of the Earth’s two natural forces, rotation and gravity, enacts
the Gyrocompass seek true north. Once settled on the true meridian the rotor indefinitely will remain
there as long as the electrical supply of the ship remains constant and unaltered and unaffected by
external forces.

Usage and Errors


Gyro compasses are pre-eminently used in most ships in order to detect true north, steer, and find
positions and record courses. But due to the ship’s course, speed and latitude, there could appear
some steaming errors. It has been found that on Northerly courses the Gyro compass north is
slightly deflected to the West of the true meridian whereas on Southerly courses it is deflected to the
East.
Modern ships use a GPS system or other navigational aids feed data to the Gyrocompass for
correcting the error. An orthogonal triad of fiber optic design and also ring laser gyroscopes which
apply the principles of optical path difference to determine rate of rotation, instead of depending
upon mechanical parts, may help eliminate the flaws and detect true north.

Confidential to United Marine Training Center, Inc. Page 5-56


Pre Sea Review Materials <course code>
Version Number
Deck
5 November 2018

Nautical Charts and Publications

Chart Constructions

Chart Projections
Chart is a representation intended primarily for navigation. Chart projections:
 represent a spherical surface on a plane
 are expressed as mathematical formula for converting geographical coordinates on the
spheroids to plane coordinates on the charts.

Figure 5-10: Chart Projection – Source: ttps://centrodemidias.am.gov.br/dmdocuments/18J4GEO004P2.pdf

Properties of Chart Projection


 True shape of a physical features
 Correct angular relationship
 Equal areas (represents areas in proper proportions)
 Constant scale values
 Great circles represented as straight lines
 Rhumb lines represented as straight lines

Classifications of Chart Projections


Charts projections are classified according to:
 Class (cylindrical, conical or azimuthal)
 Point of secancy (Tangent or secant)
 Aspect (normal, transverse or oblique)
 Distortion Property (equivalent, equidistant or conformal).

Confidential to United Marine Training Center, Inc. Page 5-57


Pre Sea Review Materials <course code>
Version Number
Deck
5 November 2018

Distortions
Converting a sphere to a flat surface results in distortion. This is the most profound single fact about
map projections—they distort the world—a fact that you will investigate in more detail in Module 4,
Understanding and Controlling Distortion.
Imagine a map projection as an attempt to reconstruct your face in two dimensions. Some maps will
get the shapes of all your features just right, but not the sizes—your forehead and chin, for instance,
may come out huge. Other maps will get the sizes right, but the shapes will be stretched—maybe
your full, round mouth will appear wide, thin, and rather mean.
Some maps preserve distances. Measurements from the tip of your nose to your chin, ears, and
eyes will be right, even though the size and shape of your features is wrong. Other maps preserve
direction. Your features may look weird, and they may be scrunched up or set too far apart, but their
relative positions will be correct.
Finally, some maps are compromises—they get nothing exactly right but nothing too far wrong. In
particular, compromise projections try to balance shape and area distortion.
So the four spatial properties subject to distortion in a projection are
 Shape
 Bearing
 Scale
 Area

Figure 5-11: Distortion – Source: https://www.e-education.psu.edu/geog160/node/1918

Confidential to United Marine Training Center, Inc. Page 5-58


Pre Sea Review Materials <course code>
Version Number
Deck
5 November 2018

Distortion Property
1. Conformal (Orthomorphic). The angles are identical between lines in the map to the angles
between the original lines on the curved reference surface.
2. Equidistant. The lengths are the same of particular lines in the map as the length of the
original lines on the curved reference surface (distances are preserved).

Information on Nautical Charts

Chart Number
Shown outside the bottom right hand and top left hand corner of the chart, and in the thumb-level on
the reverse of the chart

Chart Title
Shown in the most convenient place so that no essential navigational information is obscured, and
in the thumb-level on the reverse of the chart

Soundings
Given under the title of the chart

Source Data Diagram


Indicates the source, date and scale of the survey in each part of the chart

Satellite Derived Position


The datum shift, indicating the amount by which a position obtained from satellite navigation system
should be move to agree with the chart

Date of Publication
Shown outside the bottom border of the chart in the middle

Large Correction
Until 1972, chart where revised by either: New Edition - revised throughout
Large Correction - portion of the chart was revised appears to the right of the date of publication
under the date of new edition (if any) Example: Large Correction 10th February 1969

Small Correction
Gives essential information for navigation published in Admiralty Notice to Mariner or, information of
secondary importance which is added to the chart plates by bracketed correction appears outside
the bottom left hand corner of the chart

Date of Printing
Shown by the date in the thumb-label on the reverse of the chart. Example: Printed November 1980

Confidential to United Marine Training Center, Inc. Page 5-59


Pre Sea Review Materials <course code>
Version Number
Deck
5 November 2018

Chart Dimensions
the figure in the parentheses shown outside the lower right hand border of the chart.

Corner Coordinates
Coordinates expressing the latitude and longitude of the limits of admiralty charts published after
1972. shown on the upper right and lower left corner of the chart

Chart Scales

 The natural scale is shown beneath the title


a. Harbor Charts
 for navigation and anchorage in harbor and small waterways
 larger than 1:50,000
b. Coastal Charts
 inshore coastwise navigation
 for entering or leaving bays and harbor of considerable width
 navigating in large inland waters
 smaller than 1:50,000 to 1: 150,000
c. General Charts
 coastwise navigation outside outlying reefs and shoal
 plotting dead reckoning on long voyage
 1:150,000 to 1: 600,000
d. Sailing Charts
 smallest scale chart
 planning, fixing position at sea
 plotting dead reckoning on long voyage
 smaller than 1:600,000
 shore line and topography are generalized
 only principal navigational light, outer buoy, landmarks are shown

Colors
 Blue tint - shallow water areas
 Green tint - drying height
 Magenta - TSS, pilot station, chart limit, maritime limit, etc.
 Yellow tint - land mass or land area

How Navigational Aids Are Charted

 Little circle at the bottom indicates the charted position of the buoy
 Q.R = describes the light characteristics (i.e. Quick Red)

Confidential to United Marine Training Center, Inc. Page 5-60


Pre Sea Review Materials <course code>
Version Number
Deck
5 November 2018

 R = the color of the buoy


 No 4 = name of the buoy which may be written on it
 Magenta Flash shows it is lit
 Black star with Magenta Flash = Indicates the charted position of lighthouse
 Iso.WR.15s = Light characteristics
 22m = Height of light above reference plane
 16/14M = Range of Light

Figure 5-12: Navigational Aids - Source: Snip from MARES

Nautical Publications
 The Mariner’s Handbook (NP100)
 Admiralty Ocean Passages for the World (NP136)
 Admiralty Distance Tables (NP350)
 Admiralty Sailing Directions (NP 1-74)
 Admiralty List of Lights and Fog Signals
 Admiralty List of Radio Signals Vol. 1-6
 Admiralty List Tide Tables
 Catalogue of Admiralty Charts and Publications
 Chart 5011

Chart Correction

SOLAS Chapter V – Regulation 27


Nautical charts and nautical publications, such as sailing directions, lists of lights, notices to
mariners, tide tables and all other nautical publications necessary for the intended voyage, shall be
adequate and up to date.

Tools Needed
 Pens
 0.18 mm - insert information
 0.25 mm - delete information
 Ink - color should be violet

Confidential to United Marine Training Center, Inc. Page 5-61


Pre Sea Review Materials <course code>
Version Number
Deck
5 November 2018

 Pencils - use soft pencil e.g. 2B; 7H


 Soft Eraser
 Adhesive - for NM block
 Parallel Ruler
 Compass divider
 Hacksaw Blades
 Straight Edge
 Templates

Publications Needed
 NP 133A - Chart Correction Log
 NP 234 - Cumulative List of Admiralty Notice to Mariners
 NP 247 - Annual Summary of Admiralty Notice to Mariners
 NP 131 - Chart Catalogue
 CHART 5011 - Symbols and Abbreviations

How to Apply Updates

 NM should be logged in NP 133A


 Always refer to NTM - primary source
 Always INSERT before you DELETE information
 TRACING is only a guide to illustrate the update and to pin point a position
 TRACING is a tool to be used to position an update accurately and quickly

How to Avoid Mistakes

 using the information in the NM (in conjunction with tracing) apply the update to the chart
 check what you have done
 write the NM number in the bottom left hand of the chart
 cross through the NM number adjacent to the chart in NP 133A

Terms used in Notice to Mariners

 INSERT - insertion of new data


 AMEND - changes in characteristics
 SUBSTITUTE - when one feature replaces an existing feature
 MOVE - position to be moved
 DELETE - features are to be removed

Confidential to United Marine Training Center, Inc. Page 5-62


Pre Sea Review Materials <course code>
Version Number
Deck
5 November 2018

IALA

International Association of Marine Aids to Navigation and Lighthouse Authorities

Aids to Navigation
 IALA Maritime Buoyage System
 Sector Light
 Leading Marks
 Lighthouses
 Beacons

Ranges of Lights
 Geographic Range
 Nominal Range
 Luminous Range

Applies to all fixed or floating marks other than:


 light house
 sector lights
 leading lights and marks
 light vessels
 Large Automatic Navigational Buoy (Lanby buoy)

IALA indicates the following:


 Center line of the channel and their sides
 Natural dangers such as sand bank and wreck
 Areas where navigation is subject to regulation

Position Fixing Method

Lines of Position

The modern chart shows us positions of many recognizable aids to navigation like churches and
lighthouses, which facilitate the approach to a coastal area. This concept originated from a chart by
Waghenaer and proved a milestone in the development of European cartography. This work was
called “Spieghel der Zeevaerdt” and included coastal profiles and tidal information much like the
modern chart. It enables us to find the angle between the North and for example an offshore
platform, as seen from our position.
Refer to Figure 5-13. Taking a bearing on this oil rig with a compass provides us with a compass
course. This course first needs correction for both variation and - via ship's heading - deviation
before plotting a Line of Position (LOP) in the chart as a true course.
Our position is somewhere along this line.

Confidential to United Marine Training Center, Inc. Page 5-63


Pre Sea Review Materials <course code>
Version Number
Deck
5 November 2018

Compass courses True courses


Figure 5-13: Line of Position – Source: http://www.sailingissues.com/pdfs.html

Ranges

A precise way to obtain a LOP, and without a compass, is to locate two aids to navigation in line.
The map of Laura Island on the right shows four examples of ranges, each consisting of two aids to
navigation

Figure 5-14: To get the LOP - Source: www.sailingissues.com

More distance between the two landmarks enhances accuracy and less distance between the
vessel and the closest aid to navigation also enhances accuracy.
One of these four ranges consists of two lights that are intentionally placed to provide a LOP. These
pairs of lights are called range lights or leading lights. In this case they indicate the approach
towards the marina and mark the channel between the dangerous rocks along a true course of 50°.
When looking towards any leading lights, the nearest one will be lower. Therefore, in the middle of
the channel both lights will appear vertically above each other.

Figure 5-15: Avoiding dangerous wreck - Source: http://www.sailingissues.com/pdfs.html

Confidential to United Marine Training Center, Inc. Page 5-64


Pre Sea Review Materials <course code>
Version Number
Deck
5 November 2018

Position Fix

If two LOPs intersect we can construct a position fix: the ship's position on the earth.
Often however, a triangle occurs when a third LOP is added in the construction. This indicates that
there are errors involved in at least one of the bearings taken. In practice, we should consider each
LOP as the average bearing in a wider sector of for instance 10°.

Figure 5-16: The position fix - Source: http://www.sailingissues.com/pdfs.html

The optimum angular spread is 90° (two objects) or 120° (three objects). Moreover, bearings on
distant objects bring about more uncertainty in our position fix as the sector widens. Finally, if
moving fast you should not put any time between the bearings.
The next example features a nocturnal landfall on Willemsen Island - you are welcome to visit, but
mind the rocks. The position fix is plotted by taking bearings at two light-vessels as their lights
appear over the horizon. The variation is -1° and the ship's compass heading is 190°. Since we use
our steering compass for our bearings, we can use the same deviation table. That means a
deviation of -4° with which we can calculate (cc + var + dev = tc) the true courses.

Construction of LOP
 Compass bearing on Will. N is 72°
 True course is 067°T
 Plot LOP with time & true course
 Compass bearing on Will. S is 173°
 Plot LOP with time
 Draw an ellipse where the LOPs intersect
 Notate time and “Fix” alongside
 Position is 32° 04,2' N, 024° 46,7' E

Confidential to United Marine Training Center, Inc. Page 5-65


Pre Sea Review Materials <course code>
Version Number
Deck
5 November 2018

Figure 5-17: Construction of LOP - Source: http://www.sailingissues.com/pdfs.html

Without a third LOP - forming the dreaded triangle - there is the false suggestion of accuracy.
Yet, instrument errors, erroneous identification of an aid to navigation, sloppy plotting, etc. can and
will cause navigation errors. Therefore, if close to e.g. rocks, you should assume to be at the worst
possible position (i.e. closest to the navigational hazard).
The lines plotted in the chart are always true courses and these are labelled with true courses by
default; the “T” is optional. If labelled with the corresponding magnetic course or compass course
add an “M” or “C”, respectively.

Estimated Position
It is sometimes impossible to obtain more than one LOP at a time. To determine the ship's position
with one aid to navigation we can use a running fix. However, if a running fix is not possible, we can
determine an estimated position.
An estimated position is based upon whatever incomplete navigational information is available, such
as a Single LOP, a series of depth measurements correlated to charted depths, or a visual
observation of the surroundings.

Figure 5-18: Estimate Position - Source: http://www.sailingissues.com/pdfs.html

In the example on the right we see an estimated position constructed using a single LOP and the
ship's dead reckoning position (DR). This is done by drawing a line from the DR position at the time
of the LOP perpendicular to the LOP. An EP is denoted by a square instead of an ellipse.

Confidential to United Marine Training Center, Inc. Page 5-66


Pre Sea Review Materials <course code>
Version Number
Deck
5 November 2018

Do not rely on an EP as much as a fix. The scale of reliability, from best to worst:
 Fix
 Running fix
 Estimated position
 DR position

Dead Reckoning
Dead reckoning is a technique to determine a ship's approximate position by applying to the last
established charted position a vector or series of vectors representing true courses and speed. This
means that if we have an earlier fix, we plot from that position our course and “distance travelled
Since then” and deduce our current position.

09:30 We start off with a Fix and plot a DR position for 15 minutes
later.
09:45 Our estimation about our speed and course was correct, so we
don't have to charge the DR position.
10:00 and so on…

S = Speed through water (not over ground)


C = Course through water (not over ground)
T = True course (default)
M = Magnetic course for handheld compass (no deviation
correction)
C = Compass course for steering compass (deviation correction)
Mark with an arrow, a semi-circle (circular arc) and “DR”.

Figure 5-19: Dead Reckoning- Sources: http://www.sailingissues.com/pdfs.html

Dead reckoning is crucial since it provides an approximate position in the future. Each time a fix or
running fix is plotted, a vector representing the ordered course and speed originate from it. The
direction of this course line represents the ship's course, and the length represents the distance one
would expect the ship to travel in a given time. This extrapolation is used as a safety precaution: a
predicted DR position that will place the ship in water 1-meter-deep should raise an eyebrow…
In the example above the true courses are plotted in the chart, and to assist the helmsman these
course lines are labelled with the corresponding compass courses.

Guidelines for dead reckoning:


 Plot a new course line from each new fix or running fix (Single LOP).
 Never draw a new course line from an EP.
 Plot a DR position every time course or speed changes.
 Plot a corrected DR position if the predicted course line proofed wrong, and continue from
there.

Confidential to United Marine Training Center, Inc. Page 5-67


Pre Sea Review Materials <course code>
Version Number
Deck
5 November 2018

Running Fix
Under some circumstances, such as low visibility, only one line of position can be obtained at a
time. In this event, a line of position obtained at an earlier time may be advanced to the time of the
later LOP. These two LOPs should not be parallel to each other; remember that the optimal angular
spread is 90°. The position obtained is termed a running fix because the ship has “run” a certain
distance during the time interval between the two LOPs.
To use the LOP obtained at an earlier time, we must advance it to the time of the second LOP. This
is done by using the dead reckoning plot. First, we measure the distance between the two DR
positions and draw a construction line, which is parallel to a line connecting the two DR positions.

Note that if there are no intervening course changes between the two DR positions, it's easiest just
to use the course line itself as the construction line.

Now, using the parallel rulers we advance the first LOP along this construction line over the distance
we measured, the intersection is our RFix.

If there is an intervening course change, it appears to make our problem harder. Not so! The only
DR positions that matter are the two corresponding with the LOPs.

Guidelines for advancing a LOP:


 The distance: equal to the distance between the two corresponding DR positions.
 The direction: equal to the direction between the two corresponding DR positions.
 Draw the advanced LOP with a dotted line and mark with both times.
 Label the Running Fix with an ellipse and "RFix" without underlining.

Confidential to United Marine Training Center, Inc. Page 5-68


Pre Sea Review Materials <course code>
Version Number
Deck
5 November 2018

09:16 We obtain a Single LOP on LANBY 1 and plot a corresponding (same time) dead
reckoning position. The estimated position is constructed by drawing the shortest line
between the DR and the LOP: perpendicular.

09:26 No LOPs at all. We tack and plot a DR position.

09:34 We obtain a LOP on LANBY 2. To use the first LOP we advance it over a construction
line between the two corresponding DR positions. We use both its direction &
distance.

Figure 5-20: Running Fix - Sources: http://www.sailingissues.com/pdfs.html

Sailings

Methods of Calculations

Sailings refer to the methods of calculating the distance and course made good by a vessel.
Different types of sailing are used depending on the situation and criterion when they are applicable.
Listed are the basic methods of sailing calculations
 Plane Sailing
 Parallel Sailing
 Traverse Sailing
 Mid-Latitude Sailing
 Mercator Sailing
 Great Circle Sailing
 Composite Great Circle Sailing

Confidential to United Marine Training Center, Inc. Page 5-69


Pre Sea Review Materials <course code>
Version Number
Deck
5 November 2018

Use of Norie’s Table


Contents:
 Traverse Tables
 Meridional Parts
 Diff. Logarithmic Tables
 Other Miscellaneous Tables

Procedure to use for Parallel Sailing


1. Locate the table that has the whole number of degrees of the latitude.

Rule:
 If the latitude is greater than 45º, then it’s located at the bottom of the table and the
column header should be read from the bottom.

2. In the column headed in Italics “D. Lon”, locate the value of Dlon.
3. Extract the vale for Departure in the column header “Dep.” against the value of Dlon.. This
value of Dep. is also the distance in Parallel Sailing

Procedure to use for Plane Sailing


1. Change the course to quadrantal notation
2. Locate the table that has the whole number in degrees of the course
3. In the column headed Dist., locate the value of the given distance.
 If the value distance is not a whole number, then the distance can be multiplied by 10
or 100 to remove the decimal place and the new value used as the distance as the
distance value.
4. Extract the D. Lat and Dep. values against the distance value.
 If the distance value was multiplied by 10 or 100 to remove the decimal place, then the
value of D. Lat and Dep. Must be divided 10 or 100 to get the correct values.
 The D. Lat and Dep. are named, accordingly, the quadrantal course.

Procedure to find Course and Distance from given Dlat and Departure
1. Note the difference between the given D. Lat and Departure

 If the values of Dep. and D. Lat are near to one another, then the course is
approaching 45º.
 If Dep. is greater than D. Lat, the course is greater than 45º;
 If Dep. Is less than the D. Lat, the course is less than 45º
 If the values of D. Lat is nearly double the value of Dep., then the course is
approaching 26º.
 If the value of Dep. is greater than ½ the value of D. Lat, then the course is greater
than 26º; and if the value of Dep. is less than ½ the value of D. Lat, then the course is
less than 26º.

Confidential to United Marine Training Center, Inc. Page 5-70


Pre Sea Review Materials <course code>
Version Number
Deck
5 November 2018

 If the values of Dep. are nearly double the value of D.Lat., then the course is
approaching 64º. Additionally,
 If the value of ½ of Dep. is greater than D. Lat, then the course is greater than 64º; and
if the value of Dep. is less than D. Lat, then the course is less than 64º.

2. Pick out the table you think near the course. Locate the value of D. Lat as closely as
possible, then check the value of Dep. against the value of D.Lat.

3. Compare the extracted value of Dep. From the table with the given one. Remember that
as the course increases, the value of Dep. also increases. Extract the value of distance
and course; some interpolation is needed to get the accurate distance and course.

Procedure to find Departure from corresponding Dlo and vice-versa


1. Calculate the mean latitude or middle latitude; then locate the table which has the whole
number of degrees of the latitude.
2. Use the column with the header D. Lon Dep., which is printed in italics. Look down the
column to locate the nearest value of Departure and extract the value for D. Lon or vice
versa. The interpolation might be needed to get an accurate result.
 If the latitude is not whole number, then repeat the procedure with the table 1º higher,
then interpolate two results to get the final answers.

Traverse Table Limitations


 Applicable to Parallel and Plane Sailings only
 Tables are tabulated from right-angled triangles and covers a distance up to 600 NM
 Tables are named from 0º to 45º at the top of the pages and 45º to 090º at the bottom of
the pages
 Courses expressed in appropriate quadrant

Parallel Sailing
 the inter-conversion of departure and difference of longitude when the vessel is
proceeding due east or due west (090⁰ or 270⁰)
 simplest form of spherical sailing
 the distance travelled due east or west is known as Departure

Mid-Latitude Sailing
 involves the use of the mid or mean latitude for converting departure to difference of
longitude

Mercator Sailing
 provides a mathematical solution of the plot as made on a Mercator chart.
 similar to plane sailing but uses difference of meridional parts and difference of longitude
in place of difference of latitude and departure.

Confidential to United Marine Training Center, Inc. Page 5-71


Pre Sea Review Materials <course code>
Version Number
Deck
5 November 2018

Great Circle Sailing


 involves the solution of courses, distances and points along a great circle between two
points, the earth being regarded as a sphere.
 used when it is desired to take advantage of the shorter distance along the great circle.

Composite Great Circle Sailing


 a modification of great circle sailing to limit the maximum latitude.
 the limiting Latitude is decided by the navigator, which will be used as the vertex for
calculations and formula derivation
 combination of parallel sailing and great circle sailing.

Example 5-2

MV Antares departed from position 37° 08’ 00” N 009°33’ 00” W and arrived at 36° 10’ 00” N
006°02’ 00” W. Calculate the course and distance made by MV Antares using plane sailing.

Solution
Lat1 37° 08’ 00” N Long1 009° 13’ 00” W
Lat2 36° 10’ 00” N Long2 006°02’ 00” W
Dlat 00° 58’ 00” S Dlo 003° 11’ 00” E

𝐿𝑎𝑡1 + 𝐿𝑎𝑡2
𝑀𝑒𝑎𝑛 𝑙𝑎𝑡 =
2
37° 08’ 00” 𝑁 + 36° 10’ 00” 𝑁
𝑀𝑒𝑎𝑛 𝑙𝑎𝑡 =
2
𝑀𝑒𝑎𝑛 𝑙𝑎𝑡 = 36° 39’ 00” 𝑁
𝐷𝑒𝑝 = 𝐷𝑙𝑜 𝑥 cos(𝑀𝑒𝑎𝑛 𝑙𝑎𝑡)
𝐷𝑒𝑝 = 191’ 𝑥 𝐶𝑜𝑠36° 39’ 00”
𝐷𝑒𝑝 = 153.239’

𝐷𝑒𝑝
tan 𝐶0 =
𝐷𝑙𝑎𝑡
153.239’
tan 𝐶0 𝑐𝑜𝑢𝑟𝑠𝑒 =
58’
𝑪𝒐𝒖𝒓𝒔𝒆 = 𝑺 𝟔𝟗° 𝟏𝟔’ 𝟕. 𝟏𝟖” 𝑬
𝐷𝑙𝑎𝑡
𝐷𝑖𝑠𝑡 =
cos 𝐶0
58’
𝐷𝑖𝑠𝑡 =
cos 69° 16’ 7.18”
𝑫𝒊𝒔𝒕 = 𝟏𝟔𝟑. 𝟖𝟒𝟖’

Confidential to United Marine Training Center, Inc. Page 5-72


Pre Sea Review Materials <course code>
Version Number
Deck
5 November 2018

Example 5-3

MV EMTC Lusitania steams between position 51° 13’ 00” N 048° 13’ 00” W and 51° 13’ 00” N
037° 10’ 00” W. Calculate the departure made by MV EMTC Lusitania using parallel sailing

Solution
𝐿𝑜𝑛𝑔1 048° 13’ 00” 𝑊
𝐿𝑜𝑛𝑔2 037° 10’ 00” 𝑊
𝑫𝒍𝒐 𝟎𝟏𝟏° 𝟎𝟑’ 𝟎𝟎” 𝑬

𝐷𝑙𝑜 = 011° 03’ 00” ∙ 60


𝐷𝑙𝑜 = 663’ 𝐸

𝐷𝑒𝑝 = 𝐷𝑙𝑜 ∙ 𝐶𝑜𝑠𝑙𝑎𝑡


𝐷𝑒𝑝 = 663’ ∙ cos 51° 13’ 00”
𝑫𝒆𝒑 = 𝟒𝟏𝟓. 𝟐𝟐𝟖’ 𝑬

Example 5-4

MV EMTC Lusitania steams between position 51° 13’ 00” N 048° 13’ 00” W and 51° 13’ 00” N
037° 10’ 00” W. Calculate the departure made by MV EMTC Lusitania using parallel sailing

Solution
Find the meridional parts,
𝐿𝑎𝑡1
𝑀𝑃1 = 7915.704 log(tan ( ) + 45) − 23.3 sin(𝐿𝑎𝑡1 )
2

35⁰ 27.0’
𝑀𝑃1 = 7915.704 log(tan ( ) + 45) − 23.3 sin(35⁰ 27.0’)
2

𝑴𝑷𝟏 = 𝟐, 𝟐𝟔𝟑. 𝟖𝟒′

𝐿𝑎𝑡2
𝑀𝑃2 = 7915.704 log(tan ( ) + 45) − 23.3 sin(𝐿𝑎𝑡2 )
2

37° 48.5’
𝑀𝑃2 = 7915.704 log(tan ( ) + 45) − 23.3 sin(37° 48.5’)
2

𝑴𝑷𝟐 = 𝟐, 𝟒𝟑𝟗. 𝟒𝟐’

Note: The values of the Meridional Parts can be extracted directly from Table 5 of Norie’s Table

Find 𝐷𝑙𝑎𝑡 and 𝐷𝑙𝑜,

Lat1 = 35⁰ 27.0’ N MP1 = 2263.84 Long1 = 139⁰ 39.0’E


Lat2 = 37⁰ 48.5’ N MP2 = 2439.42 Long2 = 122⁰ 24.0’W
DLat = 02⁰21.5’ N DMP = 175.58’ Dlo = 262⁰ 03.0’W
x 60 -360⁰ 00.0’
Dlat = 141.5’ Dlo = 097⁰ 57.0’E x 60
Dlo = 5, 877’

Confidential to United Marine Training Center, Inc. Page 5-73


Pre Sea Review Materials <course code>
Version Number
Deck
5 November 2018

Find the true course 𝐶0,


𝐷𝑙𝑜
tan 𝐶0 =
𝐷𝑀𝑃

5 877
tan 𝐶0 =
175.58

𝐶0 = 𝑁 88.3° 𝐸 (Direction of course is based on Dlat and Dlo)

𝑪𝟎 = 𝟎𝟖𝟖° 𝑻

Distance 𝐷𝑖𝑠𝑡,

𝐷𝑙𝑎𝑡
𝐷𝑖𝑠𝑡 =
cos 𝐶0
141.5
𝐷𝑖𝑠𝑡 =
cos 88°17.3′

𝑫𝒊𝒔𝒕 = 𝟒 𝟕𝟑𝟕. 𝟐 𝒏𝒂𝒖𝒕𝒊𝒄𝒂𝒍 𝒎𝒊𝒍𝒆𝒔

Confidential to United Marine Training Center, Inc. Page 5-74


Pre Sea Review Materials <course code>
Version Number
Deck
5 November 2018

Supplementary Exercises
Solve the following the problems using mid-lat sailing
Answer
1] MV BCR Prince departed from position 35° 26’ 00” N
178° 13’ 00” W and arrived at 37° 13’ 00” N 177° 01’ 00” Course: N 65° 05’ 29.31” W
E. Calculate the course and distance made by MV BCR Distance: 254.054’
Prince using Plane sailing.

2] Find the arrival position of MV Condor using plane 65° 04’ 45.36” N
sailing if she departed from position 50° 31’ 00” N 007°
13’ 00” W and steams 331° T for 313 miles. 011° 23’ 58.38” W

3] Find the arrival position of MV Dublin Express using


plane sailing if she departed from position 45° 12’ 00” N 39° 31’ 30” N
161° 12’ 00” W and steams 213° T for 406 miles. 166° 11’ 15.66” W

Solve the following the problems using parallel sailing

4] MV Falcon has a difference of longitude of 003° 13’ 00”


corresponding to a departure of 130 miles. Calculate in 47° 39’ 23.05” N or S
what latitude did MV Falcon navigated.

5] MV German Sky steams 090° T from position 31° 21’ 31° 21’ 00” S
00” S 023° 13’ 00” W. Calculate the arrival position of
MV German Sky if she steams 281 miles. 017° 43’ 57.73” W

6] MV Hector steams 270° T from position 33° 00’ 00” N 33° 00’ 00” N
033° 13’ 00” W at 13 knots for 2 days and 13 hours.
Calculate the DR position of MV Hector. 048° 58’ 32.64” W
Solve the following the problems using Mercator sailing

7] Calculate by Mercator sailing the true course and True course =198.6 ° T
distance from position 39° 11’N, 009° 43’W to position
11° 10’N, 020° 15’W. Dist =1773.6 '

8] Calculate by Mercator sailing the True course and Course = 302.4 ° T


distance from position 017° 22’ S 046° 31’ E to position
13° 10’ N 001° 55’ W. Dist =3419'

9] A vessel steams 178° T for 2555 NM from position 32° 21.4 S


10°12’ N 002°01’ E. Find by Mercator sailing the arrival
position. 003° 33.6 E
Solve the following the problems using great circle sailing

10] Your vessel departs Yokohama from position Lat 35⁰


27.0’N, Long 139⁰ 39.0’E bound for San Francisco, CA
at position Lat 37⁰ 48.5’N, Long 122⁰ 24.0’W. Determine GCD = 4472.5 miles
the following: I.C. = 054⁰ T
a. GCD F.C. = 123⁰ T
b. I.C.
c. F.C.

11] M.V. Antares will conduct Great Circle sailing from


position 17° 19’ N, 025° 01’ W to position 25° 42’ N, 075° GCD = 2850.5 NM
35’W. Show your calculations to find the below. Initial course = 289° 17.6’ T
a. GCD (N070°42’ 26”W)
b. I.C. Final course = 269° 35.7’ T
(S089° 35’ 41”W)
c. F.C.

Confidential to United Marine Training Center, Inc. Page 5-75


Pre Sea Review Materials <course code>
Version Number
Deck
5 November 2018

12] Find the GCD and I/Co when sailing from position
latitude 41° 41’ S, longitude 175° 21’ E to latitude 52° 22’
S, longitude 075° 05’ W. Determine the following: GCD = 4082.5 NM
a. GCD Initial Course = 141° 39’ 29” T
b. I.C.

13] Determine your initial course and distance by great circle


sailing when going from Lat 11° 14’ N, Long 125° 03’ E GCD = 9331.58 NM
to Lat 08° 01’ S, Long 079° 34’ W. Show your Initial course = N 84° 39’ 42” E
calculations to find the below. = 84° 39.7’ T
a. GCD Final course = S 99° 31’ 41” E
b. I.C. = 080° 28.3’ T
c. F.C.

Confidential to United Marine Training Center, Inc. Page 5-76


Pre Sea Review Materials <course code>
Version Number
Deck
5 November 2018

Section 6
Bibliography

Note Taking and Interview Tips


Broggy, J., & McClelland, G. (2009). Integrating Concept Mapping into Higher Education: A Case
study with Physics. British Education Research Association (BERA) Conference, 6.
California Polytechnic State University. (n.d.). Note Taking Systems. Retrieved September 2018,
from Cal Poly: https://asc.calpoly.edu/ssl/notetakingsystems
Director, C. (2018, February 8). Note-taking methods. Retrieved September 2018, from Massey
University.
Doyle, A. (2018, September 11). Interview Q & A. Retrieved September 13, 2018, from the balance
careers: https://www.thebalancecareers.com/panel-interview-questions-and-answers-2061158
Friedman, M. C. (2014, October 15). Note-taking tolls and tips. Retrieved September 2018, from
Harvard Initiative for Learning and Teaching: https://hilt.harvard.edu/blog/note-taking-tools-and-
tips
Jackson, A. E. (2017, September 26). 9 Things to Never Do in a Panel Interview. Retrieved
September 14, 2018, from glass door: https://www.glassdoor.com/blog/9-things-to-never-do-in-a-
panel-interview/
Jones, G., & Mort, P. (n.d.). Note Taking Skills. Sydney, Australia.
Lindsay, N. (n.d.). The Firing Squad: How to Survive a Panel Interview. Retrieved September 14,
2018, from the muse: https://www.themuse.com/advice/the-firing-squad-how-to-survive-a-panel-
interview
McLaughlin Library. (n.d.). Note-Taking Skills. Retrieved September 2018, from University of
Guelph.
Oxford Learning. (2017, May 3). How to Take Study Notes: 5 Effective Note Taking Methods.
Retrieved September 2018, from Oxford Learning Never Stop Learning:
https://www.oxfordlearning.com/5-effective-note-taking-methods/
Quast, L. (2014, May 19). Job Seekers: 7 Tips for a Successful Panel Interview. Retrieved
September 13, 2018, from Forbes: https://www.forbes.com/sites/lisaquast/2014/05/19/job-
seekers-7-tips-for-a-successful-panel-interview/#31f52b6a6089
Ross, A. (2015, December). 10 Great Tips to Make a Good Impression at Your Interview. Retrieved
September 14, 2018, from Bite Size Bio: https://bitesizebio.com/6696/10-great-tips-to-make-a-
good-impression-at-your-interview/
Utah State University. (n.d.). Note Taking: Cornell Method. Utah, United States of America.
Western Sydney University Library. (2017, February). Note-taking techniques. Sydney, Australia.

Mathematics
Angle - Units of Measurement of an Angle. Retrieved from:
http://science.jrank.org/pages/368/Angle-Units-measurement-an-
angle.html#ixzz5UMxpwpXJhttp://science.jrank.org/pages/368/Angle-Units-measurement-an-
angle.html
Anwar, N. (2006). Navigation. Livingston, West Lothian: Witherbys Seamanship International Ltd.
Bird, J. (2007). Engineering Mathematics (5th Edition). Netherlands: Elsevier Ltd.
Corner, K., Jackson, L. & Embelton, W. Reed's Marine Engineering and Technology: Mathematics
for Marine Engineers, Vol. 1 (7th Edition). London: Adlard Coles Nautical, 1997.

Confidential to United Marine Training Center, Inc. Page 6-1


Pre Sea Review Materials <course code>
Version Number
Deck
5 November 2018

Definitions of the Trigonometric Functions of an Acute Angle. (2018). Retrieved from:


https://www.themathpage.com/aTrig/definitions-trigonometric.htm
Degree. (2018). Retrieved from: https://en.wikipedia.org/wiki/Degree_(angle)
Directions and Bearings. (2018). Retrieved from:
(https://www.mathsteacher.com.au/year10/ch15_trigonometry/11_directions/23dir.htm
Earl G. E. & Robinson, D. M. (2007). Munro's Mathematics for Deck Officers. Glasgow: Brown, Son
& Ferguson, Ltd., 1999.
Eather, J. (n.d.). Quick Reference. A Math’s Dictionary for Kids. Retrieved 8 10, 2018, from A Math’s
Dictionary for Kids: http://www.amathsdictionaryforkids.com/qr/qr.html
Ellis, W., & Burzynski, D. (2008). Elementary Algebra. Texas: Rice University.
Finding an Angle in a Right Angled Triangle. Retrieved from:
https://www.mathsisfun.com/algebra/trig-finding-angle-right-triangle.html
Frost, A. (2004). Practical Navigation for Officers of the Watch. Glasgow: Brown, Son &Ferguson,
Ltd.
Great Circle Sailing. Retrieved from: http://shipofficer.com/so/wp-content/uploads/2015/02/10.-
Great-Circle-Sailing.pdf
Great Circle Sailing. Retrieved from:
https://geodesyattamucc.pbworks.com/f/GreatCircleComputations.pdf
Hart, W. Plane and Spherical Trigonometry with Applications. Boston: D. C. Heath and Company,
1964.
Johnson, M. How to Solve Word Problems in Algebra: A Solved Problem Approach (2nd Edition).
New York: McGraw-Hill, 2000.
Mercator Sailing. Retrieved from: http://shipofficer.com/so/wp-content/uploads/2015/02/8.-Mercator-
Sailing.pdf
Meridional Difference. Retrieved from: http://thaimaritime1.blogspot.com/
Parallel Sailing. Retrieved from: http://shipofficer.com/so/wp-content/uploads/2015/02/5.-Parallel-
Sailing.pdf
Plane Sailing. Retrieved from: http://shipofficer.com/so/wp-content/uploads/2015/02/6.-Plane-
Sailing.pdf
Pythagoras' Theorem. Retrieved from: https://www.mathsisfun.com/pythagoras.html
Pierce, R. (n.d.). Math Is Fun. Retrieved August 9, 2018, from Illustrated Mathematics Dictionary:
http://www.mathsisfun.com/definitions/index.html
Radian Measure. Retrieved from: http://www.zeihen.com/lesson-13-3-radian-measure.html
Santiago, D. A. (2005). Navigation (A Comprehensive Study Guide for Marine Deck Officers).
Manila, Philippines: SBR Marine Services Corporation.
The Sphere. (2018). Retrieved from: https://www.mathalino.com/reviewer/solid-mensuration-solid-
geometry/sphere
Unit Circle Trigonometry. Retrieved from:
http://www.montereyinstitute.org/courses/DevelopmentalMath/COURSE_TEXT2_RESOURCE/U1
9_L1_T3_text_final.html
Uy, F., Feliciano, F., De La Fuente, R., & Ocampo, J. (2005). Plane and Spherical Trigonometry.
Manila: Merriam & Webster Bookstore, Inc.
Young, Cynthia Y. Trigonometry (2nd Edition). Hoboken, New Jersey: John Wiley & Sons, Inc.,
2010.

Confidential to United Marine Training Center, Inc. Page 6-2


Pre Sea Review Materials <course code>
Version Number
Deck
5 November 2018

Physics
Aaravjindal. (2018). System of Measurement. Retrieved from Wikipedia:
https://en.wikipedia.org/wiki/System_of_measurement
Alshara, D. A. (2015). Fluid Mechanics.
Definition and Mathematics of Work. (1996-2018). Retrieved from The Physics Classroom:
https://www.physicsclassroom.com/class/energy/Lesson-1/Definition-and-Mathematics-of-Work
Drawing Free-Body Diagrams. (1996-2018). Retrieved from The Physics Classroom:
https://www.physicsclassroom.com/class/newtlaws/Lesson-2/Drawing-Free-Body-Diagrams
Hewitt, P. G. (2012). Conceptual Physics. In P. G. Hewitt, Conceptual Physics (11th ed., pp. 104-
105). Florida: Dartmouth Publishing, Inc.
KC, S. (2018). Measurement and Measurement Units in Physics. Retrieved from Sciencetopia:
https://www.sciencetopia.net/physics/measurement-physical-quantity
Newton's Second Law. (1996-2018). Retrieved from The Physics Classroom:
https://www.physicsclassroom.com/class/newtlaws/Lesson-3/Newton-s-Second-Law
Newton's Third Law. (1996-2018). Retrieved from The Physics Classroom:
https://www.physicsclassroom.com/class/newtlaws/Lesson-4/Newton-s-Third-Law
Potential Energy. (1996-2018). Retrieved from The Physics Classroom:
https://www.physicsclassroom.com/class/energy/Lesson-1/Potential-Energy
Power. (1996-2018). Retrieved from The Physics Classroom:
https://www.physicsclassroom.com/class/energy/Lesson-1/Power
The Meaning of Force. (1996-2018). Retrieved from The Physics Classroom:
https://www.physicsclassroom.com/Class/newtlaws/u2l2a.cfm#category
Types of Forces. (1996-2018). Retrieved from The Physics Classroom:
https://www.physicsclassroom.com/class/newtlaws/Lesson-2/Types-of-Forces
Work-Energy Theorem. (2013). Retrieved from Lumen Learning:
https://courses.lumenlearning.com/boundless-physics/chapter/work-energy-theorem/
Young, H. D., Freedman, R. A., & Ford, A. L. (2012). Sear's and Zemanky's University Physics with
Modern Physics. In H. D. Young, R. A. Freedman, & A. L. Ford, Sear's and Zemanky's University
Physics with Modern Physics (13th ed., p. 6). Philippines: Pearson Education South Asia PTE.
LTD.

Terrestrial Navigation
Bowditch, N. (2002). The American Practical Navigator (2002 Bicentennial Ed). Maryland: National
Imagery and Mapping Agency.
Calcutt, D. and Tetley L. (2001). Electronic Navigation Systems (3rd Ed). Great Britain: Lightning
Source UK Ltd.
Frost, A. (2004). Practical Navigation for Officers of the Watch (1st Ed). Great Britain: Brown, Son
and Ferguson, Ltd.
Gnomonic Projections. Retrieved from: http://mathworld.wolfram.com/GnomonicProjection.html
Gyro Compass. Retrieved from:
http://thenauticalsite.com/nauticalnotes/magcompass/mymagcompass- lesson03-
gyrocompass.html
House, D.J. (2006). Navigation for Masters (3rd Ed). London: Witherbyand Co Ltd.
International Maritime Organization. (2011). International Conventions on Standards of Training,
Certification and Watchkeeping for Seafarers as amended (2011 Ed). UK: CPI Books, Ltd.
International Maritime Organization. (2011). International Convention for the Safety of Life at Sea,
as amended (5th Ed). UK: CPI Books, Ltd.

Confidential to United Marine Training Center, Inc. Page 6-3


Pre Sea Review Materials <course code>
Version Number
Deck
5 November 2018

International Maritime Organization. (2000). Resolution MSC.116(73) Recommendation on


Performance Standards for Marine Transmitting Heading Devices (THDs).
International Maritime Organization. (1977). Resolution A.382(X) Magnetic Compasses Carriage
and Performance Standards.
International Maritime Organization. (1979). Resolution A.424(XI) Performance Standards for Gyro-
Compasses.
Magnetic Compass. Retrieved from: http://www.compassadjustment.com/#1
Map Projections. Retrieved from:
http://www.progonos.com/furuti/MapProj/Normal/TOC/cartTOC.html
Nautical Charts. Retrieved from: http://www.nauticalcharts.noaa.gov/mcd/
Nicholls and Brown (1987). Nicholls’s Concise Guide to Navigation Vol. 1 (10th Ed). Great Britain:
Brown, Son and Ferguson, Ltd.
United Kingdom Hydrographic Office. (2006). IALA Maritime Buoyage System (6th Ed). UK: Crown.
United Kingdom Hydrographic Office. (2017). How to Keep Your Admiralty Products Up-To-Date
(10th Ed). UK: Crown.
United Kingdom Hydrographic Office. (2012). Symbols and Abbreviations Used on Admiralty Paper
Charts (5th Ed). UK: Crown

Confidential to United Marine Training Center, Inc. Page 6-4

You might also like